Peds Final Exam

Réussis tes devoirs et examens dès maintenant avec Quizwiz!

In what age group should the nurse expect a child to develop gross motor skills? 1 Birth through infancy 2 Early childhood 3 Later childhood 4 Middle childhood

2 Children between 1 and 6 years of age are considered to be in early childhood. This is when motor skills such as walking, talking, and climbing develop. The nurse should assess the child to determine whether the child is progressing appropriately. The period from birth to 12 months is called infancy. This is when the relationship between parent and child develops. This increases trust in the baby, and gross motor activity is not prominent at this growth stage. Later childhood is from 11 to 19 years of age. It is a period of rapid maturation and a point of entry into adulthood. Middle childhood encompasses the age group from 6 to 10 years old. This is when steady physical, mental, and social development is seen in children.

How should the nurse obtain cultural knowledge? 1 By interacting with people of different cultures 2 By seeking formal and informal education about various cultures and cultural practices 3 By learning to appreciate different cultural values 4 By seeking training in various nursing interventions

2 Cultural competence has five components. Cultural knowledge is formal and informal education about various cultures and cultural practices. It does not involve interacting with people from different cultures -- this is a cultural encounter. It also does not involve cultural appreciation by the nurse; this is cultural awareness. It does not consist of knowledge of nursing interventions.

Often parents are confused by the terms growth and development and use the terms interchangeably. Based on the nurse's knowledge of growth and development, the most appropriate explanation of development is: 1 a child grows taller all through early childhood. 2 a child learns to throw a ball overhand. 3 a child's weight triples during the first year. 4 a child's brain increases in size until school age.

2 Development is the mental and cognitive attainment of skills. Growth is the increase in physical size—both height and weight.

The nurse finds that a child under care for a gastrostomy experienced fecal incontinence. What is a priority intervention by the nurse? 1 Use a disinfectant immediately on the skin. 2 Gently clean the skin and remove moisture. 3 Elevate the bed no more than 30 degrees. 4 Use an adhesive remover to remove fecal matter

2 Fecal incontinence, especially when mixed with urine, wound drainage, or gastric drainage around gastrostomy tubes can erode the epidermis. The nurse should gently clean the skin and remove the excess moisture. A disinfectant is used when there is a risk of infection and the skin barrier is damaged. Elevating the bed to no more than 30 degrees for short periods prevents friction injuries. Adhesive remover is used to remove adhesives and may not be effective for removing fecal matter.

The nursing instructor is teaching a group of students about gastrostomy feeding. Which statement by the student indicates a need for additional teaching? "It is used: 1 "...when tube passage is not possible through the mouth." 2 "...when tube passage is possible through the pharynx." 3 "...when tube passage is not possible through the cardiac sphincter." 4 "....to avoid the constant irritation of an NG tube."

2 Feeding by way of gastrostomy, or G tube, is often used for children in whom passage of a tube through the pharynx, mouth, esophagus, and cardiac sphincter of the stomach is contraindicated or impossible. It is also used to avoid the constant irritation of an NG tube in children who require tube feeding over an extended period.

The nurse is caring for an infant on gastrostomy feeding. The nurse gives the infant a small and safe pacifier to suck on. What is the rationale behind this? 1 Nutritive sucking is essential during gastrostomy feeding. 2 It keeps the child from crying too much. 3 It prevents the risk of aspiration during gastrostomy feeding. 4 It enhances the nutritive value of gastrostomy feeding.

2 Infants are giving a pacifier as non-nutritive sucking can help decrease crying and aid in weight gain. Pacifiers do not increase the risk of aspiration as children suck on them. Pacifiers do not mix with gastrostomy feedings and cannot enhance the nutritive value of gastronomy feeding.

Early detection of a hearing impairment is critical because of its effect on areas of a child's life. The nurse should evaluate further for effects of the hearing impairment on: 1 reading development. 2 speech development. 3 relationships with peers. 4 performance at school

2 The ability to hear sounds is essential for the development of speech. Babies imitate the sounds that they hear. The child will have greater difficulty learning to read, but the primary issue of concern is the effect on speech. Relationships with peers and performance at school will be affected by the child's lack of hearing. The effect will be augmented by difficulties with oral communication.

Which statement helps explain the growth and development of children? 1 Development proceeds at a predictable rate. 2 The sequence of developmental milestones is predictable. 3 Rates of growth are consistent among children. 4 At times of rapid growth, there is also acceleration of development

2 There is a fixed, precise order to development. There are periods of both accelerated and decelerated growth and development. Each child develops at his or her own rate. Physical growth and development proceed at differing rates. Test-Taking Tip: Do not worry if you select the same numbered answer repeatedly, because there usually is no pattern to the answers.

Several blood tests are ordered for a preschool child with severe anemia. The child is crying and upset because of memories of the venipuncture done at the clinic 2 days ago. The nurse should explain: a. venipuncture discomfort is very brief. b. only one venipuncture will be needed. c. topical application of local anesthetic can eliminate venipuncture pain. d. most blood tests on children require only a finger puncture because a small amount of blood is needed.

ANS: C Preschool children are concerned with both pain and the loss of blood. When preparing the child for venipuncture, the nurse will use a topical anesthetic to eliminate any pain. This is a traumatic experience for preschool children. They are concerned about their bodily integrity. A local anesthetic should be used, and a bandage should be applied to maintain bodily integrity. The nurse should not promise one attempt in case multiple attempts are required. Both finger punctures and venipunctures are traumatic for children. Both require preparation.

The nurse is conducting a staff in-service on sickle cell anemia. Which describes the pathologic changes of sickle cell anemia? a. Sickle-shaped cells carry excess oxygen. b. Sickle-shaped cells decrease blood viscosity. c. Increased red blood cell destruction occurs. d. Decreased adhesion of sickle-shaped cells occurs.

ANS: C The clinical features of sickle cell anemia are primarily the result of increased red blood cell destruction and obstruction caused by the sickle-shaped red blood cells. Sickled red cells have decreased oxygen-carrying capacity and transform into the sickle shape in conditions of low oxygen tension. When the sickle cells change shape, they increase the viscosity in the area where they are involved in the microcirculation. Increased adhesion and entanglement of cells occurs.

Which clinical manifestations in an infant would be suggestive of spinal muscular atrophy (Werdnig-Hoffmann disease)? a. Hyperactive deep tendon reflexes b. Hypertonicity c. Lying in the frog position d. Motor deficits on one side of body

ANS: C The infant lies in the frog position with the legs externally rotated, abducted, and flexed at knees. The deep tendon reflexes are absent. The child has hypotonia and inactivity as the most prominent features. The motor deficits are bilateral.

A child is playing in the playroom. The nurse needs to take a blood pressure on the child. Which is the appropriate procedure for obtaining the blood pressure? a. Take the blood pressure in the playroom. b. Ask the child to come to the exam room to obtain the blood pressure. c. Ask the child to return to his or her room for the blood pressure, then escort the child back to the playroom. d. Document that the blood pressure was not obtained because the child was in the playroom.

ANS: C The playroom is a safe haven for children, free from medical or nursing procedures. The child can be returned to his or her room for the blood pressure and then escorted back to the playroom. The exam room is reserved for painful procedures that should not be performed in the childs hospital bed. Documenting that the blood pressure was not obtained because the child was in the playroom is inappropriate.

The nurse is planning care for an 8-year-old child with a concussion. Which is descriptive of a concussion? a. Petechial hemorrhages cause amnesia. b. Visible bruising and tearing of cerebral tissue occur. c. It is a transient and reversible neuronal dysfunction. d. A slight lesion develops remotely from the site of trauma.

ANS: C A concussion is a transient, reversible neuronal dysfunction with instantaneous loss of awareness and responsiveness resulting from trauma to the head. Petechial hemorrhages along the superficial aspects of the brain along the point of impact are a type of contusion, but are not necessarily associated with amnesia. A contusion is visible bruising and tearing of cerebral tissue. Contrecoup is a lesion that develops remote from the site of trauma as a result of an acceleration-deceleration injury.

2. A nursing student asks the faculty member to explain an oncogene. Which response by the faculty member is the most appropriate? A. A cell that changes into a malignancy after environmental stress B. Any gene found inside a solid tumor that can be removed for biopsy C. A gene in a virus that encourages malignant transformation in cells D. An inherited gene that is programmed to become a malignant cell

ANS: C An oncogene is a gene found inside a virus that has the ability to encourage a normal cell to become malignant.

The parents of a terminally ill child are overwhelmed with the volume of telephone calls asking about the child's condition. What suggestion should the nurse give to the family to make communication easier? 1 "Allow open visiting hours for all family members and the child's friends." 2 "Designate one family member or friend to relay information to everyone else." 3 "Instruct the family to create a website for information and to post updates." 4 "Tell the parents that answering calls and e-mails will help keep them busy."

2 To minimize stress on parents, the family should designate one family member or friend to relay information to others. Open visiting hours would increase the workload of the staff and interfere with care. In addition, the parents would not get any rest because of the continual need to entertain friends and family. Suggesting that the family create a website to post updates or asking parents to answer calls to keep busy may not be helpful. The parents do not need more work to do while dealing with the child's condition.

The nurse is teaching a student nurse about a child who only has one X chromosome. What abnormality does the child have? 1 Down syndrome 2 Turner syndrome 3 Fragile X syndrome 4 Contiguous gene syndrome

2 Turner syndrome is the only viable condition that happens as a result of the child missing one X chromosome. Down syndrome occurs when the child has an extra autosome, chromosome 21. Fragile X syndrome is a condition in which the chromosomes are fragile or weak, and it is associated with other changes in the autosomes. Microdeletion or microduplication of chromosome segments is called contiguous gene syndrome.

A child is 50 cm (20 inches) long in the second month of infancy. The nurse checks the baby 2 months later and finds healthy growth in the child. Approximately how long would the baby be at 4 months? 1. 52 cm 2. 55 cm 3. 57 cm 4. 60 cm

2 Until 6 months after birth, infants should grow 2.5 cm every month, so this 50-cm baby would to grow by 5 cm in 2 months. Therefore, the baby should be 55 cm in length by 4 months of age. If the child is only 52 cm, then the nurse should assess the child's nutritional status to determine whether caloric needs are being met. If the child is 57 cm or 60 cm, the nurse should assess the parents' height first. The baby may be longer because of greater than average parental height. If this is not the case, then the nurse should assess the child's endocrine system for growth problems.

The nurse is reviewing Erikson's theory about the autonomy versus shame and doubt stage. The nurse is trying to correlate it to Freud's psychosexual theory. Which stage would the nurse review in Freud's theory? 1 Oral 2 Anal 3 Phallic 4 Latency

2 When the nurse is reviewing the autonomy versus shame and doubt stage in Erikson theory, it refers to a toddler. The corresponding level in Freud's theory for the toddler's psychosexual developmental stage is the anal stage, when the toddler is toilet trained. The oral stage in Freud's theory represents infancy, from birth to 1 year, and is the trust versus mistrust stage in Erikson's theory. The phallic stage in Freud's theory represents early childhood, 3-6 years of age, or initiative versus guilt in Erikson's theory. Latency in Freud's theory represents middle childhood, 6-12 years, or industry versus inferiority in Erikson's theory.

The nurse is preparing to insert a nasogastric (NG) tube for a child with impaired swallowing capacity. Arrange the steps of the procedure in the correct order. 1. Flush the tube with sterile water. 2. Place child supine with head slightly hyperflexed. 3. Measure the tube for approximate length. 4. Stabilize the tube by holding or taping it to the cheek. 5. Warm the formula to room temperature.

2, 3, 4, 5, 1 The child should be in a supine position with head slightly hyperflexed. The tube is measured for approximate length of insertion and marked. After insertion, the tube is stabilized by holding or taping it to the cheek. Warm formula to room temperature before starting the flow. The tube is flushed with sterile water after the feeding.

A neonate had corrective surgery 3 days ago for esophageal atresia. The nurse notices that after the child receives his gastrostomy feeding, there is often a backup of formula feeding into the tube. As a result, the nurse should: 1 position the child in a supine position after feedings. 2 position the child on his or her left side after feedings. 3 leave the gastrostomy tube open and suspended after feedings. 4 leave the gastrostomy tube clamped after feedings.

3 By keeping the tube open to air, the buildup of pressure on the operative site will be prevented. The child should be positioned on the right side with head elevated at approximately 30 degrees. The formula is backing up into the tube because of the delayed emptying. Leaving the tube clamped will create pressure on the operative site.

The nurse is caring for a Buddhist patient. The nurse finds that the patient has not touched the food tray. The food tray holds chicken, bread, soup, and fruits. What is the most appropriate action by the nurse? 1 Start an intravenous (IV) infusion because the patient is not eating. 2 Advise the patient to eat the food because it is essential for health. 3 Ask the patient whether there is a problem with the food provided. 4 Ignore the behavior.

3 Many Buddhists are strictly vegetarian. This may explain why the patient does not touch the meal tray. The nurse should ask the patient what his or her dietary preferences are and provide what the patient wants. However, the nurse should not provide food choices that go against any dietary restrictions placed by the health care provider. It is inappropriate to start an IV line because IV fluids do not provide nutrition. The nurse should not force the patient to eat or ignore the patient.

30. A child needs surgery to resect a tumor, but is scheduled for several weeks of radiation therapy first. The parents are frustrated and want to know why the surgery that can cure the cancer is being delayed. Which response by the nurse is the most appropriate? A. "Children who have radiation first generally do better than others." B. "If the radiation destroys the tumor, surgery will not be needed." C. "Radiation will shrink the tumor, making it easier to get all of it out." D. "The surgeons must be worried that they cannot get the whole tumor."

ANS: C Often radiation or chemotherapy is used prior to surgical resection to shrink the size of the tumor, maximizing the chances of complete removal. The other responses are not accurate.

8. A group of boys ages 9 and 10 years have formed a "boys-only" club that is open to neighborhood and school friends who have skateboards. How should this behavior be interpreted? a. Behavior that encourages bullying and sexism. b. Behavior that reinforces poor peer relationships. c. Characteristic of social development of this age. d. Characteristic of children who later are at risk for membership in gangs.

ANS: C One of the outstanding characteristics of middle childhood is the creation of formalized groups or clubs. Peer-group identification and association are essential to a child's socialization. Poor relationships with peers and a lack of group identification can contribute to bullying. A boys-only club does not have a direct correlation with later gang activity.

The nurse is taking care of a 10-year-old child who has osteomyelitis. Which treatment plan is considered the primary method of treating osteomyelitis? a. Joint replacement b. Bracing and casting c. Intravenous antibiotic therapy d. Long-term corticosteroid therapy

ANS: C Osteomyelitis is an infection of the bone, most commonly caused by Staphylococcus aureus. The treatment of choice is antibiotics. Joint replacement, bracing and casting, and long-term corticosteroid therapy are not indicated for infectious processes.

To establish evidence-based practice, the nurse has to collect high-quality evidence from various sources. Where does the nurse find the best quality of evidence? 1 Observational studies 2 Hospital patient records 3 Randomized clinical trials 4 Direct interview with patients

3 The nurse knows the best source for consistence and unbiased evidence is from well-performed randomized clinical trials (RCT). Observational studies have biased information, so are not reliable, unless the study is done meticulously. RCTs are preferred to hospital records or patient interviews. As biases can affect the establishment of evidence-based practice, the nurse should take precautions to avoid biased or low-quality evidence.

The nurse is recommending how to prevent iron deficiency anemia in a healthy, term, breast-fed infant. Which should be suggested? a. Iron (ferrous sulfate) drops after age 1 month b. Iron-fortified commercial formula by age 4 to 6 months c. Iron-fortified infant cereal by age 2 months d. Iron-fortified infant cereal by age 4 to 6 months

ANS: D Breast milk supplies inadequate iron for growth and development after age 5 months. Supplementation is necessary at this time. The mother can supplement the breastfeeding with iron-fortified infant cereal. Iron supplementation or the introduction of solid foods in a breast-fed baby is not indicated. Providing iron-fortified commercial formula by age 4 to 6 months should be done only if the mother is choosing to discontinue breastfeeding.

Therapeutic management of a child with tetanus includes the administration of: a. nonsteroidal anti-inflammatory drugs (NSAIDs) to reduce inflammation. b. muscle stimulants to counteract muscle weakness. c. bronchodilators to prevent respiratory complications. d. tetanus immunoglobulin therapy.

ANS: D Tetanus immunoglobulin therapy, to neutralize toxins, is the most specific therapy for tetanus. Tetanus toxin acts at the myoneural junction to produce muscular stiffness and lowers the threshold for reflex excitability. NSAIDs are not routinely used. Sedatives or muscle relaxants are used to help reduce titanic spasm and prevent seizures. Respiratory status is carefully evaluated for any signs of distress because muscle relaxants, opioids, and sedatives that may be prescribed may cause respiratory depression. Bronchodilators would not be used unless specifically indicated.

Which is a type of skin traction with legs in an extended position? a. Dunlop b. Bryant c. Russell d. Buck extension

ANS: D Buck extension traction is a type of skin traction with the legs in an extended position. It is used primarily for short-term immobilization, preoperatively with dislocated hips, for correcting contractures, or for bone deformities such as Legg-Calvé-Perthes disease. Dunlop traction is an upper-extremity traction used for fractures of the humerus. Bryant traction is skin traction with the legs flexed at a 90-degree angle at the hip. Russell traction uses skin traction on the lower leg and a padded sling under the knee. The combination of longitudinal and perpendicular traction allows realignment of the lower extremity and immobilizes the hips and knees in a flexed position.

5. The parent of an infant with nasopharyngitis should be instructed to notify the health care professional if the infant: a. Becomes fussy. c. Has a fever over 99 F. b. Has a cough. d. Shows signs of an earache.

ANS: D If an infant with nasopharyngitis has a fever over 101 F, there is early evidence of respiratory complications. Irritability and a slight fever are common in an infant with a viral illness. Cough can be a sign of nasopharyngitis

3. Which statement describes the cognitive abilities of school-age children? a. Have developed the ability to reason abstractly b. Become capable of scientific reasoning and formal logic c. Progress from making judgments based on what they reason to making judgments based on what they see d. Have the ability to classify, group and sort, and hold a concept in their minds while making decisions based on that concept

ANS: D In Piaget's stage of concrete operations, children have the ability to group and sort and make conceptual decisions. Children cannot reason abstractly until late adolescence. Scientific reasoning and formal logic are skills of adolescents. Making judgments on what the child sees versus what he or she reasons is not a developmental skill.

34. Which information should the nurse teach workers at a day care center about respiratory syncytial virus (RSV)? a. RSV is transmitted through particles in the air. b. RSV can live on skin or paper for up to a few seconds after contact. c. RSV can survive on nonporous surfaces for about 60 minutes. d. Frequent hand washing can decrease the spread of the virus

ANS: D Meticulous hand washing can decrease the spread of organisms. RSV infection is not airborne. It is acquired mainly through contact with contaminated surfaces. RSV can live on skin or paper for up to 1 hour and on cribs and other nonporous surfaces for up to 6 hours

Which medication is usually tried first when a child is diagnosed with juvenile idiopathic arthritis (JIA)? a. Aspirin b. Corticosteroids c. Cytotoxic drugs such as methotrexate d. Nonsteroidal anti-inflammatory drugs (NSAIDs)

ANS: D NSAIDs are the first drugs used in JIA. Naproxen, ibuprofen, and tolmetin are approved for use in children. Aspirin, once the drug of choice, has been replaced by the NSAIDs because they have fewer side effects and easier administration schedules. Corticosteroids are used for life-threatening complications, incapacitating arthritis, and uveitis. Methotrexate is a second-line therapy for JIA.

38. Parents have understood teaching about prevention of childhood otitis media if they make which statement? a. We will only prop the bottle during the daytime feedings. b. Breastfeeding will be discontinued after 4 months of age. c. We will place the child flat right after feedings. d. We will be sure to keep immunizations up to date.

ANS: D Parents have understood the teaching about preventing childhood otitis media if they respond they will keep childhood immunizations up to date. The child should be maintained upright during feedings and after. Otitis media can be prevented by exclusively breastfeeding until at least 6 months of age. Propping bottles is discouraged to avoid pooling of milk while the child is in the supine position.

The nurse is caring for a child with severe head trauma after a car accident. Which is an ominous sign that often precedes death? a. Papilledema b. Delirium c. Doll's head maneuver d. Periodic and irregular breathing

ANS: D Periodic or irregular breathing is an ominous sign of brainstem (especially medullary) dysfunction that often precedes complete apnea. Papilledema is edema and inflammation of optic nerve. It is commonly a sign of increased ICP. Delirium is a state of mental confusion and excitement marked by disorientation for time and place. The doll's head maneuver is a test for brainstem or oculomotor nerve dysfunction.

25.An advantage of peritoneal dialysis is that: a. Treatments are done in hospitals. b. Protein loss is less extensive. c. Dietary limitations are not necessary. d. Parents and older children can perform treatments.

ANS: D Peritoneal dialysis is the preferred form of dialysis for parents, infants, and children who wish to remain independent. Parents and older children can perform the treatments themselves. Treatments can be done at home. Protein loss is not significantly different. The dietary limitations are necessary, but they are not as stringent as those for hemodialysis.

The nurse is assessing a child who was just admitted to the hospital for observation after a head injury. Which is the most essential part of the nursing assessment to detect early signs of a worsening condition? a. Posturing b. Vital signs c. Focal neurologic signs d. Level of consciousness

ANS: D The most important nursing observation is assessment of the child's level of consciousness. Alterations in consciousness appear earlier in the progression of an injury than do alterations of vital signs or focal neurologic signs. Neurologic posturing is indicative of neurologic damage. Vital signs and focal neurologic signs are later signs of progression when compared with level-of-consciousness changes.

The nurse is leading an educational program for parents of 5- to 9-year-old children. Which topic should the nurse include in the teaching plan to prevent childhood mortality in children of this age? 1 Suicide 2 Being overweight 3 Heart diseases 4 Unintentional injuries

4 Unintentional injuries and accidents are the leading cause of death in children ages 5 to 9 because this age group have the ability to run and climb and may experience falls, burns, and collisions. Therefore, the nurse should focus mainly on reducing the risk of accidents. Suicide is not a significant cause of mortality in children under the age of 10. Being overweight is a significant problem in childhood, but not the leading cause of deaths in the 5-9 age group. Heart diseases are the most significant causes of mortality in children ages 1-4 years and 15-19 years.

14. The school nurse has been asked to begin teaching sex education in the 5th grade. Which statement should be the foundation for the information the nurse should present? a. Children in 5th grade are too young for sex education. b. Children should be discouraged from asking too many questions. c. Correct terminology should be reserved for children who are older. d. Sex can be presented as a normal part of growth and development.

ANS: D When sex information is presented to school-age children, sex should be treated as a normal part of growth and development. Fifth graders are usually 10 to 11 years old. This age is not too young to speak about physiologic changes in their bodies. They should be encouraged to ask questions. Preadolescents need precise and concrete information.

29. What should the nurse stress in a teaching plan for the mother of an 11-year-old diagnosed with ulcerative colitis? a. Preventing the spread of illness to others b. Nutritional guidance and preventing constipation c. Teaching daily use of enemas d. Coping with stress and avoiding triggers

ANS: D Coping with the stress of chronic illness and the clinical manifestations associated with ulcerative colitis (diarrhea, pain) are important teaching foci. Avoidance of triggers can help minimize the impact of the disease and its effect on the child. Ulcerative colitis is not infectious. Although nutritional guidance is a priority teaching focus, diarrhea is a problem with ulcerative colitis, not constipation. Daily enemas are not part of the therapeutic plan of care.

10. Therapeutic management of most children with Hirschsprung's disease is primarily: a. daily enemas. b. low-fiber diet. c. permanent colostomy. d. surgical removal of affected section of bowel.

ANS: D Most children with Hirschsprung's disease require surgical rather than medical management. Surgery is done to remove the aganglionic portion of the bowel, relieve obstruction, and restore normal bowel motility and function of the internal anal sphincter. Preoperative management may include enemas and low-fiber, high-calorie, high-protein diet until the child is physically ready for surgery. The colostomy that is created in Hirschsprung's disease is usually temporary.

6. A stool specimen from a child with diarrhea shows the presence of neutrophils and red blood cells. This is most suggestive of which condition? a. Protein intolerance b. Parasitic infection c. Fat malabsorption d. Bacterial gastroenteritis

ANS: D Neutrophils and red blood cells in stool indicate bacterial gastroenteritis. Protein intolerance is suspected in the presence of eosinophils. Parasitic infection is indicated by eosinophils. Fat malabsorption is indicated by foul-smelling, greasy, bulky stools

19. Type 1 diabetes mellitus is suspected in an adolescent. Which clinical manifestation may be present? a. Moist skin b. Weight gain c. Fluid overload d. Poor wound healing

ANS: D Poor wound healing is often an early sign of type 1 diabetes mellitus. Dry skin, weight loss, and dehydration are clinical manifestations of type 1 diabetes mellitus.

23. What are the manifestations of hypoglycemia? a. Lethargy b. Thirst c. Nausea and vomiting d. Shaky feeling and dizziness

ANS: D Some of the clinical manifestations of hypoglycemia include shaky feelings; dizziness; difficulty concentrating, speaking, focusing, and coordinating; sweating; and pallor. Lethargy, thirst, and nausea and vomiting are manifestations of hyperglycemia.

25. The nurse is discussing various sites used for insulin injections with a child and her family. Which site usually has the fastest rate of absorption? a. Arm b. Leg c. Buttock d. Abdomen

ANS: D The abdomen has the fastest rate of absorption but the shortest duration. The arm has a fast rate of absorption but short duration. The leg has a slow rate of absorption but a long duration. The buttock has the slowest rate of absorption and the longest duration.

5. Which immunization should be considered carefully before being given to a child receiving chemotherapy for cancer? a. Tetanus vaccine b. Inactivated poliovirus vaccine c. Diphtheria, pertussis, tetanus (DPT) d. Measles, rubella, mumps

ANS: D The vaccine used for measles, mumps, and rubella is a live virus and can result in an overwhelming infection. Tetanus vaccine, inactivated poliovirus vaccine, and diphtheria, pertussis, tetanus (DPT) are not live virus vaccines.

A school nurse is conducting a class with adolescents on suicide. Which true statement about suicide should the nurse include in the teaching session? a. A sense of hopelessness and despair are a normal part of adolescence. b. Gay and lesbian adolescents are at a particularly high risk for suicide. c. Problem-solving skills are of limited value to the suicidal adolescent. d. Previous suicide attempts are not an indication of risk for completed suicides.

B A significant number of teenage suicides occur among homosexual youths. Gay and lesbian adolescents who live in families or communities that do not accept homosexuality are likely to suffer low self-esteem, self-loathing, depression, and hopelessness as a result of a lack of acceptance from their family or community. At-risk teenagers include those who are depressed, have poor problem-solving skills, or use drugs and alcohol. History of previous suicide attempt is a serious indicator for possible suicide completion in the future.

In terms of language and cognitive development, a 4-year-old child would be expected to: a. Think in abstract terms. b. Follow simple commands. c. Understand conservation of matter. d. Comprehend another person's perspective.

B Children ages 3 to 4 years can give and follow simple commands. Children cannot think abstractly at age 4 years. Conservation of matter is a developmental task of the school-age child. A 4-year-old child cannot comprehend another's perspective.

What is descriptive of the preschooler's understanding of time? a. Has no understanding of time b. Associates time with events c. Can tell time on a clock d. Uses terms like "yesterday" appropriately

B In a preschooler's understanding, time has a relation with events such as, "We'll go outside after lunch." Preschoolers develop an abstract sense of time at age 3 years. Children can tell time on a clock at age 7 years. Children do not fully understand use of time-oriented words until age 6 years.

A normal characteristic of the language development of a preschool-age child is: a. Lisp. c. Echolalia. b. Stammering. d. Repetition without meaning.

B Stammering and stuttering are normal dysfluencies in preschool-age children. Lisps are not a normal characteristic of language development. Echolalia and repetition are traits of toddlers' language.

A nurse is caring for an adolescent hospitalized for cellulitis. The nurse notes that the adolescent experiences many "mood swings" throughout the day. The nurse interprets this behavior as: a. Requiring a referral to a mental health counselor. b. Requiring some further lab testing. c. Normal behavior. d. Related to feelings of depression.

C Adolescents vacillate in their emotional states between considerable maturity and childlike behavior. One minute they are exuberant and enthusiastic; the next minute they are depressed and withdrawn. Because of these mood swings, adolescents are frequently labeled as unstable, inconsistent, and unpredictable, but the behavior is normal. The behavior would not require a referral to a mental health counselor or further lab testing. The mood swings do not indicate depression.

Which type of play is most typical of the preschool period? a. Solitary c. Associative b. Parallel d. Team

C Associative play is group play in similar or identical activities but without rigid organization or rules. Solitary play is that of infants. Parallel play is that of toddlers. School-age children play in teams.

A nurse is reviewing hormone changes that occur during adolescence. The hormone that is responsible for the growth of beard, mustache, and body hair in the male is: a. Estrogen. c. Androgen. b. Pituitary hormone. d. Progesterone.

C Beard, mustache, and body hair on the chest, upward along the linea alba, and sometimes on other areas (e.g., back and shoulders) appears in males and is androgen dependent. Estrogen and progesterone are produced by the ovaries in the female and do not contribute to body hair appearance in the male. The pituitary hormone does not have any relationship to body hair appearance in the male.

Vitamin A supplementation may be recommended for the young child who has: a. Mumps. c. Measles (rubeola). b. Rubella. d. Erythema infectiosum.

C Evidence shows that vitamin A decreases morbidity and mortality associated with measles. Vitamin A will not lessen the effects of mumps, rubella, or fifth disease.

Which is the most commonly used method in completed suicides? a. Firearms c. Self-inflected laceration b. Drug overdose d. Carbon monoxide poisoning

A Firearms are the most commonly used instruments in completed suicides among both males and females. For adolescent boys, firearms are followed by hanging and overdose. For adolescent females, overdose and strangulation are the next most common means of completed suicide. The most common method of suicide attempt is overdose or ingestion of potentially toxic substances such as drugs. The second most common method of suicide attempt is self-inflicted laceration. Carbon monoxide poisoning is not one of the more frequent forms of suicide completion.

A nurse is teaching adolescent boys about pubertal changes. The first sign of pubertal change seen with boys is: a. Testicular enlargement. c. Scrotal enlargement. b. Facial hair. d. Voice deepens.

A The first sign of pubertal changes in boys is testicular enlargement in response to testosterone secretion, which usually occurs in Tanner stage 2. Slight pubic hair is present and the smooth skin texture of the scrotum is somewhat altered. As testosterone secretion increases, the penis, testes, and scrotum enlarge. During Tanner stages 4 and 5, rising levels of testosterone cause the voice to deepen and facial hair appears at the corners of the upper lip and chin.

The nurse is caring for an adolescent brought to the hospital with acute drug toxicity. Cocaine is believed to be the drug involved. Data collection should include the: a. Mode of administration. b. Actual content of the drug. c. Function the drug plays in the adolescent's life. d. Adolescent's level of interest in rehabilitation.

A When the drug is questionable or unknown, every effort must be made to determine the type, amount of drug taken, the mode and time of administration, and factors relating to the onset of presenting symptoms. Because the actual content of most street drugs is highly questionable, this information would be difficult to obtain. It is helpful to know the pattern of use but not essential during this emergency. This is an inappropriate time for an evaluation about the level of interest in rehabilitation.

D (casein hydrolysate milk formula.)

A 3-month-old bottle-fed infant is allergic to cow's milk. The nurse's BEST option for a substitute is: A. goat's milk. B. soy-based formula. C. skim milk diluted with water. D. casein hydrolysate milk formula.

D (Peanut butter)

A 9-month-old infant is seen in the emergency department after developing a urticaric rash with cough and wheezing. When collecting the history of events before the sudden onset of the rash with cough and wheezing, the mother states they were "feeding the baby new foods." Which food is the possible cause of this type of reaction in the infant? A. Potatoes B. Green beans C. Spinach D. Peanut butter

C (kwashiorkor.)

A nurse is providing education to a community group in preparation for a mission trip to a third world country with limited access to protein-based food sources. The nurse is aware that children in this country are at increased risk for: A. rickets. B. marasmus. C. kwashiorkor. D. pellagra.

D. ("We stopped using the car seat now that my child is older.")

A parent has a 2-year-old in the clinic for a well-child checkup. Which statement by the parent would indicate to the nurse that the parent needs more instruction regarding accident prevention? A. "We locked all the medicines in the bathroom cabinet." B. "We turned the thermostat down on our hot water heater." C. "We placed gates at the top and bottom of the basement steps." D. "We stopped using the car seat now that my child is older."

B (this is a normal reaction for this age.)

A parent of an 8-month-old infant tells the nurse that the baby cries and screams whenever he or she is left with the grandparents. The nurse's reply should be based on knowledge that: A. the infant is most likely spoiled. B. this is a normal reaction for this age. C. this is an abnormal reaction for this age. D. grandparents are not responsive to that infant.

Which toys should a nurse provide to promote imaginative play for a 3-year-old hospitalized child (select all that apply)? a. Plastic telephone b. Hand puppets c. Jigsaw puzzle (100 pieces) d. Farm animals and equipment e. Jump rope

A, B, D To promote imaginative play for a 3-year-old child, the nurse should provide: dress-up clothes, dolls and dollhouses, housekeeping toys, play-store toys, telephones, farm animals and equipment, village sets, trains, trucks, cars, planes, hand puppets, and medical kits. A 100-piece jigsaw puzzle and a jump rope would be appropriate for a young, school-age child but not a 3-year-old child.

An adolescent teen has bulimia. Which assessment finding should the nurse expect? a. Diarrhea c. Cold intolerance b. Amenorrhea d. Erosion of tooth enamel

D Some of the signs of bulimia include erosion of tooth enamel, increased dental caries from vomited gastric acid, throat complaints, fluid and electrolyte disturbances, and abdominal complaints from laxative abuse. Diarrhea is not a result of the vomiting. It may occur in patients with bulimia who also abuse laxatives. Amenorrhea and cold intolerance are characteristics of anorexia nervosa, which some bulimics have. These symptoms are related to the extreme low weight.

B, C, D (B. who sleep prone C. who were premature D. with prenatal drug exposure)

Infants most at risk for sudden infant death syndrome (SIDS) are those: (Select all that apply.) A. who sleep supine B. who sleep prone C. who were premature D. with prenatal drug exposure E. with a cousin that died of SIDS

The number of deaths of children in community are 24, 86, 100, and 200 respectively for the children under the age of 4 weeks, 4 months, 28 weeks, and 8 months per 2,000 live births. What is the neonatal mortality rate? Record your answer using whole number. _____/1,000 live births

12 Neonatal mortality rate is the number of deaths during the first 28 days of life per 1,000 live births. In this case, death rate of children under 4 weeks (28 days) in this community is 24/2,000 live births so this it is equal to 12/1,000 live births.

Which behavior would most likely be manifested in a young child experiencing the protest phase of separation anxiety? 1 Inactivity 2 Clings to parent 3 Depressed, sad 4 Regression to earlier behavior

2 In the protest phase, the child aggressively responds to separation from parents (such as clinging to a parent). Inactivity is characteristic of despair. Depression and sadness are characteristics of despair. Regression to earlier behavior is characteristic of despair.

B (It is now recommended that all infants and toddlers ride in rear-facing car safety seats until they reach the age of 2 years or the height or weight recommended by the car seat manufacturer. Children 2 years old and older who have outgrown the rear-facing height or weight limit for their car safety seat should use a forward-facing car safety seat with a harness up to the maximum height or weight recommended by the manufacturer. One year is too young to switch to a forward-facing position.)

Kimberly's parents have been using a rearward-facing, convertible car seat since she was born. The parents should be taught that most car seats can be safely switched to the forward-facing position when the child reaches which age? a. 1 year b. 2 years c. 3 years d. 4 years

25. The nurse is caring for a child with persistent hypoxia secondary to a cardiac defect. The nurse recognizes that a risk exists of cerebrovascular accidents (strokes). Which of the following is an important objective to decrease this risk? 1. Minimize seizures 2. Prevent dehydration 3. Promote cardiac output 4. Reduce energy expenditure

ANS: 2 2. In children with persistent hypoxia, polycythemia develops. Dehydration must be prevented in hypoxemic children because it potentiates the risk of strokes

12. Which of the following defects results in decreased pulmonary blood flow? 1. Atrial septal defect 2. Tetralogy of Fallot 3. Ventricular septal defect 4. Patent ductus arteriosus

ANS: 2 2. Tetralogy of Fallot results in decreased blood flow to the lungs. The pulmonic stenosis increases the pressure in the right ventricle, causing the blood to go from right to left across the ventricular septal defect

18. When a child has chronic renal failure, the progressive deterioration produces a variety of clinical and biochemical disturbances that eventually are manifested in the clinical syndrome known as: a. Uremia. b. Oliguria. c. Proteinuria. d. Pyelonephritis.

ANS: A Uremia is the retention of nitrogenous products, producing toxic symptoms. Oliguria is diminished urine output. Proteinuria is the presence of protein, usually albumin, in the urine. Pyelonephritis is an inflammation of the kidney and renal pelvis

3. Decongestant nose drops are recommended for a 10-month-old infant with an upper respiratory tract infection. Instructions for nose drops should include: a. Avoiding use for more than 3 days. b. Keeping drops to use again for nasal congestion. c. Administering drops until nasal congestion subsides. d. Administering drops after feedings and at bedtime.

ANS: A Vasoconstrictive nose drops such as Neo-Synephrine should not be used for more than 3 days to avoid rebound congestion. Drops should be discarded after one illness because they may become contaminated with bacteria. Vasoconstrictive nose drops can have a rebound effect after 3 days of use. Drops administered before feedings are more helpful.

An important nursing intervention when caring for a child who is experiencing a seizure would be to: a. describe and record the seizure activity observed. b. restrain the child when seizure occurs to prevent bodily harm. c. place a tongue blade between the teeth if they become clenched. d. suction the child during a seizure to prevent aspiration.

ANS: A When a child is having a seizure, the priority nursing care is observation of the child and seizure. The nurse then describes and records the seizure activity. The child should not be restrained, and nothing should be placed in the child's mouth. This may cause injury. To prevent aspiration, if possible, the child should be placed on the side, facilitating drainage.

9. A goiter is an enlargement or hypertrophy of which gland? a. Thyroid b. Adrenal c. Anterior pituitary d. Posterior pituitary

ANS: A A goiter is an enlargement or hypertrophy of the thyroid gland. Goiter is not associated with the adrenals or the anterior and posterior pituitaries.

C (9 months Most infants can pull themselves to a standing position at age 9 months. Any infant who cannot pull to a standing position by age 11 to 12 months should be referred for further evaluation for developmental dysplasia of the hip. At 6 months, the infant has just obtained coordination of arms and legs. By age 8 months, infants can bear full weight on their legs.)

By what age should the nurse expect that an infant will be able to pull to a standing position? a. 6 months b. 8 months c. 9 moths d. 11 to 12 months

By what age should concerns about pubertal delay be considered in boys? a. 12 to 12.5 years c. 13 to 13.5 years b. 12.5 to 13 years d. 13.5 to 14 years

D Concerns about pubertal delay should be considered for boys who exhibit no enlargement of the testes or scrotal changes by 13.5 to 14 years of age. Ages younger than 13.5 years are too young for initial concern.

A (transfer objects from one hand to the other By age 7 months, infants can transfer objects from one hand to the other, crossing the midline. The crude pincer grasp is apparent at about age 9 months. The infant can scribble spontaneously at age 15 months. At age 12 months, the infant can release cubes into a cup.)

In terms of fine motor development, the infant of 7 months should be able to: a. Transfer objects from one hand to the other. b. Use thumb and index finger in a crude pincer grasp. c. Hold a crayon and make a mark on paper. d. Release cubes into a cup.

D. (control of anal and urethral sphincters.)

Myelination of the spinal cord is almost complete by 2 years of age. As a result of this, the toddler can gradually achieve: A. throwing a ball without falling. B. slowing of gastrointestinal transit time. C. visual acuity of 20/20. D. control of anal and urethral sphincters.

B (storing poisonous substances in a locked cabinet.)

Poisoning in toddlers can best be prevented by: A. consistently using safety caps. B. storing poisonous substances in a locked cabinet. C. keeping ipecac syrup in the home. D. storing poisonous substances out of reach.

A (Animism is the attribution of lifelike qualities to inanimate objects. By scolding the stairs, the toddler is attributing human characteristics to them. Ritualism is the need to maintain sameness and reliability. It provides a sense of comfort to the toddler. Irreversibility is the inability to reverse or undo actions initiated physically. Steven is acting in an age-appropriate manner.)

Steven, 16 months old, falls down a few stairs. He gets up and "scolds" the stairs as if they caused him to fall. This is an example of which of the following? a. Animism b. Ritualism c. Irreversibility d. Delayed cognitive development

C (give child a frozen teething ring to relieve inflammation.)

The MOST appropriate recommendation for relief of teething pain is to instruct the parents to: A. rub gums with aspirin to relieve inflammation. B. apply hydrogen peroxide to gums to relieve irritation. C. give child a frozen teething ring to relieve inflammation. D. have child chew on a warm teething ring to encourage tooth eruption.

A (Allow to splash in bath The feel of the water while the infant is splashing provides tactile stimulation. Various colored blocks provide visual stimulation for a 4- to 6-month-old infant. A music box, tapes, and CDs provide auditory stimulation. Swings and strollers provide kinesthetic stimulation.)

The best play activity to provide tactile stimulation for a 6-month-old infant is to: a. Allow to splash in bath. b. Give various colored blocks. c. play music box, tapes, or CDs d. Use infant swing or stroller.

B (take a thorough, detailed history of usual daily events.)

The exhausted parents of a 2-month-old infant with colic ask the nurse what is the best method to promote comfort and sleep for the infant. The nurse's initial action is to: A. advise the mother to follow a milk-free diet for 3 to 5 days. B. take a thorough, detailed history of usual daily events. C. administer simethicone drops to provide relief from gas pains. D. explain that the parents need to stay calm so the infant will remain calm.

A (Injuries are the single most common cause of death in children ages 1 through 4 years. It is the period of highest death rate from injuries of any childhood age-group except adolescence. Infectious and childhood diseases are less common cause of deaths in this age-group. Congenital disorders are the second leading cause of death in this age-group.)

The leading cause of death during the toddler period is: a. Injuries. b. Infectious diseases. c. Congenital disorders. d. Childhood diseases.

A (Flame burns from matches and lighters represent one of the most fatal types of burns in the toddler age-group. Scald burns from water, hot object burns from cigarettes or irons, and electric burns from outlets are all significant causes of burn injury. The child should be protected from these causes by reducing the temperature of the hot water in the home, keeping objects such as cigarettes and irons away from children, and placing protective guards over electrical outlets when not in use.)

The most fatal type of burn in the toddler age-group is: a. Flame burn from playing with matches. b. Scald burn from high-temperature tap water. c. Hot object burn from cigarettes or irons. d. Electric burn from electrical outlets.

C (Brush teeth with plain water if child does not like toothpaste.)

The nurse is teaching the parent of a 2-year-old child how to care for the child's teeth. Which of the following should be included? A. Flossing is not recommended at this age. B. The child is old enough to brush teeth effectively. C. Brush teeth with plain water if child does not like toothpaste. D. Toothbrush should be small and have hard, rounded, nylon bristles.

A (soft and flexible shoes are generally better The main purpose of the shoe is protection. Soft, well-constructed, athletic-type shoes are best for infants and children. High-top shoes are not necessary for support but may be helpful keeping the child's foot in the shoe. Inflexible shoes can delay walking, aggravate in-toeing and out-toeing, and impede development of the supportive foot muscles.)

The nurse notices that a 10-month-old infant being seen in the clinic is wearing expensive, inflexible, high-top shoes. The nurse should explain that: a. Soft and flexible shoes are generally better. b. High-top shoes are necessary for support. c. Inflexible shoes are necessary to prevent in-toeing and out-toeing. d. This type of shoe will encourage the infant to walk sooner.

C (Inability to conserve)

The nurse notices that a toddler is more cooperative when taking medicine from a small cup rather than from a large cup. This is an example of which characteristic of preoperational thought? A. Egocentrism B. Irreversibility C. Inability to conserve D. Transductive reasoning

A (Push-pull toys encourage large muscle activity and are appropriate for toddlers. Nesting blocks are more appropriate for a 12- to 15-month-old child. A bicycle with training wheels is appropriate for a preschool or young school-age child. A computer can be appropriate as early as the preschool years.)

Which toy is the most developmentally appropriate for an 18- to 24-month-old child? a. A push-pull toy b. Nesting blocks c. A bicycle with training wheels d. A computer

A 10-year-old patient is talking to the nurse about wanting to try contact lenses instead of wearing glasses. She states that the other children at her school call her "four-eyes." Contact lenses should be prescribed for a child who is: a. At least 12 years of age. b. Able to read all the written information and instructions. c. Able to independently care for the lenses in a responsible manner. d. Confident that she really wants contact lenses.

c. Able to independently care for the lenses in a responsible manner.

The child with Down syndrome should be evaluated for what characteristic before participating in some sports? a. Hyperflexibility b. Cutis marmorata c. Atlantoaxial instability d. Speckling of iris (Brushfield's spots)

c. Atlantoaxial instability

Which term refers to opacity of the crystalline lens that prevents light rays from entering the eye and refracting on the retina? a. Myopia b. Amblyopia c. Cataract d. Glaucoma

c. Cataract

When caring for a newborn with Down syndrome, the nurse should be aware that the most common congenital anomaly associated with Down syndrome is: a. Hypospadias. c. Congenital heart disease. b. Pyloric stenosis. d. Congenital hip dysplasia.

c. Congenital heart disease.

A nurse is preparing to perform a dressing change on a 6-year-old child with mild cognitive impairment (CI) who sustained a minor burn. Which strategy should the nurse use to prepare the child for this procedure? a. Verbally explain what will be done. b. Have the child watch a video on dressing changes. c. Demonstrate a dressing change on a doll. d. Explain the importance of keeping the burn area clean.

c. Demonstrate a dressing change on a doll.

2. Which of the following is a complication that may occur after a cardiac catheterization? 1. Cardiac arrhythmia 2. Hypostatic pneumonia 3. Congestive heart failure 4. Rapidly increasing blood pressure

ANS: 1 1. Because a catheter is introduced into the heart, a risk exists of catheter-induced dysrhythmias occurring during the procedure. These are usually transient.

16. Which of the following drugs is an angiotensin-converting enzyme (ACE) inhibitor? 1. Captopril (Capoten) 2. Furosemide (Lasix) 3. Spironolactone (Aldactone) 4. Chlorothiazide (Diuril)

ANS: 1 1. Capoten is a drug which is an ACE inhibitor.

33. What painful, tender, pea-sized nodules may appear on the pads of the fingers or toes in bacterial endocarditis? 1. Osler nodes 2. Janeway lesions 3. Subcutaneous nodules 4. Aschoff nodes

ANS: 1 1. Osler nodes are red, painful, intradermal nodes found on pads of the phalanges in bacterial endocarditis.

37. Therapeutic management of the child with rheumatic fever includes: 1. Administration of penicillin. 2. Avoid salicylates (aspirin). 3. Strict bed rest for 4 to 6 weeks. 4. Administration of corticosteroids if chorea develops.

ANS: 1 1. The goal of medical management is the eradication of the hemolytic streptococci. Penicillin is the drug of choice

14. Which of the following is a clinical manifestation of the systemic venous congestion that can occur with congestive heart failure? 1. Tachypnea 2. Tachycardia 3. Peripheral edema 4. Pale, cool extremities

ANS: 3 3. Peripheral edema, especially periorbital edema, is a clinical manifestation of systemic venous congestion

5. Nursing interventions for the child after a cardiac catheterization would include which of the following? 1. Allow ambulation as tolerated. 2. Monitor vital signs every 2 hours. 3. Assess the affected extremity for temperature and color. 4. Check pulses above the catheterization site for equality and symmetry.

ANS: 3 3. The extremity that was used for access for the cardiac catheterization must be checked for temperature and color. Coolness and blanching may indicate arterial occlusion.

6. After the child returns from cardiac catheterization, the nurse monitors the child's vital signs. The heart rate should be counted for how many seconds? 1. 10 2. 15 3. 30 4. 60

ANS: 4 4. After cardiac catheterization, the heart rate should be counted for a full minute to detect evidence of dysrhythmias or bradycardia.

7. The nurse is caring for a school-age girl who has had a cardiac catheterization. The child tells the nurse that her bandage is "too wet." The nurse finds the bandage and bed soaked with blood. The most appropriate initial nursing action is which of the following? 1. Notify physician. 2. Apply new bandage with more pressure. 3. Place in Trendelenburg position. 4. Apply direct pressure above catheterization site.

ANS: 4 4. If bleeding occurs, direct continuous pressure is applied 2.5 cm (1 inch) above the percutaneous skin site to localize pressure over the vessel puncture.

28. Seventy-two hours after cardiac surgery, a young child has a temperature of 101° F. The nurse should do which of the following? 1. Keep child warm with blankets. 2. Apply a hypothermia blanket. 3. Record temperature on nurses' notes. 4. Report findings to physician.

ANS: 4 4. In the first 24 to 48 hours after surgery, the body temperature may increase to 37.7° C or 100° F as part of the inflammatory response to tissue trauma. If the temperature is higher or continues after this period, it is most likely a sign of an infection and immediate investigation is indicated

23. Which of the following is a sign of hypokalemia? 1. Apnea 2. Oliguria 3. Twitching 4. Muscle weakness

ANS: 4 4. Muscle weakness is a characteristic clinical manifestation of hypokalemia.

3. José is a 4-year-old child scheduled for a cardiac catheterization. Preoperative teaching should be: 1. Directed at his parents, because he is too young to understand. 2. Detailed in regard to the actual procedures so he will know what to expect. 3. Done several days before the procedure so that he will be prepared. 4. Adapted to his level of development so that he can understand.

ANS: 4 4. Preoperative teaching should always be directed at the child's stage of development. The caregivers also benefit from the same explanations.

32. What is the most common causative agent of bacterial endocarditis? 1. Staphylococcus albus 2. Streptococcus hemolyticus 3. Staphylococcus albicans 4. Streptococcus viridans

ANS: 4 4. Staphylococcus viridans is the most common causative agent in bacterial (infective) endocarditis

A, B, E (A good pacifier should be easily grasped by the infant. One-piece construction is necessary to avoid having the nipple and guard separate. The material should be sturdy and flexible. An attached ribbon or string and soft, pliable material are not characteristics of a good pacifier.)

Pacifiers can be extremely dangerous because of the frequency of use and the intensity of the infant's suck. In teaching parents about appropriate pacifier selection, the nurse should explain that a pacifier should have which characteristics (select all that apply)? a. Easily grasped handle b. One-piece construction c. Ribbon or string to secure to clothing d. Soft, pliable material e. Sturdy, flexible material

C (Parents need to be taught to always use the restraint even for short trips. Further teaching is needed if they make this statement. Parents have understood the teaching if they encourage the child to help attach buckles, straps, and shields but always double-check fastenings; do not start the car until everyone is properly restrained; and anchor the car safety seat securely to the car's anchoring system and apply the harness snugly to the child.)

Parents need further teaching about the use of car safety seats if they make which statement? a. "Even if our toddler helps buckle the straps, we will double-check the fastenings." b. "We won't start the car until everyone is properly restrained." c. "We won't need to use the car seat on short trips to the store." d. "We will anchor the car seat to the car's anchoring system."

B (The toddler's activities begin to produce purposeful results.)

The nurse is teaching the parents of a 24-month-old about motor skill development. The nurse should include which statement in the teaching? A. The toddler walks alone but falls easily. B. The toddler's activities begin to produce purposeful results. C. The toddler is able to grasp small objects but cannot release them at will. D. The toddler's motor skills are fully developed but occur in isolation from the environment.

D (Foreign-body aspiration is common during the second year of life. Although they chew well, children at this age may have difficulty with large pieces of food such as meat and whole hot dogs and with hard foods such as nuts or dried beans. Peanuts have many beneficial nutrients but should be avoided because of the risk of aspiration in this age-group. The sodium level may be a concern, but the risk of aspiration is more important. Many foods pass through the gastrointestinal tract incompletely digested. This is not necessarily detrimental to the child.)

The nurse recommends to parents that peanuts are not a good snack food for toddlers. The nurse's rationale for this action is that: a. They are low in nutritive value. b. They are very high in sodium. c. They cannot be entirely digested. d. They can be easily aspirated.

What represents the major stressor of hospitalization for children from middle infancy throughout the preschool years? a. Separation anxiety b. Loss of control c. Fear of bodily injury d. Fear of pain

ANS: A The major stress for children from infancy through the preschool years is separation anxiety, also called anaclitic depression. This is a major stressor of hospitalization. Loss of control, fear of bodily injury, and fear of pain are all stressors associated with hospitalization. However, separation from family is a primary stressor in this age-group.

18. Which activity is most appropriate for developing fine motor skills in the school-age child? a. Drawing b. Singing c. Soccer d. Swimming

ANS: A Activities such as drawing, building models, and playing a musical instrument increase the school-age child's fine motor skills. Singing is an appropriate activity for the school-age child, but it does not increase fine motor skills. The school-age child needs to participate in group activities to increase both gross motor skills and social skills, but group activities do not increase fine motor skills. Swimming is an activity that also increases gross motor skills.

11. A 9 year old often comes to the school nurse complaining of stomach pains. The teacher says that the child has lately been somewhat aggressive and stubborn in the classroom. What should the school nurse recognize as the possible trigger for these behaviors? a. Signs of stress b. Developmental delay c. A physical problem causing emotional stress d. Lack of adjustment to the school environment

ANS: A Signs of stress include stomach pains or headache, sleep problems, bed-wetting, changes in eating habits, aggressive or stubborn behavior, reluctance to participate, or regression to early behaviors. This child is exhibiting signs of stress, not developmental delay, a physical problem, or lack of adjustment.

13. A school-age child has had an upper respiratory tract infection for several days and then began having a persistent dry, hacking cough that was worse at night. The cough has become productive in the past 24 hours. This is most suggestive of: a. Bronchitis. c. Viral-induced asthma. b. Bronchiolitis. d. Acute spasmodic laryngitis.

ANS: A Bronchitis is characterized by these symptoms and occurs in children older than 6 years. Bronchiolitis is rare in children older than 2 years. Asthma is a chronic inflammation of the airways that may be exacerbated by a virus. Acute spasmodic laryngitis occurs in children between 3 months and 3 years.

23. Which statement expresses accurately the genetic implications of cystic fibrosis (CF)? a. If it is present in a child, both parents are carriers of this defective gene. b. It is inherited as an autosomal dominant trait. c. It is a genetic defect found primarily in non-Caucasian population groups. d. There is a 50% chance that siblings of an affected child also will be affected.

ANS: A CF is an autosomal recessive gene inherited from both parents and is found primarily in Caucasian populations. An autosomal recessive inheritance pattern means that there is a 25% chance that a sibling will be infected but a 50% chance a sibling will be a carrier.

19. A child has a chronic, nonproductive cough and diffuse wheezing during the expiratory phase of respiration. This suggests: a. Asthma. c. Bronchiolitis. b. Pneumonia. d. Foreign body in the trachea

ANS: A Children with asthma usually have these chronic symptoms. Pneumonia appears with an acute onset and fever and general malaise. Bronchiolitis is an acute condition caused by respiratory syncytial virus. Foreign body in the trachea will manifest with acute respiratory distress or failure and maybe stridor

44. A nurse is interpreting the results of a tuberculin skin test (TST) on an adolescent who is human immunodeficiency virus (HIV) positive. Which induration size indicates a positive result for this child 48 to 72 hours after the test? a. 5 mm c. 15 mm Test Bank - Maternal Child Nursing Care by Perry (6th Edition, 2017) 663 b. 10 mm d. 20 mm

ANS: A Clinical evidence of a positive TST in children receiving immunosuppressive therapy, including immunosuppressive doses of steroids, or who have immunosuppressive conditions, including HIV infection, is an induration of 5 mm. Children younger than 4 years of age (a) with other medical risk conditions, including Hodgkins disease, lymphoma, diabetes mellitus, chronic renal failure, or malnutrition; (b) born or whose parents were born in high-prevalence tuberculosis (TB) regions of the world; (c) frequently exposed to adults who are HIV infected, homeless, users of illicit drugs, residents of nursing homes, incarcerated or institutionalized, or migrant farm workers; and (d) who travel to high-prevalence TB regions of the world are positive when the induration is 10 mm. Children 4 years of age or older without any risk factors are positive when the induration is 20 mm.

8.Therapeutic management of nephrosis includes: a. Corticosteroids. b. Antihypertensive agents. c. Long-term diuretics. d. Increased fluids to promote diuresis.

ANS: A Corticosteroids are the first line of therapy for nephrosis. Response is usually seen within 7 to 21 days. Antihypertensive agents and long-term diuretic therapy are usually not necessary. A diet that has fluid and salt restrictions may be indicated.

4.What should the nurse recommend to prevent urinary tract infections in young girls? a. Wearing cotton underpants b. Limiting bathing as much as possible c. Increasing fluids; decreasing salt intake d. Cleansing the perineum with water after voiding

ANS: A Cotton underpants are preferable to nylon underpants. No evidence exists that limiting bathing, increasing fluids, decreasing salt intake, or cleansing the perineum with water decreases urinary tract infections in young girls.

37. A nurse is charting that a hospitalized child has labored breathing. Which describes labored breathing? a. Dyspnea c. Hypopnea b. Tachypnea d. Orthopnea

ANS: A Dyspnea is labored breathing. Tachypnea is rapid breathing. Hypopnea is breathing that is too shallow. Orthopnea is difficulty breathing except in upright position.

10. The nurse closely monitors the temperature of a child with nephrosis. The purpose of this is to detect an early sign of: a. Infection. b. Hypertension. c. Encephalopathy. d. Edema.

ANS: A Infection is a constant source of danger to edematous children and those receiving corticosteroid therapy. An increased temperature could be an indication of an infection, but it is not an indication of hypertension or edema. Encephalopathy is not a complication usually associated with nephrosis. The child will most likely have neurologic signs and symptoms.

A 10-year-old child, without a history of previous seizures, experiences a tonic-clonic seizure at school. Breathing is not impaired, but some postictal confusion occurs. The most appropriate initial action by the school nurse is to: a. stay with child and have someone call emergency medical service (EMS). b. notify parent and regular practitioner. c. notify parent that child should go home. d. stay with child, offering calm reassurance.

ANS: A The EMS should be called to transport the child because this is the child's first seizure. Because this is the first seizure, evaluation should be performed as soon as possible. The nurse should stay with the child while someone else notifies the EMS.

16.The primary clinical manifestations of acute renal failure are: a. Oliguria and hypertension. b. Hematuria and pallor. c. Proteinuria and muscle cramps. d. Bacteriuria and facial edema.

ANS: A The principal feature of acute renal failure is oliguria. Hematuria and pallor, proteinuria and muscle cramps, and bacteriuria and facial edema are not principal features of acute renal failure

31. A mother shares with the clinic nurse that she has been giving her 4 year old the antidiarrheal drug loperamide. What conclusion should the nurse arrive at based on knowledge of this classification of drugs? a. Not indicated b. Indicated because it slows intestinal motility c. Indicated because it decreases diarrhea d. Indicated because it decreases fluid and electrolyte losses

ANS: A Antimotility medications are not recommended for the treatment of acute infectious diarrhea. These medications have adverse effects and toxicity, such as worsening of the diarrhea because of slowing of motility and ileus, or a decrease in diarrhea with continuing fluid losses and dehydration. Antidiarrheal medications are not recommended in infants and small children.

1. Nurses must be alert for increased fluid requirements when a child presents with which possible concern? a. Fever b. Mechanical ventilation c. Congestive heart failure d. Increased intracranial pressure (ICP)

ANS: A Fever leads to great insensible fluid loss in young children because of increased body surface area relative to fluid volume. Respiratory rate influences insensible fluid loss and should be monitored in the mechanically ventilated child. Congestive heart failure is a case of fluid overload in children. ICP does not lead to increased fluid requirements in children.

15. Which statement appropriately describes a neuroblastoma? a. It is considered to be a "silent" tumor. b. Early diagnosis is usually possible because of the obvious clinical manifestations. c. It is the most common brain tumor in young children. d. It is the most common benign tumor in young children

ANS: A Neuroblastoma is a silent tumor with few symptoms. In more than 70% of cases, diagnosis is made after metastasis occurs, with the first signs caused by involvement in the nonprimary site. In only 30% of cases is diagnosis made before metastasis. Neuroblastomas are the most common malignant extracranial solid tumors in children. The majority of tumors develop in the adrenal glands or the retroperitoneal sympathetic chain. They are not benign; they metastasize.

16. A nurse is conducting a staff in-service on childhood cancers. Which is the primary site of osteosarcoma? a. Femur b. Humerus c. Pelvis d. Tibia

ANS: A Osteosarcoma is the most frequently encountered malignant bone cancer in children. The peak incidence is between ages 10 and 25 years. More than half occur in the femur. After the femur, most of the remaining sites are the humerus, tibia, pelvis, jaw, and phalanges

16. What condition is characterized by a chronic inflammatory process that may involve any part of the gastrointestinal (GI) tract from mouth to anus? a. Crohn's disease b. Ulcerative colitis c. Meckel's diverticulum d. Irritable bowel syndrome

ANS: A The chronic inflammatory process of Crohn's disease involves any part of the GI tract from the mouth to the anus but most often affects the terminal ileum. Ulcerative colitis, Meckel's diverticulum, and irritable bowel syndrome do not affect the entire GI tract.

12. The nurse is admitting a child with a Wilms' tumor. Which is the initial assessment finding associated with this tumor? a. Abdominal swelling b. Weight gain c. Hypotension d. Increased urinary output

ANS: A The initial assessment finding with a Wilms' (kidney) tumor is abdominal swelling. Weight loss, not weight gain, may be a finding. Hypertension occasionally occurs with a Wilms' tumor. Urinary output is not increased, but hematuria may be noted.

7. A child is being admitted with an infratentorial brain tumor. Which anatomical regions of the brain does the nurse know this tumor might include? (Select all that apply.) A. Brainstem B. Cerebellum C. Cerebrum D. Frontal lobe E. Parietal lobe

ANS: A, B Brain tumors in children are classified as either supratentorial or infratentorial. Infratentorial tumors are located in the posterior third of the brain, below the tentorium, and involve the brainstem and cerebellum. The cerebrum, frontal, and parietal lobes are located in the supratentorial section of the brain, which is the anterior two-thirds of the brain structure.

18. Bismuth subsalicylate may be prescribed for a child with a peptic ulcer to effect what result? a. Eradicate Helicobacter pylori b. Coat gastric mucosa c. Treat epigastric pain d. Reduce gastric acid production

ANS: A This combination of drug therapy is effective in the treatment and eradication of H. pylori. It does not bring about any of the results.

A school-age child is diagnosed with systemic lupus erythematosus (SLE). The nurse should plan to implement which interventions for this child? (Select all that apply.) a. Instructions to avoid exposure to sunlight b. Teaching about body changes associated with SLE c. Preparation for home schooling d. Restricted activity

ANS: A, B Key issues for a child with SLE include therapy compliance; body-image problems associated with rash, hair loss, and steroid therapy; school attendance; vocational activities; social relationships; sexual activity; and pregnancy. Specific instructions for avoiding exposure to the sun and UVB light, such as using sunscreens, wearing sun-resistant clothing, and altering outdoor activities, must be provided with great sensitivity to ensure compliance while minimizing the associated feeling of being different from peers. The child should continue school attendance in order to gain interaction with peers and activity should not be restricted, but promoted.

The nurse is evaluating the laboratory results on cerebral spinal fluid (CSF) from a 3-year-old child with bacterial meningitis. Which findings confirm bacterial meningitis? (Select all that apply.) a. Elevated white blood cell (WBC) count b. Decreased glucose c. Normal protein d. Elevated red blood cell (RBC) count

ANS: A, B The cerebrospinal fluid analysis in bacterial meningitis shows elevated WBC count, decreased glucose, and increased protein content. There should not be RBCs evident in the CSF fluid.

The nurse in the neonatal intensive care unit is caring for an infant with myelomeningocele scheduled for surgical repair in the morning. Which early signs of infection should the nurse monitor on this infant? (Select all that apply.) a. Temperature instability b. Irritability c. Lethargy d. Bradycardia e. Hypertension

ANS: A, B, C The nurse should observe an infant with unrepaired myelomeningocele for early signs of infection, such as temperature instability (axillary), irritability, and lethargy. Bradycardia and hypertension are not early signs of infection in infants.

12. The staff nurse is educating nursing students on the long-term effects of childhood chemotherapy. Which problems does the nurse include in the educational session? (Select all that apply.) A. Cardiac dysfunction B. Hearing loss C. Increased risk of multiple-gestation pregnancies D. Learning disabilities E. Peripheral neuropathy

ANS: A, B, D, E The list of long-term effects of chemotherapy is lengthy and includes cardiac dysfunction, hearing loss, learning disabilities, and peripheral neuropathy, among others. Sterility, not an increased risk for multiple-gestation pregnancies, is also an effect.

A 14-year-old girl is in the intensive care unit after a spinal cord injury 2 days ago. Nursing care for this child includes which action(s)? (Select all that apply.) a. Monitoring and maintaining systemic blood pressure b. Administering corticosteroids c. Minimizing environmental stimuli d. Discussing long-term care issues with the family e. Monitoring for respiratory complications

ANS: A, B, E Spinal cord injury patients are physiologically labile, and close monitoring is required. They may be unstable for the first few weeks after the injury. Corticosteroids are administered to minimize the inflammation present with the injury. It is not necessary to minimize environmental stimuli for this type of injury. Discussing long-term care issues with the family is inappropriate. The family is focusing on the recovery of their child. It will not be known until the rehabilitation period how much function the child may recover.

9. The nurse is teaching parents about the importance of good nutrition for their child who has cancer. Which components does the nurse include as important for this child's diet? (Select all that apply.) A. High calories B. High carbohydrates C. High vitamins D. Low minerals E. Low protein

ANS: A, C The child with cancer needs optimal nutrition, including a diet high in calories, fatty acids, vitamins, protein, and minerals.

6. The nurse is explaining types of solid tumors to a group of students. Which information does the nurse include? (Select all that apply.) A. A sarcoma is found in bone or muscle. B. "Carcinoma" means any cancerous tumor. C. Epithelial cells give rise to carcinomas. D. Lymphoma might originate in the thymus. E. Pediatric and adult solid tumors are similar.

ANS: A, C, D Pediatric and adult solid tumors are very different. A sarcoma arises from connective or supporting tissues, such as bones or muscle. A carcinoma is cancer arising from glandular and/or epithelial cells. Lymphomas originate in lymphoid organs, such as the lymph nodes, spleen, and thymus.

2. The treatment of brain tumors in children consists of which therapies? (Select all that apply.) a. Surgery b. Bone marrow transplantation c. Chemotherapy d. Stem cell transplantation e. Radiation f. Myelography

ANS: A, C, E Treatment for brain tumors in children may consist of surgery, chemotherapy, and radiotherapy alone or in combination. Bone marrow and stem cell transplantation therapies are used for leukemia, lymphoma, and other solid tumors where myeloablative therapies are used. Myelography is a radiographic examination after an intrathecal injection of contrast medium. It is not a treatment.

2. The nurse working with pediatric oncology patients educates the patients and families regarding best long-term follow-up practices. Which recommendations does this include? (Select all that apply.) A. Continued care by an interdisciplinary team B. Height measurements until puberty is reached C. Genetic testing prior to having children D. Risk-based follow-up appointments E. Thyroid screening for 5 years after remission

ANS: A, D Best-practice recommendations for follow-up include risk-based referrals and continued involvement of an interdisciplinary team of specialists. Height measurements are important for children until their adult height is achieved. Genetic testing is only recommended for certain types of cancer. Thyroid screening is important throughout the lifetime of survivors who were treated with radiotherapy to the neck, spine, or brain.

A young boy will receive a bone marrow transplant (BMT). This is possible because one of his older siblings is a histocompatible donor. Which is this type of BMT called? a. Syngeneic b. Allogeneic c. Monoclonal d. Autologous

ANS: B Allogeneic transplants are from another individual. Because he and his sibling are histocompatible, the BMT can be done. Syngeneic marrow is from an identical twin. There is no such thing as a monoclonal BMT. Autologous refers to the individuals own marrow.

A young child with human immunodeficiency virus (HIV) is receiving several antiretroviral drugs. The purpose of these drugs is to: a. cure the disease. b. delay disease progression. c. prevent spread of disease. d. treat Pneumocystis carinii pneumonia.

ANS: B Although not a cure, these antiviral drugs can suppress viral replication, preventing further deterioration of the immune system and delaying disease progression. At this time, cure is not possible. These drugs do not prevent the spread of the disease. P. carinii prophylaxis is accomplished with antibiotics.

The nurse is caring for an infant with myelomeningocele scheduled for surgical closure in the morning. Which interventions should the nurse plan for the care of the myelomeningocele sac? a. Open to air b. Covered with a sterile moist nonadherent dressing c. Reinforcement of the original dressing if drainage noted d. A diaper secured over the dressing

ANS: B Before surgical closure, the myelomeningocele is prevented from drying by the application of a sterile, moist, nonadherent dressing over the defect. The moistening solution is usually sterile normal saline. Dressings are changed frequently (every 2 to 4 hours), and the sac is closely inspected for leaks, abrasions, irritation, and any signs of infection. The sac must be carefully cleansed if it becomes soiled or contaminated. The original dressing would not be reinforced but changed as needed. A diaper is not placed over the dressing because stool contamination can occur.

A (Never heating a bottle in a microwave oven. Neither infant formula nor breast milk should be warmed in a microwave oven as this may cause oral burns as a result of uneven heating in the container. The bottle may remain cool while hot spots develop in the milk. Warming expressed milk in a microwave decreases the availability of antiinfective properties and causes separation of the fat content. Milk should be warmed in a lukewarm water bath.)

The parents of a 4-month-old infant tell the nurse that they are getting a microwave oven and will be able to heat the baby's formula faster. The nurse should recommend: a. Never heating a bottle in a microwave oven. b. Heating only 10 ounces or more. c. Always leaving the bottle top uncovered to allow heat to escape. d. Shaking the bottle vigorously for at least 30 seconds after heating.

D (The preschooler has a great imagination. Sounds and shadows can have a negative effect on sleeping behavior. Night-lights provide the child with the ability to visualize the environment and decrease the fear felt in a dark room. A dark, quiet room may be scary to a preschooler. High-carbohydrate snacks increase energy and do not promote relaxation. Most 2-year-olds take one nap each day. Many give up the habit by age 3. Insufficient rest during the day can lead to irritability and difficulty sleeping at night.)

What is helpful to tell a mother who is concerned about preventing sleep problems in her 2-year-old child? a. Have the child always sleep in a quiet, darkened room. b. Provide high-carbohydrate snacks before bedtime. c. Communicate with the child's daytime caregiver about eliminating the afternoon nap. d. Use a night-light in the child's room.

B (4 to 6 months Physiologically and developmentally, the 4- to 6-month-old is in a transition period. The extrusion reflex has disappeared, and swallowing is a more coordinated process. In addition, the gastrointestinal tract has matured sufficiently to handle more complex nutrients and is less sensitive to potentially allergenic food. Infants of this age will try to help during feeding. Two to 3 months is too young. The extrusion reflex is strong, and the infant will push food out with the tongue. No research base indicates that the addition of solid food to bottle-feeding has any benefit. Tooth eruption can facilitate biting and chewing; most infant foods do not require this ability.)

When is the best age for solid food to be introduced into the infant's diet? a. 2 to 3 months b. 4 to 6 months c. when birth weight has tripled d. When tooth eruption has started

B (During the second year of life, level of comprehension and understanding of speech increases and is far greater than the child's vocabulary. This is also true for bilingual children, who are able to achieve this linguistic milestone in both languages. The 18-month-old child has a vocabulary of 10 or more words. At this age, the child does not use one-word sentences or phrases. The child has a limited vocabulary of single words that are comprehensible.)

Which should the nurse expect for a toddler's language development at age 18 months? a. Vocabulary of 25 words b. Increasing level of comprehension c. Use of phrases d. Approximately one third of speech understandable

When should children with cognitive impairment be referred for stimulation and educational programs? a. As young as possible b. As soon as they have the ability to communicate in some way c. At age 3 years, when schools are required to provide services d. At age 5 or 6 years, when schools are required to provide services

a. As young as possible

Prevention of hearing impairment in children is a major goal for the nurse. This can be achieved through: a. Being involved in immunization clinics for children. b. Assessing a newborn for hearing loss. c. Answering parents' questions about hearing aids. d. Participating in hearing screening in the community.

a. Being involved in immunization clinics for children.

The most common type of hearing loss, which results from interference of transmission of sound to the middle ear, is called: a. Conductive. c. Mixed conductive-sensorineural. b. Sensorineural. d. Central auditory imperceptive.

a. Conductive.

A nurse is providing a parent information regarding autism. Which statement made by the parent indicates understanding of the teaching? a. "Autism is characterized by periods of remission and exacerbation." b. "The onset of autism usually occurs before 3 years of age." c. "Children with autism have imitation and gesturing skills." d. "Autism can be treated effectively with medication."

b. "The onset of autism usually occurs before 3 years of age."

A nurse would suspect possible visual impairment in a child who displays: a. Excessive rubbing of the eyes. b. Rapid lateral movement of the eyes. c. Delay in speech development. d. Lack of interest in casual conversation with peers.

a. Excessive rubbing of the eyes.

The school nurse is caring for a child with a penetrating eye injury. Emergency treatment includes: a. Applying a regular eye patch. b. Applying a Fox shield to the affected eye and any type of patch to the other eye. c. Applying ice until the physician is seen. d. Irrigating the eye copiously with a sterile saline solution.

b. Applying a Fox shield to the affected eye and any type of patch to the other eye.

Parents have learned that their 6-year-old child has autism. The nurse may help the parents to cope by explaining that the child may: a. Have an extremely developed skill in a particular area. b. Outgrow the condition by early adulthood. c. Have average social skills. d. Have age-appropriate language skills.

a. Have an extremely developed skill in a particular area.

Mark, a 9-year-old with Down syndrome, is mainstreamed into a regular third-grade class for part of the school day. His mother asks the school nurse about programs such as Cub Scouts that he might join. The nurse's recommendation should be based on knowing that: a. Programs such as Cub Scouts are inappropriate for children who are cognitively impaired. b. Children with Down syndrome have the same need for socialization as other children. c. Children with Down syndrome socialize better with children who have similar disabilities. d. Parents of children with Down syndrome encourage programs such as scouting because they deny that their children have disabilities.

b. Children with Down syndrome have the same need for socialization as other children.

An implanted ear prosthesis for children with sensorineural hearing loss is a(n): a. Hearing aid. c. Auditory implant. b. Cochlear implant. d. Amplification device.

b. Cochlear implant.

A father calls the emergency department nurse saying that his daughter's eyes burn after getting some dishwasher detergent in them. The nurse recommends that the child be seen in the emergency department or by an ophthalmologist. What should the nurse recommend before the child is transported? a. Keep the eyes closed. b. Apply cold compresses. c. Irrigate eyes copiously with tap water for 20 minutes. d. Prepare a normal saline solution (salt and water) and irrigate eyes for 20 minutes.

c. Irrigate eyes copiously with tap water for 20 minutes

Which action is contraindicated when a child with Down syndrome is hospitalized? a. Determine the child's vocabulary for specific body functions. b. Assess the child's hearing and visual capabilities. c. Encourage parents to leave the child alone for extended periods of time. d. Have meals served at the child's usual mealtimes.

c. Encourage parents to leave the child alone for extended periods of time.

An adolescent male visits his primary care provider complaining of difficulty with his vision. When the nurse asks the adolescent to explain what visual deficits he is experiencing, the adolescent states, "I am having difficulty seeing distant objects; they are less clear than things that are close." What disorder does the nurse suspect the adolescent has? a. Hyphema b. Astigmatism c. Amblyopia d. Myopia

d. Myopia

An adolescent gets hit in the eye during a fight. The school nurse, using a flashlight, notes the presence of gross hyphema (hemorrhage into anterior chamber). The nurse should: a. Apply a Fox shield. b. Instruct the adolescent to apply ice for 24 hours. c. Have adolescent rest with eye closed and ice applied. d. Notify parents that adolescent needs to see an ophthalmologist.

d. Notify parents that adolescent needs to see an ophthalmologist.

An appropriate nursing diagnosis for a child with a cognitive dysfunction who has a limited ability to anticipate danger is: a. Impaired Social Interaction. b. Deficient Knowledge. c. Risk for Injury. d. Ineffective Coping.

c. Risk for Injury.

The nurse is talking with a 10-year-old boy who wears bilateral hearing aids. The left hearing aid is making an annoying whistling sound that the child cannot hear. The most appropriate nursing action is to: a. Ignore the sound. b. Ask him to reverse the hearing aids in his ears. c. Suggest that he reinsert the hearing aid. d. Suggest that he raise the volume of the hearing aid.

c. Suggest that he reinsert the hearing aid

The pediatric nurse understands that fragile X syndrome is: a. A chromosome defect affecting only females. b. A chromosome defect that follows the pattern of X-linked recessive disorders. c. The second most common genetic cause of cognitive impairment. d. The most common cause of noninherited cognitive impairment.

c. The second most common genetic cause of cognitive impairment.

The teaching plan for the parents of a 3-year-old child with amblyopia ("lazy eye") should include what instruction? a. Apply a patch to the child's eyeglass lenses. b. Apply a patch only during waking hours. c. Apply a patch over the "bad" eye to strengthen it. d. Cover the "good" eye completely with a patch.

d. Cover the "good" eye completely with a patch.

Which is often administered to prevent or control hemorrhage in a child with cancer? a. Nitrosoureas b. Platelets c. Whole blood d. Corticosteroids

ANS: B Most bleeding episodes can be prevented or controlled with the administration of platelet concentrate or platelet-rich plasma. Nitrosoureas, whole blood, and corticosteroids would not prevent or control hemorrhage.

11. Which type of croup is always considered a medical emergency? a. Laryngitis c. Spasmodic croup b. Epiglottitis d. Laryngotracheobronchitis (LTB)

ANS: B Epiglottitis is always a medical emergency needing antibiotics and airway support for treatment. Laryngitis is a common viral illness in older children and adolescents, with hoarseness and upper respiratory infection symptoms. Spasmodic croup is treated with humidity. LTB may progress to a medical emergency in some children

33. The nurse is completing an admission assessment on a 3-year-old child. The child's Humpty Dumpty score is 15. Which action by the nurse is the most appropriate? A. Allow the child access to the play room. B. Classify the child as at high risk for falls. C. Place the child on seizure precautions. D. Put the child in isolation precautions.

ANS: B A Humpty Dumpty score of 12 or above indicates a high risk for falls. The child has been classified as at high risk for falls, and nursing care should be implemented to prevent them. Access to the play room can be accomplished with almost any child. Seizures and isolation actions are not related.

40. The nurse is assessing a child with acute epiglottitis. Examining the childs throat by using a tongue depressor might precipitate which symptom or condition? a. Inspiratory stridor c. Sore throat b. Complete obstruction d. Respiratory tract infection

ANS: B If a child has acute epiglottitis, examination of the throat may cause complete obstruction and should be performed only when immediate intubation can take place. Stridor is aggravated when a child with epiglottitis is supine. Sore throat and pain on swallowing are early signs of epiglottitis. Epiglottitis is caused by Haemophilus influenzae in the respiratory tract.

29. Cardiopulmonary resuscitation is begun on a toddler. Which pulse is usually palpated because it is the most central and accessible? a. Radial c. Femoral b. Carotid d. Brachia

ANS: B In a toddler, the carotid pulse is palpated. The radial pulse is not considered a central pulse. The femoral pulse is not the most central and accessible. The brachial pulse is felt in infants younger than 1 year.

1. Which statement accurately describes physical development of a child during the school-age years? a. The child's weight almost triples. b. A child grows an average of 2 inches/year. c. Few physical differences are apparent among children at the end of middle childhood. d. Fat gradually increases, which contributes to the child's heavier appearance.

ANS: B In middle childhood, growth in height and weight occur at a slower pace. Between the ages of 6 and 12 years, children grow 2 inches/year. In middle childhood, children's weight will almost double; they gain 3 kg/year. At the end of middle childhood, girls grow taller and gain more weight than boys. Children take on a slimmer look with longer legs in middle childhood.

Which is an appropriate nursing intervention when caring for a child in traction? a. Remove adhesive traction straps daily to prevent skin breakdown. b. Assess for tightness, weakness, or contractures in uninvolved joints and muscles. c. Provide active range-of-motion exercises to affected extremity three times a day. d. Keep the child in one position to maintain good alignment.

ANS: B Traction places stress on the affected bone, joint, and muscles. The nurse must assess for tightness, weakness, or contractures developing in the uninvolved joints and muscles. The adhesive straps should be released or replaced only when absolutely necessary. Active, passive, or active with resistance exercises should be carried out for the unaffected extremity only. Movement is expected with children. Each time the child moves, the nurse should check to ensure that proper alignment is maintained.

26. A child with cystic fibrosis is receiving recombinant human deoxyribonuclease (rhDNase). This drug: a. May cause mucus to thicken. b. May cause voice alterations. c. Is given subcutaneously. d. Is not indicated for children younger than 12 years.

ANS: B Two of the only adverse effects of rhDNase are voice alterations and laryngitis. rhDNase decreases viscosity of mucus, is given in an aerosolized form, and is safe for children younger than 12 years of age.

21. One of the clinical manifestations of chronic renal failure is uremic frost. What best describes this term? a. Deposits of urea crystals in urine b. Deposits of urea crystals on skin c. Overexcretion of blood urea nitrogen d. Inability of body to tolerate cold temperatures

ANS: B Uremic frost is the deposition of urea crystals on the skin, not in the urine. The kidneys are unable to excrete blood urea nitrogen, leading to elevated levels. There is no relation between cold temperatures and uremic frost.

12. A child is admitted with acute glomerulonephritis. The nurse would expect the urinalysis during this acute phase to show: a. Bacteriuria and hematuria. b. Hematuria and proteinuria. c. Bacteriuria and increased specific gravity. d. Proteinuria and decreased specific gravity.

ANS: B Urinalysis during the acute phase characteristically shows hematuria and proteinuria. Bacteriuria and changes in specific gravity are not usually present during the acute phase.

18. Asthma in infants is usually triggered by: a. Medications. c. Exposure to cold air. b. A viral infection. d. Allergy to dust or dust mites.

ANS: B Viral illnesses cause inflammation that causes increased airway reactivity in asthma. Medications such as aspirin, nonsteroidal antiinflammatory drugs, and antibiotics may aggravate asthma, but not frequently in infants. Exposure to cold air may exacerbate already existing asthma. Allergy is associated with asthma, but 20% to 40% of children with asthma have no evidence of allergic disease.

The senior nurse instructed the student nurse to check the website, www.ndvh.org. Information about which topic will the student nurse find on this website? 1 Domestic violence 2 Malnourishment 3 Infant mortality rate 4 Hospital administration

1 In USA, www.ndvh.org is the official website of the National Domestic Violence Hotline. Pediatric nurses can find valuable information on this website related to the signs of exposure to violence in children. This website also provides nonviolent problem-solving strategies, counseling, and referrals for children who are the victims of domestic abuse. Information about malnourishment, infant mortality rates, and hospital administration is not available at this website.

24. The nurse is caring for an 11-year-old boy who has recently been diagnosed with diabetes. What should be included in the teaching plan for daily injections? a. The parents do not need to learn the procedure. b. He is old enough to give most of his own injections. c. Self-injections will be possible when he is closer to adolescence. d. He can learn about self-injections when he is able to reach all injection sites.

ANS: B School-age children are able to give their own injections. Parents should participate in learning and giving the insulin injections. He is already old enough to administer his own insulin. The child is able to use thighs, abdomen, part of the hip, and arm. Assistance can be obtained if other sites are used.

Which should the nurse expect to find in the cerebral spinal fluid (CSF) results of a child with Guillain-Barr syndrome (GBS)? (Select all that apply.) a. Decreased protein concentration b. Normal glucose c. Fewer than 10 white blood cells (WBCs/mm3) d. Elevated red blood cell (RBC) count

ANS: B, C Diagnosis of GBS is based on clinical manifestations, CSF analysis, and EMG findings. CSF analysis reveals an abnormally elevated protein concentration, normal glucose, and fewer than 10 WBCs/mm3. CSF fluid should not contain RBCs.

Parents of a school-age child with hemophilia ask the nurse, Which sports are recommended for children with hemophilia? Which sports should the nurse recommend? (Select all that apply.) a. Soccer b. Swimming c. Basketball d. Golf e. Bowling

ANS: B, D, E Because almost all persons with hemophilia are boys, the physical limitations in regard to active sports may be a difficult adjustment, and activity restrictions must be tempered with sensitivity to the childs emotional and physical needs. Use of protective equipment, such as padding and helmets, is particularly important, and noncontact sports, especially swimming, walking, jogging, tennis, golf, fishing, and bowling, are encouraged. Contact sport such as soccer and basketball are not recommended.

The nurse is monitoring an infant for signs of increased intracranial pressure (ICP). Which are late signs of increased intracranial pressure (ICP) in an infant? (Select all that apply.) a. Tachycardia b. Alteration in pupil size and reactivity c. Increased motor response d. Extension or flexion posturing e. Cheyne-Stokes respirations

ANS: B, D, E Late signs of ICP in an infant or child include bradycardia, alteration in pupil size and reactivity, decreased motor response, extension or flexion posturing, and Cheyne-Stokes respirations.

3. A child is diagnosed with juvenile hypothyroidism. The nurse should expect to assess which symptoms associated with hypothyroidism? (Select all that apply.) a. Weight loss b. Sleepiness c. Diarrhea d. Puffiness around the eyes e. Spare hair

ANS: B, D, E A child diagnosed with juvenile hypothyroidism will display sleepiness; dry, periorbital puffiness; and spare hair growth. Weight loss and diarrhea are signs of hyperthyroidism.

Which is caused by a virus that primarily infects a specific subset of T lymphocytes, the CD4+ T cells? a. Wiskott-Aldrich syndrome b. Idiopathic thrombocytopenic purpura c. Acquired immunodeficiency syndrome (AIDS) d. Severe combined immunodeficiency disease

ANS: C AIDS is caused by the human immunodeficiency virus (HIV), which primarily attacks the CD4+ T cells. Wiskott-Aldrich syndrome, idiopathic thrombocytopenic purpura, and severe combined immunodeficiency disease are not viral illnesses.

A 9-year-old patient is scheduled for a surgical procedure next week. What teachings will the nurse include to ensure the patient's assent? Select all that apply: 1 Inform the patient about the nature of the condition. 2 Tell the patient what can be expected. 3 Inform the patient how consent is obtained. 4 Assess patient understanding. 5 Solicit an expression of the patient's willingness.

1, 2, 4, 5 Assent of the patient should include four key teachings. The nurse should help the patient achieve a developmentally appropriate awareness of the nature of their condition. The nurse should tell the patient what can be expected. The nurse should also make a clinical assessment of the patient's understanding, and solicit an expression of the patient's willingness to accept the proposed procedure. Such measures help reduce anxiety in the patient and are important for their assent. Information about the legal procedures of obtaining consent is not related to the surgical procedure and will not be as helpful in reducing anxiety.

The nurse plans to assess the role-relationship pattern in a child. Which questions should the nurse ask the parents? Select all that apply. 1 "Does the child have any security objects at home?" 2 "How do you handle discipline problems at home?" 3 "Have you ever noticed that your child sweats a lot?" 4 "How does your child usually handle disappointments?" 5 "Have any major changes in the family occurred lately?"

1, 2, 5 For assessing the role-relationship pattern in the child, the nurse should ask the parents about any security objects the child may have at home that provide comfort, discipline problems of the child, and family changes. From this information, the nurse can understand the relationship between the parents and the child. Information about sweating gives an idea about the elimination pattern in the child. Information about the disappointment handling potential of the child gives an idea about the child's coping-stress tolerance pattern.

A pregnant woman is diagnosed with a rubella infection during a prenatal checkup. What does the nurse expect the health care provider will tell the patient? Select all that apply. "The newborn may: 1 Have vision difficulties." 2 Have growth impairment." 3 Have difficulty hearing." 4 Develop breathing problems." 5 Not be able to concentrate."

1, 3 Rubella infections during pregnancy may cause hearing and visual loss in the newborn. However, these impairments may disappear as the child grows. Rubella infections do not cause growth retardation. Growth hormone deficiency or Turner syndrome can lead to growth impairment. Respiratory disorders or allergic reactions can result from hypersensitivities and can cause difficulty breathing in the newborn. A decreased ability to concentrate indicates impaired cognition. It usually results from inadequate intake of omega-3 fatty acids by the mother during pregnancy.

The nurse is explaining the health care bill of rights for children to parents and children in a pediatric ward. What key information does the nurse discuss with the group? Select all that apply. 1 Quality health care 2 Economic assistance 3 Respect and personal dignity 4 Making choices and decisions 5 Complex information

1, 3, 4 The bill of rights emphasizes the quality of health care to the children and explains that children and teens should be treated with respect and dignity. The bill also states that children have the right to make choices and decisions in their health care. Children have right to get emotional support from the health care professionals. Children cannot always expect economic assistance from the hospital. The nurse need not explain complicated information such as pathologic process of the disease to the child since the child will not be able to understand. Such information should be provided to the caregivers of the child.

Parents inform the nurse that they had noticed some needle injuries on their child's left elbow and some syringes and needles in the child's school bag. What should the nurse suggest to the child's parent? Select all that apply. 1 Encourage the child to participate in scouts. 2 Discourage the child from participating in sports due to injury. 3 Encourage the child to participate in church activities. 4 Provide first aid to the child and apply bandage to elbow. 5 Educate the parent and children about the ill effects of drugs.

1, 3, 5 The nurse should understand that the child is taking to illicit drugs. Preventive measures to reduce the youth's illicit drug use include encouraging participation in organized sports, scouts, and other church activities. The children and parents should be educated on the ill effects of drugs. Sports are not contraindicated and first aid is not a preventive measure.

A hospitalized child is being released for home health care. What suggestions should the nurse provide to prepare the family for transporting the child home? Select all that apply. 1 Take a basin in case of vomiting 2 Avoid using the restraint system 3 Keep a blanket and pillow in the car 4 Discourage the use of a straw for drinking fluids 5 Administer prescribed pain medication before leaving

1, 3, 5 The parents should use a basin or plastic bag for managing vomiting in the child. A blanket and pillow should be kept in the car to provide comfort. Pain medication can be administered before leaving to provide a pain-free journey home. The use of a car safety restraint system should be encouraged for the child's safety. Also, the use of a straw for drinking fluids should be encouraged except for children with oral facial surgeries.

Which child should the nurse document as being anemic? a. 7-year-old child with a hemoglobin of 11.5 g/dl b. 3-year-old child with a hemoglobin of 12 g/dl c. 14-year-old child with a hemoglobin of 10 g/dl d. 1-year-old child with a hemoglobin of 13 g/dl

ANS: C Anemia is a condition in which the number of red blood cells, or hemoglobin concentration, is reduced below the normal values for age. Anemia is defined as a hemoglobin level below 10 or 11 g/dl. The child with a hemoglobin of 10 g/dl would be considered anemic. The normal hemoglobin for a child after 2 years of age is 11.5 to 15.5 g/dl.

The nurse is planning activity for a 4-year-old child with anemia. Which activity should the nurse plan for this child? a. Game of hide and seek in the childrens outdoor play area b. Participation in dance activities in the playroom c. Puppet play in the childs room d. A walk down to the hospital lobby

ANS: C Because the basic pathologic process in anemia is a decrease in oxygen-carrying capacity, an important nursing responsibility is to assess the childs energy level and minimize excess demands. The childs level of tolerance for activities of daily living and play is assessed, and adjustments are made to allow as much self-care as possible without undue exertion. Puppet play in the childs room would not be overly tiring. Hide and seek, dancing, and walking to the lobby would not conserve the anemic childs energy.

What is the primary disadvantage associated with outpatient and day facility care? a. Increased cost b. Increased risk of infection c. Lack of physical connection to the hospital d. Longer separation of the child from family

ANS: C Outpatient and day facility care do not provide extended care; therefore, a child requiring extended care must be transferred to the hospital, causing increased stress to the child and parents. Outpatient care decreases cost and reduces the risk of infection. Outpatient care also minimizes separation of the child from family.

A 10-year-old female child requires daily medications for a chronic illness. Her mother tells the nurse that she is always nagging her to take her medicine before school. What is the most appropriate nursing action to promote the child's compliance? 1 Establishing a contract with her, including rewards 2 Suggesting time-outs when she forgets her medicine 3 Discussing with her mother the damaging effects of nagging 4 Asking the child to bring her medicine containers to each appointment so they can be counted

1 For school-age children, behavior contracting associated with desirable rewards is an effective method for achieving compliance. Time-outs should be used only if the behavioral contracting is not successful. Although nagging is not an effective strategy, the nurse needs to assist the mother in problem-solving rather than criticize the actions. Monitoring the medicine supply may be tried if the contracting is not successful.

The nurse is assessing a newborn with Down syndrome. The newborn's parent tells the nurse, "We are having a hard time holding our baby. We didn't have this hard of a time with our other children." What would be the nurse's best response? 1 "Children with Down syndrome have lower muscle tone." 2 "This happens in some children because of undeveloped bonding." 3 "Are you more apprehensive because your child has Down syndrome?" 4 "You should see a counselor to help you cope with your child's condition."

1 Newborns with Down syndrome have joint hyperflexibility and low muscle tone. This can make it difficult to hold the newborn because he or she can go limp like a rag doll. This makes it difficult for the parents to embrace and provide warmth to their newborn. This may make parents feel that the newborn is not bonding with them, but difficulty holding the child does not indicate impaired bonding between the child and parents. Inability to understand the child's needs and nonverbal communication indicates undeveloped bonding. Asking the parents whether they are more apprehensive does not answer their question. It is also a closed-ended question, which is not therapeutic communication. Telling the parents they need to see a counselor is not appropriate. They just need support and teaching.

The best explanation for why pulse oximetry is used on young children is that it: 1 is noninvasive. 2 is better than capnography. 3 is more accurate than arterial blood gases. 4 provides intermittent measurements of O2.

1 Pulse oximetry is a noninvasive method to determine oxygen saturation. Capnography measures carbon dioxide exhalation. It does not reflect oxygen perfusion. Pulse oximetry is less invasive and easier to test than arterial blood gases. Pulse oximetry provides continuous or intermittent measurements of oxygen saturation.

A nurse is instructing a nursing assistant on techniques to facilitate lipreading with a hearing-impaired child who lip-reads. Which techniques should the nurse include (select all that apply)? a. Speak at eye level. b. Stand at a distance from the child. c. Speak words in a loud tone. d. Use facial expressions while speaking. e. Keep sentences short.

A, D, E

A child is scheduled for a tonsillectomy and is afraid of the surgery. The child asks the nurse, "Will I need another operation when I have a sore throat again?" Which response should the nurse give to the child? 1 "Once your tonsils are taken out, you will not need the surgery again." 2 "You will need to repeat the surgery when you have another infection." 3 "You will need to have another surgery when you turn 14 years old." 4 "Once your tonsils are fixed, you will not have any more sore throats."

1 The child does not have enough knowledge about the tonsillectomy. Therefore the child may have fear about the surgery. The nurse should explain to the child that once the tonsils are removed, they do not need "fixing" again. It helps relieve the child's fear about the operation, and the child may feel comfortable. Once the tonsillectomy has been done in the child, a second operation is not required after another throat infection. There will actually not be a need for repeating the operation at any age. The child needs to be instructed that there may be other sore throats in the future. However, the child needs to be reassured that future sore throats will not require surgery.

The nurse is preparing to administer a vaccine to a child. The child is refusing to take the vaccination because of fear of bleeding. What should the nurse do in this situation? 1 Tell the child he or she can pick the bandage color. 2 Tell the child bleeding will stop in a few seconds. 3 Request a staff member sit beside the child. 4 Give a favorite toy to the child for distraction.

1 The child is refusing to take vaccination because of fear of bleeding and pain. The nurse should ask the child to select the color of the bandage to be used. This reassures the child and will make him or her feel better. Even if the nurse tells the child that the bleeding will stop when the needle is removed, it does little to help relieve the child's fear. The nurse should not scold the child in a firm tone because the child may get frightened. Giving a favorite toy to the child for playing is not helpful for relieving the fear. A favorite toy may help the child sleep at night. Requesting a staff member sit beside the child may not be helpful for relieving the child's fear. It may be needed to help hold the child still during a procedure.

The nurse is obtaining the admission history of a recently admitted adolescent. The nurse notes the patient requires help inserting contact lenses. Under which functional health pattern should the nurse record this observation? 1 Activity-exercise pattern 2 Cognitive-perceptual pattern 3 Nutrition-metabolic pattern 4 Health perception-health management pattern

1 The nurse records the admission history of the patient in terms of different functional health patterns. This helps in documenting all the required information about the patient. The patient requires help inserting his or her contact lenses. This implies that the patient needs support to perform an activity. The nurse should record this information under the activity-exercise pattern. The cognitive-perceptual pattern recognizes the cognitive development in the child and includes information such as defects in vision, hearing, or grading in the school. The nutrition-metabolic pattern is used in the assessment of nutrition in the patient, food allergies, and food intake habits. The health perception-health management pattern reports the medication and the health history of the child.

A couple who is going through a divorce asks the nurse how to disclose this news to their 4-year-old child. What would be the nurse's best response? 1 "You should sit down and calmly explain the situation to your child." 2 "You should not discuss it with the child until after the divorce is final." 3 'You should ask the grandparents or another relative to break the news." 4 "You should take your child to see a psychiatrist to break the news."

1 The parents should set aside time and explain the separation to their child. They should answer the child's questions and give the child time to absorb the information. The parents should not hide the information from the child because it can damage trust between the parents and the child. It is healthier when the child finds this out from parents rather than somebody else, whether it is the grandparents or a psychiatrist.

The nurse is educating new parents about the prevention of sudden infant death syndrome (SIDS). What position does the nurse tell the parents is the best sleeping position for their infant? 1 Supine 2 Prone 3 On the side 4 On a chair

1 The safest sleeping position to prevent SIDS is wholly supine. No pillows should be placed in a young infant's crib while the infant is sleeping. Lying prone or lying on the side can raise the risk of sudden infant death syndrome as the infant's nose may get covered by the bed. Putting the child on a chair may be unsafe as the child may fall off of the chair.

Which pattern of growth and development would be seen when a parent is working with a toddler to read the alphabet? 1 Sensitive 2 Sequential 3 Directional 4 Developmental

1 When the child is learning or training and going through psychologic changes, the child is going through the sensitive phase of development and growth pattern. Sequential growth patterns are definite growth patterns where the child crawls before standing and stands before walking. Directional development refers to early development and symmetrical growth along with central and peripheral nervous system development. Developmental pace focuses on growth patterns of the child as toddler, preschooler, and adolescent.

Which child behaviors indicate maladaptive coping toward chronic illness? Select all that apply. 1 Is irritable both at home and during school 2 Demonstrates feelings of pessimism and inadequacy 3 Expresses feelings of worthlessness to others 4 Expresses anxiety during remissions 5 Demonstrates confidence during an exacerbation

1, 2, 3 Children with negative and declining attitudes toward their chronic illness use maladaptive coping patterns. These maladaptive coping patterns are characterized by irritability at home and school. The child has a pessimistic outlook toward the disease and feels inadequate in managing it. The child may feel different, withdrawn, and less worthy than others. The child who has healthy coping skills may expresses appropriate emotions, such as anxiety, sadness, and anger, during periods of illness exacerbations. Demonstrating confidence during remissions indicates healthy coping skills

What information should the nurse provide to a group of nursing students when teaching them about the analgesic stepladder for pain management in children with terminal illness? Select all that apply. 1 "It was implemented by the World Health Organization (WHO)." 2 It provides for the use of adjunctive drug therapy to ease pain." 3 It outlines the principles of analgesic selection and titration." 4 "It describes the use of combination therapy to decrease side effects." 5 It was created to prevent drug dependence and drug addiction."

1, 2, 3 Children with terminal illness experience some amount of pain in the last stage of their illness. Nurses should follow the analgesic stepladder framed by World Health Organization (WHO) for pain management in children. It outlines the use of adjunctive therapy depending on the severity of the pain as mild or severe/persistent. It outlines the principles of analgesic selection such as administration of nonnarcotic analgesics to ease mild pain and administration of narcotic analgesics (opioids) to ease severe or persistent pain. It outlines the use of combination therapy to counteract adverse effects, not side effects. A patient with a terminal illness will require increased levels of pain medication to manage pain. The patient will not become dependent or addicted.

The nurse is assessing a child with autism for prognostic factors. What findings in the child suggest a better prognosis? Select all that apply. 1 Male sex 2 Early recognition 3 Functional speech 4 Lower intelligence 5 Behavioral impairment

1, 2, 3 Male sex carries a more favorable prognosis than female sex. Early recognition allows early intervention to help the child recover. Children with functional speech have a better prognosis than those who do not have functional speech. Children with higher intelligence have a more favorable prognosis than children with lesser intelligence. Children who do not have behavioral impairment have a better prognosis than children with behavioral impairment.

The nurse is instructing the parents of a 6-month-old child about the dietary requirements and factors that may influence the eating habits of the child. Which statement made by the nurse is appropriate? Select all that apply. 1 "Culture will have some influence on children's eating habits." 2 "Cholesterol is required for the synthesis of neurons in child's brain." 3 "During adolescence, children tend to make food choices for sociability." 4 "First 3 years of life are crucial in establishing eating habits of children." 5 "Cholesterol content is high in nuts and vegetable oils so use them sparingly."

1, 2, 3, 4 The nurse should inform the parents that culture has some influence on children's eating habits, and the child is likely to follow it. Cholesterol is required for the synthesis of neurons in the child's brain and should be included in the diet. During adolescence, children tend to make food choices for sociability and the first 3 years of life are crucial in establishing eating habits of children. Cholesterol is present only in animal products such as meat, milk, and eggs but not present in plant products.

The nurse is assessing a school-age child. The child stays with a parent who is recently divorced and has a meager income. The child does not like to mingle with other students at school. The child's performance is poor in studies and is cruel toward pets at home. Which factors in the child could most likely lead to pediatric social illness? Select all that apply. 1 Poverty 2 Pet cruelty 3 Single parent 4 Going to school 5 Behavior with others

1, 2, 3, 5 Pediatric social illness is a new morbidity in children. It refers to "the behavior, social, and educational problems that the children face". Poor socioeconomic status is a social problem. Animal cruelty is a behavioral problem. Problem within the family is a social problem Failure at school is an educational problem, and behavior with other children is also a behavioral problem. Any of these could cause pediatric social illness. Going to school does not cause social illness. It helps the child to gain knowledge, learn moral values, and to lead a successful life.

The parents of a 12-year-old child are informed that the child is in the terminal stages of osteosarcoma. The nurse explains to the parents about the care that needs to be provided for the child. Which characteristics in the parents indicate that they are in denial? Select all that apply. 1 Want the child to be admitted for more treatment 2 Refuse to believe the results of the tests 3 Pleased to receive the medical diagnosis 4 Ask questions about treatment or prognosis 5 Insist that nobody is telling them the truth

1, 2, 3, 5 The initial diagnosis of a terminal illness in a child is met with shock or denial. Parents experience this reaction, and it can last for days, weeks, or months. If the parents ask questions about the child's treatment or prognosis, this indicates that they have accepted the diagnosis. Parents in denial may want the child to be admitted for more medical treatment. They may feel the terminal diagnosis is wrong and refuse to believe the test results. People are usually not pleased to receive a terminal diagnosis; they are upset and angry. Insisting that nobody is telling them the truth is also an indicator of denial.

The nurse suspects tissue injury in an infant on intravenous therapy. What parameters will the nurse assess to determine tissue injury? Select all that apply. 1 The amount of redness 2 Blanching 3 The amount of swelling 4 Quality of pulses above infiltration 5 Coolness of the area

1, 2, 3, 5 The nurse adheres to certain guidelines available for determining the severity of tissue injury. Staging characteristics, such as the amount of redness, blanching, the amount of swelling, pain, capillary refill, and warmth or coolness of the area, are used to determine severity. The quality of pulses below infiltration is assessed and not above it.

The parents of a teenaged child tell the nurse they are worried that their child is hanging out with the wrong type of friends. What can the nurse advise the parents to do to help the adolescent make better choices? Select all that apply. 1 Provide support and love at home to make the child feel wanted. 2 Bring the adolescent to an inpatient psychiatric facility. 3 Establish clear and concise boundaries, rules, and expectations. 4 Talk to the child and allow the child to make her own decisions. 5 Ground the child until she shows appropriate behavior with friends.

1, 3, 4 In order to be an effective parent, the family should convey four external assets to the child so that the child can learn to make the correct choices. The parents should provide support, care, and love at home and in the community. It is also important to establish clear and concise boundaries and expectations both at home and in the community. It is also important for parents to discuss good decision-making strategies with their children to empower them to make correct choices. The adolescent is not displaying any behaviors that warrant an inpatient psychiatric admission, nor does she need to be grounded.

The nurse is discharging a young child from the hospital. The nurse should instruct the parents to look for which posthospital child behaviors? Select all the apply. 1 Tendency to cling to parents 2 Jealousy toward others 3 Demands for parents' attention 4 Anger toward parents 5 New fears such as nightmares

1, 3, 5 Young children's posthospital behaviors include: They show initial aloofness toward parents; this may last from a few minutes (most common) to a few days. This is frequently followed by dependency behaviors: tendency to cling to parents; demands for parents' attention; vigorous opposition to any separation (e.g., staying at preschool or with a babysitter). Other negative behaviors include: new fears (e.g., nightmares); resistance to going to bed, night waking; withdrawal and shyness; hyperactivity; temper tantrums; food peculiarities; attachment to blanket or toy; regression in newly learned skills (e.g., self-toileting). Posthospital behaviors for older children include negative behaviors: emotional coldness followed by intense, demanding dependence on parents; anger toward parents; jealousy toward others (e.g., siblings).

The nurse is reviewing mortality indicators in a city with a population of 30 million. In 2011, there were 3000 total live births, 60 stillbirths, 60 deaths younger than 28 weeks of age, 30 deaths younger than 4 weeks of age, and 90 deaths younger than 1 year (includes all subcategories of deaths younger than 1 year). What is the neonatal mortality rate of the city? Record your answer using a whole number. _____ per 1000 live births

10 Neonate refers to a baby who is younger than 28 days of age. The number of deaths younger than 28 days, or 4 weeks, was 30. Therefore, neonatal mortality rate = number of deaths younger than 28 days (4 weeks) / number of live births or 30 / 3000 = 10 / 1000.

A visually impaired child is hospitalized for eye surgery. What nursing intervention should be included in the plan of care to encourage the child to be independent? The nurse: 1 Does not keep a stool or small desk near the bed. 2 Instructs the cleaner not to move the furniture around. 3 Gives the child work to do while the child is in the hospital. 4 Does not educate the child about the treatment procedures.

2 Changing furniture positions can result in accidents, so this must be avoided. A small stool or a desk should be placed near the bed to support the child so that he or she can climb into bed easily. The child is a patient in the hospital and should not be asked to work while there. Educating the child about the procedures that will be carried out for the treatment will help the child understand and mentally prepare for them.

A patient who is undergoing stem cell therapy asks the nurse about undifferentiated cells. Which response given by the nurse is most appropriate? "These cells:" 1 are able to divide at a very rapid rate." 2 multiply to form any part of the body." 3 can perform specialized functions." 4 are similar to all other cells in the body."

2 Differentiation is the process by which immature cells transform into mature cells to form tissues. Thus undifferentiated cells are immature and not developed. These cells would be able to reproduce to form any part of the body. These cells are not similar to cancer cells and do not multiply rapidly. These cells are immature and are not able to perform specialized functions. These cells are not well developed and thus do not resemble other mature cells of the body.

The relative of a child receiving oxygen therapy brings the child a remote-controlled airplane as a gift. What safety risk does the toy present to the patient? 1 The toy may cause suffocation. 2 The toy can cause fire. 3 The toy may have toxic lead paint. 4 The toy may distract the child.

2 Electrical or friction toys are not safe because sparks can cause oxygen to ignite. Bags made of plastic or other material are more likely to cause suffocation. Toxic lead paint is usually a concern for toys from unknown manufacturers. Distraction can be an advantage for the child as it is likely to bring some joy and solace.

The nurse works in a pediatric unit. Which child would have an increased vulnerability to the stresses of hospitalization? 1 A female child 2 A child with a difficult temperament 3 A child with an average intelligence 4 A child older than 6 years of age

2 Hospitalization is a stressor in children and so they may react differently to it. Certain children are more susceptible to the stressful effects of hospitalization than others. Children who have difficult temperament may not readily adjust with the unfamiliar environment of the hospital. These children may experience adverse effects of hospitalization. Female children are able to withhold stress more when compared to male children and thus are less likely to experience stressors. Children with average intelligence may be able to understand their condition and the importance of hospitalization and thus may be more adaptable. Children with lower IQ would not understand the purpose of hospital admission and thus would be extremely stressed due to hospitalization. Children who are older than 6 years of age have developed the maturity to understand their condition and the purpose of hospitalization. Thus, they would be more adaptable to their condition, and experience less stress related to hospitalization.

The nurse is caring for a child dying from cancer. Physical signs that the child is approaching death include: 1 rapid pulse. 2 change in respiratory pattern. 3 sensation of cold, although body feels hot. 4 loss of hearing followed by loss of other senses.

2 In the final hours of life the respiratory pattern may become labored, with periods of apnea. The pulse becomes weak and slowed. There is a sensation of heat, although the body feels cold. Hearing is the last sense to fail.

What does the nurse keep in mind while administering an enema to a child? 1 The nurse should not give details about the procedure. 2 The buttocks of the child should be held together briefly. 3 Pillows should not be used during the procedure. 4 Administration of enemas should be noninvasive in children

2 Infants and young children are unable to retain the solution after it is administered, so the buttocks must be held together for a short time to retain the fluid. A careful explanation may help ease any concerns or fears the child may have about the procedure. The enema is administered and expelled while the child is lying with the buttocks over the bedpan and with the head and back supported by pillows. An enema is an intrusive procedure.

The nurse is caring for a dying boy whose religion is Islam (Muslim/Moslem). An important nursing consideration related to his impending death and religion is that: 1 there are no special rites. 2 there are specific practices to be followed. 3 the family is expected to "wait" away from the dying person. 4 baptism should be performed if it has not been done previously.

2 Islam has specific rituals for bathing and wrapping the body in cloth before it is to be moved. The nurse should contact someone from the person's mosque to assist. Family may be present. No baptism is performed at this time. Test-Taking Tip: Attempt to select the answer that is most complete and includes the other answers within it. For example, a stem might read, "A child's intelligence is influenced by:" and three options might be genetic inheritance, environmental factors, and past experiences. The fourth option might be multiple factors, which is a more inclusive choice and therefore the correct answer.

The nurse has been assigned to the pediatric respiratory unit. What is the preliminary requirement for the nurse to evaluate improvement in the respiratory function of the child with treatment? 1 The child's feedback 2 The baseline data 3 The parents' opinion 4 The primary health care provider's opinion

2 It is impossible to evaluate the improvement in the respiratory function of the child without having any baseline data. The child's feedback provides only subjective assessment. Evaluation of respiratory system function requires formal knowledge of the respiratory system assessment. Therefore, parent opinion is not reliable for improvement in the child's respiratory function. Evaluation of the improvement in the respiratory function requires objective assessment. Obtaining the primary health care provider's opinion is a type of subjective assessment.

Autism is a complex developmental disorder. The diagnostic criteria for autism include delayed or abnormal functioning in which area(s) with onset before age 3 years (select all that apply)? a. Language as used in social communication b. Gross motor development c. Growth below the 5th percentile for height and weight d. Symbolic or imaginative play e. Social interaction

A, D, E

The pediatric nurse has recorded the birth weight, head circumference, axillary temperature, and crown to rump length of a newborn baby. What does the nurse consider while assessing the risk for mortality in this infant? 1 Measure head circumference on alternate weeks. 2 Use the birth weight for the assessment of infant mortality rate. 3 Prefer rectal temperature to axillary temperature in the new born. 4 Crown to rump length is the best indicator of infant mortality rate

2 Low birth weight is associated with high mortality rate so birth weight is considered in predicting the infant mortality rate. Head circumference, rectal temperature or axillary temperature, and crown to rump length are generally not preferred indicators as compared to birth weight. Large head may increase the susceptibility of the child to acquire a head injury. Body temperature fluctuates due to many physiologic and pathological reasons. Alterations in rectal or axillary temperature of a newborn do not indicate that the child has high infant mortality rate. The height (crown-rump length) of the newborn is not an indicator of infant mortality rate. Test-Taking Tip: Start by reading each of the answer options carefully. Usually at least one of them will be clearly wrong. Eliminate this one from consideration. Now you have reduced the number of response choices by one and improved the odds. Continue to analyze the options. If you can eliminate one more choice in a four-option question, you have reduced the odds to 50/50. While you are eliminating the wrong choices, recall often occurs. One of the options may serve as a trigger that causes you to remember what a few seconds ago had seemed completely forgotten.

What important information should the nurse include when teaching the parents of an adolescent about nutrition? 1 Adolescents are usually mature enough to make healthy food choices. 2 Resources are available to assist lower income families to obtain enough protein. 3 Behavior problems in this age group are not related to nutritional deficiencies. 4 Parental influence has the greatest impact on food choices at this age.

2 Lower income families may need resources and information about how to obtain assistance in getting expensive foods such as meats to get enough protein intake. During adolescence, parental influence diminishes and the adolescent makes food choices related to peer acceptability and sociability. Occasionally these choices are detrimental to adolescents with chronic illnesses, such as diabetes, obesity, chronic lung disease, hypertension, cardiovascular risk factors, and renal disease. Families that struggle with lower incomes, homelessness, and migrant status generally lack the resources to provide their children with adequate food intake; nutritious foods, such as fresh fruits and vegetables; and appropriate protein intake. The result is nutritional deficiencies with subsequent growth and developmental delays, depression, and behavior problems. Behavior problems can indeed be related to nutritional deficiencies.

The nurse is assessing a traditional Hindu woman. What should the nurse ask in order to learn about the patient's health traditions? 1 "Do you have an advance directive for medical decisions?" 2 "What are your beliefs about health and illness?" 3 "How often do you pray or visit a place of worship?" 4 "Can you tell me when you developed the symptoms?"

2 Many cultures have health practices and traditions different from those of the Western world. The nurse can ask questions about their health and illness beliefs. These include all the health practices, medicines, and food of the patient's culture. This can help the nurse and patient develop mutually acceptable goals. Asking a patient whether he or she had an advance directive is important but will not provide information about cultural health practices. How often the patient prays is not related to cultural health practices. The nurse should ask questions about when the symptoms developed, but this does not relate to cultural health practices.

Several nurses tell their nursing supervisor that they want to be able to attend the funeral of a child for whom they had cared. They say they felt especially close to both the child and the family. The supervisor should recognize that attending the funeral is: 1 appropriate because families expect this expression of concern. 2 appropriate because it can assist in the resolution of personal grief. 3 inappropriate because it is unprofessional. 4 inappropriate because it increases burnout.

2 Nurses should attend the funeral of a child if they felt closeness with the family. This will help the nurses grieve and gain closure. Families may or may not expect this expression of concern. The behavior is appropriate if a relationship existed between the nurses and family. This may prevent burnout.

The nurse is teaching the nursing students about functions of play in the hospital. Which statement made by the nursing student indicates the need for further teaching? "Play: 1 Can lessen the stress of separation from the family." 2 Makes the child nervous in a strange environment." 3 Helps the child develop a positive attitude for others." 4 Provides an expressive outlet for the child's creative ideas."

2 Play is one of the most important aspects of a child's life and one of the most effective tools for managing stress. It is helpful for the child to relieve stress. It is also essential for the child's mental, emotional, and social well-being. Play does not make the child anxious in an unfamiliar environment. It helps the child feel more secure in a strange environment. Play lessens the stress of separation from the family because the child is busy. During play, the child communicates with others, which helps develop a positive attitude toward others. It also stimulates thinking in the child by allowing the child to express creative ideas.

Bronchial (postural) drainage generally is performed: 1 immediately before all aerosol therapy. 2 before meals and at bedtime. 3 immediately on arising and at bedtime. 4 thirty minutes after meals and at bedtime.

2 The most effective time for bronchial drainage is before meals and at bedtime. It is more effective after other respiratory therapy, such as bronchodilators or nebulizer treatments. The procedure should be done 3 to 4 times each day. When drainage is done after meals, it may cause the child to vomit.

The nurse is working with a family who has a child with a chronic illness. The nurse notices that the parents watch television and play on their electronic devices instead of spending time with the child. The parents are frequently reprimanding the child. What should the nurse interpret from these findings? The parents: 1 Are being overprotective. 2 Are displaying rejection. 3 Are in denial of the illness. 4 Have gradual acceptance.

2 The parents' reaction of detaching themselves from the child and constantly rebuking the child indicates rejection. However, even though the parents are rejecting the child, they may still meet the child's physical needs. When parents do not let their children learn new skills for fear of injury, this indicates overprotection. Pretending that the child does not have a problem indicates denial. Placing necessary restrictions on the child indicates gradual acceptance.

The nurse is caring for a teenager scheduled for surgery. What criteria does the nurse use to obtain valid consent from the patient? The patient should: Select all that apply. 1 Be intelligent. 2 Be over the age of majority. 3 Be well-informed. 4 Act voluntarily. 5 Be healthy

2, 3, 4 To obtain valid informed consent, health care providers must meet three conditions. The person must be capable of giving consent; he or she must be over the age of majority. The person must receive the information needed to make an intelligent decision. The person must act voluntarily when exercising freedom of choice. The intelligence of the patient is not measured; the patient is only required to be competent enough to make decisions.

The nurse is preparing a child for an endotracheal tube (ET) placement. How does the nurse verify the placement of the tube? Select all that apply. 1 Visualization of unilateral chest expansion 2 Auscultation over the epigastrium 3 Examination of water vapor in the tube 4 Waveform verification with continuous capnography 5 Examination using a chest radiography

2, 3, 4, 5 ET tube placement should be verified by at least one clinical sign and at least one confirmatory technology. Such technologies include auscultation over the epigastrium and the lung fields bilaterally in the axillary region, examination of water vapor in the tube, and waveform verification with continuous capnography. Chest radiography can also be used to verify placement. Visualization of bilateral (and not unilateral) chest expansion is used to verify placement of the ET tube.

The nurse is developing a teaching plan about preventing fetal exposure to teratogens. Which teratogenic agents or conditions should the nurse include? Select all that apply. 1 acetaminophen (Tylenol) 2 isotretinoin (Accutane) 3 cocaine 4 hyperthermia 5 ethyl alcohol 6 phenytoin (Dilantin)

2, 3, 4, 5, 6 Teratogens, agents that cause birth defects when present in the prenatal environment, account for the majority of adverse intrauterine effects not attributable to genetic factors. Types of teratogens include drugs (phenytoin [Dilantin], warfarin [Coumadin], isotretinoin [Accutane]); chemicals (ethyl alcohol, cocaine, lead); infectious agents (rubella, cytomegalovirus); physical agents (maternal ionizing radiation, hyperthermia); and metabolic agents (maternal PKU). Many of these teratogenic exposures and the resulting effects are completely preventable, such as ingestion of alcohol resulting in fetal alcohol syndrome or fetal alcohol effects, which causes severe birth defects, including cognitive impairment. The incidence of fetal alcohol syndrome is estimated at 5.2 per 10,000 live births (American Academy of Pediatrics, 2000).

The nurse is caring for a child who is in the terminal stage of acute lymphocytic leukemia. What should be the goals of palliative care in the child? Select all that apply. 1 To assess the severity of disease symptoms 2 To provide effective pain management 3 To strive for the best possible quality of life 4 To mandate that the family join in the care 5 To support the family and the child

2, 3, 5 Palliative care focuses on relieving pain and managing symptoms related to a terminal disease. It also addresses the psychological, social, and spiritual problems of children living with terminal conditions. The goal of palliative care is to achieve the best quality of life for patients and their families, consistent with their values. Palliative care seeks to improve the quality of life for a patient with a terminal condition. The nurse will assess the severity of the child's symptoms, but this is not the goal of palliative care. The nurse should encourage family members to participate in the child's care but should not mandate their help.

The nursing student is caring for a child admitted to the hospital. The nursing student asks the nurse instructor, "How can we keep the child's routine habits while he is in the hospital?" What would be the best response by the nurse instructor? Select all that apply. "Ask the parents: 1 "About the use of any herbal therapies." 2 "When the child goes to sleep at night." 3 "What foods the child prefers to eat." 4 "How the child's grades are in school." 5 "Which toy the child plays with at home."

2, 3, 5 The nurse should assess the child's usual health habits at home to promote a more normal environment in the hospital. This includes the child's sleep-rest, nutritional-metabolic, and activity-exercise patterns. The nurse would assess the sleep-rest pattern by asking when the child goes to sleep at night. Assessing the nutritional-metabolic pattern would include asking about food preferences. The nurse should also ask what toy the child plays with at home as part of the activity-exercise pattern. These will help the nurse plan individualized care for the child. History about herbal and complementary therapy helps in preventing drug-drug interaction and severe adverse effects.

Which statement is true about smoking in adolescence? a. Smoking is related to other high-risk behaviors. b. Smoking is more common among athletes. c. Smoking is less common when the adolescent's parent(s) smokes. d. Smoking among adolescents is becoming more prevalent.

A Cigarettes are considered a gateway drug. Teenagers who smoke are 11.4 times more likely to use an illicit drug. Teens who refrain from smoking often have a desire to succeed in athletics. If a parent smokes, it is more likely that the teen will smoke. Cigarette smoking has declined among all groups since the 1990s.

What should the nurse do to communicate with a patient who is cognitively impaired and speaks a foreign language? 1 Insert the patient's hearing aids. 2 Use verbal expressions. 3 Use a language translator. 4 Use visual aids and drawings.

3 A language translator should be used when any patient speaks a foreign language, no matter what his or her cognitive level is. Inserting the patient's hearing aids will help the patient hear, but it will not break the language barrier. Verbal expressions can be helpful, but they are not as effective as having a translator speaking the patient's language. Visual aids and drawings may be helpful, but not everything is easily communicated with these methods.

A 10-year-old child is diagnosed with an autism spectrum disorder (ASD). The parents ask the nurse about the cause of the disorder. Which answer given by the nurse is most appropriate? 1 "Autism is caused by a high intake of proteins during pregnancy." 2 "The disorder is caused by vaccines that contain thimerosal." 3 "The exact cause of autism spectrum disorders is unknown." 4 "Alcohol consumption during pregnancy is linked to autism."

3 Although the exact cause of ASD is not known, the nurse should always help parents understand that they are not responsible for the child's condition. There are many theories about the cause of ASD, but nothing is definitive. High intake of proteins is necessary during pregnancy because it promotes proper growth and development of the fetus. Vaccines containing thimerosal are not associated with ASD. Thimerosal is a preservative found in some vaccines. Consumption of alcohol during pregnancy leads to fetal alcohol syndrome, not autism.

The Kohlberg moral development theory states that children are concerned with conformity and loyalty at a stage of their growth. When this stage is correlated with the cognitive development of children, what would the age group be? 1 0-2 years 2 2-7 years 3 7-11 years 4 11-15 years

3 According to Kohlberg's moral development theory, when children are concerned with conformity and loyalty, they are at the conventional level. Children at this level are considered to be working on concrete operations of cognitive development, where children 7-11 years old are included. Infants between 0 and 2 years of age are included in the sensory motor level of cognitive development. Children between 2 and 7 years old are in the preoperational stage of cognitive development. Children between 11 and 15 years are considered to be in the formal operation stage of cognitive development.

The nurse is caring for a patient who is on long-term catheterization. According to the National Quality Forum, what should the nurse assess in this patient? 1 Oxygen saturation using arterial or venous blood 2 Monitoring of respiratory rate while in a sitting position 3 Signs and symptoms of a urinary tract infection 4 Abnormal changes in the electrocardiogram (ECG)

3 According to National Quality Forum, the nurse has to measure the patient-centered outcome. The nurse has to assess the patients in the intensive care unit (ICU) regularly to identify the urinary tract infections associated with the urinary catheter. Signs and symptoms of a urinary tract infection are fever, burning urination, and yellow urine. The urinalysis is done to confirm the presence of urinary tract infection (UTI). Urinary catheterization has no direct association with changes in arterial or venous oxygen saturation, respiratory, or cardiac complication.

A 14-year-old patient with Ewing's sarcoma is scheduled for an amputation of the right leg. The patient has been grieving about the loss ever since the health care provider prescribed the surgery. What type of grief does the nurse identify in this patient? 1 Acute grief 2 Universal loss 3 Anticipatory grief 4 Complicated grief

3 Because the amputation has not occurred, the patient's reaction is caused by anticipation of a future loss. This is a healthy response to the anticipated loss of the leg. It will help foster healthy grieving. Acute grief is a definite syndrome characterized by psychological and somatic symptoms. It is characterized by emotional distancing in relationships with others, accompanied by erratic responses of irritability, hostility, and anger. Universal loss does not describe the emotions of grieving before an actual loss. Some people feel constantly upset and preoccupied with a person who has died, to the point where their relationships and work suffer for months on end. Such a reaction is known as complicated grief.

The nurse is caring for an 8-year-old child who has a chronic illness. The child has a tracheostomy, and a parent is rooming-in during this hospitalization. The parent insists on providing almost all of the child's care and tells the nurses how to care for the child. When planning the child's care, the primary nurse should recognize that the parent is: 1 controlling and demanding. 2 assuming the nurse's role. 3 the expert in care of the child. 4 not allowing nurses to function independently.

3 Because these parents care for this child with complex health needs at home, they are most familiar with the care requirements and routine. The nurse's role includes assessment and evaluation, not just the implementation phase. The nurse recognizes that the philosophy of family-centered care states that the parents are the experts in the care of their child. The nurse functions collaboratively with the family.

Nurses play an important role in current issues and trends in health care. What is a current trend in pediatric nursing and health care today? 1 The patient is the unit of care for the health care provider. 2 Discharge planning begins when the physician writes the order. 3 Health promotion resources enable children to achieve their full potential. 4 The focus of pediatric health care is trending toward acute hospital care.

3 Health promotion provides opportunities to reduce differences in current health status among members of different groups and provides a better chance to achieve the fullest health potential. The patient and family is the unit of care for the health care provider. Discharge planning begins when the patient is admitted. The focus of pediatric health care is trending away from acute hospital settings.

The signs and symptoms in a nursing diagnosis describe: 1 projected changes in an individual's health status, clinical conditions, or behavior. 2 an individual's response to health pattern deficits in the child, family, or community. 3 a cluster of cues and/or defining characteristics that are derived from patient assessment and indicate actual health problems. 4 physiologic, situational, and maturational factors that cause the problem or influence its development.

3 Identifying characteristics derived from patient assessment is the third part of the nursing diagnosis , the signs and symptoms. Projected changes in health status are the outcomes or goals that are established. An individual's response to health pattern deficits is the definition of the problem statement, the first component of the nursing diagnosis. The factors that cause the problem or influence its development is the definition of etiology, the second component of the nursing diagnosis.

Why does the nurse ask the parents of a hospitalized child to bring the child's blanket from home? 1 To alleviate any fears in the child 2 To decrease any allergic reactions 3 To provide comfort for the child 4 To keep the child warm at night

3 If the parents cannot stay with the child in the hospital, the nurse may ask the parents to leave an article such as a blanket or toy from home. This is because young children associate such inanimate objects with significant people, and they gain comfort and reassurance from these possessions. When a child is frightened, the nurse should provide physical contact to ease the child. If the child is allergic to the linens at the hospital, it would be the hospital's responsibility to find alternative bedding. There are plenty of blankets available in the hospital, so the parents would not bring the blanket to keep the child warm.

A 10-year-old child has moderate cognitive impairment. With which activity would a teacher expect the child to need help? 1 Copying information from the board 2 Learning safe and healthy habits 3 Performing arithmetic calculations 4 Communicating with classmates

3 Students with moderate cognitive impairment (IQ of 50-55) have difficulty with functional reading and arithmetic calculations. The student can perform simple manual skills, such as copying information from the board, learning safe and healthy habits, and communicating with classmates.

The nurse is caring for a Mexican patient with severe febrile seizures. As a part of the care, the nurse removes the child's clothing and provides a bath while maintaining privacy. The nurse also takes off an amulet and necklace worn by the child. Later in the day, the child's parents refuse to speak to the nurse. What is the likely cause for their behavior? 1 They oppose treatments in conventional medicine. 2 They did not like the nurse touching and bathing their child. 3 They did not want the amulet and necklace to be removed. 4 Their spiritual healer forbids them to talk to the nurse

3 Many religions believe in supernatural causes and cures of diseases. Many Mexican people wear amulets and necklaces, which they believe ward off evil and protect a person from evil eye and diseases. The nurse took off the amulet and necklace without speaking to the parents or asking their permission. This is the most likely cause of the parents' displeasure. The parents brought the child to the hospital, which suggests they have no objection to modern medicine or nursing care.

The parents of a 10-year-old child diagnosed with terminal cancer ask the nurse not to inform their child about the condition. What would be the most appropriate response of the nurse? 1 Report the situation to the appropriate child welfare organization. 2 Ignore the parents' request and tell the child about the condition. 3 Inform the parents that being honest with the child is important. 4 Tell the parents that treatment cannot be initiated until the child is informed.

3 Often, parents ask health care providers not to reveal the prognosis to the terminally ill child. This is a difficult situation for health care providers. Although children do not understand that they are going to die, they are in a position to understand that something is seriously wrong with them. Therefore the nurse should make the parents understand that being honest with the child is important. Being honest creates an open environment where feelings may be shared by the entire family. This often encourages the parents to communicate openly with the child. The nurse is not required to report to the child welfare organization at this stage. The parents' decision should not be ignored; rather, the parents should be encouraged to talk openly with the child. The nurse cannot withhold treatment.

A preschooler is diagnosed with terminal stage bronchial carcinoma. What would the child consider the most probable reason for the condition? 1 The child will blame the parents for acquiring the disease. 2 The child will blame the grandparents for acquiring this condition. 3 The child would consider it to be a punishment for his or her own actions. 4 The child would understand the pathophysiology of the disease.

3 Preschoolers tend to believe that their thoughts or actions are sufficient to cause death or disease. Therefore, the child is most likely to think that the disease is a punishment for the child's action. This may result in the child feeling guilty. The nurse should help parents to understand this kind of reaction of their child and encourage them to be with the child. The child will not blame the parents or grandparents for the condition. The child is too young to understand the pathophysiology of the disease.

Parents who are divorcing are worried about how the divorce will affect their child. Which symptom indicates unhealthy coping in the child? 1 Body aches 2 Increased sleep 3 Loss of appetite 4 Body rash

3 The most common manifestation of distress in children is loss of appetite. The child is emotionally distressed by the parents' separation, and this stress can be manifested by a loss of appetite. Body aches, increased sleep, and rash over the body are physical symptoms that are very rarely seen in emotional distress. These symptoms may be caused by a viral infection.

Which statement describes a reconstituted family? 1 A child who lives with another family that has legal guardianship 2 A child staying with grandparents, away from the parents 3 A child staying with the father and the new stepmother 4 A child living in a family with both parents working for a living

3 Reconstituted families are families made up of stepfathers or stepmothers with or without their respective children. Children staying with another family with legal guardianship are in foster care. Children staying with their grandparents are in kinship care. Families where both parents work are called dual-earner families.

A parent tells a 13-year-old child that he is adopted. What could happen as a result of telling the child at this age? 1 Sympathy 2 Happiness 3 Depression 4 Excitement

3 The earlier the child knows of his adoption status, the better. Generally, older children display anger and sadness. This can often be manifested as depression. The child may feel abandoned, but a feeling of sympathy is rare. Similarly, happiness and excitement are not what a child feels after learning about being adopted.

The nurse is caring for a child with cancer. What should the nurse ask the child's parents about in order to obtain information about the child's coping-stress tolerance pattern? 1 "How do you both handle discipline problems at home?" 2 "Have you ever noticed if your child has many friends?" 3 "How does your child usually handle disappointment?" 4 "Who will be staying with your child at the hospital?"

3 The nurse should ask the child's parents about how the child usually handles disappointment. This can help the nurse understand the coping-stress tolerance pattern of the child. It is also helpful for identifying stressors in the child. It is important to know how discipline problems are managed in the child. This helps to know about the child and parent role-relationship pattern. When the nurse asks about the child's friends, it is to assess the child's role and relationship patterns outside the home. The nurse can understand the role and relationship pattern between the parents and child after knowing who will stay in the hospital with the child.

During assessment of a 7-month-old child, the nurse checks the child's height and weight and compares them with previous assessment records. The nurse finds that the child's height has increased by 1.25 cm, and the weight is 140 g more than in the previous month. What does the nurse infer from this observation? 1 The child is displaying symptoms of Down syndrome. 2 The child's weight is not ideal in relation to height. 3 The child's height and weight are ideal. 4 The child has a calcium deficiency due to malnutrition.

3 The nurse should regularly check the height and weight of the child and compare them with previous assessment records. These comparisons help the nurse identify genetic defects that can affect the child's growth and development. A child gains 140 g in weight, and height increases by 1.25 cm every month from ages 6 to 12 months. Therefore, this child has an ideal height and weight. Down syndrome is characterized by a slower growth rate. The child is having age-appropriate increases in height and weight and thus does not have Down syndrome. Calcium deficiency decreases bone density and causes fractures in children. The nurse cannot determine whether the child has calcium deficiency by assessing height and weight. Test-Taking Tip: Make certain that the answer you select is reasonable and obtainable under ordinary circumstances and that the action can be carried out in the given situation.

The nurse notices that a child's spleen is quite large. To which age group does the child belong? 1 0-12 months 2 1-6 years 3 6-12 years 4 12-18 years

3 The spleen is easily palpated between the ages of 6 and 12 years. If it is palpated at any other age, such as 0-12 months, 1-6 years, or 12-18 years, this finding must be reported to the primary health care provider.

An infant's blood glucose levels are low, and the nurse instructs the mother to perform kangaroo care. Which condition would the nurse have assessed in the child? 1 Irregular sleep patterns 2 Reduced metabolism 3 Improper thermoregulation 4 Impaired maturation

3 Thermoregulation is one of the most important adaptations for an infant to develop. A hypothermic infant tends to develop conditions such as hypoglycemia and metabolic acidosis. Skin-to-skin contact, or kangaroo care, is beneficial in maintaining the infant's temperature. Sleep irregularities may develop when an infant is suffering from pain internally or externally, which would also affect the growth and development at early postnatal development. Metabolic rates are usually high in children. Neurological maturation, or a dramatic increase in the number of neurons, occurs when the infant is in the embryonic stage and the neonatal state.

A child with strabismus is undergoing treatment for impaired vision of the left eye. The nurse covers the child's right eye with an occlusion patch. Why does the nurse do so? 1 To protect the right eye from dust 2 To reduce intraocular pressure in the left eye 3 To increase vision in the left eye 4 To prevent the child from rubbing the right eye

3 While caring for a child with strabismus, the nurse should cover the unaffected eye with an occlusive patch because it helps stimulate vision and movement in the weaker eye. The main reason for applying an ocular patch is to improve vision in the left eye, not to protect the right eye from dust. Applying an ocular patch on the right eye does not reduce intraocular pressure in the left eye; antiglaucoma medications can be used to reduce intraocular pressure. Applying an occlusion patch will not prevent the child from rubbing his or her eyes. The nurse should explain to the child that rubbing the eyes may cause further damage.

A child in the clinic exhibits reduced visual acuity in one eye despite appropriate optical correction. The nurse expects the child's health care provider to diagnosis the child with: 1 myopia. 2 hyperopia. 3 amblyopia. 4 astigmatism.

3 Visual acuity in one eye despite appropriate optical correction is amblyopia. Myopia is nearsightedness, which is the ability to see objects up close but not clearly at a distance. Hyperopia is farsightedness, which is the ability to see distant objects clearly but not those up close. Astigmatism is an alteration in vision caused by unequal curvature in the refractive apparatus of the eye.

The nurse is removing the tape of an intravenous catheter in a child. What is the most appropriate instruction given by the nurse? 1 "Stay calm while I remove the catheter." 2 "This will cause just a little bit of pain." 3 "Let's remove the tape together." 4 "You are the bravest kid in the world."

3 When it is time to discontinue an intravenous infusion, many children are distressed by the thought of catheter removal. Encouraging children to remove or help remove the tape from the site provides them with a measure of control and often fosters their cooperation. The nurse should not remove the tape without first asking the child for cooperation, so asking he child to remain calm and then remove the catheter is inappropriate. The nurse should not use the word pain; saying that it would cause discomfort is more appropriate. Telling children that they are bravest in the world may actually make them fear removal of the catheter as children are used to such statements during distress.

The nurse is assessing a child with Down syndrome. What findings in the child should alert the nurse to report to the health care provider immediately? Select all that apply. 1 Loss of pain sensation 2 Loss of impulse control 3 Loss of established motor skill 4 Loss of established bowel control 5 Loss of established bladder control

3, 4, 5 Loss of established motor skill and bowel and bladder control indicate spinal cord compression and must be reported immediately. The child with Down syndrome may have persistent neck pain caused by spinal cord compression. These children do not have impaired pain sensation. Children with Down syndrome are not aggressive. Loss of impulse control is not seen in such children.

The nursing instructor is explaining the risk factors and pathogenesis of Down syndrome to a group of nursing students. What information should the nurse include in the explanation? Select all that apply. 1 It is caused by a mutation of chromosomes. 2 It is more likely to occur if the paternal age is more than 35 years. 3 It is more likely to occur if the maternal age is more than 35 years. 4 It is caused by acquisition of an extra sex chromosome. 5 It is caused by acquisition of an extra autosomal chromosome.

3, 5 Maternal age more than 35 years increases the risk of having babies with Down syndrome. Down syndrome is caused by the presence of an extra autosomal chromosome. Down syndrome is not caused by a mutation of chromosomes. Advanced paternal age is not a risk factor for Down syndrome. There is no extra sex chromosome in children with Down syndrome.

What is a common postoperative complication of anesthesia? 1 Respiratory tract infections 2 Cardiac arrest 3 Infection of the joints 4 Resistance to anesthetic agents

4

The nurse is reviewing the plan of care with the parent of a child with Crohn's disease. Which statement made by the parent indicates a need for further education? 1 "This illness is why my child has delayed puberty." 2 "This illness is caused by the intake of fiber-rich food." 3 "This illness has impaired my child's growth and development." 4 "This illness is God's way of testing our religious faith."

4 A parent who states that the child's illness is a trial sent by God to test religious faith is expressing a self-accusatory feeling that indicates a lack of understanding of the condition. Crohn's disease is an autoimmune disorder that affects the gastrointestinal tract, causing inflammation that can result in excessive diarrhea, abdominal pain, and other symptoms. Crohn's disease can stunt growth and weaken bones in children who are going through puberty. Parents are advised to refrain from serving fiber-rich food to such children because these foods are difficult to digest and may block the intestine.

The nurse is caring for a child who is in the final stages of death. What can the nurse do to provide comfort for the parents? 1 Explain to the parents that births and deaths are normal. 2 Inform the family that their child will soon be at peace. 3 Ask the parents to stay away from the child at this time. 4 Have the parents focus on the quality of the child's remaining life.

4 Acknowledging grief is often the first step toward facing the reality of the child's illness. It is important to have the parents spend the time that is left with their child. Such acceptance may help parents focus on the quality of the child's remaining life. The nurse should not be so blunt as to say that births and deaths are normal. It is not showing compassion. Even though the child will be at peace when the child dies, this is not the best response because it does not provide comfort. The nurse should not ask the parents to stay away from their child in their final stages of life.

The nursing instructor is teaching a group of students about using chest tubes in children. Which statement by the student indicates a need for additional teaching? 1 Excess fluid is removed by chest tubes. 2 Chest tubes clear air from lungs. 3 Pneumothorax may need chest tubes. 4 Chest tubes eliminate leaked blood.

4 Before chest tube insertion, hematologic and coagulation studies are assessed for any risk of bleeding during the procedure. Chest tubes therefore do not remove blood. They are used to remove fluid or air from the pleural or pericardial space. Chest tube placement is often a necessity for pneumothorax, hemothorax, chylothorax, empyema, and pleural or pericardial effusion.

A 27-year-old patient reports having difficulty sleeping, having a poor appetite, and feeling sad since the death of her spouse. What should the nurse identify in this patient as a normal response to the death of the spouse? 1 Grief 2 Mourning 3 Acceptance 4 Bereavement

4 Bereavement is a process of grief experienced by most people. Examples include feelings of sadness, insomnia, poor appetite, deprivation, and desolation. The grieving person may seek professional help for the relief of symptoms if they interfere with daily activities and do not subside within a few months of the loss. Grief is a normal, appropriate emotional response to an external or consciously recognized loss. Mourning is a term used to describe a person's outward expression of grief. The person experiences emotional detachment from the loved object or person. Acceptance is the stage where the patient has achieved inner and outer peace through a personal victory over fear.

The parent of a visually impaired infant says to the nurse, "I am afraid that my child may not be able to bond with me because my child cannot maintain eye contact with me." The nurse teaches the parent about other signs that indicate that the child is responding. What should the nurse include in the explanation? 1 "The child compensates by increasing listening to your voice and smiling." 2 "The child's attention span decreases when a parent is trying to communicate." 3 "The child does not make throaty sounds when a parent is trying to communicate." 4 "The child's breathing or activity increases when the child is in contact with or near a parent."

4 Changes in respiratory patterns and increasing activity reflect the child's excitement about being close to the parent. The child is able to hear the parent's voice but is not able to smile when hearing the parent's voice at this age. This happens after about 2 years of age. Decreased attention span could be caused by lack of interest in the communication. When a child makes sounds in response to the parent's communication, it is sign of intimate bonding between the child and parent.

Evidence-based practice, a current health care trend, is best described as: 1 gathering evidence of mortality and morbidity in children. 2 meeting physical and psychosocial needs of the child and family in all areas of practice. 3 using a professional code of ethics as a means for professional self-regulation. 4 questioning why something is effective and whether there is a better approach.

4 Evidence-based practice helps to focus on measurable outcomes and the use of demonstrated, effective interventions and questions whether there is a better approach. Gathering evidence of mortality and morbidity in children will assist the nurse in determining areas of concern and potential involvement. It is not possible to meet all needs of the family and child in all areas of practice. The nurse is an advocate for the family. This is part of the professional role and licensure. Test-Taking Tip: Start by reading each of the answer options carefully. Usually at least one of them will be clearly wrong. Eliminate this one from consideration. Now you have reduced the number of response choices by one and improved the odds. Continue to analyze the options. If you can eliminate one more choice in a four-option question, you have reduced the odds to 50/50. While you are eliminating the wrong choices, recall often occurs. One of the options may serve as a trigger that causes you to remember what a few seconds ago had seemed completely forgotten.

A child is hospitalized for a chronic illness. Initially, the child showed symptoms of depression but later started interacting with others. What does the nurse infer from the patient's behavior? The child is: 1 Content with the care provided. 2 Showing improved social skills. 3 Getting used to the surroundings. 4 Detached from both parents.

4 Hospitalized children undergo depression when they are separated from their parents. As they go through the stages of separation anxiety, children eventually detach from their parents and develop new and shallow relationships. Children interact with others and develop new relationships as a result of resignation, not contentment. Children who are detached begin to show increased interest in their surroundings. They are also not developing their social interaction skills. Children try not to think about the separation; hence, they start developing new interactions.

The parents of a child are grieving the loss of their child. The mother of the child has disturbed sleep. The father informs the nurse that the mother can hear their child talk. The mother wants to be left alone and does not speak to any of the family members. What should be the most appropriate response of the nurse? 1 Tell the father that his wife may have mental breakdown in near future. 2 Tell the father that his wife is coping very poorly and needs professional help. 3 Inform the father that his wife has hallucinations and needs psychiatric referral. 4 Inform the father that it is a normal part of grief and would subside over time.

4 Grief reactions vary from person to person and it is required for the healing to occur. It is important for the nurse to inform the husband that hearing the dead person talk and distancing from others is very common reaction. It does not indicate any impending mental breakdown, poor coping, or psychiatric problem. Therefore, the husband should be reassured that his wife is trying to cope with the grief and she is normal.

The parents brought their child to the emergency department after a needle penetrated the child's eye. Which action should the nurse perform while caring for the child? 1 Examine the eye to look for foreign bodies. 2 Irrigate the eye to remove the needle from the eye. 3 Evert the upper eyelid to wash the eye thoroughly. 4 Observe for hyphema and reaction of the pupil to light.

4 If a child has a penetrating eye injury of any kind, the nurse should examine the eye to determine whether any aqueous humor has leaked from the penetration site. The nurse should observe the presence of hyphema, or bleeding from the eye. The nurse should also assess for pupillary reaction to light because it helps assess the functioning of the pupil. The nurse does not need to examine the eye for foreign bodies because there is already a foreign body in the eye. If the child is experiencing a penetrating eye injury, the nurse does not irrigate the eye to remove the object because this can further damage the cornea. In the case of chemical burns, the nurse rinses the eye by everting the upper eyelid.

A child who is terminally ill with bone cancer is in severe pain. Nursing interventions should be based on knowledge that: 1 children tend to be overmedicated for pain. 2 giving large doses of opioids causes euthanasia. 3 narcotic addiction is common in terminally ill children. 4 large doses of opioids are justified when there are no other treatment options.

4 Large doses may be needed because the child has become physiologically tolerant to the drug, requiring higher doses to achieve the same degree of pain control. Continuing studies report that children are consistently undermedicated for pain. The dose is titrated to relieve pain. Addiction refers to a psychological dependence on the medication, which does not happen in terminal care.

According to Piaget, the adolescent is in the fourth stage of cognitive development, or period of: a. Formal operations. c. Conventional thought. b. Concrete operations. d. Postconventional thought.

A Cognitive thinking culminates with capacity for abstract thinking. This stage, the period of formal operations, is Piaget's fourth and last stage. The concrete operations stage usually develops between ages 7 and 11 years. Conventional and postconventional thought refer to Kohlberg's stages of moral development.

The role of the pediatric nurse is influenced by trends in health care. The greatest trend in health care is: 1 primary focus on treatment of disease or disability. 2 national health care planning on a distributive or episodic basis. 3 accountability to professional codes and international standards. 4 shift of focus to prevention of illness and maintenance of health.

4 Prevention is the current focus of health care, one in which nursing plays a major role. Traditionally, treating disease or disability is the role of the physician. National health care planning is not a major trend. Accountability to professional codes is an established responsibility, not a trend.

The nurse is evaluating the quality of evidence of research found in the field of infectious diseases in infants. The nurse found unusually strong evidence from unbiased observational studies. What grade should be given to this research according to the GRADE criteria? 1 Low 2 High 3 Very low 4 Moderate

4 The GRADE criteria are used for grading the evidence of research by nurses. According to this, if evidence is found to be unusually strong from unbiased observational studies, it should be graded as moderate. Low grade is given to the evidence with at least one critical outcome from the observation studies. High grade is given to the studies with exceptionally strong evidence from unbiased observational studies and very low grade is given to studies with at least one critical outcome from very indirect evidence.

The diagnosis of cognitive impairment is based on the presence of: 1 intelligence quotient (IQ) of 75 or less. 2 IQ of 70 or less. 3 subaverage intellectual functioning, deficits in adaptive skills, and onset at any age. 4 subaverage intellectual functioning, deficits in adaptive skills, and onset before 18 years of age.

4 The diagnosis of cognitive impairment includes subaverage intellectual functioning and deficits in adaptive skills, including an onset before age 18. IQ is only one component of the diagnosis of cognitive impairment. The onset of the deficit must be before age 18 to meet the diagnosis of cognitive impairment.

The parents of a child who is dying of leukemia are grieving their impending loss. What intervention should the nurse perform to provide effective care for the family? 1 Encourage the use of alcohol and drugs as a way to escape grief. 2 Refer the family to a self-help group to manage their grief. 3 Emphasize that acute grieving is a process that lasts for years. 4 Provide the child and family with time to share memories.

4 The nurse should be very sensitive to the feelings of a grieving family. The nurse should also provide the child and family with the opportunity to review special experiences or memories in their lives. This helps the family members share their emotions and bond with each other. Parents and family members should be advised to seek professional counseling or work with each other to handle grief. The nurse should never encourage people to use alcohol or drugs to escape grief. The nurse can refer the family to a self-help group, but this is not always an effective way to help grieving parents. They may need a professional counselor to help guide them through the grieving process. The nurse should emphasize to the family that acute grieving only lasts for a few weeks to months, but the entire grieving process is a painful process that often takes years to resolve.

Which intervention should the nurse incorporate to prevent hypothermia in an infant? 1 Give hot milk or hot water to the infant at regular intervals. 2 Place the unclothed, diapered infant in the sun for few hours. 3 Feed the infant formula, which is higher in calories. 4 Put the unclothed, diapered infant on the mother's bare chest

4 Thermoregulation is not well developed in infants. As a result, babies are at risk for hypothermia. Kangaroo care is an effective way to prevent hypothermia in the infant. In this method, an unclothed, diapered infant is placed on the mother's bare chest. This provides physiological warmth to the infant. It is not advisable to give hot milk or hot water to the infant because it can damage their tissues. An unclothed infant must not be kept in the sun for a long time because the sun's ultraviolet rays can cause skin damage. There is no difference in calories between breast milk and formula. Test-Taking Tip: Avoid selecting answers that state hospital rules or regulations as a reason or rationale for action.

The nurse has to follow a method of problem identification and problem solving. Arrange the steps of the nursing process model in an appropriate order. 1. Planning 2. Evaluation 3. Diagnosis 4. Assessment 5. Implementation

4, 3, 1, 5, 2 The five-step nursing process model is assessment, diagnosis, planning, implementation, and evaluation. The first step is assessment in which the nurse collects, and analyzes subjective and objective data of the patient. This step is very crucial in determining the Client Needs. The second step is the formulation of the nursing diagnosis. In this phase, the nurse interprets and makes decisions about the acquired data. The third step is planning wherein the nurse develops a care plan and establishes the desired outcomes in the patient. This care plan also includes the interventions that would help the patient to achieve the desired outcomes. The fourth step is implementation in which the interventions that have been listed in the care plan are carried out in a systematic manner. Evaluation is the last step of nursing process which establishes the efficacy of the interventions provided to the patient.

A (6 to 8 weeks The bones surrounding the posterior fontanel fuse and close by age 6 to 8 weeks. Ten weeks or longer is too late.)

By what age does the posterior fontanel usually close? a. 6 to 8 weeks b. 10 to 12 weeks c. 4 to 6 months d. 8 to 10 months

Which statement best describes a child who is abused by the parent(s)? a. Unintentionally contributes to the abusing situation b. Belongs to a low socioeconomic population c. Is healthier than the nonabused siblings d. Abuses siblings in the same way as child is abused by the parent(s)

A A child's temperament, position in the family, additional physical needs, activity level, or degree of sensitivity to parental needs unintentionally contributes to the abusing situation. Socioeconomic status is an environmental characteristic. This child is less likely to be abused than one who is premature, disabled, or very young. The abused child does not in turn abuse his or her siblings.

Preschoolers' fears can best be dealt with by which intervention? a. Actively involving them in finding practical methods to deal with the frightening experience b. Forcing them to confront the frightening object or experience in the presence of their parents c. Using logical persuasion to explain away their fears and help them recognize how unrealistic the fears are d. Ridiculing their fears so they understand that there is no need to be afraid

A Actively involving the child in finding practical methods to deal with the frightening experience is the best way to deal with fears. Forcing a child to confront fears may make the child more afraid. Preconceptual thought prevents logical understanding. Ridiculing fears does not make them go away.

Which aspect of cognition develops during adolescence? a. Capability to use a future time perspective b. Ability to place things in a sensible and logical order c. Ability to see things from the point of view of another d. Progress from making judgments based on what they see to making judgments based on what they reason

A Adolescents are no longer restricted to the real and actual. They also are concerned with the possible; they think beyond the present. During concrete operations (between ages 7 and 11 years), children exhibit the ability to place things in a sensible and logical order, the ability to see things from another's point of view, and the ability to make judgments based on what they reason rather than just what they see.

In terms of cognitive development, the 5-year-old child would be expected to: a. Use magical thinking. b. Think abstractly. c. Understand conservation of matter. d. Be able to comprehend another person's perspective.

A Magical thinking is believing that thoughts can cause events. Abstract thought does not develop until school-age years. The concept of conservation is the cognitive task of school-age children ages 5 to 7 years. Five-year-olds cannot understand another's perspective.

An adolescent girl tells the nurse that she has suicidal thoughts. The nurse asks her if she has a specific plan. Asking this should be considered: a. An appropriate part of the assessment. b. Not a critical part of the assessment. c. Suggesting that the adolescent needs a plan. d. Encouraging the adolescent to devise a plan.

A Routine health assessments of adolescents should include questions that assess the presence of suicidal ideation or intent. Questions such as "Have you ever developed a plan to hurt yourself or kill yourself?" should be part of that assessment. Threats of suicide should always be taken seriously and evaluated. Suggesting that the adolescent needs a plan and encouraging her to devise this plan would be inappropriate statements by the nurse.

A common characteristic of those who sexually abuse children is that they: a. Pressure the victim into secrecy. b. Are usually unemployed and unmarried. c. Are unknown to victims and victims' families. d. Have many victims that are each abused only once.

A Sex offenders may pressure the victim into secrecy, regarding the activity as a "secret between us" that other people may take away if they find out. Abusers are often employed upstanding members of the community. Most sexual abuse is committed by men and persons who are well known to the child. Abuse is often repeated with the same child over time. The relationship may start insidiously without the child realizing that sexual activity is part of the offer.

Which is the most significant factor in distinguishing those who commit suicide from those who make suicidal attempts or threats? a. Social isolation c. Degree of depression b. Level of stress d. Desire to punish others

A Social isolation is a significant factor in distinguishing adolescents who will kill themselves from those who will not. It is also more characteristic of those who complete suicide than of those who make attempts or threats. Level of stress, degree of depression, and desire to punish others are contributing factors in suicide, but they are not the most significant factor in distinguishing those who complete suicide from those who attempt suicide.

D (The child is exhibiting the ritualism that is characteristic at this age. Ritualism is the need to maintain sameness and reliability. It provides a sense of comfort to the toddler. It will dictate certain principles in feeding practices, including rejecting a favorite food because it is served in a different container. This does not indicate a child who has unreasonable expectations or a need to exert control, but rather normal development. Toddlers use ritualistic behaviors to maintain necessary structure in their lives. This is not regression, which is a retreat from a present pattern of functioning.)

A father tells the nurse that his daughter wants the same plate and cup used at every meal, even if they go to a restaurant. The nurse should explain that this is: a. A sign that the child is spoiled. b. A way to exert unhealthy control. c. Regression, common at this age. d. Ritualism, common at this age.

B (The blanket is an important transitional object.)

A hospitalized toddler clings to a worn, tattered blanket. She screams when anyone tries to take it away. What is the nurse's BEST explanation to the parents for the child's attachment to the blanket? A. The blanket encourages immature behavior. B. The blanket is an important transitional object. C. She has not mastered the developmental task of individuation-separation. D. She has not bonded adequately with her mother.

D (A topical anesthetic, eutectic mixture of local anesthetic (EMLA), can be applied before injections are given. Several topical anesthetic agents can be used to minimize the discomfort associated with immunization injections. These include EMLA and vapor coolant sprays. Pain associated with many procedures can be prevented or minimized by using the principles of atraumatic care. With preparation, the injection site can be properly anesthetized to decrease the amount of pain felt by the infant. Infants have the neural pathways to sense pain. Numerous research studies have indicated that infants perceive and react to pain in the same manner as do children and adults. The mother should be allowed to discuss her concerns and the alternatives available. This is part of the informed consent process.)

A mother tells the nurse that she doesn't want her infant immunized because of the discomfort associated with injections. The nurse should explain that: a. This cannot be prevented. b. Infants do not feel pain as adults do. c. This is not a good reason for refusing immunizations. d. A topical anesthetic, eutectic mixture of local anesthetic (EMLA), can be applied before injections are given.

C (commercial iron-fortified formula For children younger than 1 year, the American Academy of Pediatrics recommends the use of breast milk. If breastfeeding has been discontinued, iron-fortified commercial formula should be used. Cow's milk should not be used in children younger than 12 months. Maternal iron stores are almost depleted by this age; the iron-fortified formula will help prevent the development of iron deficiency anemia.)

A mother tells the nurse that she is discontinuing breastfeeding her 5-month-old infant. The nurse should recommend that the infant be given: a. Skim milk. b. Whole cow's milk. c. Commercial iron-fortified formula d. Commercial formula without iron.

D (Semiformed, seedy, yellow)

A nurse is caring for a 2-month-old exclusively breastfed infant with an admitting diagnosis of colic. Based on the nurse's knowledge of breastfed infants, what type of stool is expected? A. Dark brown and small hard pebbles B. Loose with green mucus streaks C. Formed and with white mucus D. Semiformed, seedy, yellow

Strict isolation is required for a child who is hospitalized with (select all that apply): a. Mumps. b. Chickenpox. c. Exanthema subitum (roseola). d. Erythema infectiosum (fifth disease). e. Parvovirus B19.

A, B, C, D Childhood communicable diseases requiring strict transmission-based precautions (Contact, Airborne, and Droplet Precautions) include diphtheria, chickenpox, measles, mumps, tuberculosis, adenovirus, Haemophilus influenzae type B, mumps, pertussis, plague, streptococcal pharyngitis, and scarlet fever. Strict isolation is not required for parvovirus B19.

While caring for a child, the nurse provides small toys and works overtime to take care of the child, and even calls the hospital during off-duty time to find out whether the child is improving. The nurse regularly meets the mother outside of the hospital. The nurse asks the mother if she is involved in care of the child. Which actions of the nurse indicate a nontherapeutic nurse-patient relationship? Select all that apply. A Giving a toy to the child B Working overtime to look after the child C Calling the hospital frequently to inquire about the child D Asking whether the mother is involved in care of the child E Meeting the mother outside of the hospital to discuss the child

A, B, C, E Giving toys, clothes, food, and other items to the patient; working overtime to take care of a particular patient; calling the hospital or patient's home frequently to inquire about the patient's health; and meeting the patient or family outside of the hospital indicate that the nurse is overinvolved with the child and the family. Thus, such actions should be avoided to develop a good nurse-patient relationship. Asking if the mother is involved in the child's care indicates that the nurse is concerned about the patient and the family. It indicates a therapeutic relationship between the nurse and the child and family.

Which play patterns does a 3-year-old child typically display (select all that apply)? a. Imaginary play b. Parallel play c. Cooperative play d. Structured play e. Associative play

A, B, C, E Children between ages 3 and 5 years enjoy parallel and associative play. Children learn to share and cooperate as they play in small groups. Play is often imitative, dramatic, and creative. Imaginary friends are common around age 3 years. Structured play is typical of school-age children.

Which assessment findings indicate to the nurse a child has Down syndrome (select all that apply)? a. High-arched, narrow palate b. Protruding tongue c. Long, slender fingers d. Transverse palmar crease e. Hypertonic muscle tone

A, B, D

A 9-month-old child has been treated after a choking incident. Which advice does the nurse give to the parents to prevent further incidents? Select all that apply. A "Never leave your child unattended." B "Your child is too young to be allowed to eat solid food." C "Make sure all cabinets, drawers, and containers are childproof." D "Marbles and LEGOs are not appropriate toys for children at that age." E "Allowing your child to crawl on the floor increases the risk for injury."

A, C, D Crawling infants may explore their environments through taste and touch. They tend to put everything in their mouths. Therefore, the nurse should advise the parents to always keep a watch on their child and never leave their child unattended. Children at 9 months of age will begin to be able to pull themselves up to standing positions, potentially giving them access to cabinets, drawers, and containers that they were unable to access before. Therefore, the parents need to thoroughly childproof the home. The parents should be warned about providing small, colorful toys or leaving small objects on the floor because the child will be attracted to such objects and may attempt to swallow them, which may result in choking. The child is 8 months old. By this age, solid foods are permitted. The child should be allowed to play and crawl on the floor, as this is helpful for the development of the child's gross motor skills.

Research has shown that the most successful smoking cessation programs among teens include (select all that apply): a. Peer-led education and support. b. Information on the long-term effects of smoking. c. Programs including the media. d. School-based programs. e. Information on the immediate effects of smoking.

A, C, D, E Two areas of antismoking campaigns that have shown success are those that are peer-led and use media in education related to smoking prevention. School-based programs have also shown success and can be strengthened by expansion into the community and youth groups. Teens respond much better to education that focuses on the immediate effects of smoking. For the most part, smoking prevention programs that focus on the negative long-term effects of smoking have been ineffective.

Which actions of the nurse indicate that the nurse is providing atraumatic care to the patient who is admitted in the intensive care unit and his or her family? Select all that apply. A The nurse patiently listens to the parent's concerns. B The nurse spends off-duty time playing with the child. C The nurse allows a parent to stay with the child at all times. D The nurse gives the appropriate pain medications to the child. E The nurse explains the treatment given to the child to the parents

A, D, E One important goal of atraumatic care is to reduce psychological distress of patients and their families. It is important that the nurse patiently listens to the parents' concerns and provides emotional support. Preventing and minimizing bodily pain is another important aspect of atraumatic care. Therefore, the nurse should give appropriate pain medications to the child. Another goal of atraumatic care is to keep the parents informed about the child's disease, treatments given, and the disease progression. Thus, the nurse should explain the treatments given to the parents. Spending off-duty time with the patient indicates that the nurse is overinvolved with the patient. Allowing a parent to stay with the child at all times is not part of atraumatic care.

The nurse is planning care for an adolescent with AIDS. Which is the priority nursing goal? a. Preventing infection b. Preventing secondary cancers c. Restoring immunologic defenses d. Identifying source of infection

ANS: A Because the child is immunocompromised in association with HIV infection, the prevention of infection is paramount. Although certain precautions are justified in limiting exposure to infection, these must be balanced with the concern for the childs normal developmental needs. Preventing secondary cancers is not currently possible. Current drug therapy is affecting the disease progression; although not a cure, these drugs can suppress viral replication, preventing further deterioration. Case finding is not a priority nursing goal.

Latex allergy is suspected in a child with spina bifida. Appropriate nursing interventions include which action? a. Avoid using any latex product. b. Use only nonallergenic latex products. c. Administer medication for long-term desensitization. d. Teach family about long-term management of asthma.

ANS: A Care must be taken that individuals who are at high risk for latex allergies do not come in direct or secondary contact with products or equipment containing latex at any time during medical treatment. There are no nonallergenic latex products. At this time, desensitization is not an option. The child does not have asthma. The parents must be taught about allergy and the risk of anaphylaxis.

The nurse is teaching parents about the importance of iron in a toddlers diet. Which explains why iron deficiency anemia is common during toddlerhood? a. Milk is a poor source of iron. b. Iron cannot be stored during fetal development. c. Fetal iron stores are depleted by age 1 month. d. Dietary iron cannot be started until age 12 months.

ANS: A Children between the ages of 12 and 36 months are at risk for anemia because cows milk is a major component of their diet and it is a poor source of iron. Iron is stored during fetal development, but the amount stored depends on maternal iron stores. Fetal iron stores are usually depleted by age 5 to 6 months. Dietary iron can be introduced by breastfeeding, iron-fortified formula, and cereals during the first 12 months of life.

A possible cause of acquired aplastic anemia in children is: a. drugs. b. injury. c. deficient diet. d. congenital defect.

ANS: A Drugs, such as chemotherapeutic agents and several antibiotics (e.g., chloramphenicol), can cause aplastic anemia. Injury, deficient diet, and congenital defect are not causative agents in acquired aplastic anemia.

The nurse is conducting a staff in-service on common problems associated with myelomeningocele. Which common problem is associated with this defect? a. Hydrocephalus b. Craniosynostosis c. Biliary atresia d. Esophageal atresia

ANS: A Hydrocephalus is a frequently associated anomaly in 80% to 90% of children. Craniosynostosis is the premature closing of the cranial sutures and is not associated with myelomeningocele. Biliary and esophageal atresia is not associated with myelomeningocele.

The nurse is planning a staff in-service on childhood spastic cerebral palsy. Spastic cerebral palsy is characterized by: a. hypertonicity and poor control of posture, balance, and coordinated motion. b. athetosis and dystonic movements. c. wide-based gait and poor performance of rapid, repetitive movements. d. tremors and lack of active movement.

ANS: A Hypertonicity and poor control of posture, balance, and coordinated motion are part of the classification of spastic cerebral palsy. Athetosis and dystonic movements are part of the classification of dyskinetic (athetoid) cerebral palsy. Wide-based gait and poor performance of rapid, repetitive movements are part of the classification of ataxic cerebral palsy. Tremors and lack of active movement may indicate other neurologic disorders.

When a preschool child is hospitalized without adequate preparation, the nurse should recognize that the child may likely see hospitalization as: a. Punishment. b. Threat to childs self-image. c. An opportunity for regression. d. Loss of companionship with friends

ANS: A If a toddler is not prepared for hospitalization, a typical preschooler fantasy is to attribute the hospitalization to punishment for real or imagined misdeeds. Threat to childs self-image and loss of companionship with friends are reactions typical of school-age children. Regression is a response characteristic of toddlers when threatened with loss of control.

Which of the following types of seizures may be difficult to detect? a. Absence b. Generalized c. Simple partial d. Complex partial

ANS: A Absence seizures may go unrecognized because little change occurs in the child's behavior during the seizure. Generalized, simple partial, and complex partial seizures all have clinical manifestations that are observable.

The nurse is teaching a group of nursing students about newborns born with the congenital defect of myelomeningocele. Which common problem is associated with this defect? a. Neurogenic bladder b. Mental retardation c. Respiratory compromise d. Cranioschisis

ANS: A Myelomeningocele is one of the most common causes of neuropathic (neurogenic) bladder dysfunction among children. Risk of mental retardation is minimized through early intervention and management of hydrocephalus. Respiratory compromise is not a common problem in myelomeningocele. Cranioschisis is a skull defect through which various tissues protrude. It is not associated with myelomeningocele.

A 4-year-old child has just been diagnosed with pseudohypertrophic (Duchenne) muscular dystrophy. The management plan should include which action? a. Recommend genetic counseling. b. Explain that the disease is easily treated. c. Suggest ways to limit use of muscles. d. Assist family in finding a nursing facility to provide his care.

ANS: A Pseudohypertrophic (Duchenne) muscular dystrophy is inherited as an X-linked recessive gene. Genetic counseling is recommended for parents, female siblings, maternal aunts, and their female offspring. No effective treatment exists at this time for childhood muscular dystrophy. Maintaining optimal function of all muscles for as long as possible is the primary goal. It has been found that children who remain as active as possible are able to avoid wheelchair confinement for a longer time. Assisting the family in finding a nursing facility to provide his care is inappropriate at the time of diagnosis. When the child becomes increasingly incapacitated, the family may consider home-based care, a skilled nursing facility, or respite care to provide the necessary care.

A 10-year-old girl needs to have another intravenous (IV) line started. She keeps telling the nurse, Wait a minute, and, Im not ready. The nurse should recognize that: a. This is normal behavior for a school-age child. b. This behavior is usually not seen past the preschool years. c. The child thinks the nurse is punishing her. d. The child has successfully manipulated the nurse in the past.

ANS: A This school-age child is attempting to maintain control. The nurse should provide the girl with structured choices about when the IV will be inserted. This can be characteristic behavior when an individual needs to maintain some control over a situation. The child is trying to have some control in the hospital experience.

Which can result from the bone demineralization associated with immobility? a. Osteoporosis b. Urinary retention c. Pooling of blood d. Susceptibility to infection

ANS: A Bone demineralization leads to a negative calcium balance, osteoporosis, pathologic fractures, extraosseous bone formation, and renal calculi. Urinary retention is secondary to the effect of immobilization on the urinary tract. Pooling of blood is a result of the cardiovascular effects of immobilization. Susceptibility to infection can result from the effects of immobilization on the respiratory and renal systems.

Which drug should the nurse expect to administer to a preschool child who has increased intracranial pressure (ICP) resulting from cerebral edema? a. Mannitol (Osmitrol) b. Epinephrine hydrochloride (Adrenalin) c. Atropine sulfate (Atropine) d. Sodium bicarbonate (Sodium bicarbonate)

ANS: A For increased ICP, mannitol, an osmotic diuretic, administered intravenously, is the drug used most frequently for rapid reduction. Epinephrine hydrochloride, atropine sulfate, and sodium bicarbonate are not used to decrease ICP.

An adolescent with osteosarcoma is scheduled for a leg amputation in 2 days. The nurse's approach should include which action? a. Answering questions with straightforward honesty b. Avoiding discussing the seriousness of the condition c. Explaining that, although the amputation is difficult, it will cure the cancer d. Assisting the adolescent in accepting the amputation as better than a long course of chemotherapy

ANS: A Honesty is essential to gain the child's cooperation and trust. The diagnosis of cancer should not be disguised with falsehoods. The adolescent should be prepared for the surgery so he or she has time to reflect on the diagnosis and subsequent treatment. This allows questions to be answered. To accept the need for radical surgery, the child must be aware of the lack of alternatives for treatment. Amputation is necessary, but it will not guarantee a cure. Chemotherapy is an integral part of the therapy with surgery. The child should be informed of the need for chemotherapy and its side effects before surgery.

Which term is used to describe a type of fracture that does not produce a break in the skin? a. Simple b. Compound c. Complicated d. Comminuted

ANS: A If a fracture does not produce a break in the skin, it is called a simple, or closed, fracture. A compound, or open, fracture is one with an open wound through which the bone protrudes. A complicated fracture is one in which the bone fragments damage other organs or tissues. A comminuted fracture occurs when small fragments of bone are broken from the fractured shaft and lie in the surrounding tissue. These are rare in children.

The nurse is preparing an adolescent with scoliosis for a Luque-rod segmental spinal instrumentation procedure. Which consideration should the nurse include? a. Nasogastric intubation and urinary catheter may be required. b. Ambulation will not be allowed for up to 3 months. c. Surgery eliminates the need for casting and bracing. d. Discomfort can be controlled with nonpharmacologic methods.

ANS: A Luque-rod segmental spinal instrumentation is a surgical procedure. Nasogastric intubation and urinary catheterization may be required. Ambulation is allowed as soon as possible. Depending on the instrumentation used, most patients walk by the second or third postoperative day. Casting and bracing are required postoperatively. The child usually has considerable pain for the first few days after surgery. Intravenous opioids should be administered on a regular basis.

14.What should the nurse include in a teaching plan for the parents of a child with vesicoureteral reflux? a. The importance of taking prophylactic antibiotics b. Suggestions for how to maintain fluid restrictions c. The use of bubble baths as an incentive to increase bath time d. The need for the child to hold urine for 6 to 8 hours

ANS: A Prophylactic antibiotics are used to prevent urinary tract infections (UTIs) in a child with vesicoureteral reflux, although this treatment plan has become controversial. Fluids are not restricted when a child has vesicoureteral reflux. In fact, fluid intake should be increased as a measure to prevent UTIs. Bubble baths should be avoided to prevent urethral irritation and possible UTI. To prevent UTIs, the child should be taught to void frequently and never resist the urge to urinate.

Which is the priority nursing intervention for an unconscious child after a fall? a. Establish adequate airway. b. Perform neurologic assessment. c. Monitor intracranial pressure. d. Determine whether a neck injury is present.

ANS: A Respiratory effectiveness is the primary concern in the care of the unconscious child. Establishment of an adequate airway is always the first priority. A neurologic assessment and determination of whether a neck injury is present will be performed after breathing and circulation are stabilized. Intracranial, not intercranial, pressure is monitored if indicated after airway, breathing, and circulation are maintained.

30. The nurse is conducting an assessment on a school-age child with urosepsis. Which assessment finding should the nurse expect? a. Fever with a positive blood culture b. Proteinuria and edema c. Oliguria and hypertension d. Anemia and thrombocytopenia

ANS: A Symptoms of urosepsis include a febrile urinary tract infection coexisting with systemic signs of bacterial illness; blood culture reveals the presence of a urinary pathogen. Proteinuria and edema are symptoms of minimal change nephrotic syndrome. Oliguria and hypertension are symptoms of acute glomerulonephritis. Anemia and thrombocytopenia are symptoms of hemolytic uremic syndrome.

35. Prior to administering IV chemotherapy, which action by the nurse is most important? A. Ensure the IV has a good blood return. B. Provide diversionary activities. C. Take and record a set of vital signs. D. Weigh the child.

ANS: A To prevent extravasation of IV chemotherapy it is important to make sure the line flushes easily and has a good blood return. This is a critical action to maintain patient safety. The other actions may also be utilized, but would not take priority over ensuring patient safety.

24. The earliest recognizable clinical manifestation of cystic fibrosis (CF) is: a. Meconium ileus. b. History of poor intestinal absorption. c. Foul-smelling, frothy, greasy stools. d. Recurrent pneumonia and lung infections.

ANS: A The earliest clinical manifestation of CF is a meconium ileus, which is found in about 10% of children with CF. Clinical manifestations include abdominal distention, vomiting, failure to pass stools, and rapid development of dehydration. History of malabsorption is a later sign that manifests as failure to thrive. Foulsmelling stools and recurrent respiratory infections are later manifestations of CF.

32. Which vaccine is now recommended for the immunization of all newborns? a. Hepatitis A vaccine b. Hepatitis B vaccine c. Hepatitis C vaccine d. Hepatitis A, B, and C vaccines

ANS: B Universal vaccination for hepatitis B is now recommended for all newborns. A vaccine is available for hepatitis A, but it is not yet universally recommended. No vaccine is currently available for hepatitis C. Only hepatitis B vaccine is recommended for newborns.

14. A 4-year-old child is several days postoperative after a resection of a brain tumor. The nurse finds the child irritable and lethargic, and notes that she has vomited. Which medication does the nurse anticipate administering? A. Dexamethasone (Decadron) B. Fosphenytoin (Cerebyx) C. Odansetron (Zofran) D. Phenytoin (Dilantin)

ANS: A This child has manifestations of increased intracranial pressure, a possible outcome after brain surgery. The nurse prepares to administer a corticosteroid to decrease the edema. Fosphenytoin and phenytoin are for seizures. Odansetron is for nausea.

37. A child has the following laboratory values: WBC, 7.2 mm3; bands, 4%; and neutrophils, 60%. Based on these values, which action by the nurse is the most appropriate? A. Continue monitoring the child for infection. B. Place the child on protective isolation. C. Obtain two sets of blood cultures. D. Restrict visitors to the child.

ANS: A This child's absolute neutrophil count is 4,608; therefore, the child is not neutropenic. The nurse should continue to monitor. The other actions are not necessary.

24. A parent confides to the nurse that a friend, who is 32, has been diagnosed with Hodgkin's disease. The parent says "I thought only children get that!" What response by the nurse is the most appropriate? A. "No, there are both young adult and older adult forms." B. "Usually people over the age of 50 do not get this." C. "Yes, only children under the age of 10 are affected." D. "You are right; your friend must have misspoken."

ANS: A Three groups are affected by Hodgkin's disease: children younger than 14, young adults 15-34 years of age, and older adults 55-74 years of age. The parent's friend could certainly be correct about the diagnosis.

18. An adolescent with osteosarcoma is scheduled for a leg amputation in 2 days. The nurse's approach should include which action? a. Answering questions with straightforward honesty b. Avoiding discussing the seriousness of the condition c. Explaining that, although the amputation is difficult, it will cure the cancer d. Assisting the adolescent in accepting the amputation as better than a long course of chemotherapy

ANS: A Honesty is essential to gain the child's cooperation and trust. The diagnosis of cancer should not be disguised with falsehoods. The adolescent should be prepared for the surgery so he or she has time to reflect on the diagnosis and subsequent treatment. This allows questions to be answered. To accept the need for radical surgery, the child must be aware of the lack of alternatives for treatment. Amputation is necessary, but it will not guarantee a cure. Chemotherapy is an integral part of the therapy with surgery. The child should be informed of the need for chemotherapy and its side effects before surgery.

7. The nurse is administering an IV chemotherapeutic agent to a child diagnosed with leukemia. The child suddenly begins to wheeze and have severe urticaria. Which is the most appropriate nursing action? a. Stop drug infusion immediately. b. Recheck rate of drug infusion. c. Observe child closely for next 10 minutes. d. Explain to child that this is an expected side effect.

ANS: A If an allergic reaction is suspected, the drug should be immediately discontinued. Any drug in the line should be withdrawn, and a normal saline infusion begun to keep the line open. Rechecking the rate of drug infusion, observing the child closely for next 10 minutes, and explaining to the child that this is an expected side effect can all be done after the drug infusion is stopped and the child is evaluated.

2. A child with growth hormone (GH) deficiency is receiving GH therapy. What is the best time for the GH to be administered? a. At bedtime b. After meals c. Before meals d. On arising in the morning

ANS: A Injections are best given at bedtime to more closely approximate the physiologic release of GH. Before or after meals and on arising in the morning are times that do not mimic the physiologic release of the hormone.

8. A young child is brought to the emergency department with severe dehydration secondary to acute diarrhea and vomiting. Therapeutic management of this child will begin with which intervention? a. Intravenous fluids b. Oral rehydration solution (ORS) c. Clear liquids, 1 to 2 ounces at a time d. Administration of antidiarrheal medication

ANS: A Intravenous fluids are initiated in children with severe dehydration. ORS is acceptable therapy if the dehydration is not severe. Diarrhea is not managed by using clear liquids by mouth. These fluids have a high carbohydrate content, low electrolyte content, and high osmolality. Antidiarrheal medications are not recommended for the treatment of acute infectious diarrhea.

20. What is the earliest clinical manifestation of biliary atresia? a. Jaundice b. Vomiting c. Hepatomegaly d. Absence of stooling

ANS: A Jaundice is the earliest and most striking manifestation of biliary atresia. It is first observed in the sclera and may be present at birth, but is usually not apparent until ages 2 to 3 weeks. Vomiting is not associated with biliary atresia. Hepatomegaly and abdominal distention are common but occur later. Stools are large and lighter in color than expected because of the lack of bile.

1. Which demonstrates the school-age child's developing logic in the stage of concrete operations? (Select all that apply.) a. The school-age child is able to recognize that he can be a son, brother, or nephew at the same time. b. The school-age child understands the principles of adding, subtracting, and reversibility. c. The school-age child understands the principles of adding, subtracting, and reversibility. d. The school-age child has thinking that is characterized by egocentrism and animism

ANS: A, B, C The school-age child understands that the properties of objects do not change when their order, form, or appearance does. Conservation occurs in the concrete operations stage. Comprehension of class inclusion occurs as the school-age child's logic increases. The child begins to understand that a person can be in more than one class at the same time. This is characteristic of concrete thinking and logical reasoning. The school-age child is able to understand principles of adding, subtracting, and the process of reversibility, which occurs in the stage of concrete operations. Thinking that is characterized by egocentrism and animism occurs in the intuitive thought stage, not the concrete operations stage of development.

The nurse is conducting discharge teaching to parents of a preschool child with myelomeningocele, repaired at birth, being discharged from the hospital after a urinary tract infection (UTI). Which should the nurse include in the discharge instructions related to management of the childs genitourinary function? (Select all that apply.) a. Continue to perform the clean intermittent catheterizations (CIC) at home. b. Administer the oxybutynin chloride (Ditropan) as prescribed. c. Reduce fluid intake in the afternoon and evening hours. d. Monitor for signs of a recurrent urinary tract infection. e. Administer furosemide (Lasix) as prescribed.

ANS: A, B, D Discharge teaching to prevent renal complications in a child with myelomeningocele include: (1) regular urologic care with prompt and vigorous treatment of infections; (2) a method of regular emptying of the bladder, such as clean intermittent catheterization (CIC) taught to and performed by parents and self-catheterization taught to children; (3) medications to improve bladder storage and continence, such as oxybutynin chloride (Ditropan) and tolterodine (Detrol). Fluids should not be limited and Lasix is not used to improve renal function for children with myelomeningocele.

8. A child has been admitted with a paraspinal Ewing's sarcoma. The nursing instructor questions the student about assessing for signs of spinal cord compression. Which manifestations does this include? (Select all that apply.) A. Burning pain down the legs B. Difficulty with swallowing C. "Foot drop," causing a limp D. Respiratory depression E. Weakness in the hands

ANS: A, C, E Common manifestations of spinal cord compression include burning pain, often down the legs; foot drop causing difficulty with ambulation; and weakness in the hands. Other manifestations include numbness, cramping, and loss of sensation in the feet, and sexual dysfunction in the older patient. Swallowing problems and respiratory depression are not related.

4. The nurse should expect to assess which clinical manifestations in an adolescent with Cushing's syndrome? (Select all that apply.) a. Hyperglycemia b. Hyperkalemia c. Hypotension d. Cushingoid features e. Susceptibility to infections

ANS: A, D, E In Cushing's syndrome, physiologic disturbances seen are cushingoid features, hyperglycemia, susceptibility to infection, hypertension, and hypokalemia.

The nurse is conducting a staff in-service on casts. Which is an advantage to using a fiberglass cast instead of a plaster of Paris cast? a. Cheaper b. Dries rapidly c. Molds closely to body parts d. Smooth exterior

ANS: B A synthetic casting material dries in 5 to 30 minutes as compared with a plaster cast, which takes 10 to 72 hours to dry. Synthetic casts are more expensive and have a rough exterior, which may scratch surfaces. Plaster casts mold closer to body parts.

Amy, age 6 years, needs to be hospitalized again because of a chronic illness. The clinic nurse overhears her school-age siblings tell her, We are sick of Mom always sitting with you in the hospital and playing with you. It isnt fair that you get everything and we have to stay with the neighbors. The nurses best assessment of this situation is that: a. The siblings are immature and probably spoiled. b. Jealousy and resentment are common reactions to the illness or hospitalization of a sibling. c. The family has ineffective coping mechanisms to deal with chronic illness. d. The siblings need to better understand their sisters illness and needs.

ANS: B Siblings experience loneliness, fear, worry, anger, resentment, jealousy, and guilt. The siblings experience stress equal to that of the hospitalized child. These are not uncommon responses by normal siblings. There is no evidence that the family has maladaptive coping or that the siblings lack understanding.

Which immunization should be given with caution to children infected with human immunodeficiency virus (HIV)? a. Influenza b. Varicella c. Pneumococcal d. Inactivated poliovirus (IPV)

ANS: B The children should be carefully evaluated before being given live viral vaccines such as varicella, measles, mumps, and rubella. The child must be immunocompetent and not have contact with other severely immunocompromised individuals. Influenza, pneumococcal, and inactivated poliovirus (IPV) are not live vaccines.

A nurse in the emergency department is assessing a 5-year-old child with symptoms of pneumonia and a fever of 102 F. Which intervention can the nurse implement to promote a sense of control for the child? a. None, this is an emergency and the child should not participate in care. b. Allow the child to hold the digital thermometer while taking the childs blood pressure. c. Ask the child if it is OK to take a temperature in the ear. d. Have parents wait in the waiting room.

ANS: B The nurse should allow the child to hold the digital thermometer while taking the childs blood pressure. Unless an emergency is life threatening, children need to participate in their care to maintain a sense of control. Because emergency departments are frequently hectic, there is a tendency to rush through procedures to save time. However, the extra few minutes needed to allow children to participate may save many more minutes of useless resistance and uncooperativeness during subsequent procedures. The child may not give permission, if asked, for a procedure that is necessary to be performed. It is better to give choices such as, Which ear do you want me to do your temperature in? instead of, Can I take your temperature? Parents should remain with their child to help with decreasing the childs anxiety.

The parents of a child hospitalized with sickle cell anemia tell the nurse that they are concerned about narcotic analgesics causing addiction. Which is appropriate for the nurse to explain about narcotic analgesics? a. Are often ordered but not usually needed b. Rarely cause addiction because they are medically indicated c. Are given as a last resort because of the threat of addiction d. Are used only if other measures, such as ice packs, are ineffective

ANS: B The pain of sickle cell anemia is best treated by a multidisciplinary approach. Mild to moderate pain can be controlled by ibuprofen and acetaminophen. When narcotics are indicated, they are titrated to effect and are given around the clock. Patient-controlled analgesia reinforces the patients role and responsibility in managing the pain and provides flexibility in dealing with pain. Few, if any, patients who receive opioids for severe pain become behaviorally addicted to the drug. Narcotics are often used because of the severe nature of the pain of vasoocclusive crisis. Ice is contraindicated because of its vasoconstrictive effects.

1. Which statement best describes why children have fewer respiratory tract infections as they grow older? a. The amount of lymphoid tissue decreases. b. Repeated exposure to organisms causes increased immunity. c. Viral organisms are less prevalent in the population. d. Secondary infections rarely occur after viral illnesses.

ANS: B Children have increased immunity after exposure to a virus. The amount of lymphoid tissue increases as children grow older. Viral organisms are not less prevalent, but older children have the ability to resist invading organisms. Secondary infections after viral illnesses include Mycoplasma pneumoniae and groups A and B streptococcal infections

A school-age child has sustained a head injury and multiple fractures after being thrown from a horse. The child's level of consciousness is variable. The parents tell the nurse that they think their child is in pain because of periodic crying and restlessness. The most appropriate nursing action is to: a. discuss with parents the child's previous experiences with pain. b. discuss with practitioner what analgesia can be safely administered. c. explain that analgesia is contraindicated with a head injury. d. explain that analgesia is unnecessary when child is not fully awake and alert.

ANS: B A key nursing role is to provide sedation and analgesia for the child. Consultation with the appropriate practitioner is necessary to avoid conflict between the necessity to monitor the child's neurologic status and the promotion of comfort and relief of anxiety. Information on the child's previous experiences with pain should be obtained as part of the assessment, but because of the severity of injury, analgesia should be provided as soon as possible. Analgesia can be safely used in individuals who have sustained head injuries and can decrease anxiety and resultant increased ICP.

Which statement best describes a subdural hematoma? a. Bleeding occurs between the dura and the skull. b. Bleeding occurs between the dura and the cerebrum. c. Bleeding is generally arterial, and brain compression occurs rapidly. d. The hematoma commonly occurs in the parietotemporal region.

ANS: B A subdural hematoma is bleeding that occurs between the dura and the cerebrum as a result of a rupture of cortical veins that bridge the subdural space. An epidural hemorrhage occurs between the dura and the skull, is usually arterial with rapid brain concussion, and occurs most often in the parietotemporal region.

17. The nurse is caring for a child with carbon monoxide (CO) poisoning associated with smoke inhalation. What is essential in this childs care? a. Monitor pulse oximetry. b. Monitor arterial blood gases. c. Administer oxygen if respiratory distress develops. d. Administer oxygen if childs lips become bright, cherry red

ANS: B Arterial blood gases and COHb levels are the best way to monitor CO poisoning. PaO2 monitored with pulse oximetry may be normal in the case of CO poisoning. Oxygen at 100% should be given as quickly as possible, not only if respiratory distress or other symptoms develop.

The nurse is preparing a school-age child for computed tomography (CT scan) to assess cerebral function. The nurse should include which statement in preparing the child? a. "Pain medication will be given." b. "The scan will not hurt." c. "You will be able to move once the equipment is in place." d. "Unfortunately, no one can remain in the room with you during the test."

ANS: B For CT scans, the child must be immobilized. It is important to emphasize to the child that at no time is the procedure painful. Pain medication is not required; however, sedation is sometimes necessary. Someone is able to remain with the child during the procedure.

When taking the history of a child hospitalized with Reye syndrome, the nurse should not be surprised that a week ago the child had recovered from: a. measles. b. varicella. c. meningitis. d. hepatitis.

ANS: B Most cases of Reye syndrome follow a common viral illness such as varicella or influenza. Measles, meningitis, and hepatitis are not associated with Reye syndrome.

6. A nurse is caring for a child who is scheduled to have intrathecal chemotherapy today. Which action by the nurse is most important when providing care to this patient and family? A. Educating family on side effects of chemotherapy B. Ensuring a signed consent is on the chart C. Providing distraction techniques during the process D. Reassuring the child the parents will be present

ANS: B Intrathecal chemotherapy (introducing chemotherapy into the subarachnoid space of the spinal cord) is an invasive procedure and requires a signed consent. Although all actions are important for this child, the priority is ensuring the consent is executed appropriately and on the chart.

16. The nurse is caring for a child with acute respiratory distress syndrome (ARDS) associated with sepsis. Nursing actions should include: a. Force fluids. c. Institute seizure precautions. b. Monitor pulse oximetry. d. Encourage a high-protein diet

ANS: B Monitoring cardiopulmonary status is an important evaluation tool in the care of the child with ARDS. Maintenance of vascular volume and hydration is important and should be done parenterally. Seizures are not a side effect of ARDS. Adequate nutrition is necessary, but a high-protein diet is not helpful

36. A school-age child with chronic renal failure is admitted to the hospital with a serum potassium level of 5.2 mEq/L. Which prescribed medication should the nurse plan to administer? a. Spironolactone (Aldactone) b. Sodium polystyrene sulfonate (Kayexalate) c. Lactulose (Cephulac) d. Calcium carbonate (Calcitab)

ANS: B Normal serum potassium levels in a school-age child are 3.5 to 5 mEq/L. Sodium polystyrene sulfonate is administered to reduce serum potassium levels. Spironolactone is a potassium-sparing diuretic and should not be used if the serum potassium is elevated. Lactulose is administered to reduce ammonia levels in patients with liver disease. Calcium carbonate may be prescribed as a calcium supplement, but it will not reduce serum potassium levels.

22.Calcium carbonate is given with meals to a child with chronic renal disease. The purpose of this is to: a. Prevent vomiting. b. Bind phosphorus. c. Stimulate appetite. d. Increase absorption of fat-soluble vitamins.

ANS: B Oral calcium carbonate preparations combine with phosphorus to decrease gastrointestinal absorption and the serum levels of phosphate; serum calcium levels are increased by the calcium carbonate, and vitamin D administration is necessary to increase calcium absorption. Calcium carbonate does not prevent vomiting, stimulate appetite, or increase the absorption of fat-soluble vitamins.

21. A child has been cured of a retinoblastoma. When the parents ask how long monitoring for bone-related complications of radiation therapy should continue, which is the most appropriate response by the nurse? A. "After 5 years, you can stop worrying about this." B. "Cancers of the bone can occur up to 15 years later." C. "Probably all complications will occur within 3 years." D. "Radiation complications do not occur in bones."

ANS: B Osteosarcoma can occur as a consequence of radiation therapy up to 15 years later.

39. A nurse works on a pediatric oncology unit. After receiving report, which child should the nurse assess first? A. Having infusion of D5 NS and sodium bicarbonate B. On high-dose methotrexate (Rheumatrex), urine pH of 7.8 C. Receiving cyclophosphamide (Cytoxan), urine specific gravity of 1.008 D. 2 days post-tumor resection, complaining of pain

ANS: B Patients on high-dose methotrexate need their urine pH to be higher than 7.0. This child needs the nurse's attention first. An IV with NaHCO3 is common prior to receiving methotrexate. A urine specific gravity of 1.010 is required for children on chemotherapy. Pain would be an expected finding 2 days postoperatively, and should be treated, but not before the nurse assesses the other child.

28. The narrowing of preputial opening of foreskin is called: a. Chordee b. Phimosis c. Epispadias d. Hypospadias

ANS: B Phimosis is the narrowing or stenosis of the preputial opening of the foreskin. Chordee is the ventral curvature of the penis. Epispadias is the meatal opening on the dorsal surface of the penis. Hypospadias is a congenital condition in which the urethral opening is located anywhere along the ventral surface of the penis.

6.The narrowing of the preputial opening of the foreskin is called: a. Chordee. b. Phimosis. c.Epispadias d.Hypospadias.

ANS: B Phimosis is the narrowing or stenosis of the preputial opening of the foreskin. Chordee is the ventral curvature of the penis. Epispadias is the meatal opening on the dorsal surface of the penis. Hypospadias is a congenital condition in which the urethral opening is located anywhere along the ventral surface of the penis.

9. Which statement is descriptive of the play of school-age children? a. Individuality in play is better tolerated than at earlier ages. b. Knowing the rules of a game gives an important sense of belonging. c. They like to invent games, making up the rules as they go. d. Team play helps children learn the universal importance of competition and winning.

ANS: B Play involves increased physical skill, intellectual ability, and fantasy. Children form groups and cliques and develop a sense of belonging to a team or club. At this age, children begin to see the need for rules. Conformity and ritual permeate their play. Their games have fixed and unvarying rules, which may be bizarre and extraordinarily rigid. With team play, children learn about competition and the importance of winning, an attribute highly valued in the United States.

A 5-year-old boy is being prepared for surgery to remove a brain tumor. Nursing actions should be based on which statement? a. Removal of tumor will stop the various symptoms. b. Usually the postoperative dressing covers the entire scalp. c. He is not old enough to be concerned about his head being shaved. d. He is not old enough to understand the significance of the brain.

ANS: B The child should be told what he will look and feel like after surgery. This includes the size of the dressing. The nurse can demonstrate on a doll the expected size and shape of the dressing. Some of the symptoms may be alleviated by the removal of the tumor, but postsurgical headaches and cerebellar symptoms such as ataxia may be aggravated. Children should be prepared for the loss of their hair, and it should be removed in a sensitive, positive manner if the child is awake. Children at this age have poorly defined body boundaries and little knowledge of internal organs. Intrusive experiences are frightening, especially those that disrupt the integrity of the skin.

12. The nurse encourages the mother of a toddler with acute laryngotracheobronchitis to stay at the bedside as much as possible. The nurses rationale for this action is primarily that: a. Mothers of hospitalized toddlers often experience guilt. b. The mothers presence will reduce anxiety and ease the childs respiratory efforts. c. Separation from the mother is a major developmental threat at this age. d. The mother can provide constant observations of the childs respiratory efforts.

ANS: B The familys presence will decrease the childs distress. The mother may experience guilt, but this is not the best answer. Although separation from the mother is a developmental threat for toddlers, the main reason to keep parents at the childs bedside is to ease anxiety and therefore respiratory effort. The child should have constant cardiorespiratory monitoring and noninvasive oxygen saturation monitoring, but the parent should not play this role in the hospital.

3.Which factor predisposes a child to urinary tract infections? a. Increased fluid intake b. Short urethra in young girls c. Prostatic secretions in males d. Frequent emptying of the bladder

ANS: B The short urethra in females provides a ready pathway for invasions of organisms. Increased fluid intake and frequent bladder emptying offer protective measures against urinary tract infections. Prostatic secretions have antibacterial properties that inhibit bacteria.

34. The nurse is conducting teaching for an adolescent being discharged to home after a renal transplantation. The adolescent needs further teaching if which statement is made? a. "I will report any fever to my primary health care provider." b. "I am glad I only have to take the immunosuppressant medication for two weeks." c. "I will observe my incision for any redness or swelling." d. "I won't miss doing kidney dialysis every week."

ANS: B The immunosuppressant medications are taken indefinitely after a renal transplantation, so they should not be discontinued after 2 weeks. Reporting a fever and observing an incision for redness and swelling are accurate statements. The adolescent is correct in indicating dialysis will not need to be done after the transplantation.

15. What is an important consideration for the school nurse who is planning a class on bicycle safety to consider? a. Most bicycle injuries involve collision with an automobile. b. Head injuries are the major causes of bicycle-related fatalities. c. Children should wear bicycle helmets if they ride on paved streets. d. Children should not ride double unless the bicycle has an extra-large seat.

ANS: B The most important aspect of bicycle safety is to encourage the rider to use a protective helmet. Head injuries are the major cause of bicycle-related fatalities. Although motor vehicle collisions do cause injuries to bicyclists, most injuries result from falls. The child should always wear a properly fitted helmet approved by the U.S. Consumer Product Safety Commission. Children should not ride double.

8. A neutropenic child is admitted to the hospital and placed in protective isolation. Which instruction does the nurse give the family to help maintain a safe environment for the child? A. Do not let the child have chewing gum B. Flowers, plants, and produce are not allowed C. The child can only have one visitor at a time D. Toys and items from home cannot be brought in

ANS: B The neutropenic child should not have fresh flowers, plants, fruits, or vegetables because they can harbor infectious microorganisms. The other instructions are not needed.

The nurse has received report on four children. Which child should the nurse assess first? a. A school-age child in a coma with stable vital signs b. A preschool child with a head injury and decreasing level of consciousness c. An adolescent admitted after a motor vehicle accident is oriented to person and place d. A toddler in a persistent vegetative state with a low-grade fever

ANS: B The nurse should assess the child with a head injury and decreasing level of consciousness first (LOC). Assessment of LOC remains the earliest indicator of improvement or deterioration in neurologic status. The next child the nurse should assess is a toddler in a persistent vegetative state with a low-grade fever. The school-age child in a coma with stable vital signs and the adolescent adm

7.An objective of care for the child with nephrosis is to: a. Reduce blood pressure. b. Reduce excretion of urinary protein. c. Increase excretion of urinary protein. d. Increase ability of tissues to retain fluid.

ANS: B The objectives of therapy for the child with nephrosis include reduction of the excretion of urinary protein, reduction of fluid retention, prevention of infection, and minimizing of complications associated with therapy. Blood pressure is usually not elevated in nephrosis. Increased excretion of urinary protein and increased ability of tissues to retain fluid are part of the disease process and must be reversed.

21. Which behavior by parents or teachers will best assist the child in negotiating the developmental task of industry? a. Identifying failures immediately and asking the child's peers for feedback b. Structuring the environment so the child can master tasks c. Completing homework for children who are having difficulty in completing assignments d. Decreasing expectations to eliminate potential failures

ANS: B The task of the caring teacher or parent is to identify areas in which a child is competent and to build on successful experiences to foster feelings of mastery and success. Structuring the environment to enhance self-confidence and to provide the opportunity to solve increasingly more complex problems will promote a sense of mastery. Asking peers for feedback reinforces the child's feelings of failure. When teachers or parents complete children's homework for them, it sends the message that they do not trust the children to do a good job. Providing assistance and suggestions and praising their best efforts are more appropriate. Decreasing expectations to eliminate failures will not promote a sense of achievement or mastery.

21. b-Adrenergic agonists and methylxanthines are often prescribed for a child with an asthma attack. What is their action? a. Liquefy secretions c. Reduce inflammation of the lungs b. Dilate the bronchioles d. Reduce infection

ANS: B These medications work to dilate the bronchioles in acute exacerbations. These medications do not liquefy secretions or reduce infection. Corticosteroids and mast cell stabilizers reduce inflammation in the lungs.

32. A nurse assesses a toddler using the FLACC score. The child is kicking and crying steadily. The mother is upset, as she is unable to console the child. Which action by the nurse is most appropriate? A. Administer acetaminophen (Tylenol). B. Give a dose of morphine (Duramorph). C. Play soothing, quiet music. D. Prepare a dose of propofol (Diprovan).

ANS: B This child exhibits several behaviors seen in the severe pain category according to the FLACC score. The best medication for this level of pain is an opioid analgesic, such as morphine. Tylenol is used for mild to moderate pain. Nonpharmacological measures can be used as an adjunct, but it will not relieve this degree of pain alone. Propofol is usually used for procedures.

5. A parent brings a child to the clinic and reports that the child is reluctant to walk and has a new limp. The parent also reports that the child seems lethargic and tired all the time. The nurse notes that the child appears pale. Which other finding would warrant immediate notification of the health-care provider? A. Difficulty staying asleep at night B. Left-sided abdominal enlargement C. Polyphagia and polydipsia D. Swelling of the legs and feet

ANS: B This child has some manifestations of acute lymphocytic leukemia (ALL). Left-sided abdominal enlargement could be indicative of splenomegaly, which is another manifestation of this disease. The nurse should report these findings immediately. Difficulty staying asleep at night is vague and could be related to a number of causes, both physical and behavioral. Polydipsia and polyphagia are two of the three classic signs of diabetes. Swelling of the legs and feet is not a manifestation of ALL.

26. What should a nurse advise the parents of a child with type 1 diabetes mellitus who is not eating as a result of a minor illness? a. Give the child half his regular morning dose of insulin. b. Substitute simple carbohydrates or calorie-containing liquids for solid foods. c. Give the child plenty of unsweetened, clear liquids to prevent dehydration. d. Take the child directly to the emergency department.

ANS: B A sick-day diet of simple carbohydrates or calorie-containing liquids will maintain normal serum glucose levels and decrease the risk of hypoglycemia. The child should receive his regular dose of insulin even if he does not have an appetite. If the child is not eating as usual, he needs calories to prevent hypoglycemia. During periods of minor illness, the child with type 1 diabetes mellitus can be managed safely at home.

4. What is a common cause of acute diarrhea? a. Hirschsprung's disease b. Antibiotic therapy c. Hypothyroidism d. Meconium ileus

ANS: B Acute diarrhea is a sudden increase in frequency and change in consistency of stools and may be associated with antibiotic therapy. Hirschsprung's disease, hypothyroidism, and meconium ileus are usually manifested with constipation rather than diarrhea.

14. Chronic adrenocortical insufficiency is also referred to as what? a. Graves' disease b. Addison's disease c. Cushing's syndrome d. Hashimoto's disease

ANS: B Addison's disease is chronic adrenocortical insufficiency. Graves' and Hashimoto's diseases involve the thyroid gland. Cushing's syndrome is a result of excessive circulation of free cortisol.

4. A youngster will receive a bone marrow transplant (BMT) made possible because an older siblings is a histocompatible donor. Which is this type of BMT called? a. Syngeneic b. Allogeneic c. Monoclonal d. Autologous

ANS: B Allogeneic transplants are from another individual. Because he and his sibling are histocompatible, the BMT can be done. Syngeneic marrow is from an identical twin. There is no such thing as a monoclonal BMT. Autologous refers to the individual's own marrow.

18. What is the most appropriate intervention for the parents of a 6-year-old girl with precocious puberty? a. Advise the parents to consider birth control for their daughter. b. Explain the importance of having the child foster relationships with same-age peers. c. Assure the child's parents that there is no increased risk for sexual abuse because of her appearance. d. Counsel parents that there is no treatment currently available for this disorder.

ANS: B Despite the child's appearance, the child needs to be treated according to her chronologic age and to interact with children in the same age-group. An expected outcome is that the child will adjust socially by exhibiting age-appropriate behaviors and social interactions. Advising the parents of a 6 year old to put their daughter on birth control is not appropriate and will not reverse the effects of precocious puberty. Parents need to be aware that there is an increased risk of sexual abuse for a child with precocious puberty. Treatment for precocious puberty is the administration of gonadotropin-releasing hormone blocker, which slows or reverses the development of secondary sexual characteristics and slows rapid growth and bone aging

10. Exophthalmos may occur in children with what diagnosis? a. Hypothyroidism b. Hyperthyroidism c. Hypoparathyroidism d. Hyperparathyroidism

ANS: B Exophthalmos (protruding eyeballs) is a clinical manifestation of hyperthyroidism. Hypothyroidism, hypoparathyroidism, and hyperparathyroidism are not associated with exophthalmos.

14. A 5 year old is being prepared for surgery to remove a brain tumor. Nursing actions should be based on which statement? a. Removal of tumor will stop the various symptoms. b. Usually the postoperative dressing covers the entire scalp. c. He is not old enough to be concerned about his head being shaved. d. He is not old enough to Comprehension the significance of the brain.

ANS: B The child should be told what he will look and feel like after surgery. This includes the size of the dressing. The nurse can demonstrate on a doll the expected size and shape of the dressing. Some of the symptoms may be alleviated by the removal of the tumor, but postsurgical headaches and cerebellar symptoms such as ataxia may be aggravated. Children should be prepared for the loss of their hair, and it should be removed in a sensitive, positive manner if the child is awake. Children at this age have poorly defined body boundaries and little knowledge of internal organs. Intrusive experiences are frightening, especially those that disrupt the integrity of the skin.

35. A nurse is reviewing the laboratory results on a school-age child diagnosed with hypoparathyroidism. Which results are consistent with this condition? a. Decreased serum phosphorus b. Decreased serum calcium c. Increased serum glucose d. Decreased serum cortisol

ANS: B The diagnosis of hypoparathyroidism is made on the basis of clinical manifestations associated with decreased serum calcium and increased serum phosphorus. Decreased serum phosphorus would be seen in hyperparathyroidism, elevated glucose in diabetes, and decreased serum cortisol in adrenocortical insufficiency (Addison's disease).

2. A boy with leukemia screams whenever he needs to be turned or moved. Which is the most probable cause of this pain? a. Edema b. Bone involvement c. Petechial hemorrhages d. Changes within the muscles

ANS: B The invasion of the bone marrow with leukemic cells gradually causes weakening of the bone and a tendency toward fractures. As leukemic cells invade the periosteum, increasing pressure causes severe pain. Edema, petechial hemorrhages, and changes within the muscles would not cause severe pain.

15. A neonate born with ambiguous genitalia is diagnosed with congenital adrenogenital hyperplasia. Therapeutic management includes administration of which medication? a. Vitamin D b. Cortisone c. Stool softeners d. Calcium carbonate

ANS: B The most common biochemical defect with congenital adrenal hyperplasia is partial or complete 21-hydroxylase deficiency. With complete deficiency, insufficient amounts of aldosterone and cortisol are produced, so circulatory collapse occurs without immediate replacement. Vitamin D, stool softeners, and calcium carbonate have no role in the therapy of adrenogenital hyperplasia.

The nurse is planning care for a school-age child admitted to the hospital with hemophilia. Which interventions should the nurse plan to implement for this child? (Select all that apply.) a. Finger sticks for blood work instead of venipunctures b. Avoidance of IM injections c. Acetaminophen (Tylenol) for mild pain control d. Soft tooth brush for dental hygiene e. Administration of packed red blood cells

ANS: B, C, D Nurses should take special precautions when caring for a child with hemophilia to prevent the use of procedures that may cause bleeding, such as IM injections. The subcutaneous route is substituted for IM injections whenever possible. Venipunctures for blood samples are usually preferred for these children. There is usually less bleeding after the venipuncture than after finger or heel punctures. Neither aspirin nor any aspirin-containing compound should be used. Acetaminophen is a suitable aspirin substitute, especially for controlling mild pain. A soft toothbrush is recommended for dental hygiene to prevent bleeding from the gums. Packed red blood cells are not administered. The primary therapy for hemophilia is replacement of the missing clotting factor. The products available are factor VIII concentrates.

4. A nurse teaches parents that team play is important for school-age children. Which abilities can children develop by experiencing team play? (Select all that apply.) a. Achieve personal goals over group goals. b. Learn complex rules. c. Experience competition. d. Learn about division of labor.

ANS: B, C, D Team play helps stimulate cognitive growth because children are called on to learn many complex rules, make judgments about those rules, plan strategies, and assess the strengths and weaknesses of members of their own team and members of the opposing team. Team play can also contribute to children's social, intellectual, and skill growth. Children work hard to develop the skills needed to become team members, to improve their contribution to the group, and to anticipate the consequences of their behavior for the group. Team play teaches children to modify or exchange personal goals for goals of the group; it also teaches them that division of labor is an effective strategy for attaining a goal.

3. A clinic nurse is conducting a staff in-service for other clinic nurses about signs and symptoms of a rhabdomyosarcoma tumor. Which should be included in the teaching session? (Select all that apply.) a. Bone fractures b. Abdominal mass c. Sore throat and ear pain d. Headache e. Ecchymosis of conjunctiva

ANS: B, C, E The initial signs and symptoms of rhabdomyosarcoma tumors are related to the site of the tumor and compression of adjacent organs. Some tumor locations, such as the orbit, manifest early in the course of the illness. Other tumors, such as those of the retroperitoneal area, only produce symptoms when they are relatively large and compress adjacent organs. Unfortunately, many of the signs and symptoms attributable to rhabdomyosarcoma are vague and frequently suggest a common childhood illness, such as "earache" or "runny nose." An abdominal mass, sore throat and ear pain, and ecchymosis of conjunctiva are signs of a rhabdomyosarcoma tumor. Bone fractures would be seen in osteosarcoma, and a headache is a sign of a brain tumor.

15. A pediatric patient is receiving asparaginase (Elspar). What manifestations would lead the nurse to determine that the child is having a possible side effect from this drug? (Select all that apply.) A. Blistering at infusion site B. Increased PT and INR C. Potassium of 2.7 mEq/L D. Seizures E. Shortness of breath

ANS: B, D Some common side effects of Elspar include seizures, hyperglycemia, nausea/vomiting, rashes, coagulation abnormalities, hepatotoxicity, pancreatitis, and anaphylaxis. Blistering is common with daunorubicin (Daunomycin). Hypokalemia is seen with carboplatin (Paraplatin). Shortness of breath could be seen with bleomycin (Blenoxane), which causes pulmonary fibrosis and pneumonitis.

5. A nurse is caring for several patients with acute lymphocytic leukemia (ALL). Which children does the nurse understand have the best prognosis? A. Infant B. < 10 years of age C. > 25% abnormal cells in bone marrow aspirate D. White count 4,200/mm3 E. White count 25,000/mm3

ANS: B, D The best prognosis for ALL occurs in children 2 to 9 years of age and in children whose initial white blood cell count is < 5,000/mm3. Children 10 and older and whose initial white blood cell counts are ?= 50,000/mm3 have worse prognoses. Infants have a very poor prognosis.

6. A mother who intended to breastfeed has given birth to an infant with a cleft palate. Which nursing interventions should be included in the plan of care? (Select all that apply.) a. Giving medication to suppress lactation. b. Encouraging and helping mother to breastfeed. c. Teaching mother to feed breast milk by gavage. d. Recommending use of a breast pump to maintain lactation until infant can suck.

ANS: B, D The mother who wishes to breastfeed may need encouragement and support because the defect does present some logistical issues. The nipple must be positioned and stabilized well back in the infant's oral cavity so that the tongue action facilitates milk expression. The suction required to stimulate milk, absent initially, may be useful before nursing to stimulate the let-down reflex. Because breastfeeding is an option, if the mother wishes to breastfeed, medications should not be given to suppress lactation. Because breastfeeding can usually be accomplished, gavage feedings are not indicated.

1. Which expected appearance will the nurse explain to parents of an infant returning from surgery after an enucleation was performed to treat retinoblastoma? (Select all that apply.) a. A lot of drainage will come from the affected socket. b. The face may be edematous or ecchymotic. c. The eyelids will be sutured shut for the first week. d. There will be an eye pad dressing taped over the surgical site. e. The implanted sphere is covered with conjunctiva and resembles the lining of the mouth.

ANS: B, D, E After enucleation surgery, the parents are prepared for the child's facial appearance. An eye patch is in place, and the child's face may be edematous or ecchymotic. Parents often fear seeing the surgical site because they imagine a cavity in the skull. A surgically implanted sphere maintains the shape of the eyeball, and the implant is covered with conjunctiva. When the eyelids are open, the exposed area resembles the mucosal lining of the mouth. The dressing, consisting of an eye pad taped over the surgical site, is changed daily. The wound itself is clean and has little or no drainage. So expecting a lot of drainage is not accurate to tell parents. The eyelids are not sutured shut after enucleation surgery.

4. The nurse is preparing to care for an infant returning from pyloromyotomy surgery. Which prescribed orders should the nurse anticipate implementing? (Select all that apply.) a. Nothing by mouth for 24 hours b. Administration of analgesics for pain c. Ice bag to the incisional area d. Intravenous (IV) fluids continued until tolerating fluids by mouth e. Clear liquids as the first feeding

ANS: B, D, E Feedings are usually instituted soon after a pyloromyotomy surgery, beginning with clear liquids and advancing to formula or breast milk as tolerated. IV fluids are administered until the infant is taking and retaining adequate amounts by mouth. Appropriate analgesics should be given round the clock because pain is continuous. Ice should not be applied to the incisional area as it vasoconstricts and would reduce circulation to the incisional area and impair healing.

13. What is the most common clinical manifestation(s) of brain tumors in children? a. Irritability b. Seizures c. Headaches and vomiting d. Fever and poor fine motor control

ANS: C Headaches, especially on awakening, and vomiting that is not related to feeding are the most common clinical manifestation(s) of brain tumors in children. Irritability, seizures, and fever and poor fine motor control are clinical manifestations of brain tumors, but headaches and vomiting are the most common.

Iron dextran is ordered for a young child with severe iron deficiency anemia. Nursing considerations include to: a. administer with meals. b. administer between meals. c. inject deeply into a large muscle. d. massage injection site for 5 minutes after administration of drug.

ANS: C Iron dextran is a parenteral form of iron. When administered intramuscularly, it must be injected into a large muscle. Iron dextran is for intramuscular or intravenous (IV) administration. The site should not be massaged to prevent leakage, potential irritation, and staining of the skin.

Which is most descriptive of the pathophysiology of leukemia? a. Increased blood viscosity occurs. b. Thrombocytopenia (excessive destruction of platelets) occurs. c. Unrestricted proliferation of immature white blood cells (WBCs) occurs. d. First stage of coagulation process is abnormally stimulated.

ANS: C Leukemia is a group of malignant disorders of the bone marrow and lymphatic system. It is defined as an unrestricted proliferation of immature WBCs in the blood-forming tissues of the body. Increased blood viscosity may occur secondary to the increased number of WBCs. Thrombocytopenia may occur secondary to the overproduction of WBCs in the bone marrow. The coagulation process is unaffected by leukemia.

A school-age child, admitted for intravenous antibiotic therapy for osteomyelitis, reports difficulty in going to sleep at night. Which intervention should the nurse implement to assist the child in going to sleep at bedtime? a. Request a prescription for a sleeping pill. b. Allow the child to stay up late and sleep late in the morning. c. Create a schedule similar to the one the child follows at home. d. Plan passive activities in the morning and interactive activities right before bedtime.

ANS: C Many children obtain significantly less sleep in the hospital than at home; the primary causes are a delay in sleep onset and early termination of sleep because of hospital routines. One technique that can minimize the disruption in the childs routine is establishing a daily schedule. This approach is most suitable for noncritically ill school-age and adolescent children who have mastered the concept of time. It involves scheduling the childs day to include all those activities that are important to the child and nurse, such as treatment procedures, schoolwork, exercise, television, playroom, and hobbies. The school-age child with osteomyelitis would benefit from a schedule similar to the one followed at home. Requesting a prescription for a sleeping pill would be inappropriate, and allowing the child to stay up late and sleep late would not be keeping the child in a routine followed at home. Passive activities in the morning and interactive activities at bedtime should be reversed; it would be better to keep the child active in the morning hours and plan quiet activities at bedtime.

23.The diet of a child with chronic renal failure is usually characterized as: a. High in protein. b. Low in vitamin D. c. Low in phosphorus. d. Supplemented with vitamins A, E, and K.

ANS: C Dietary phosphorus is controlled to prevent or control the calcium/phosphorus imbalance by the reduction of protein and milk intake. Protein should be limited in chronic renal failure to decrease intake of phosphorus. Vitamin D therapy is administered in chronic renal failure to increase calcium absorption. Supplementation with vitamins A, E, and K is not part of dietary management in chronic renal disease.

A home care nurse is caring for an adolescent with a T1 spinal cord injury. The adolescent suddenly becomes flushed, hypertensive, and diaphoretic. Which intervention should the nurse perform first? a. Place the adolescent in a flat right side-lying position. b. Place a cool washcloth on the adolescents forehead and continue to monitor the blood pressure. c. Implement a standing prescription to empty the bladder with a sterile in and out Foley catheter. d. Take a full set of vital signs and notify the health care provider.

ANS: C The adolescent is experiencing an autonomic dysreflexia episode. The paralytic nature of autonomic function is replaced by autonomic dysreflexia, especially when the lesions are above the mid-thoracic level. This autonomic phenomenon is caused by visceral distention or irritation, particularly of the bowel or bladder. Sensory impulses are triggered and travel to the cord lesion, where they are blocked, which causes activation of sympathetic reflex action with disturbed central inhibitory control. Excessive sympathetic activity is manifested by a flushing face, sweating forehead, pupillary constriction, marked hypertension, headache, and bradycardia. The precipitating stimulus may be merely a full bladder or rectum or other internal or external sensory input. It can be a catastrophic event unless the irritation is relieved. Placing a cool washcloth on the adolescents forehead, continuing to monitor blood pressure and vital signs, and notifying the healthcare provider would not reverse the sympathetic reflex situation.

A nurse is preparing to complete an admission assessment on a 2-year-old child. The child is sitting on the parents lap. Which technique should the nurse implement to complete the physical exam? a. Ask the parent to place the child in the hospital crib. b. Take the child and parent to the exam room. c. Perform the exam while the child is on the parents lap. d. Ask the child to stand by the parent while completing the exam.

ANS: C The nurse should complete the exam while the child is on the parents lap. For young children, particularly infants and toddlers, preserving parent-child contact is the best means of decreasing the need for or stress of restraint. The entire physical examination can be done in a parents lap with the parent hugging the child for procedures such as an otoscopic examination. Placing the child in the crib, taking the child to the exam room, or asking the child to stand by the parent would separate the child from the parent and cause anxiety.

The nurse is reviewing prenatal vitamin supplements with an expectant client. Which supplement should be included in the teaching? a. Vitamin A throughout pregnancy b. Multivitamin preparations as soon as pregnancy is suspected c. Folic acid for all women of childbearing age d. Folic acid during the first and second trimesters of pregnancy

ANS: C The widespread use of folic acid among women of childbearing age has decreased the incidence of spina bifida significantly. Vitamin A is not related to the prevention of spina bifida. Folic acid supplementation is recommended for the preconception period and during the pregnancy. Only 42% of women actually follow these guidelines.

The nurse has documented that a child's level of consciousness is obtunded. Which describes this level of consciousness? a. Slow response to vigorous and repeated stimulation b. Impaired decision making c. Arousable with stimulation d. Confusion regarding time and place

ANS: C Obtunded describes a level of consciousness in which the child is arousable with stimulation. Stupor is a state in which the child remains in a deep sleep, responsive only to vigorous and repeated stimulation. Confusion is impaired decision making. Disorientation is confusion regarding time and place.

6. It is generally recommended that a child with acute streptococcal pharyngitis can return to school: a. When the sore throat is better. c. After taking antibiotics for 24 hours. b. If no complications develop. d. After taking antibiotics for 3 days.

ANS: C After children have taken antibiotics for 24 hours, even if the sore throat persists, they are no longer contagious to other children. Complications may take days to weeks to develop

A 10-year-old boy on a bicycle has been hit by a car in front of the school. The school nurse immediately assesses airway, breathing, and circulation. The next nursing action: should be to a. place on side. b. take blood pressure. c. stabilize neck and spine. d. check scalp and back for bleeding.

ANS: C After determining that the child is breathing and has adequate circulation, the next action is to stabilize the neck and spine to prevent any additional trauma. The child's position should not be changed until the neck and spine are stabilized. Blood pressure is a later assessment. Less urgent, but an important assessment, is inspection of the scalp for bleeding.

13. A child is 2 hours postoperative after a resection of a brain tumor. Which assessment by the nurse takes priority? A. Blood pressure B. Intake and output C. Neurological exam D. Temperature

ANS: C All actions are appropriate for a child postoperatively. However, the answer that is most specific to this child's procedure is the neurological exam.

1. A nursing faculty member explains to the class that which item is the most important for tumor cell growth? A. Age of transforming cells B. Programmed cell death C. Proximity to a capillary D. Rapidity of cell growth

ANS: C All cells, including tumor cells, need a consistent supply of oxygen and nutrients, delivered via the capillaries. Neoplastic cells must be in close enough proximity to a capillary to provide these required elements. The other factors do not have such an important role, if any, in neoplastic growth.

12. An 8-year-old child has been diagnosed with a brain tumor. Based on knowledge of childhood cancers, which intervention does the nurse plan to implement when the child is admitted to the hospital? A. Aspiration precautions B. Protective isolation C. Safety precautions D. Seizure precautions

ANS: C Brain tumors in children 1 to 10 years of age are usually infratentorial and involve the brainstem and cerebellum. Manifestations of brainstem tumors result from involvement of the cranial nerves and include hemiparesis, spastic gait, and frequent stumbling and falling. The nurse implements safety precautions for this child. The other precautions may or may not be needed depending on the child's specific condition, treatment, and side effects of treatment.

23. A child is in the hospital receiving chemotherapy for Hodgkin's lymphoma. What action by a new nurse causes the precepting nurse to intervene? A. Assesses the need for anti-emetics prior to starting chemotherapy B. Checks the IV for blood return before giving the chemotherapy C. Double wraps the chemotherapy bags and places in the trash can D. Performs hand hygiene prior to and after caring for the patient

ANS: C Chemotherapeutic agents are considered hazardous waste and must be disposed of in specific containers, not the trash can. The other actions are appropriate.

11. A nurse is caring for a child who has acute lymphocytic leukemia and has been treated with doxorubicin (Adriamycin). Which assessment finding would the nurse report immediately? A. Loss of appetite B. Low WBC count C. Peripheral edema D. Temperature of 100.6°F (38.1°C), once

ANS: C Doxorubicin and other anthracycline drugs are known to cause heart damage. Peripheral edema may signal heart failure and should be reported right away. Loss of appetite and low WBC count are common findings for a child on chemotherapy. A single temperature of 100.6°F does not need to be reported.

The nurse is teaching a family how to care for their infant in a Pavlik harness to treat developmental dysplasia of the hip. Which should be included? a. Apply lotion or powder to minimize skin irritation. b. Remove harness several times a day to prevent contractures. c. Return to clinic every 1 to 2 weeks. d. Place diaper over harness, preferably using a superabsorbent disposable diaper that is relatively thin.

ANS: C Infants have a rapid growth pattern. The child needs to be assessed by the practitioner every 1 to 2 weeks for possible adjustments. Lotions and powders should not be used with the harness. The harness should not be removed, except as directed by the practitioner. A thin disposable diaper can be placed under the harness.

19. A nurse is looking at photographs of a friend's infant. The nurse notes a whitish glow in the child's eyes, and the friend asks why the baby's eyes look so odd. Which response by the nurse is the most appropriate? A. "If his eyes look like this by 6 months, he needs to see a doctor." B. "Take him to the doctor to see what's wrong with his eyes." C. "This is called leukocoria and may signify retinoblastoma." D. "Your baby may have a brain tumor; take him to the hospital."

ANS: C Leukocoria (also known as the cat's-eye reflex) is a whitish glow in the pupil, often noticed on photographs, and is seen in children with retinoblastoma. The child needs to be seen by his health-care provider. The mother should not wait 6 months. Advising the mother to find out what's wrong with his eyes is not as accurate as explaining the manifestation. This sign is not seen in brain tumors.

36. A child is receiving chemotherapy. The nurse assesses the child's oral cavity and notes the following: raspy voice, thick saliva, and debris on the teeth. Which action by the nurse is the most appropriate? A. Have the child use commercial mouthwash. B. Hold the next dose of chemotherapy. C. Increase the frequency of oral care. D. Place the child on NPO status.

ANS: C Mucositis is a diffuse inflammation of the mouth and oral mucous membranes, and is common during chemotherapy. The nurse should increase the frequency of oral care in the child who is manifesting signs of this problem. Commercial mouthwash contains alcohol, which would burn the tissues. The chemotherapy would not be interrupted. The child should be encouraged to eat and drink as tolerated.

18. A nurse sees the term "proptosis" in a child's medical record. Which physical assessment does the nurse plan to incorporate into the child's exam based on this finding? A. Balance testing B. Hearing screen C. Visual acuity D. Strength testing

ANS: C Proptosis is a downward displacement of the eyeball that can affect visual acuity and is frequently seen in children with rhabdomyosarcoma. The other assessments are not related.

2. Generally what is the earliest age at which puberty begins? a. 13 years in girls, 13 years in boys b. 11 years in girls, 11 years in boys c. 10 years in girls, 12 years in boys d. 12 years in girls, 10 years in boys

ANS: C Puberty signals the beginning of the development of secondary sex characteristics. This begins in girls earlier than in boys. Usually a 2-year difference occurs in the age at onset. Girls and boys do not usually begin puberty at the same age; girls usually begin earlier than boys do.

Which type of traction uses skin traction on the lower leg and a padded sling under the knee? a. Dunlop b. Bryant c. Russell d. Buck extension

ANS: C Russell traction uses skin traction on the lower leg and a padded sling under the knee. The combination of longitudinal and perpendicular traction allows realignment of the lower extremity and immobilizes the hips and knees in a flexed position. Dunlop traction is an upper-extremity traction used for fractures of the humerus. Bryant traction is skin traction with the legs flexed at a 90-degree angle at the hip. Buck extension traction is a type of skin traction with the legs in an extended position. It is used primarily for short-term immobilization, preoperatively with dislocated hips, for correcting contractures, or for bone deformities such as Legg-Calvé-Perthes disease.

20. It is now recommended that children with asthma who are taking long-term inhaled steroids should be assessed frequently because they may develop: a. Cough. c. Slowed growth. b. Osteoporosis. d. Cushings syndrome.

ANS: C The growth of children on long-term inhaled steroids should be assessed frequently to assess for systemic effects of these drugs. Cough is prevented by inhaled steroids. No evidence e

The nurse is caring for a school-age child diagnosed with juvenile idiopathic arthritis (JIA). Which intervention should be a priority? a. Apply ice packs to relieve stiffness and pain. b. Administer acetaminophen to reduce inflammation. c. Teach the child and family correct administration of medications. d. Encourage range-of-motion exercises during periods of inflammation.

ANS: C The management of JIA is primarily pharmacologic. The family should be instructed regarding administration of medications and the value of regular schedule of administration to maintain a satisfactory blood level in the body. They need to know that NSAIDs should not be given on an empty stomach and to be alert for signs of toxicity. Warm moist heat is best for relieving stiffness and pain. Acetaminophen does not have antiinflammatory effects. Range-of-motion exercises should not be done during periods of inflammation.

4. A nurse hears that a new admission to the hospital was recently diagnosed with the most common kind of childhood cancer. Which collaborative care does the nurse prepare to provide to this patient? A. Antibiotic administration B. Bone marrow transplant C. Chemotherapy D. Liver transplant

ANS: C The most common type of childhood cancer is acute lymphocytic leukemia (ALL). First-line treatment for ALL is inducing remission with chemotherapy. Antibiotics are not used unless the child has an infection. Bone marrow transplant may be considered later in the child's course of care. A liver transplant would not be a treatment for ALL.

29. The nurse is admitting a school-age child in acute renal failure with reduced glomerular filtration rate. Which urine test is the most useful clinical indication of glomerular filtration rate? a. pH b. Osmolality c. Creatinine clearance d. Protein level

ANS: C The most useful clinical indication of glomerular filtration is the clearance of creatinine. It is a substance that is freely filtered by the glomerulus and secreted by the renal tubule cells. The pH and osmolality are not estimates of glomerular filtration. Although protein in the urine demonstrates abnormal glomerular permeability, it is not a measure of filtration rate.

29. A child has cancer, is unresponsive, and is doing poorly. Which action by the nursing student causes the faculty to intervene? A. Allows the parents to hold the child B. Places the child on NPO status C. Takes the child's rectal temperature D. Turns the child even if she moans

ANS: C The nurse avoids the rectal route for anything: temperatures, suppositories, and enemas are not allowed, as the rectal mucosa is fragile and prone to injury, which can lead to infection. The other actions are appropriate.

A 5-year-old girl sustained a concussion when she fell out of a tree. In preparation for discharge, the nurse is discussing home care with her mother. Which statement made by the mother indicates a correct understanding of the teaching? a. "I should expect my child to have a few episodes of vomiting." b. "If I notice sleep disturbances, I should contact the physician immediately." c. "I should expect my child to have some behavioral changes after the accident." d. "If I notice diplopia, I will have my child rest for 1 hour."

ANS: C The parents are advised of probable posttraumatic symptoms that may be expected. These include behavioral changes and sleep disturbances. If the child has these clinical signs, they should be immediately reported for evaluation. Sleep disturbances are to be expected.

A nurse is conducting discharge teaching for parents of an infant with osteogenesis imperfecta (OI). Further teaching is indicated if the parents make which statement? a. "We will be very careful handling the baby." b. "We will lift the baby by the buttocks when diapering." c. "We're glad there is a cure for this disorder." d. "We will schedule follow-up appointments as instructed."

ANS: C The treatment for OI is primarily supportive. Although patients and families are optimistic about new research advances, there is no cure. The use of bisphosphonate therapy with IV pamidronate to promote increased bone density and prevent fractures has become standard therapy for many children with OI; however, long bones are weakened by prolonged treatment. Infants and children with this disorder require careful handling to prevent fractures. They must be supported when they are being turned, positioned, moved, and held. Even changing a diaper may cause a fracture in severely affected infants. These children should never be held by the ankles when being diapered but should be gently lifted by the buttocks or supported with pillows. Follow-up appointments for treatment with bisphosphonate can be expected.

The nurse is caring for a 12-year-old child with a left leg below the knee amputation (BKA). The child had the surgery 1 week ago. Which intervention should the nurse plan to implement for this child? a. Elevate the left stump on a pillow. b. Place ice pack on the stump. c. Encourage the child to use an overhead bed trapeze when repositioning. d. Replace the ace wrap covering the stump with a gauze dressing.

ANS: C Use of the overhead bed trapeze should be encouraged to begin to build up the arm muscles necessary for walking with crutches. Stump elevation may be used during the first 24 hours, but after this time, the extremity should not be left in this position because contractures in the proximal joint will develop and seriously hamper ambulation. Ice would not be an appropriate intervention and would decrease circulation to the stump. Stump shaping is done postoperatively with special elastic bandaging using a figure-eight bandage, which applies pressure in a cone-shaped fashion. This technique decreases stump edema, controls hemorrhage, and aids in developing desired contours so the child will bear weight on the posterior aspect of the skin flap rather than on the end of the stump. This wrap should not be replaced with a gauze dressing.

3. What is the priority nursing goal for a 14 year old diagnosed with Graves' disease? a. Relieving constipation b. Allowing the adolescent to make decisions about whether or not to take medication c. Verbalizing the importance of monitoring for medication side effects d. Developing alternative educational goals

ANS: C Children being treated with propylthiouracil or methimazole must be carefully monitored for side effects of the drug. Because sore throat and fever accompany the grave complication of leukopenia, these children should be seen by a health care practitioner if such symptoms occur. Parents and children should be taught to recognize and report symptoms immediately. The adolescent with Graves' disease is not likely to be constipated. Adherence to the medication schedule is important to ensure optimal health and wellness. Medications should not be skipped and dose regimens should not be tapered by the child without consultation with the child's medical provider. The management of Graves' disease does not interfere with school attendance and does not require alternative educational plans.

9. Constipation has recently become a problem for a school-age child who is being treated for seasonal allergies. The nurse should focus the assessment on what possibly related factor? a. Diet b. Allergies c. Antihistamines d. Emotional factors

ANS: C Constipation may be associated with drugs such as antihistamines, antacids, diuretics, opioids, antiepileptics, and iron. Because this is the only known recent change in her habits, the addition of antihistamines is most likely the etiology of the diarrhea, rather than diet, allergies, or emotional factors. With a change in bowel habits, the presence and role of any recently prescribed medications should be assessed.

17. What is used to treat moderate-to-severe inflammatory bowel disease? a. Antacids b. Antibiotics c. Corticosteroids d. Antidiarrheal medications

ANS: C Corticosteroids such as prednisone and prednisolone are used in short bursts to suppress the inflammatory response in inflammatory bowel disease. Antacids and antidiarrheals are not drugs of choice to treat the inflammatory process of inflammatory bowel disease. Antibiotics may be used as adjunctive therapy to treat complications.

30. The nurse is admitting a toddler with the diagnosis of juvenile hypothyroidism. Which is a common clinical manifestation of this disorder? a. Insomnia b. Diarrhea c. Dry skin d. Accelerated growth

ANS: C Dry skin, mental decline, and myxedematous skin changes are associated with juvenile hypothyroidism. Children with hypothyroidism are usually sleepy. Constipation is associated with hypothyroidism. Decelerated growth is common in juvenile hypothyroidism.

8. What is a common clinical manifestation of juvenile hypothyroidism? a. Insomnia b. Diarrhea c. Dry skin d. Accelerated growth

ANS: C Dry skin, mental decline, and myxedematous skin changes are associated with juvenile hypothyroidism. Children with hypothyroidism are usually sleepy. Constipation is associated with hypothyroidism. Decelerated growth is common in juvenile hypothyroidism.

8. A school-age child diagnosed with leukemia experienced severe nausea and vomiting when receiving chemotherapy for the first time. Which is the most appropriate nursing action to prevent or minimize these reactions with subsequent treatments? a. Encourage drinking large amounts of favorite fluids. b. Encourage child to take nothing by mouth (remain NPO) until nausea and vomiting subside. c. Administer an antiemetic at least 30 minutes before chemotherapy begins. d. Administer an antiemetic as soon as child has nausea.

ANS: C The most beneficial regimen to minimize nausea and vomiting associated with chemotherapy is to administer the antiemetic 30 minutes to an hour before the chemotherapy is begun. The goal is to prevent anticipatory symptoms. Drinking fluids will add to the discomfort of the nausea and vomiting. Waiting until nausea and vomiting subside will help with this episode, but the child will have the discomfort and be at risk for dehydration. Administering an antiemetic as soon as the child has nausea does not prevent anticipatory nausea.

13. Glucocorticoids, mineralocorticoids, and sex steroids are secreted by which organ? a. Thyroid gland b. Parathyroid glands c. Adrenal cortex d. Anterior pituitary

ANS: C These hormones are secreted by the adrenal cortex. The thyroid gland produces thyroid hormone and thyrocalcitonin. The parathyroid glands produce parathyroid hormone. The anterior pituitary produces hormones such as growth hormone, thyroid-stimulating hormone, adrenocorticotropic hormone, gonadotropin, prolactin, and melanocyte-stimulating hormone.

23. The nurse is caring for an infant with suspected pyloric stenosis. Which clinical manifestation would indicate pyloric stenosis? a. Abdominal rigidity and pain on palpation b. Rounded abdomen and hypoactive bowel sounds c. Visible peristalsis and weight loss d. Distention of lower abdomen and constipation

ANS: C Visible gastric peristaltic waves that move from left to right across the epigastrium are observed in pyloric stenosis, as is weight loss. Abdominal rigidity and pain on palpation, and rounded abdomen and hypoactive bowel sounds, are usually not present. The upper abdomen is distended, not the lower abdomen.

12. A child diagnosed with hypoparathyroidism is receiving vitamin D therapy. The parents should be advised to watch for which sign of vitamin D toxicity? a. Headache and seizures b. Physical restlessness and voracious appetite without weight gain c. Weakness and lassitude d. Anorexia and insomnia1

ANS: C Vitamin D toxicity can be a serious consequence of therapy. Parents are advised to watch for signs including weakness, fatigue, lassitude, headache, nausea, vomiting, and diarrhea. Renal impairment is manifested through polyuria, polydipsia, and nocturia. Headaches may be a sign of vitamin D toxicity, but seizures are not. Physical restlessness and a voracious appetite with weight loss are manifestations of hyperthyroidism. Anorexia and insomnia are not characteristic of vitamin D toxicity.

28. Which description of a stool is characteristic of intussusception? a. Ribbon-like stools b. Hard stools positive for guaiac c. "Currant jelly" stools d. Loose, foul-smelling stools

ANS: C With intussusception, passage of bloody mucus-coated stools occurs. Pressure on the bowel from obstruction leads to passage of "currant jelly" stools. Ribbon-like stools are characteristic of Hirschsprung's disease. Stools will not be hard. Loose, foul-smelling stools may indicate infectious gastroenteritis.

The nurse is caring for a neonate with suspected meningitis. Which clinical manifestations should the nurse prepare to assess if meningitis is confirmed? (Select all that apply.) a. Headache b. Photophobia c. Bulging anterior fontanel d. Weak cry e. Poor muscle tone

ANS: C, D, E Assessment findings in a neonate with meningitis include bulging anterior fontanel, weak cry, and poor muscle tone. Headache and photophobia are signs seen in an older child.

The nurse is caring for a preschool child with a cast applied recently for a fractured tibia. Which assessment findings indicate possible compartment syndrome? (Select all that apply.) a. Palpable distal pulse b. Capillary refill to extremity less than 3 seconds c. Severe pain not relieved by analgesics d. Tingling of extremity e. Inability to move extremity

ANS: C, D, E Indications of compartment syndrome are severe pain not relieved by analgesics, tingling of extremity, and inability to move extremity. A palpable distal pulse and capillary refill to the extremity less than 3 seconds are expected findings.

6. The nurse is caring for a school-age child with hyperthyroidism (Graves' disease). Which clinical manifestations should the nurse monitor that may indicate a thyroid storm? (Select all that apply.) a. Constipation b. Hypotension c. Hyperthermia d. Tachycardia e. Vomiting

ANS: C, D, E A child with a thyroid storm will have severe irritability and restlessness, vomiting, diarrhea, hyperthermia, hypertension, severe tachycardia, and prostration.

The nurse is preparing a child for possible alopecia from chemotherapy. Which should be included? a. Explain to child that hair usually regrows in 1 year. b. Advise child to expose head to sunlight to minimize alopecia. c. Explain to child that wearing a hat or scarf is preferable to wearing a wig. d. Explain to child that when hair regrows, it may have a slightly different color or texture.

ANS: D Alopecia is a side effect of certain chemotherapeutic agents. When the hair regrows, it may be a different color or texture. The hair usually grows back within 3 to 6 months after cessation of treatment. The head should be protected from sunlight to avoid sunburn. Children should choose the head covering they prefer.

A child with leukemia is receiving triple intrathecal chemotherapy consisting of methotrexate, cytarabine, and hydrocortisone. The purpose of this is to prevent: a. infection. b. brain tumor. c. drug side effects. d. central nervous system (CNS) disease.

ANS: D For certain children, CNS prophylactic therapy is indicated. This drug regimen is used to prevent CNS leukemia and will not prevent infection or drug side effects. If the child has a brain tumor in addition to leukemia, additional therapy would be indicated.

The nurse is conducting a staff in-service on childhood blood disorders. Which describes the pathology of idiopathic thrombocytopenic purpura? a. Bone marrow failure in which all elements are suppressed b. Deficiency in the production rate of globin chains c. Diffuse fibrin deposition in the microvasculature d. An excessive destruction of platelets

ANS: D Idiopathic thrombocytopenic purpura is an acquired hemorrhagic disorder characterized by an excessive destruction of platelets, discolorations caused by petechiae beneath the skin, and a normal bone marrow. Aplastic anemia refers to a bone marrowfailure condition in which the formed elements of the blood are simultaneously depressed. Thalassemia major is a group of blood disorders characterized by deficiency in the production rate of specific hemoglobin globin chains. Disseminated intravascular coagulation is characterized by diffuse fibrin deposition in the microvasculature, consumption of coagulation factors, and endogenous generation of thrombin and plasma.

Myelosuppression, associated with chemotherapeutic agents or some malignancies such as leukemia, can cause bleeding tendencies because of a(n): a. decrease in leukocytes. b. increase in lymphocytes. c. vitamin C deficiency. d. decrease in blood platelets.

ANS: D The decrease in blood platelets secondary to the myelosuppression of chemotherapy can cause an increase in bleeding. The child and family should be alerted to avoid risk of injury. Decrease in leukocytes, increase in lymphocytes, and vitamin C deficiency would not affect bleeding tendencies.

The nurse is caring for an intubated infant with botulism in the pediatric intensive care unit. Which health care provider prescriptions should the nurse clarify with the health care provider before implementing? a. Administer 250 mg botulism immune globulin intravenously (BIG-IV) one time. b. Provide total parenteral nutrition (TPN) at 25 ml/hr intravenously. c. Titrate oxygen to keep pulse oximetry saturations greater than 92. d. Administer gentamicin sulfate (Garamycin) 10 mg per intravenous piggyback every 12 hours.

ANS: D The nurse should clarify the administration of an aminoglycoside antibiotic. Antibiotic therapy is not part of the management of infant botulism because the botulinum toxin is an intracellular molecule, and antibiotics would not be effective; aminoglycosides in particular should not be administered because they may potentiate the blocking effects of the neurotoxin. Treatment consists of immediate administration of botulism immune globulin intravenously (BIG-IV) without delaying for laboratory diagnosis. Early administration of BIG-IV neutralizes the toxin and stops the progression of the disease. The human-derived botulism antitoxin (BIG-IV) has been evaluated and is now available nationwide for use only in infant botulism. Approximately 50% of affected infants require intubation and mechanical ventilation; therefore, respiratory support is crucial, as is nutritional support because these infants are unable to feed.

An 8-year-old girl is receiving a blood transfusion when the nurse notes that she has developed precordial pain, dyspnea, distended neck veins, slight cyanosis, and a dry cough. These manifestations are most suggestive of: a. air emboli. b. allergic reaction. c. hemolytic reaction. d. circulatory overload.

ANS: D The signs of circulatory overload include distended neck veins, hypertension, crackles, dry cough, cyanosis, and precordial pain. Signs of air embolism are sudden difficulty breathing, sharp pain in the chest, and apprehension. Allergic reactions are manifested by urticaria, pruritus, flushing, asthmatic wheezing, and laryngeal edema. Hemolytic reactions are characterized by chills, shaking, fever, pain at infusion site, nausea, vomiting, tightness in chest, flank pain, red or black urine, and progressive signs of shock and renal failure.

4. When caring for an infant with an upper respiratory tract infection and elevated temperature, an appropriate nursing intervention is to: a. Give tepid water baths to reduce fever. b. Encourage food intake to maintain caloric needs. c. Have child wear heavy clothing to prevent chilling. d. Give small amounts of favorite fluids frequently to prevent dehydration.

ANS: D Preventing dehydration by small frequent feedings is an important intervention in the febrile child. Tepid water baths may induce shivering, which raises temperature. Food should not be forced; it may result in the child vomiting. The febrile child should be dressed in light, loose clothing.

19. Which comment is most developmentally typical of a 7-year-old boy? a. "I am a Power Ranger, so don't make me angry." b. "I don't know whether I like Mary or Joan better." c. "My mom is my favorite person in the world." d. "Jimmy is my best friend."

ANS: D School-age children form friendships with peers of the same sex, those who live nearby, and other children who have toys that they enjoy sharing. Magical thinking is developmentally appropriate for the preschooler. Opposite-sex friendships are not typical for the 7-year-old child. Seven-year-old children socialize with their peers, not their parents.

38. A child has nausea after chemotherapy despite anti-emetics. However, the child complains that "my tummy is growling." Which other action should the nurse take to promote comfort for this child? A. Avoid hard, difficult-to-chew foods. B. Encourage a high fluid intake with meals. C. Offer the child hard candy to suck on. D. Provide bland items, such as plain mashed potatoes.

ANS: D Several actions can help the child with nausea: offering plain, bland foods; avoiding spicy, heavy, or fatty foods; decreasing the odor associated with foods if that bothers the child; and having the child take food separately from liquids. Liquid together with food can make the child feel full, inducing nausea. The other options are good choices for other nutritional problems.

2. A child has had cold symptoms for more than 2 weeks, a headache, nasal congestion with purulent nasal drainage, facial tenderness, and a cough that increases during sleep. The nurse recognizes that these symptoms are characteristic of which respiratory condition? a. Allergic rhinitis c. Asthma b. Bronchitis d. Sinusitis

ANS: D Sinusitis is characterized by signs and symptoms of a cold that do not improve after 14 days, a low-grade fever, nasal congestion and purulent nasal discharge, headache, tenderness, a feeling of fullness over the affected sinuses, halitosis, and a cough that increases when the child is lying down. The classic symptoms of allergic rhinitis are watery rhinorrhea; itchy nose, eyes, ears, and palate; and sneezing. Symptoms occur as long as the child is exposed to the allergen. Bronchitis is characterized by a gradual onset of rhinitis and a cough that is initially nonproductive but may change to a loose cough. The manifestations of asthma may vary, with wheezing being a classic sign. The symptoms presented in the question do not suggest asthma.

The nurse is performing a Glasgow Coma Scale on a school-age child with a head injury. The child opens eyes spontaneously, obeys commands, and is oriented to person, time, and place. Which is the score the nurse should record? a. 8 b. 11 c. 13 d. 15

ANS: D The Glasgow Coma Scale (GCS) consists of a three-part assessment: eye opening, verbal response, and motor response. Numeric values of 1 through 5 are assigned to the levels of response in each category. The sum of these numeric values provides an objective measure of the patient's level of consciousness (LOC). A person with an unaltered LOC would score the highest, 15. The child who opens eyes spontaneously, obeys commands, and is oriented is scored at a 15.

36. Why do infants and young children quickly have respiratory distress in acute and chronic alterations of the respiratory system? a. They have a widened, shorter airway. b. There is a defect in their sucking ability. c. The gag reflex increases mucus production. d. Mucus and edema obstruct small airways.

ANS: D The airway in infants and young children is narrower, not wider, and respiratory distress can occur quickly because mucus and edema can cause obstruction to their small airways. Sucking is not necessarily related to problems with the airway. The gag reflex is necessary to prevent aspiration. It does not produce mucus.

27. The parents of a child with cancer ask the nurse why the child is losing weight even though he is eating what he normally does. Which response by the nurse is the most appropriate? A. "Cancer consumes body tissues, causing weight loss." B. "He may be going through a growth spurt right now." C. "How do you know he is eating like he normally does?" D. "When you are sick, you need more nutrition than usual."

ANS: D The demands of illness lead to increased nutritional needs. A child with cancer needs increased nutrition. Cancer does not consume body tissues. The child may be going through a growth spurt, but this is not always the case and is not the best answer. Asking the parents how they know the child's eating habits have changed may put them on the defensive.

A 4-year-old child is newly diagnosed with Legg-Calvé-Perthes disease. Nursing considerations should include which action? a. Encouraging normal activity for as long as is possible b. Explaining the cause of the disease to the child and family c. Preparing the child and family for long-term, permanent disabilities d. Teaching the family the care and management of the corrective appliance

ANS: D The family needs to learn the purpose, function, application, and care of the corrective device and the importance of compliance to achieve the desired outcome. The initial therapy is rest and non-weight bearing, which helps reduce inflammation and restore motion. Legg-Calvé-Perthes is a disease with an unknown etiology. A disturbance of circulation to the femoral capital epiphysis produces an ischemic aseptic necrosis of the femoral head. The disease is self-limiting, but the ultimate outcome of therapy depends on early and efficient therapy and the child's age at onset.

A (playing peek-a-boo Because object permanence is a new achievement, peek-a-boo is an excellent activity to practice this new skill for visual stimulation. Playing pat-a-cake and showing how to clap hands will help with kinesthetic stimulation. Imitating animal sounds will help with auditory stimulation.)

An appropriate play activity for a 7-month-old infant to encourage visual stimulation is: a. Playing peek-a-boo. b. Playing pat-a-cake. c. Imitating animal sounds d. Showing how to clap hands.

33. A preschool child is being admitted to the hospital with dehydration and a urinary tract infection (UTI). Which urinalysis result should the nurse expect with these conditions? a. WBC <1; specific gravity 1.008 b. WBC <2; specific gravity 1.025 c. WBC >2; specific gravity 1.016 d. WBC >2; specific gravity 1.030

ANS: D The white blood cell count (WBC) in a routine urinalysis should be <1 or 2. Over that amount indicates a urinary tract inflammatory process. The urinalysis specific gravity for children with normal fluid intake is 1.016 to 1.022. When the specific gravity is high, dehydration is indicated. A low specific gravity is seen with excessive fluid intake, distal tubular dysfunction, or insufficient antidiuretic hormone secretion.

28. A nurse works on the pediatric oncology floor. After receiving the handoff report, which child does the nurse assess first? A. Child on protective isolation B. 4 hours post-bone marrow biopsy C. Not eating an hour after chemotherapy D. Temperature of 101.5°F (38.5°C)

ANS: D This fever indicates a probable infection. The nurse will see this child first and provide report to the physician, if this has not already been done. This child is the sickest and should be seen first; one might be tempted to see the child in protective isolation first to avoid cross-contamination, but by following isolation precautions, this risk is minimized. Not eating after chemotherapy is not cause for concern, and the child 4 hours post-bone marrow biopsy should be stable.

13. The father of 12 year old tells the nurse that he is concerned about his child getting "fat." The child's body mass index for age is at the 60th percentile. What is the most appropriate nursing action to address the father's concern? a. Reassure the father that his child is not "fat." b. Reassure the father that the weight is just a growing child. c. Suggest a low-calorie, low-fat diet, and provide food suggestions. d. Explain that this is typical of the growth pattern of children at this age.

ANS: D This is a characteristic pattern of growth in preadolescent boys, in which the growth in height has slowed in preparation for the pubertal growth spurt but weight is still gained. This should be reviewed with both the father and child, and a plan should be developed to maintain physical exercise and a balanced diet. Saying that the child is not "fat" is false reassurance. The child's weight is high for his/her height. The child needs to maintain his/her physical activity. The father is concerned; an explanation is required. A nutritional diet with physical activity should be sufficient to maintain his balance.

A child is unconscious after a motor vehicle accident. The watery discharge from the nose tests positive for glucose. The nurse should recognize that this suggests: a. diabetic coma. b. brainstem injury. c. upper respiratory tract infection. d. leaking of cerebrospinal fluid (CSF).

ANS: D Watery discharge from the nose that is positive for glucose suggests leaking of CSF from a skull fracture and is not associated with diabetes or respiratory tract infection. The fluid is probably CSF from a skull fracture and does not signify whether the brainstem is involved.

The nurse uses the palms of the hands when handling a wet cast for which reason? a. To assess dryness of the cast b. To facilitate easy turning c. To keep the patient's limb balanced d. To avoid indenting the cast

ANS: D Wet casts should be handled by the palms of the hands, not the fingers, to avoid creating pressure points. Assessing dryness, facilitating easy turning, and keeping the patient's limb balanced are not reasons for using the palms of the hand rather than the fingers when handling a wet cast.

22. Which type of hernia has an impaired blood supply to the herniated organ? a. Hiatal hernia b. Incarcerated hernia c. Omphalocele d. Strangulated hernia

ANS: D A strangulated hernia is one in which the blood supply to the herniated organ is impaired. A hiatal hernia is the intrusion of an abdominal structure, usually the stomach, through the esophageal hiatus. An incarcerated hernia is a hernia that cannot be reduced easily. Omphalocele is the protrusion of intraabdominal viscera into the base of the umbilical cord. The sac is covered with peritoneum and not skin.

10. The nurse is preparing a child for possible alopecia from chemotherapy. Which intervention should be included? a. Explain to child that hair usually regrows in 1 year. b. Advise child to expose head to sunlight to minimize alopecia. c. Explain to child that wearing a hat or scarf is preferable to wearing a wig. d. Explain to child that when hair regrows, it may have a slightly different color or texture.

ANS: D Alopecia is a side effect of certain chemotherapeutic agents. When the hair regrows, it may be a different color or texture. The hair usually grows back within 3 to 6 months after cessation of treatment. The head should be protected from sunlight to avoid sunburn. Children should choose the head covering they prefer.

11. Which is a common clinical manifestation of Hodgkin disease? a. Petechiae b. Bone and joint pain c. Painful, enlarged lymph nodes d. Enlarged, firm, nontender lymph nodes

ANS: D Asymptomatic, enlarged, cervical or supraclavicular lymphadenopathy is the most common presentation of Hodgkin disease. Petechiae are usually associated with leukemia. Bone and joint pain are not likely in Hodgkin disease. The enlarged nodes are rarely painful.

31. Which clinical manifestation may occur in the child who is prescribed methimazole for the treatment of hyperthyroidism (Graves' disease)? a. Seizures b. Enlargement of all lymph glands c. Pancreatitis or cholecystitis d. Sore throat or fever

ANS: D Children being treated with propylthiouracil or methimazole must be carefully monitored for side effects of the drug. Because sore throat and fever accompany the grave complication of leukopenia, these children should be seen by a health care practitioner if such symptoms occur. Neither seizures, cholecystitis nor pancreatitis are associated with the administration of methimazole. Enlargement of the salivary and cervical lymph glands may occur.

C (During the tertiary circular reactions stage, children have only a rudimentary sense of the classification of objects. The appearance of an object denotes its function for these children. The slot of an outlet is for putting things into. Her cognitive development is appropriate for her age and represents typical behavior for a toddler. Only some awareness exists of a causal relation between events.)

Although a 14-month-old girl received a shock from an electrical outlet recently, her parents find her about to place a paper clip in another outlet. The best interpretation of this behavior is that: a. Her cognitive development is delayed. b. This is typical behavior because toddlers are not very developed. c. This is typical behavior because of inability to transfer knowledge to new situations. d. This is not typical behavior because toddlers should know better than to repeat an act that caused pain.

B (2 months At age 2 months, the infant has a social, responsive smile. A reflex smile is usually present at age 1 month. The 3-month-old can recognize familiar faces. At age 4 months, the infant can enjoy social interactions.)

At what age should the nurse expect an infant to begin smiling in response to pleasurable stimuli? a. 1 month b. 2 months c. 3 months d. 4 months

C (10 months)

At what age would the nurse advise parents to expect their infant to be able to say "mama" and "dada" with meaning? A. 4 months B. 6 months C. 10 months D. 14 months

D (Earlier-than-normal tooth eruption This is earlier than expected. Most infants at age 6 months have two teeth. Six teeth at 6 months is not delayed; it is early tooth eruption. Although unusual, it is not dangerous.)

Austin, age 6 months, has six teeth. The nurse should recognize that this is: a. Normal tooth eruption. b. Delayed tooth eruption. c. Unusual and dangerous d. Earlier-than-normal tooth eruption.

C (By age 12 months, most children are eating the same food that is prepared for the rest of the family. Using a spoon usually is not mastered until age 18 months. The parents should be engaged in weaning a child from a bottle if that is the source of liquid. Toddlers should be encouraged to drink from a cup at the first birthday and weaned totally by 14 months. The child should be weaned from a milk/formula-based diet to a balanced diet that includes iron-rich sources of food.)

Developmentally, most children at age 12 months: a. Use a spoon adeptly. b. Relinquish the bottle voluntarily. c. Eat the same food as the rest of the family. d. Reject all solid food in preference to the bottle.

A, C, E (Toddlers younger than 2 years should be secured in a rear-facing, upright, approved car safety seat. After the age of 2 years, a forward-facing car seat can be used. Harness straps should be adjusted to provide a snug fit. The car safety seat should be placed in the middle of the rear seat. Children younger than 13 years should not ride in a front passenger seat that is equipped with an air bag.)

Motor vehicle injuries are a significant threat to young children. Knowing this, the nurse plans a teaching session with a toddler's parents on car safety. Which will she teach (select all that apply)? a. Secure in a rear-facing, upright, car safety seat. b. Place the car safety seat in the rear seat, behind the driver's seat. c. Harness safety straps should be fit snugly. d. Place the car safety seat in the front passenger seat equipped with an air bag. e. After the age of 2 years, toddlers can be placed in a forward-facing car seat.

B (Rear facing in back seat The rear-facing car seat provides the best protection for an infant's disproportionately heavy head and weak neck. Infants should face the rear from birth to 20 pounds and as close to 1 year of age as possible. The middle of the back seat provides the safest position. Severe injuries and deaths in children have occurred from air bags deploying on impact in the front passenger seat.)

The clinic is lending a federally approved car seat to an infant's family. The nurse should explain that the safest place to put the car seat is: a. Front facing in back seat. b. Rear facing in back seat. c. Front facing in front seat if an air bag is on the passenger side. d. Rear facing in front seat if an air bag is on the passenger side.

B (For young children, the most effective cleaning of teeth is done by the parents. Different positions can be used if the child's back is to the adult. The adult should use one hand to stabilize the chin and the other to brush the child's teeth. The child can participate in brushing, but for a thorough cleaning adult intervention is necessary.)

The most effective way to clean a toddler's teeth is for the: a. Child to brush regularly with toothpaste of his or her choice. b. Parent to stabilize the chin with one hand and brush with the other. c. Parent to brush the mandibular occlusive surfaces, leaving the rest for the child. d. Parent to brush the front labial surfaces, leaving the rest for the child.

A (fluids in addition to breast milk are not needed.)

The mother of a 3-month-old breastfed infant asks about giving her baby water since it is summer and very warm. The nurse should recommend that: A. fluids in addition to breast milk are not needed. B. water should be given if the infant seems to nurse longer than usual. C. water once or twice a day will make up for losses caused by environmental temperature. D. clear juices would be better than water to promote adequate fluid intake.

A (trust.)

The nurse educator instructs a nursing student that according to Erikson, infancy is concerned with acquiring a sense of: A. trust. B. industry. C. initiative. D. separation.

D (Able to grasp object voluntarily.)

The nurse expects which characteristic of fine motor skills in a 5-month-old infant? A. Strong grasp reflex B. Neat pincer grasp C. Able to build a tower of two cubes D. Able to grasp object voluntarily

C (Those using yogurt as primary source of milk.)

The nurse in the pediatric clinic identifies which infants at risk for developing vitamin D-deficient rickets? A. Lacto-ovo vegetarians B. Those who are breastfed exclusively C. Those using yogurt as primary source of milk D. Those exposed to daily sunlight

B (15 pounds Birth weight doubles at about age 5 to 6 months. At 6 months, an infant who weighed 7 pounds at birth would weigh approximately 15 pounds. Ten pounds is too little; the infant would have gone from the 50th percentile at birth to below the 5th percentile. Twenty pounds or more is too much; the infant would have tripled the birth weight at 6 months.)

The nurse is assessing a 6-month-old healthy infant who weighed 7 pounds at birth. The nurse should expect the infant to now weigh approximately: a. 10 pounds. b. 15 pounds. c. 20 pounds d. 25 pounds.

A, C, D, E (A. "I only smoke in the kitchen." C. "I have my baby sleep with me instead of alone in the crib." D. "I make sure my baby wears a flannel sleeper and has two blankets to keep warm in her crib." E. "I always leave my baby's favorite stuffed bunny rabbit in the crib to keep her from crying at night.")

The nurse should provide further teaching about sudden infant death syndrome (SIDS) prevention when hearing the mother of an 8-week-old make which statement? (Select all that apply.) A. "I only smoke in the kitchen." B. "I put my baby to sleep on her back." C. "I have my baby sleep with me instead of alone in the crib." D. "I make sure my baby wears a flannel sleeper and has two blankets to keep warm in her crib." E. "I always leave my baby's favorite stuffed bunny rabbit in the crib to keep her from crying at night."

C (12 months The American Academy of Pediatrics does not recommend the use of cow's milk for children younger than 12 months. At 6 and 9 months, the infant should be receiving commercial infant formula or breast milk. At age 18 months, milk and formula are supplemented with solid foods, water, and some fruit juices.)

The nurse should teach parents that which age is safe to give infants whole milk instead of commercial infant formula? a. 6 months b. 9 months c. 12 months d. 18 months

A, B, C, D, E (A. allow for catch-up growth. B. correct nutritional deficiencies. C. achieve ideal weight for height. D. restore optimum body composition. E. educate the parents or primary caregivers on child's nutritional requirements.)

The primary goals in the nutritional management of children with failure to thrive (FTT) are: (Select all that apply.) A. allow for catch-up growth. B. correct nutritional deficiencies. C. achieve ideal weight for height. D. restore optimum body composition. E. educate the parents or primary caregivers on child's nutritional requirements. F. educate the parents or primary caregivers that the child will need tube feedings first.

B, C (Usually by age 30 months, the primary dentition of 20 teeth is completed, and the child has sphincter control in preparation for bowel and bladder control. A doubling of birth weight, opening of the anterior fontanel, and doubling of length are not characteristic of the physical development of a 30-month-old child.)

Which are characteristic of the physical development of a 30-month-old child (select all that apply)? a. Birth weight has doubled. b. Primary dentition is complete. c. Sphincter control is achieved. d. Anterior fontanel is open. e. Length from birth is doubled.

D (The 24-month-old child can go up and down stairs alone with two feet on each step. Skipping and the ability to broad jump are skills acquired at age 3. Tricycle riding is achieved at age 4.)

Which characteristic best describes the gross motor skills of a 24-month-old child? a. Skips b. Rides tricycle c. Broad jumps d. Walks up and down stairs

D (Gently stimulate trunk by patting or rubbing. If the infant is apneic, the infant's trunk should be gently stimulated by patting or rubbing. If the infant is prone, turn onto the back. The infant should not be shaken vigorously, have the head rolled side to side, or be held by the feet upside down with the head supported. These actions can cause injury.)

Which is the most appropriate action when an infant becomes apneic? a. Shake vigorously. b. Roll head side to side. c. Hold by feet upside down with head supported. d. Gently stimulate trunk by patting or rubbing.

Which statement by a parent about a child's conjunctivitis indicates that further teaching is needed? a. "I'll have separate towels and washcloths for each family member." b. "I'll notify my doctor if the eye gets redder or the drainage increases." c. "When the eye drainage improves, we'll stop giving the antibiotic ointment." d. "After taking the antibiotic for 24 hours, my child can return to school."

c. "When the eye drainage improves, we'll stop giving the antibiotic ointment."

The most common clinical manifestation of retinoblastoma is: a. Glaucoma. c. White eye reflex. b. Amblyopia. d. Sunken eye socket.

c. White eye reflex.

How should the nurse respond to a 53-year-old woman who appears anxious and distressed who wants to have children and approaches the nurse for counseling? 1 Refer her to local adoption agencies. 2 Advise the woman to attend prenatal classes. 3 Explain the consequences of having children later in life. 4 Explain the process of having children through surrogate mother.

1 A 53-year-old woman is likely to have attained menopause and may be able to have children only through adoption. Therefore, the nurse should refer the woman to a local adoption agency. Prenatal classes are recommended for women who are of childbearing age. Explaining the woman's inability to have children after menopause might not solve the problem. The woman likely will have attained menopause and would be unable to produce ovum, so surrogacy is not possible. Test-Taking Tip: Look for options that are similar in nature. If all are correct, either the question is poor or all options are incorrect; the latter of which is more likely. Example: If the answer you are seeking is directed to a specific treatment and all but one option deal with signs and symptoms, you would be correct in choosing the treatment-specific option.

A preschool child watches a nurse pour medication from a tall, thin glass to a short, wide glass. Which statement is appropriate developmentally for this age group? 1 The amount of medicine is less. 2 The amount of medicine did not change, only its appearance. 3 Pouring medicine makes the medicine hot. 4 The glass changed shape to accommodate the medicine

1 A preschool child does not have the ability to understand the concept of conservation . This concept is not developed until school age. Understanding conservation occurs between 7 to 10 years of age, when a child begins to realize that physical factors, such as volume, weight, and number, remain the same even though outward appearances are changed. Children are able to deal with a number of different aspects of a situation simultaneously. This is not an expected response by a child. A preschool child will not typically believe the glass changed shape to accommodate the medicine but rather that the amount of medicine is less in the short, wide glass.

A mother tells the nurse that she will visit her 2-year-old son tomorrow about noon. During the child's bath, he asks for mommy. The nurse's best reply is: 1 "Mommy will be here after lunch." 2 "Mommy always comes back to see you." 3 "Your mommy told me yesterday that she would be here today about noon." 4 "Mommy had to go home for a while, but she will be here today."

1 Because toddlers have a limited concept of time, the nurse should translate the mother's statement about being back around noon by linking the arrival time to a familiar activity that takes place at that time. Saying that the child's mother will always return does not give the child any information about when his mother will visit. Twelve noon is a meaningless concept for a toddler. Saying generally that the child's mother will visit does not give the child specific information about when his mother will visit.

The nurse is teaching a pregnant woman to eat a nutritious diet and to attend regular antenatal health check-ups for the assessment of fetal well-being. The primary purpose of this nursing intervention is to reduce the neonatal mortality rate due to: 1 birth weight less than 2.5g. 2 gestational diabetes in mother. 3 birth weight of more than 3.5 g. 4 febrile convulsions in neonate

1 Birth weight of less than 2,500 grams or 5.5 pounds in considered low birth weight (LBW). LBW is associated with higher neonatal mortality rate in the United States when compared with other countries. The lower the birth weight, the higher the mortality rate. Birth weight of the neonates born to uncontrolled diabetic mother can be high. The mortality rate of neonates born with a birth weight of more than 3.5 g is lower than that of neonates born with LBW. Febrile convulsions seldom cause death in neonates. Test-Taking Tip: Key words or phrases in the stem of the question such as first, primary, early, or best are important. Similarly, words such as only, always, never, and all in the alternatives are frequently evidence of a wrong response. As in life, no real absolutes exist in nursing; however, every rule has its exceptions, SO answer with care.

During their school-age years, children best understand concepts that can be seen or illustrated. The nurse knows this type of thinking is termed as: 1 concrete operations. 2 preoperational. 3 school-age rhetoric. 4 formal operations.

1 Black-and-white reasoning involves a situation in which only two alternatives are considered, when in fact there are additional options. Preoperational thinking is concrete and tangible. During the school-age years, children deal with thoughts and learn through observation. They do not have the ability to do abstract reasoning and learn best with illustration. Thought at this time is dominated by what the school-age child can see, hear, or otherwise experience. School-age rhetoric simply refers to the type of ideas that arise out of the years children attend school. Formal operations are characterized by the adaptability and flexibility that occurs during the adolescent years. Test-Taking Tip: Answer every question because, on the NCLEX exam, you must answer a question before you can move on to the next question.

A child has recently been admitted to the hospital. The child's parents have not yet arrived at the hospital. What behavior is the child exhibiting that leads the nurse to believe the child is exhibiting the stage of protest? The child: 1 Screams and hits the nurse. 2 Is withdrawn from others. 3 Has the habit of bed-wetting. 4 Sits in a corner with a toy.

1 Children often undergo separation anxiety when they are separated from their parents. This separation anxiety manifests in different stages such as protest, despair, and detachment. Protest is the first stage of separation anxiety, during which the child screams, cries, or hits the other person for separating him or her from the parents. After this stage, the child enters the stage of despair, where the child begins to withdraw from others and stay depressed. During this stage, the child starts wetting the bed and sucking the thumb because of fear and anxiety. After the stage of despair, the child enters the stage of detachment. The child starts interacting with strangers and takes an increased interest in the surroundings or sits in a corner and plays with a toy.

A child with autism spectrum disorder is hospitalized for a treatment that will last about 1 week. How should the nurse make the child comfortable? 1 Ask the parents to accompany the child. 2 Modify the room according to the child's needs. 3 Explain the surroundings of the room. 4 Help the child perform daily routine tasks.

1 Children with autism spectrum disorders often are uncomfortable in a new environment and may not like to be with strangers. Therefore children with an autism spectrum disorder must be accompanied by their parents during hospitalization. While caring for a visually impaired child, the nurse modify the room according to the needs of the child. This helps prevent accidents. Because the child is not visually impaired, the nurse need not explain the surroundings of the room. Children with autism spectrum disorders often do not like assistance and prefer to perform their daily chores by themselves. Therefore the nurse should not help the child with such activities.

The nurse is speaking with the parents of a child with a very high fever. Which statement by the child's parent indicates a need for additional teaching? 1 "The temperature is quite high. It's life-threatening." 2 "I guess chills are common during high fever." 3 "Antipyretics should bring down the temperature." 4 "Fever has its own advantages for the body."

1 Contrary to popular belief, neither the rise in temperature nor its response to antipyretics indicates the severity or etiology of the infection. This casts doubt on the value of using fever as a diagnostic or prognostic indicator. Chills are common as the body tries to conserve heat and raise the central temperature to a new set point. Antipyretics bring down the temperature but they do not treat the condition. Fever has physiologic benefits, including increased white blood cell activity, interferon production and effectiveness, and antibody production and enhancement of some antibiotic effects.

Parents ask the nurse for advice when telling their 4-year-old about a grandmother's death. The nurse's best response involves teaching the parents that the child's concept of death is: 1 temporary. 2 permanent. 3 personified in various forms. 4 inevitable at some age.

1 Death is seen as a temporary departure. Death is thought of as not permanent; life and death can change places with each other. Personification is typical of school-age children. Children 9 to 10 years old have this understanding of death.

A week-old newborn is assessed for body weight, birth marks, and height. The birth weight is lower than what it should be for height. Which physical feature of the newborn makes the nurse conclude that the newborn is affected by Down syndrome? 1 Short and broad neck 2 Long and thin fingers 3 Short and thin lips 4 Broad and long nose

1 One of the characteristics of Down syndrome is a short, broad neck. These children have an impaired immune system and are at risk for spinal cord compression. Physical features such as long and thin fingers, short and thin lips, and broad and long nose are all common in a normal child and do not indicate any abnormality.

A patient is put on a ventilator in the intensive care unit of a tertiary hospital for long-term care. While caring for the patient, the nurse continuously assesses the health status of the patient. How does this intervention affect the patient's outcome? It prevents: 1 pneumonia. 2 lung cancer. 3 cystic fibrosis. 4 pulmonary edema.

1 One of the major disadvantages of long-term ventilator care is ventilator-associated pneumonia. The nurse has to assess the patient for ventilator-associated pneumonia or for the early detection of respiratory complications due to ventilators. Lung cancer is not associated with ventilator support. Cystic fibrosis is due to defect in chromosome number 7. Pulmonary edema is not the main concern associated with ventilators in the ICU.

The nurse finds that the parent of a terminally ill child is unusually optimistic about the child's condition. What should the nurse interpret from this? The parent: 1 Is in the denial stage of grief. 2 Believes the diagnosis is wrong. 3 Normally has a cheerful nature. 4 Is optimistic about the treatment

1 Parents may be in a state of shock when first hearing about the terminal illness of their child. The parent's abnormal optimism about the child's condition indicates that the parent is in denial and has not accepted the child's illness. It is unlikely that the parent believes that the diagnosis is wrong. The diagnosis of a terminally ill condition in a child is depressing for any parent, even one with a cheerful nature. Being optimistic when there is no cure indicates the stage of shock and denial.

On assessing a large family, the nurse finds one of the adolescents is less peer-oriented, lacks autonomous inner controls, and does not tend to participate democratically in family interactions or depend on parents for advice. Which behavior of the adolescent will the nurse think to be uncharacteristic of an adolescent belonging to a large family? 1 Less peer orientation 2 Democratic participation 3 Autonomous inner controls 4 Depending on parents for advice

1 Peer orientation is typically seen in adolescents from large families. Democratic participation, developed autonomous inner controls, and depending on parents for advice are not normal behavioral patterns seen in adolescents from large families.

A 4-year-old child is seen in a clinic for a hearing impairment. What action does the nurse observe in the child to confirm hearing impairment? Select all that apply. The child: 1 Screeches happily when looking at a toy. 2 Has difficulty trying to read a book. 3 Does not respond when an alarm sounds. 4 Points at his tummy to indicate hunger. 5 Speaks fast, stutters, and has speech delay.

1, 3, 4 A child with a hearing impairment yells or screeches in pleasure because the child cannot hear how loud these sounds are. The child also does not respond to loud sounds and prefers nonverbal communication such as pointing. A child who has difficulty reading a book may have a visual impairment. Rapid speech with stuttering and speech delay are symptoms of fragile X syndrome.

A child is hospitalized for treatment of the flu. Once the child's parents leave, the child starts crying, looks for parents, attempts to leave, refuses to take medicine, hits other children, and breaks toys. What should the nurse conclude from the child's behavior? The child is in the: 1 Protest stage. 2 Despair stage. 3 Denial stage. 4 Detachment stage.

1 The child's behavior indicates that the child is in the protest stage of separation anxiety. The child is less able to cope with separation because of stress from the illness and wants to stay with the parents. The child expresses anger indirectly by showing behavioral changes. These behavioral changes are observed in the protest stage of separation anxiety. In the despair stage, the child appears less active, depressed, and uninterested in play and refuses to eat food. The denial stage is also called the detachment stage. In this stage the child is interested in the surroundings, plays with others, and forms new but superficial relationships with others.

The nurse finds that a patient has developed tachycardia and tachypnea after administration of a muscle relaxant. What is an appropriate nursing action? 1 Administer dantrolene sodium intravenously. 2 Use hot compresses on the neck and axillae. 3 Assess the patient's history of surgical procedures. 4 Administer an inhaled anesthetic.

1 The nurse should administer dantrolene sodium intravenously as the patient is showing signs of malignant hyperthermia (MH). Symptoms of MH include hypercarbia, elevated temperature, tachycardia, tachypnea, acidosis, muscle rigidity, and rhabdomyolysis. The nurse uses ice packs on the groin, axillae, and neck as MH is usually accompanied by hyperthermia. A family or previous history of sudden high fever associated with a surgical procedure and myotonia increase the risk for MH. But the patient will not be assessed for it now as MH has already set in. Use of inhaled anesthetics increase the risk of MH; therefore, they should not be administered as the patient is exhibiting symptoms of MH.

A child who was terminally ill died. What intervention should the nurse provide for the child and the family? 1 Allow the family be with and hold or rock the child. 2 Do not remove any of the tubes or catheters. 3 Follow the hospital policy to administer last rites. 4 Instruct the family that the nurse bathes the body.

1 The nurse should be very sensitive to the emotions of the family members who are grieving their loss and try to support them as much as possible. The nurse should allow privacy to the family with the child and allow them to hold or rock the child if they want. The nurse should remove the tubes and catheters from the child's body and let the family hold the child. The nurse should allow the family to provide last rites according to their culture rather than following hospital policy. It helps the family cope. The family should be allowed to bathe and dress the child, as per their cultural practice.

The nurse is administering an antipyretic medication to a child with a high fever. What action does the nurse take in the first hour after giving the medication? 1 Check the temperature again. 2 Administer another dose. 3 Check the child's weight. 4 Check for aspirin toxicity.

1 The temperature is usually retaken 30 minutes after the antipyretic is given to assess its effect but should not be measured repeatedly. Another dose is not administered before 4 hours but no more than 5 times in 24 hours. The child's weight is taken if fluid imbalance is suspected. Aspirin toxicity can cause hyperthermia and is only assessed for if such toxicity is suspected.

The community health nurse is performing an assessment of an adolescent. Which group does the nurse ask the adolescent about when assessing the primary social group? 1 Peers 2 Church group 3 Work colleagues 4 Professional seniors

1 There are two types of social groups, primary and secondary. The primary group consists of peers and family who have a direct contact with and influence on the person. Secondary groups are indirectly related to the person. The examples include social organizations such as the church group, and professional groups such as office colleagues and professional seniors. Test-Taking Tip: Read carefully and answer the question asked; pay attention to specific details in the question.

The nurse is caring for a child recovering from ankle surgery. The child was administered an anesthetic for pain relief. What does the nurse do to prevent respiratory complications during the postoperative care? 1 Encourage respiratory movement with incentive spirometers. 2 Conduct hyperventilation with 100% oxygen. 3 Initiate cooling measures such as ice packs to the groin and axillae. 4 Change the child's position every 24 hours.

1 To prevent pneumonia and other respiratory complications after surgery, the nurse encourages respiratory movement with incentive spirometers or other motivating activities. Hyperventilation with 100% oxygen and cooling measures such as applying ice packs to the groin and axillae are done when malignant hyperthermia (MH) is diagnosed. The child's position is changed every 2 hours and deep breathing is encouraged.

Which approaches should the nurse adopt to provide support and guidance to a grieving family? Select all that apply. 1 Follow up regularly with telephone calls. 2 Provide a referral to a support group. 3 Recommend bereavement counseling. 4 Refrain from sending them cards. 5 Ask the family come for an office visit.

1, 2, 3 Nurses play an important role in helping the grieving family cope with their loss. The nurse should plan regular follow-ups through phone calls or meetings to assess whether they are coping well. Support groups and bereavement counseling may help the family vent their feelings and learn coping skills. The nurse should initiate contact by sending cards. The nurse should not expect the family to come in for a visit. The nurse should ask permission to make a home visit to make it easier on the family. This helps the family feel cared for.

The nurse is assessing a toddler's psychosocial development using Erikson's theory. What should the nurse include in the evaluation? Select all that apply. 1 Gross and fine motor skills 2 Mental acuity and capability 3 Level of doubt and shame 4 Competition with others 5 Inadequacy or inferior feelings

1, 2, 3 The stage in Erikson theory of psychosocial development that is used for toddlers (1-3 years) is autonomy versus shame and doubt. In this stage the toddler's motor skills, such as walking and climbing, are evaluated. The toddler's mental acuity and thought processes are also evaluated. The toddler develops negative feelings of doubt and shame when feeling low at this stage of growth. In the middle childhood growth pattern, the child tends to compete with others, aiming to accomplish tasks. This stage is referred to as industry versus inferiority in child. Inadequacy or inferiority complexes arise in this stage when parents impose huge expectations on the child. They tend to feel inferior in this stage of development.

The nurse is caring for a child who is scheduled to undergo an ostomy procedure. What are possible causes for a child to need undergo an ostomy procedure? Select all that apply. 1 Necrotizing enterocolitis 2 Hirschsprung disease 3 Crohn disease 4 Diseases of the bladder 5 Difficulty urinating

1, 2, 3, 4 A stoma is an opening, either natural or surgically created, which connects a portion of the body cavity to the outside environment. Children may require stomas for various health problems such as necrotizing enterocolitis, imperforate anus, and Hirschsprung disease. In older children, the most frequent causes are Crohn's disease and ureterostomies due to bladder defects. Difficulty in urinating is not reason enough for an ostomy unless the health care provider has diagnosed an underlying disorder that requires an ostomy.

The pediatric nurse is providing first aid to a child. The child sustained minor injuries while playing on the ground, and has severe pain in the knee joint. Which of the nursing interventions in the care of the child come under atraumatic care? Select all that apply. 1 Controlling pain 2 Allowing the child's privacy 3 Respecting cultural differences 4 Fostering the parent-child relationship 5 Giving vaccination for preventing tetanus

1, 2, 3, 4 Atraumatic care refers to the provision of the therapeutic interventions that would minimize or eliminate a patient's physical and psychologic distress. Atraumatic care includes interventions to reduce physical distress such as pain control measures. Interventions such as allowing the child privacy, respecting cultural differences, and fostering the parent-child relationship are examples of interventions those are helpful in minimizing psychologic distress in the child. Preventive measures such as vaccination for preventing tetanus are not an intervention included in a traumatic care.

What are the various guidelines that the nurse has to follow for the admission of a child into the hospital? Select all that apply. 1 Apply an identification band on the child's wrist. 2 Obtain the nursing admission history of the child. 3 Specimens for lab tests should not be taken. 4 Orient the parents about the inpatient facilities. 5 Hospital regulations should not be disclosed.

1, 2, 4 The nurse has to follow a few guidelines while admitting the child to the hospital. The nurse should apply an identification band on the child's wrist. This helps in providing appropriate care to the child. The nurse should take the nursing admission history in order to help to identify needs of the child. The nurse should explain to the parents as well as the child about the inpatient facilities. This would make them comfortable within the hospital. Specimens should be collected and may even be ordered for other specimens upon admission. The nurse should orient the parents and the child about the hospital regulations and schedules like visiting hours and food timings.

The nurse is assessing a child with autism. What characteristic features of autism does the nurse expect to find in the child? Select all that apply. 1 Verbal impairment 2 Stereotyped behavior patterns 3 Hearing and visual impairment 4 Nonrepetitive behavioral patterns 5 Decreased involvement in play

1, 2, 5 Children with autism usually have verbal impairment caused by poor language development. Autistic children exhibit stereotyped behavioral patterns caused by impaired neuromuscular function. Such children show decreased interest in functional play activities. Autistic children do not usually have hearing and visual impairment. Autistic children exhibit repetitive behavioral patterns.

Which behaviors of the family members indicate avoidance coping behavior? Select all that apply. 1 Refuses to agree to the treatment 2 Demonstrates anger and hostility 3 Expresses deep feelings of sorrow 4 Makes realistic plans for the future 5 Seeks cures from the faith healers

1, 2, 5 Coping reactions to stressors include those that indicate avoidance. Avoidance includes behaviors that help the person deny the truth or escape from it. Refusing to agree to treatment, being angry toward the staff, and seeking nontraditional medical treatments such as faith healing are examples of avoidance behavior. Expressing feelings of sorrow and planning realistically for the future are approach behaviors that indicate acceptance of the condition.

The parents of a terminally ill child ask the nurse how hospice care is different from hospital care. What information should the nurse give them? Select all that apply. 1 No extraordinary efforts would be made to prolong the child's life. 2 Pain and control of symptoms are the primary concerns of treatment. 3 The child's needs are considered to be the priority over the family's needs. 4 Hospice professionals are solely responsible for providing care to the child. 5 Family members are the principal caregivers supported by the hospice team.

1, 2, 5 Nurses should offer information about hospice care to the parents of a terminally ill child. Hospice is a community health care organization which specializes in caring for a dying child. It combines principles of hospice care and palliative care. The client is taken care of at home in the final stages of life. The concepts that make it different from hospital care include: no extraordinary efforts are made to prolong the child's life, and pain and control of symptoms are the primary concerns of treatment. The family members are usually the principal caregivers supported by a team of professionals, and the family's needs are given the same importance as the child's needs.

What are the unique features of a primary social group as part of social and cultural roles? Select all that apply. 1 The members of the social group are intimate with each other. 2 The members of the social group are mutually supportive. 3 The members of the social group a part of professional association. 4 The member of the social group has less concern for other member's behavior. 5 The behaviorof the individual member of a social group is constrained

1, 2, 5 Primary and secondary are the two types of social groups. The primary group includes family and peer, and is characterized by intimate and face-to-face contacts. The features of primary social group are intimacy, mutual support, and constraint of individual members' behavior. Professional association and less concern for members' behavior are features of a secondary social group. STUDY TIP: Avoid planning other activities that will add stress to your life between now and the time you take the licensure examination. Enough will happen spontaneously; do not plan to add to it.

What are the ways the nurse may help a family meet its needs based on family strength and functioning styles? Select all that apply. 1 Providing individualized support 2 Strengthening the family resources 3 Helping to delineate the individualized work 4 Training the family members to avoid stressful events 5 Building on qualities that make family function in a better manner

1, 2, 5 Providing individualized support, strengthening the family resources, and building on qualities that makes the family work better are the ways a nurse may help the family to meet its demands. Efforts should be made to share work in groups rather than delineating work. Families actually cope and respond to stressful events, which has to be identified and appreciated.

Children are taught the values of their culture through observation and feedback on their own behavior. A nurse teaching a class on cultural awareness-competence should be aware of which factor(s) that may be culturally determined? Select all that apply. 1 Social roles 2 Racial variation 3 Degree of competition 4 Determination of status 5 Geographic

1, 3, 4 Social roles are influenced by culture. Cultures that value individual resourcefulness/competition of status is acceptable. Determination of status is culturally determined and varies according to each culture. Racial variation refers to transmissible traits. Culture is composed of beliefs, values, practices, and social relationships that are learned. Cultural development may be limited by geography. The geographic boundaries are not culturally determined.

The nurse is assessing a patient with strabismus. Which finding would suggest the cause of strabismus? Select all that apply. 1 Poor vision 2 Short eyeball 3 Congenital defect 4 Muscle imbalance 5 Unequal curvature in the lens

1, 3, 4 Strabismus may result from poor vision and the resulting straining of eye muscles. Strabismus may result from a congenital defect as a developmental anomaly. Strabismus may also result from muscle imbalance caused by neuromuscular disorders. Short eyeball results in development of hyperopia, not strabismus. Unequal curvature of lens results in astigmatism, not strabismus.

What instructions should the nurse give to parents who have adopted a child? Select all that apply. 1 "You should reveal the adoption information as soon as possible." 2 "You should ask a third party to reveal the adoption information." 3 "You should always be honest and open toward the child." 4 "You should never reveal to the child that the child is adopted." 5 "Revealing adoption information causes concern and anxiety in children."

1, 3, 5 Discussing adoption with the adopted child requires utmost care. The parents should always be honest with their children. This strengthens the relationship between the parents and the adopted child. Revealing information early leads to less chance of misunderstanding and better continuance of relationship. Adoption information causes concern and anxiety in children but should not be avoided. Information about the adoption has to be revealed before any third party reveals it. Adoption information should not be withheld; the child has the right to know about the parents.

A baby is diagnosed with Turner syndrome shortly after birth. The parent is showing signs of depression and is finding it difficult to bond with the baby. Which characteristics indicate that the parent is in the adjustment stage? Select all that apply. 1 Assumes that the disability is genetically inherited 2 Acts happy and optimistic despite the diagnosis 3 Focuses on caregiving as the condition progresses 4 Believes the disability happened during pregnancy 5 Believes the disability is a punishment from God

1, 4, 5 The adjustment stage is the next phase after shock and is most often characterized by an open admission that the condition exists. This stage is accompanied by responses such as guilt. Many parents may feel responsible for the disability because it was genetically inherited. The mother may also believe she caused the disability by doing something wrong during pregnancy or may believe God is punishing her for previous misdeeds. Acting happy and optimistic despite the revealed diagnosis indicates the parent is still in denial and has not accepted the child's disability. Focusing on new caregiving practices as the condition progresses indicates total acceptance of the condition.

A female patient tells the nurse, "I am well educated and would like to start working full-time like my husband." How should the nurse respond? Select all that apply. 1 "You might potentially get really stressed with work and house chores." 2 "You may at times direct your stress toward children." 3 "Dual income would help you to lead a joyous life." 4 "You may not have adequate time for social activities." 5 "You may have problems fulfilling time demands."

1, 4, 5 Work overload is a common source of stress in a dual-earner family as both partners need to share the household chores. Social activities are significantly curtailed as most time is devoted in meeting the responsibilities of the household. Time demands and scheduling are major problems for all individuals who work. Dual income provides more economic stability rather than making them happy. It is usually an indirect stress to the child.

The nurse is teaching a group of student nurses about developmental patterns of neonates. Which information should the nurse include in the teaching plan? Select all that apply. Infants: 1 use their eyes before they use their hands. 2 gain control of their feet before their hands. 3 use their hands to observe things around them. 4 stand erect first to get control of their back. 5 have structural control of the head before the trunk

1, 5 Postnatal development happens in a cephalocaudal pattern, from the head to the feet. Infants use or develop their eyes before gaining control of their hands. Infants are able to first hold their head up, and then they can stabilize their trunk. In this development pattern, infants have control of their hands first and then gain control of their feet. Infants observe objects first using their eyes and then with their hands. Infants must be able to control their back before they can stand erect.

The nurse finds that a newborn infant weighs approximately 3 kg (7 lb). Approximately how much would the child weigh when he reaches 2.5 years of age? 1. 9 kg (20 lb) 2. 12 kg (27 lb) 3. 15 kg (33 lb) 4. 17 kg (38 lb)

2 A baby typically quadruples in weight by the time the child reaches 2 to 2.5 years of age. A child who weighed 3 kg (7 lb) at birth should weigh four times as much as that by 2.5 years, or 12 kg (27 lb). Birth weight triples by the age of 6-12 months, so the child's weight would be 9 kg (20 lb) at 6-12 months. The child would attain a body weight of 15 kg (33 lb) and 17 kg (38 lb) after 2.5 years of age.

The nurse is assessing a newborn who weighs 3 kg (7 lb). At what growth stage would the child weigh 12 kg (26 lb)? 1 Infancy 2 Toddlerhood 3 Preschool age 4 School age

2 A child's weight should quadruple by the toddler stage. The child's weight would be 12 kg if it was 3 kg at birth. Birth weight triples by the end of infancy, so it would be 9 kg if it is 3 kg at birth. After the toddler weight is achieved, an annual weight increase of 2-3 kg is seen in both preschool- and school-age children. STUDY TIP: You have a great resource in your classmates. We all have different learning styles, strengths, and perspectives on the material. Participating in a study group can be a valuable addition to your nursing school experience.

A nurse is examining a toddler and is discussing with the mother psychosocial development according to Erikson's theories. Based on the nurse's knowledge of Erikson, the most age-appropriate activity to suggest to the mother at this stage is to: 1 feed lunch. 2 allow the toddler to start making choices about what to wear. 3 allow the toddler to pull a talking-duck toy. 4 turn on a TV show with bright colors and loud songs

2 A toddler is developing autonomy and is able to start making some choices about what he or she can wear. A toddler is developing autonomy and focusing on doing things for himself or herself and therefore would not want the mother to feed him or her. The child is at the stage of autonomy versus shame and doubt, as defined by Erikson. At this age, the mother should provide opportunities for the child to be active and learn by experience and imitation. Providing toys the child can control will help achieve this stage. A toddler might easily become overstimulated by images from TV and loud sounds. Toddlers are more interested in manipulating and learning from objects in the environment.

The nurse is caring for a child after surgery. The child refuses to eat any food for lunch. What is an appropriate intervention by the nurse? 1 Insist that the child eat some more food. 2 Give the child a favorite food. 3 Call the child's parents. 4 Refer the matter to the dietician.

2 Although it is best to provide high-quality, nutritious foods, the child may desire foods and liquids that contain mostly empty or non-nutritional calories. Some well-tolerated foods include gelatin, diluted clear soups, carbonated drinks, flavored ice pops, dry toast, and crackers. Even though these substances are not nutritious, they can provide necessary fluid and calories. The nurse should not force the child to eat. Forcing a child to eat meets with rebellion and reinforces the behavior as a control mechanism. Calling the child's parents may make the child more irritated. The dietician is referred to before meal planning, and is consulted only if the child refuses to have any hospital food.

Which antipyretic is associated with Reye syndrome in children? 1 Acetaminophen (Tylenol) 2 Aspirin (Bayer) 3 Ibuprofen (Advil) 4 Norfloxacin (Noroxin)

2 Aspirin should not be given to children because of its association in children with influenza virus or chickenpox and Reye syndrome. Other antipyretics include acetaminophen (Tylenol), and nonsteroidal antiinflammatory drugs (NSAIDs). Acetaminophen (Tylenol) is the preferred drug. One nonprescription NSAID, ibuprofen (Advil), is approved for fever reduction in children as young as 6 months of age. Norfloxacin (Noroxin) is an antibiotic and is usually prescribed for bacterial infection of the gastrointestinal system.

The nurse assesses an infant at birth for height, weight, and other vital signs. What should the nurse include in the assessment to identify a conductive hearing disorder? The nurse assesses: 1 To see whether the infant's eyes move toward a flashlight. 2 The infant's response to an auditory stimulus. 3 The infant's vocal expressions during vocal communication. 4 The infant's physical activity toward a large moving object.

2 Assessment of an infant's response to auditory stimulation is used to detect a conductive hearing impairment in the newborn. Because the nurse suspects hearing impairment, the infant might have failed to respond to auditory stimulus. Eye movement following a flashlight helps in assessing the infant's vision. Vocal expressions or sounds produced in response to communication may indicate whether the infant has a speech or hearing impairment. Assessment of infant's physical activity toward a large moving object is useful to assess the child's muscle coordination. It also tests the infant's visual abilities.

The parent of a 2-year-old child tell the nurse that the child likes to play alone and asks people to repeat questions several times. The parent also says that the child uses gestures to communicate. What should the nurse infer from this? The child has: 1 Cognitive impairment. 2 Difficulty hearing. 3 Normal development. 4 Chronic mental illness.

2 Children 2 to 3 years old understand the common language used at home, and they try to communicate with family members in the same language. If the child has difficulty understanding and responding after the parent repeats a statement several times, this may indicate the child has a hearing problem. The child does not have lack of orientation, so the nurse should not infer that the child has cognitive impairment. Children stop using gestures and start communicating verbally around the age of 15 months. Therefore the child does not have normal development. The child is not bullying or being aggressive, so the nurse should not infer that the child has a chronic mental illness.

A camp nurse is assessing a group of children attending summer camp. Based on the nurse's knowledge of special parenting situations, which group of children is at risk for a sense of belonging? 1 Children adopted as infants 2 Children recently placed in foster care 3 Children whose parents recently divorced 4 Children who recently gained a stepparent

2 Children placed in foster care are at greater risk to have problems perceiving a sense of belonging. Children adopted at birth have fewer problems with acceptance when parents follow preadoption counseling about disclosure. Children of divorced parents often fear abandonment. Children who gain a stepparent are at risk for having trust problems with the new parent.

17. An 8-year-old child is receiving digoxin (Lanoxin). The nurse should notify the practitioner and withhold the medication if the apical pulse is less than which of the following? 1. 60 2. 70 3. 90 4. 100

ANS: 2 2. If a 1-minute apical pulse is less than 70 for an older child, the digoxin is withheld. 1. This is the cut-off for holding the digoxin dose in an adult.

The nurse observes a child having difficulty getting a mobile phone to work and looking puzzled. What type of play is the child demonstrating? 1 Skill play 2 Dramatic or pretend play 3 Social-effective play 4 Sense-pleasure play

2 Children's play activities are categorized during each stage of development. Pretend play, which is also called dramatic play, is seen in 11- to 13-month-old children when they perform activities that might be puzzling or frustrating to them. Skill play is seen after infants have developed the ability to grasp objects with their hands and manipulate them. This is when they use their skills to do things they observe such as putting paper in and out of a toy. Social-effective play is seen in infancy, when infants take pleasure in relationships with people. Sense-pleasure play happens when infants become attracted to natural colors or things and focus intently on them. An example is playing with sand.

The nurse needs to start an intravenous (IV) line on an 8-year-old child to begin administering intravenous antibiotics. The child starts to cry and tells the nurse, "Do it later, O.K.?" The nurse should: 1 start the IV line because allowing the child to manipulate the nurse is bad. 2 start the IV line because unlimited procrastination results in heightened anxiety. 3 postpone starting the IV line until the child is ready so that the child experiences a sense of control. 4 postpone starting the IV line until the child is ready so that the child's anxiety is reduced.

2 Intravenous antibiotics are a priority action for the nurse. A short delay may be possible to allow the child some choice, but a prolonged delay only serves to increase the anxiety. The nurse should start the IV line, recognizing that the child is attempting to gain control. If the timing of the IV line start was not essential for the start of IV antibiotics, postponing might be acceptable. The child may never be ready. The anxiety is likely to increase with prolonged delay.

The nurse is caring for a traditional Chinese patient. The patient avoids direct eye contact with the nurse. What should the nurse conclude from this behavior? 1 The patient has major depression. 2 The behavior is acceptable and normal. 3 The patient demonstrates low self-esteem. 4 The nurse should get a psychiatric referral.

2 Many traditional Chinese people avoid direct eye contact as a sign of respect. It may be normal for this patient. The nurse cannot conclude that the patient has major depression or low self-esteem just from this behavior. Other assessment data are needed. There is also no evidence that this patient needs a psychiatric referral because he or she does not make eye contact. Test-Taking Tip: Pace yourself during the testing period and work as accurately as possible. Do not be pressured into finishing early. Do not rush! Students who achieve higher scores on examinations are typically those who use their time judiciously.

A nursing professor is teaching a group of graduate nurses about anticipatory grief reactions. What information should be included in this presentation? 1 It may last for weeks or up to a few months. 2 It can happen when the death is expected. 3 It happens more than 1 year after the death. 4 It can result in feelings of excessive loneliness.

2 Nurses should be able to identify family members having anticipatory grief reactions. When death is expected, family members may experience anticipatory grief. Anticipatory grief may be manifested in varying behaviors and intensities and may include denial, anger, depression, and other psychological and physical symptoms. Anticipatory grief lasts while the loved one is preparing to die. There is no standard time frame for anticipatory grief. Complicated grief reactions are seen more than 1 year after the loss, whereas anticipatory grief happens before the death. The patient with complicated grief can display symptoms such as unusual sleep disturbances and feelings of excessive loneliness or emptiness.

The nurse is assessing a 14-year-old child who has recently discovered he or she is adopted. What may be a sign that the child is not handling this information well? 1 Nausea and vomiting 2 Decline in school grades 3 Increased thirst and polyuria 4 Changes in elimination

2 Teenagers do not always verbalize their feelings, so one sign may be a decline in school grades. The child's temper and social behaviors may also be altered. Nausea and vomiting may be a sign of a viral infection. Increased thirst and polyuria are signs diabetes mellitus. Disturbances in elimination are common in infants after stress; however, they are uncommon in adolescence. Test-Taking Tip: The computerized NCLEX exam is an individualized testing experience in which the computer chooses your next question based on the ability and competency you have demonstrated on previous questions. The minimum number of questions will be 75 and the maximum 265. You must answer each question before the computer will present the next question, and you cannot go back to any previously answered questions. Remember that you do not have to answer all of the questions correctly to pass.

The nurse is caring for a 2-day-old neonate who is healthy but has a low body temperature. The nurse instructs the infant's mother to place the unclothed infant on her bare chest. Which finding in the infant indicates ineffective management of the infant's condition? 1 Hyperglycemia 2 Metabolic acidosis 3 Body weight of 21 lbs 4 Body weight of 7.5 lbs

2 The neonate has hypothermia, and therefore the nurse instructs the mother to perform kangaroo care, in which the unclothed infant is placed on the mother's bare chest. This ensures improved thermoregulation and improves the complications of hypothermia. The presence of metabolic acidosis is a symptom of hypothermia. Hypoglycemia is caused by hypothermia in the infant. The normal weight of a healthy neonate at birth is approximately 7.5 lbs. By one year of age, the infant's weight normally triples to 21 lbs. Test-Taking Tip: Be alert for details. Details provided in the stem of the item such as behavioral changes or clinical changes (or both) within a certain time period can provide a clue to the most appropriate response or, in some cases, responses.

In 2010, there were 2000 live births in an area with a population of 20 million. There were 10 deaths among infants younger than 27 days old, 20 deaths among children ages 0 to 6 months, and 40 deaths among children younger than 1 year of age (includes all subcategories of deaths under 1 year). What is the infant mortality rate (IMR)? Record your answer using a whole number. _____________ per 1000 live births.

20 The infant mortality rate is the number of deaths during the first year of life per 1000 live births. The number of deaths of infants younger than 1 year of age was 40. Thus, IMR = number of deaths under 1 year / number of live births or 40 / 2000 = 20 / 1000.

The nurse who works in a pediatric ward wants to explore his/her own ability to develop a therapeutic relationship with children and their families. The nurse does a self-assessment to evaluate the caregiving style by using an assessment questionnaire. In the questionnaire the nurse answers "yes" to a question, which indicates a positive action. What was the question to which the nurse answered as "yes"? 1 "Do you control visitor access to children by using excuses?" 2 "Do you periodically interview children to determine their current issues?" 3 "Do you become critical when parents do not visit their children?" 4 "Does the senior nurse appreciate you for being close to the child?"

2 The nurse should maintain a therapeutic relationship with the children and their families to provide effective care. While performing the self assessment of the ability to develop therapeutic relationship with children, the nurse should evaluate her or his positive actions and negative actions. To evaluate the positive action the nurse should check whether she or he is taking periodical interviews of the children. It helps to evaluate their health, emotions, and feelings. The negative actions include restricting the parents not to see their children. Children feel comfortable with their parents, so the nurse should not restrict them. The nurse should not be critical and judgmental, as it is unprofessional. The nurse can suggest the parents spend time with children. The nurse should not be too close with any child.

The nurse is assessing a child's level of self-care. The nurse documents a rating of II for dressing and grooming. What can be inferred from this rating? The child: 1 Is independent on all aspects of personal care. 2 Depends on the supervision of another person. 3 Needs to use equipment or another adaptive device. 4 Requires direction from a person and uses equipment.

2 The self-care scale can be used for rating the functional self-care abilities of the child. The score ranges from 0 to IV. If the child is scored a II, this implies that the child requires assistance or supervision from another person. A child who is independent with activities of daily living would receive a 0. A score of I implies that the child requires equipment or a device for self-care. A score of III implies that the child requires assistance or supervision from another person and equipment or a device. A score of IV implies that the child is totally dependent and does not participate.

Which statement made by a child's parent supports the nurse's conclusion that the child has a difficult temperament? 1 "My child has predictable habits." 2 "My child often cries and throws tantrums." 3 "My child responds with passive resistance to new routines." 4 "My child becomes inactive and moody with change in routine."

2 The temperament of a child is said to be difficult when the child exhibits frequent episodes of crying, frustration, and tantrums. Such children are irritable and show irregularity in habits. Almost 10 percent of children fall under the category of difficult temperament. A child with an easygoing temperament has predictable habits and a positive attitude towards new stimuli. A slow-to-warm-up child responds with passive resistance to new routines and becomes inactive and moody.

When admitting a child to the inpatient pediatric unit, the nurse should assess for which risk factors that can increase the child's stress level associated with hospitalization? Select all that apply. 1 Mild temperament 2 Lack of fit between parent and child 3 Below-average intelligence 4 Age 5 Gender

2, 3, 4, 5 Risk factors for increased stress level of a child to illness or hospitalization: "Difficult" temperament; Lack of fit between child and parent; Age (especially between 6 months and 5 years old); Male gender; Below-average intelligence; Multiple and continuing stresses (e.g., frequent hospitalizations).

The nurse is educating a group of parents and children in the pediatric ward about the benefits of ambulatory care. What benefits does the nurse discuss with the group? Select all that apply. 1 Improved care 2 Increased cost-saving 3 Reduced chances of infection 4 Ambulatory care is lesser challenging 5 Minimum stressors of hospitalization

2, 3, 5 Ambulatory care is associated with an increased cost-saving as compared to hospital admissions, since there are no admission-related costs. Ambulatory care is associated with lesser chances of acquiring infections due to limited exposure to health care facilities. Ambulatory care is devoid of the stressors of hospitalization. There is deficient care due to the absence of qualified medical person for supervision. Ambulatory care is more challenging when compared to hospitalization as the child and the parents need to rely mostly on themselves for providing care to the child.

A child is brought to the clinic for a routine check-up. The parent informs the nurse that the child wants to go to school, but the parent does not want the child to go. What other characteristics in the parent would indicate that the parent is overprotective? Select all that apply. 1 Sets same rules for the child and the siblings 2 Continually helps child even if the child is capable 3 Helps the child to learn new skills to be independent 4 Sets high goals without understanding child's abilities 5 Puts restrictions on child's play due to fear of injury

2, 4, 5 A nurse should be able to identify parents who tend to be overprotective about their child with disabilities. The parent may continually help the child even when the child is capable of doing the task. Setting very high or low goals without understanding the child's capabilities makes the child feel incapable and inadequate. Putting restrictions on their child's play due to fear of injury indicates that the parent is too concerned about the child. Setting same rules for all children is not suggestive of an overprotective attitude in the parent. A parent who is not overprotective of the child helps the child to learn skills to be independent.

After assessment, the nurse notices that a child is in the detachment stage of separation anxiety. Which behavioral changes would the nurse observe in the child? Select all that apply. 1 Refuses to eat, drink, or get out of the bed 2 Shows an increased interest in the surroundings 3 Tries to leave the hospital to find the parents 4 Begins to form new relationships with others 5 Interacts with strangers or familiar caregivers

2, 4, 5 Detachment is the third stage of separation anxiety. It is also referred to as the denial stage. In this stage the child begins to take an interest in the surroundings. The child also forms new but superficial relationships with others and becomes more interested in interacting with strangers or familiar caregivers. The child's behavior indicates that the child has finally adjusted to the loss of the parents. This is a serious stage because reversal of the potential adverse effects is less likely to occur after detachment. Refusing to eat, drink, and get out of bed are characteristics of the despair stage of separation anxiety. Attempting to leave the hospital to find the parents is observed in protest stage of separation anxiety.

The parents of a 3-year-old child report seeing a whitish glow in the child's eyes. The nurse begins to examine the child. What information should the nurse give to the parents before assessment? "The child: 1 Needs hematologic assessment for confirmation of diagnosis." 2 May not be able to distinguish between colors." 3 May not see clearly for some time after the examination." 4 Needs immediate hospitalization after the examination."

3 A strange light in the eyes indicates that the child may have retinoblastoma. It is diagnosed by ophthalmoscopic examination, which involves dilation of the pupil. During this procedure, the eyes become sensitive, and the child may not be able to see clearly for some time. Informing the parents about it will reduce anxiety. Hematologic assessment is not used to diagnose retinoblastoma. A retinoblastoma can be diagnosed by ophthalmoscopic examination under general anesthesia and with imaging studies, including ultrasonography and computed tomography. A child with visual impairment may have difficulty distinguishing between colors, but it is not helpful for the parents to learn about it before the child's illness is diagnosed. The child may need immediate hospitalization after the examination depending on the severity of tumor, but the nurse should not tell the parents about hospitalization because it could make them panic.

Several types of long-term central venous access devices are used. A benefit of using an implanted port (e.g., Port-a-Cath) is that it: 1 is easy to use for self-administered infusions. 2 does not need to pierce the skin for access. 3 does not need to limit regular physical activity, including swimming. 4 cannot dislodge from the port, even if child plays with port site.

3 Because this device is totally under the skin, there are no activity limitations for the child. The port has to be accessed with a special needle. Because the port is totally under the skin, a needle must be used to access the port. The port site is under the skin, so there is nothing for the child to play with.

The parents of a cognitively impaired child ask the nurse for guidance with discipline. The nurse's best response is: 1 "Discipline is ineffective with cognitively impaired children." 2 "Discipline is not necessary for cognitively impaired children." 3 "Behavior modification is an excellent form of discipline." 4 "Physical punishment is the most appropriate form of discipline."

3 Behavior modification with positive reinforcement is effective in children with cognitive impairment. Discipline is essential in assisting the child in developing boundaries. Positive behaviors and desirable actions should be reinforced. Most children with cognitive impairment will not be able to understand the reason for the physical punishment; consequently behavior will not change as a result of the punishment.

A child who has undergone orofacial surgery is getting discharged. The nurse teaches the parents about how to safely transport the child on the way home. Which statement made by the parents indicates a need for additional teaching? "We should: 1 Have a blanket and pillow for our child for the car ride home." 2 Have a plastic bag for our child in case of nausea and vomiting." 3 Use a cup with a lid and a straw for giving fluids to our child." 4 Make sure our child has pain medication before discharge."

3 Children who undergo orofacial surgery should not use a straw for drinking fluids because it can damage the surgical site. Therefore the parents should not use cup with a lid and a straw for giving fluids to the child. The parents should bring a blanket and pillow for the child in the car so that the child can sit or sleep properly. Parents should bring a plastic bag, which will be helpful if the child becomes nauseated or vomits. The parents should give prescribed pain medication to the child before leaving the facility for relieving pain.

A child with fragile X syndrome was prescribed clonidine (Catapres) to improve attention and decrease hyperactivity. What other intervention may improve the child's cognitive ability? 1 Aromatherapy and hydrotherapy 2 Protein replacement and gene therapy 3 Language and occupational therapy 4 Hormone and biologic therapy

3 Children with fragile X syndrome have impaired cognitive development and may be prescribed clonidine (Catapres) to improve attention span and decrease hyperactivity. Other interventions that can improve cognitive ability in these children include speech and language therapies, occupational therapy, and special educational programs. Aromatherapy and hydrotherapy are useful for reducing stress and anxiety. Protein and gene replacement involves replacing the defected gene. It does not improve cognitive ability. Hormone therapy can be given to treat endocrine disorders. Biotherapy is given to strengthen the patient's immunity.

The nurse is speaking to a group in the community about psychosocial development according to Erikson's life-span approach. The nurse instructs the group not to impose too many expectations on a child because the child may develop an inferiority complex. What age group of children is nurse referring to here? 1 1-3 years 2 3-6 years 3 6-12 years 4 12-18 years

3 Erikson's life-span approach categorized childhood into five stages. Industry versus Inferiority is the fourth stage of development the crucial stage attained by children 6-12 years of age. Children at this stage are workers and producers, and they initiate and complete work aiming at real achievement. The child may feel inferior if parents impose many expectations on the child. The second stage is autonomy versus shame and doubt (1-3 years), when children increase their ability to control their bodies and their environment and use their mental powers in decision making. Negative feelings develop when children are made to feel low and when others shame them. Initiative versus guilt (3-6 years) is when children explore the physical world with all their senses and powers and may feel guilt when parents make their child feel their behaviors are bad. Identity versus role confusion (12-18 years) is the stage when rapid and marked physical changes occur. Adolescents struggle to fit the roles they have played and those they expect to play. When the ability to resolve these conflicts fails, it leads to role confusion.

During the assessment of a child, the nurse finds that the child is inactive, depressed, sad, and uncommunicative; refuses to eat; and generally lacks interest in everything around her. What should the nurse interpret from this assessment? The child is in the: 1 Denial stage. 2 Protest stage. 3 Despair stage. 4 Detachment stage

3 From this assessment, the nurse interprets that the child is in the despair stage. This is the second stage of separation anxiety. In the despair stage, the child appears less active, depressed, and uninterested in play or food. In this stage the child's physical condition may deteriorate from refusing to eat, drink, or get out of bed. The denial stage is the third stage of separation anxiety. In the denial stage, the child is more interested in the surroundings, plays with others, and forms new but superficial relationships with others. In the protest stage, the child reacts aggressively, cries, screams, and searches for the parents with the eyes. Detachment is the third stage of separation anxiety. It is also called denial.

A child is assessed and categorized in the industry versus inferiority stage according to Erikson's theory. The nurse compares the child with Freud's psychosexual development theory. At what stage would the child be categorized in Freud's theory? 1 Anal 2 Phallic 3 Latency 4 Genital

3 In Erikson's theory, the industry versus inferiority stage includes children 6-12 years old. The stage in Freud's theory that matches this age group is the latency stage. The anal stage of Freud's theory corresponds to the autonomy versus shame and doubt stage of Erikson's theory. The phallic stage of Freud's theory corresponds to initiative versus guilt, and the genital stage of Freud's theory corresponds to the identity versus role confusion stage of Erikson's theory.

After giving a bed bath and cleaning the feet of a 2-year-old boy, an intravenous line is inserted into a small area by shaving the scalp. His Jewish parents are upset with this procedure. What could be the most probable reason behind the parents' discontent? 1 Intravenous infusions are against Jewish culture. 2 Scalp vein insertions are against the Jewish culture. 3 Cutting the hair before 3 years of age is not permitted in Jewish culture. 4 Touching the head after touching the foot is disrespectful in Jewish culture.

3 In Jewish culture, first haircut is done for boys at age 3 years. This is called the "upsherenish ceremony." Therefore, the nurse should have gotten consent from the parents before shaving the scalp of the boy. Intravenous infusions and scalp vein insertion are allowed in the Jewish culture. Touching the head after touching the foot is considered disrespectful in the Vietnamese culture.

A family presents to the emergency room with a child with enuresis. They tell the nurse that they have tried their traditional cure of burning to alleviate the problem. What can the nurse expect as a physical sign of this practice? 1 Reddened blotches on the skin 2 Rash covering the child's body 3 Small blisters on the child's body 4 Small welt-like lesions on the skin

3 Many communities follow the practice of burning. As the name suggests, small burns are made on the child's body in an attempt to cure disorders such as enuresis and temper tantrums. The nurse is most likely to find blisters on the child's body. Reddened blotches, rashes, and welt-like lesions may be signs of an allergic reaction.

The nurse is assessing a child. The nurse asks the parents, "Has your child started sleeping less lately?" Which attribute of temperament is the nurse assessing? 1 Adaptability 2 Distractibility 3 Rhythmicity 4 Activity

3 Rhythmicity is an attribute of temperament that refers to regularity in the timing of physiologic functions such as sleep and hunger. Because the nurse is asking about changes in the child's duration of sleep, the nurse is assessing rhythmicity. Adaptability is an attribute of temperament that refers to the child's ability to adapt to new situations. Distractibility is an attribute of temperament that refers to the ease with which a child's attention is diverted. Activity is an attribute of temperament that refers to the levels of physical activity such as sleeping and eating.

A patient with two children is going through a divorce and does not understand what the term split custody means. What is the most appropriate answer? 1 Custody of both children is given to the grandparents. 2 Custody of both children is given to the parent who brings home the larger salary. 3 Custody of one child is given to the mother, and the father has custody of the other child. 4 Custody of the children is given to the father, and the mother is allowed to visit once a week.

3 Split custody means that custody of one child is given to the mother and custody of the other child is given to the father. This arrangement ensures that both parents have a child, but it separates the siblings. Custody of the children is not given to the grandparents, it is not determined by who has the larger salary, nor is it given solely to the father with visitation privileges given to the mother.

The nurse develops a plan of care based on the information documented in a child's admission assessment. The nurse instructs the health care team that they should not leave the room until the child falls asleep. What information documented under the self-perception-self-concept pattern would necessitate this nursing intervention? The child has: 1 Nightmares. 2 Disturbed sleep patterns. 3 A fear of sleeping alone. 4 The habit of bed-wetting.

3 The child has fear of sleeping alone in the room. This information is usually noted under the self-perception—self-concept pattern of the nursing history. The nurse tries to comfort the child's fear of sleeping alone by being present until the child falls asleep. Nightmares can be managed by comforting the child and preventing specific fears. Nightmares and disturbed sleep may be brought on by hospitalization and may improve once the child adapts himself or herself to the new environment. Bed-wetting is common in younger children but needs further evaluation in older children.

A child with a temporary visual impairment was admitted to the hospital for treatment. What nursing intervention would make the child feel most comfortable in the hospital? The nurse: 1 Explains the different departments of the hospital. 2 Understands the child's behavior and daily routine. 3 Describes the surroundings of the room and the unit. 4 Asks the cleaner to move the furnishings around

3 The nurse helps the child become familiar with the room so that the child knows the layout in order to avoid injury while moving around the room. Explaining to the child about the hospital departments is not necessary to make the child feel comfortable. Understanding the child's daily routine is necessary to plan activities for the child but does not increase comfort. The cleaning personnel are asked to maintain the décor of the room to avoid accidents; therefore changes should be avoided.

The pediatric nurse works efficiently in providing nursing care to an acutely ill child. After discharge, parents of the child ask the nurse to visit their home for dinner. What should the nurse do? 1 Accept it; otherwise it may adversely affect the good relationship. 2 Tell them to schedule it later as it is a busy day in hospital. 3 Reject it courteously and thank them for the invitation. 4 Ask them to invite other staff who were involved in the care as well.

3 The nurse is not supposed to develop personal relationships with the children and families during the care and after the discharge. Therefore, the nurse has to courteously reject such invitations that may lead to personnel relationships. Even if the nurse is busy, the nurse should not accept invitations for lunch, dinners, or other parties, or ask the families to invite other medical staff. This shows unprofessional behavior.

Before transporting a 16-year-old American Indian female for a magnetic resonance imaging (MRI) scan, the nurse notices the girl is wearing a decorated amulet necklace. The nurse's next best action is to: 1 remove the necklace and place it at the nurse's station. 2 explain the risks of wearing the necklace during the MRI. 3 ask the patient if there is a special reason for wearing the necklace. 4 place tape around the neck covering the necklace.

3 The nurse should first ask the patient the purpose of wearing the necklace. The amulet may be worn as a religious ritual or simply as an accessory. After assessing why the necklace is worn, the nurse can explain the reason for having to remove the necklace for the procedure. The first step is to assess. Placing tape around the neck is not an appropriate action and could be unsafe. The necklace should be left with family members if possible or in a locked cabinet, rather than at the nurse's station. Test-Taking Tip: Many times the correct answer is the longest alternative given, but do not count on it. NCLEX item writers (those who write the questions) are also aware of this and attempt to avoid offering you such "helpful hints."

The nurse is assessing the functional self-care level of a child and determines that the child requires the assistance of a caregiver for general hygiene and dressing. How would the nurse rate the child? 1. 0 2. I 3. II 4. IV

3 The nurse should rate the child as a II (two) because the child requires assistance of a caregiver for general hygiene and dressing. A grading of 0 (zero) is given to the child who is capable of taking full self-care. A grading of a I (one) is given to the child who requires the use of equipment or a device for self-care. A child who is totally dependent and does not participate in self-care would be rated a IV (four).

The nurse is caring for a 7-year-old child. The child wears an amulet because the family believes that it will protect the child from the evil eye. For a diagnostic procedure, the nurse has to take this amulet off. What should the nurse do? 1 Allow the child to continue to wear the amulet for comfort during all aspects of the procedure. 2 Do not perform the prescribed procedure because removing the amulet will hurt the family's feelings. 3 Get permission from the family to remove it for the procedure and replace it afterwards. 4 Ask the family to remove the amulet and not to let the child wear it because it obstructs medical care.

3 The nurse should try to avoid hurting the family's feelings. If the amulet is an obstacle to medical care, the nurse should explain that to the family and obtain their permission to remove the amulet for the procedure. The nurse should replace it as soon as the procedure is over. Continuing the procedure without taking off the amulet may interfere with the procedure or hurt the child. The nurse should not cancel the procedure because it is essential for the child's care. The nurse should not ask the family to remove the amulet permanently because it does not impede all medical care; affects only this procedure.

The nurse is caring for a child with an influenza viral infection. The child is anxious because the parents are unable to stay with the child. What should the nurse do to relieve the child's anxiety? The nurse should: 1 Not maintain any eye contact with the child. 2 Not speak with the child about missing the parents. 3 Use the phone to let the child talk with the parents. 4 Use a laptop to allow the child and parents to talk

3 The nurse should use a telephone to maintain contact between the child and parents so that the child can feel comfortable. It helps relieve the child's anxiety. The nurse should maintain eye contact and gently touch the child to establish rapport. The nurse should talk with the child about the parents and family to prevent detachment of the child from the parents. The nurse should not use a laptop to contact the child and parents. The laptop may not be compatible with medical equipment, and use may be restricted in certain areas.

The nurse gets out finger painting materials made from wallpaper paste for a child. After reviewing the child's medical record, the nurse decides to get out crayons and a coloring book instead. What information did the nurse find in the nursing history? The child: 1 Is on a salt-restricted diet. 2 Has lactose intolerance. 3 Has an allergy to wheat. 4 Is allergic to beetroot.

3 The painting materials made from wallpaper paste may contain wheat. If the paint comes in contact with the child's skin, it can cause an allergic reaction. Therefore the nurse changes the activity. The child who is on a salt-restricted diet need not worry about salt being in the paint. A patient is placed on salt restriction to prevent water retention, not because of an allergy. The child who is lactose intolerant cannot ingest dairy products. Beetroot is used as natural dyes in paintings; however, the wallpaper paste does not contain beetroot extract.

The nurse is caring for a child with cognitive impairment. Which statement made by the nurse to the parents is a reason for concern? 1 "I need to know more about cognitive impairment." 2 "I will ask the other staff to help with the child's care." 3 "I do not know what is going on with this child's health." 4 "I'll ask the health care provider to clarify my question."

3 The statement "I do not know what is going on with this child's health" is inappropriate because it indicates that the nurse does not understand the child's needs. The statement "I need to know more about cognitive impairment" shows the nurse's desire to learn about the disorder and help treat the child better. The statement "I will ask the other staff to help with the child's care" shows the nurse's desire to increase his or her skill level. The statement "I will ask the health care provider to clarify my question" is an inappropriate statement made by the nurse. It implies that the nurse is not aware of the care that should be given to a cognitively impaired child.

The nurse should expect to possibly incorporate which religious and cultural practices into the plan of care when caring for a 35-year-old Jewish mother who just gave birth to a healthy baby boy? Select all that apply. 1 Circumcision in hospital 2 Ordering house diet lunch tray of roasted pork with mashed potatoes 3 Allowing family, friends, and rabbi to visit patient often 4 Ask males to remove shawl and yarmulke while visiting 5 Ordering house diet with the exception of shellfish

3, 5 Family, friends, and rabbi should be allowed to visit. Individuals of the Jewish faith generally are prohibited from eating pork or shellfish. Ritual circumcision of male infants is custom on the eighth day and performed by a mohel. Asking males to remove shawls or yarmulkes is inconsistent with acceptance of religious values.

The parents of a terminally ill child are speaking with the nurse to learn about palliative care and how it differs from hospice care. What should the nurse tell them? "Palliative care: 1 Requires that a patient be a Medicare recipient." 2 Does not provide care for hospitalized patients." 3 Is provided when no effective cure is available." 4 Does not provide spiritual support to the family."

3 The terms palliative care and hospice care are associated with end-of-life care. Palliative care may be provided when no cure is available. Palliative care seeks to prevent, relieve, reduce, or soothe the symptoms of disease or disorder without effecting cure. Hospice care supports the patient and the family during the dying process (medical prognosis of 6 months or less) and the surviving family members through the process of bereavement. It provides total care for the patient in the hospital, home, and nursing home when the patient's disease is unresponsive to the curative treatment. To qualify for hospice care, the patient does not need to be a Medicare recipient. Most insurance programs cover hospice care. However, for palliative care, the patient needs to be a Medicare recipient. Palliative care can be provided to patients in a variety of settings, including hospitals. Both palliative care and hospice care include spirituality in order to provide holistic care. Nurses need to familiarize themselves with the attitudes and needs of various religious groups so that they can address the spiritual needs of their patients, families, and other members.

The nurse is caring for two children. The younger child creates complex imaginary stories using dolls and toys. The older child is engaged in building a model airplane. Which stages of development are the children likely in, according to Erikson? 1 The younger child is in the trust versus mistrust stage; the older child is in the initiative versus guilt stage. 2 The younger child is in the industry versus inferiority stage; the older child is in the identity versus role confusion stage. 3 The younger child is in the initiative versus guilt stage; the older child is in the industry versus inferiority stage. 4 The younger child is in the identity versus role confusion stage; the older child is in the trust versus mistrust stage.

3 The younger child exhibits a strong imagination and an urge to explore through her doll play, which indicates that she is in the initiative versus guilt stage of Erikson's psychosocial development theory. The older child is building a model, indicating a desire to produce something and complete a task, which is a primary characteristic of the industry versus inferiority stage according to Erikson. The trust versus mistrust stage of Erikson's theory consists of establishing trust and taking in the world using all the senses; neither child is exhibiting characteristics of the trust versus mistrust stage. The identity versus role confusion stage is characterized by rapid physical change in children and concern over how they are viewed by others; neither child is exhibiting behavior associated with this stage.

A couple visits the hospital for a prenatal checkup. On reviewing the genetic analysis report, the nurse finds that the male partner has fragile X syndrome. What should the nurse interpret from these findings? Select all that apply. 1 All of their sons will have a 50% chance of being affected. 2 All of their sons will be carriers for fragile X syndrome. 3 The chance of a daughter being affected is 50%. 4 All daughters will be carriers for fragile X syndrome. 5 All sons will be carriers and will have fragile X syndrome.

3, 4 Fragile X syndrome is an X-linked dominant syndrome with reduced penetrance. About 50% of daughters with fathers affected by fragile X syndrome will be affected because the dominant X chromosome can be from the affected father. All daughters with an affected father will be carriers. The sons get Y chromosomes from the father, so they are not necessarily carriers of the syndrome or affected by the syndrome. The sons can be carriers or affected if the syndrome is passed from the mother.

The parent of a newborn child asks the nurse the importance of breastfeeding. What should the nurse tell the parent? Select all that apply. 1 Breast milk is not rich in micronutrients. 2 Breast milk is not recommended for infants with fever. 3 Enzymes in breast milk are helpful in the digestion of milk. 4 Immunoglobulins in milk can prevent infections and diseases. 5 Breastfeeding can decrease infant mortality and morbidity.

3, 4, 5 Breast milk contains enzymes that are helpful in the digestion of milk and improve the bioavailability of all nutrients in the milk. Immunoglobulins in milk give immunity against infections and allergies. Hence, breastfeeding can decrease infant mortality and morbidity. Breast milk is rich in micronutrients. Breast milk has immunologic properties, so it can be given to the infants with fever.

Informed consent is valid when: Select all that apply. 1 universal consent is used. 2 it is completed only for major surgery. 3 a person is over the age of majority and competent. 4 information is provided to make an intelligent decision. 5 the choice exercised is free of force, fraud, duress, or coercion.

3, 4, 5 The age of majority is usually 18 years. The term competent is defined as possessing the mental capacity to make choices and understand their consequences. Enough information is provided so that the person can make an intelligent decision. The person giving consent does so voluntarily; that is, freely without coercion, any form of constraint, force, fraud, duress, or deceit. Universal consent is not sufficient. Informed consent must be obtained for each surgical or diagnostic procedure. Informed consents must be obtained for major and minor surgery, diagnostic tests, medical treatments, release of medical information, postmortem examination, removal of a child from the health care provider against medical advice, and photographs for medical, educational, or public use.

Which behaviors would characterize a child adequately coping with a chronic illness? Select all that apply. 1 Behavior problems at home and school 2 Negative attitudes about his or her condition 3 Acceptance of his or her limitations and coping accordingly 4 Taking responsibility for his or her own physical care 5 Identification with other similarly affected people

3, 4, 5 There are two ways of coping with the illness. Children with healthy coping mechanisms accept their limitations and assume responsibility for their care. These children may also identify with other similarly affected people. Maladaptive coping is evident in behavior problems at home and school. These children may also have a negative attitude toward the illness and begin acting out.

22. As part of the treatment for congestive heart failure, the child takes the diuretic furosemide. As part of teaching home care, the nurse encourages the family to give the child foods such as bananas, oranges, and leafy vegetables. These foods are recommended for this child because they are high in which of the following? 1. Chlorides 2. Potassium 3. Sodium 4. Vitamins

ANS: 2 2. Diuretics that work on the proximal and distal renal tubules contribute to increased losses of potassium. The child's diet should be supplemented with this electrolyte

A 14-year-old boy learns about his adoption from his relatives and wants to know his identity. He appears angry, embarrassed, and anxious. What should the nurse advise his parents to help resolve the conflict? 1 "Don't forget to obtain the boy's birth certificate." 2 "Don't reveal the information regarding the adoption." 3 "Don't encourage the boy to search for his identity in this situation." 4 "Don't forget to inform the child about the availability of a birth certificate."

4 Adoptive parents should inform the boy about the availability of a birth certificate. Open and honest communication between the boy and the parents is essential for the welfare of the adopted child. Legally, adoptive parents are not permitted to obtain the birth certificate. Parents should understand the need of the child to search for his identity and extend encouragement and support. Test-Taking Tip: Identify option components as correct or incorrect. This may help you identify a wrong answer

The community nurse has conducted a survey on the frequency of occurrences of various diseases and health problems such as acne, headache, diarrhea, and upper respiratory tract infections (URTI) in children. What trend would the nurse most likely notice from the survey? The frequency of: 1 Acne decreases with age 2 URTI increases with age 3 Diarrhea increases with age 4 Headaches increases with age

4 Children who have had a particular type of problem are more likely to have that problem again. Therefore, the frequency of headaches increases with age. The activity of sebaceous glands increases with age and therefore the frequency of acne also increase with age. The immune system becomes stronger with age in children. Therefore, the frequency of upper respiratory tract infections (URTI) and diarrhea decreases with age.

During the assessment of a 5-year-old child whose parents are from Southeast Asia, the nurse notices small burn injuries on the skin. The nurse learns that the child has a history of temper tantrums. Which is the immediate nursing action? 1 Call the police. 2 Report child abuse. 3 Inform the health care provider. 4 Ask the parents about the injury.

4 Communication is important to avoid wrong diagnosis and interpretations. The nurse should ask the parents about the burn injuries to understand their cultural and religious practices. If the child exhibits temper tantrums, some Southeast Asian cultural groups follow the practice of causing burn injuries on the skin for treatment. These burn injuries are not considered to be abusive by this cultural group even though the dominant culture and legal system may consider this to be child abuse. Therefore, there is no need to call the police or report child abuse. The nurse can inform the health care provider after discussing with the parents.

What is the characteristic of the type of play that is organized by children playing with other children with the purpose of accomplishing a goal? 1 The group members play independently. 2 The group members act according to their own wishes. 3 The group members do have assigned leadership roles. 4 The group members plan activities even if the group is formed loosely.

4 Cooperative play features one child supplementing another child's function with a common aim of goal completion. In cooperative play, the group may be loosely formed but the members plan activities with the aim of accomplishing a task. The group members play independently among others in parallel play. The members act according to individual wishes in associative play. There is no leadership assignment in associative play.

The nurse needs to take the blood pressure of a preschool boy for the first time. Which action is best in gaining his cooperation? 1 Take his blood pressure when a parent is there to comfort him. 2 Tell him that this procedure will help him get well more quickly. 3 Explain to him how the blood flows through the arm and why the blood pressure is important. 4 Permit him to handle equipment and see the dial move before putting the cuff in place.

4 Permitting the child to handle the equipment and see the dial move allows him to play out the experience ahead of time. The parent's presence will be helpful, but it will not alleviate fear of the unknown. The child will not be able to understand the relationship between blood pressure and feeling better. Additionally, there is no evidence that this procedure will help him get well more quickly. Explaining how blood flows through the arm is too complex for this age group.

A single working parent says that caring for a child and managing work is becoming difficult. The nurse determines that the patient is exhausted by these responsibilities. Which is the most appropriate service that would benefit the patient? 1 Hospital 2 Change of job 3 Adoption service 4 Respite child care

4 Respite child care is a service available to help parents to relieve exhaustion and avoid burnout. Therefore, the patient should seek respite child care services. Hospitals are for patients who need medical attention. It would not be appropriate to change jobs because the stress of working would still be there. Adoption services are inappropriate in this situation.

The nurse is evaluating a child for suspected autism. Which finding in the child suggests autism? 1 Limited functional play 2 Avoidance of body contact 3 Language delay at an early age 4 Inability to maintain eye contact

4 The hallmark of autism is an inability to maintain eye contact with another person. Limited functional play may be seen in children with autism, but it is not a hallmark of autism. Autistic children also avoid body contact, but it is not a hallmark finding. Language delay at an early age is not a hallmark of autism; however, children with autism may exhibit language delay at an early age.

What nursing care should be provided to a school-aged child with cognitive impairment? 1 Periodic testing of thyroid function 2 Education on sexuality 3 Education on self-care skills for the child 4 Speech therapy referral for the child

4 The nurse should refer the child with cognitive impairment for speech therapy. It helps improve communication and promotes social behavior of the child. Periodic testing of the thyroid function is done if the child has Down syndrome. Thyroid function is not altered in all children with cognitive impairment. Sexual information is given to adolescents with cognitive impairment. Younger school-age children with cognitive impairment may not understand information regarding sexuality. Self-care skills should be taught by the parents to children with cognitive impairment.

A (Toddlers use distinct behaviors in the quest for autonomy. They express their will with continued negativity and the use of the word "no." Children at this age also have rapid mood swings. The nurse should reassure the parents that their child is engaged in expected behavior for an 18-month-old.)

A parent of an 18-month-old boy tells the nurse that he says "no" to everything and has rapid mood swings. If he is scolded, he shows anger and then immediately wants to be held. The nurse's best interpretation of this behavior is that: a. This is normal behavior for his age. b. This is unusual behavior for his age. c. He is not effectively coping with stress. d. He is showing he needs more attention.

A 13-year-old patient with an ankle injury requires minor surgery. The parents of the patient have given their consent but are unable to wait during the procedure. What is the best nursing action in this context? 1 Persuade the parents to be with the patient. 2 Ask the patient's school teacher to be present. 3 Conduct the surgery when either parent is available. 4 Adhere to the parents' wishes of not participating

4 The nurse should support parents who do not want to be present in their decision and encourage them to remain close by so they can be available to support the child immediately after the procedure. The nurse should not insist that the parents sit through the procedure as it can create tension between the parents and the child. Reaching the patient's school teacher is inappropriate if parents are around and have decided not to stay. Waiting for either parent to sit through the procedure may delay treatment unnecessarily.

The pediatric nurse is working on a project to contribute to research and evidence-based practice. What should the nurse do when caring for patients of different age groups? Arrange the following steps in the correct order. 1. Develop a care plan. 2. Evaluate the effectiveness of intervention 3. Collect information. 4. Identify specific questions.

4, 3, 1, 2 The responses of the nurse to health and illness have to be followed in an order. The nurse has to identify specific questions to collect appropriate information and develop a care plan to implement. Identification of specific questions to be asked to the patient would help to formulate a clear and precise assessment plan. Collection of subjective and objective information of the patient would be helpful in determining the needs of the patient. Developing a care plan of the patient would help in establish the desired outcomes and the interventions required to achieve those outcomes. Finally, the nurse should evaluate the effectiveness of the intervention to determine if the care plan designed for the patient was successful. Test-Taking Tip: In this Question Type, you are asked to prioritize (put in order) the options presented. For example, you might be asked the steps of performing a nursing action or skill such as those involved in medication administration.

The nurse recognizes a delay in the child's developmental pattern and refers the child for further testing. Which characteristics of this child might the nurse have identified? Select all that apply: 1 Dermatological disorders 2 Impaired hearing ability 3 Immunocompromise 4 Improper communication 5 Impaired social interaction

4, 5 Inability to communicate properly or delays in speech may signify developmental delays. When a child is found to have difficulty making friends or has impaired social interactions, the child should be referred for testing. In this condition, the nurse should provide further interventions for the child to support his or her development. Dermatological disorders are assessed in a child to check for genetic abnormalities. Impaired hearing is assessed through testing of the child's auditory function. Impaired hearing does not directly affect a child's behavior. A child who is immunocompromised has a weakened immune system and is at higher risk for infections.

The student nurse is caring for a 10-year-old child in the terminal stages of acute lymphocytic leukemia. Which instructions should the nursing instructor provide to the student nurse when caring for the child? Select all that apply. "You should: 1 Challenge the child to learn ways to manage her own care." 2 Advise the child's parents to pray to God for recovery." 3 "Not involve yourself in the family's emotional distress." 4 "Urge the parents to accept the truth and spend time together." 5 "Encourage the child's parents to ask for help when they need it."

4, 5 The nurse should encourage child's parents to accept the truth and spend the remainder of the time left making memories with the sick child. The nurse should encourage the child's parents to ask for help when they need it and not try to handle everything alone. The child should not be asked to manage her own care. During terminal illness the child needs to depend on her parents and family more than ever. The nurse should not advise the child's parents to pray to God because it is unethical to force religion on a family. The nurse should not get involved in the family's emotional distress. Instead, the nurse should teach them about ways to manage the child's condition.

26. Parents of a 3-year-old child with congenital heart disease are afraid to let their child play with other children because of possible overexertion. The nurse's reply should be based on which of the following? 1. Child needs opportunities to play with peers. 2. Child needs to understand that peers' activities are too strenuous. 3. Parents can meet all the child's needs. 4. Constant parental supervision is needed to avoid overexertion.

ANS: 1 1. The child needs opportunities for social development. Children usually limit their activities if allowed to set their own pace.

A young adolescent boy tells the nurse he "feels gawky." The nurse should explain that this occurs in adolescents because: a. Growth of the extremities and neck precedes growth in other areas. b. Growth is in the trunk and chest. c. The hip and chest breadth increases. d. The growth spurt occurs earlier in boys than it does in girls.

A Growth in length of the extremities and neck precedes growth in other areas, and, because these parts are the first to reach adult length, the hands and feet appear larger than normal during adolescence. Increases in hip and chest breadth take place in a few months, followed several months later by an increase in shoulder width. These changes are followed by increases in length of the trunk and depth of the chest. This sequence of changes is responsible for the characteristic long-legged, gawky appearance of early adolescent children. The growth spurt occurs earlier in girls than in boys.

Which statement is correct about childhood obesity? a. Heredity is an important factor in the development of obesity. b. Childhood obesity in the United States is decreasing. c. Childhood obesity is the result of inactivity. d. Childhood obesity can be attributed to an underlying disease in most cases.

A Heredity is an important fact that contributes to obesity. Identical twins reared apart tend to resemble their biologic parents to a greater extent than their adoptive parents. It is difficult to distinguish between hereditary and environmental factors. The rate of childhood obesity has increased so dramatically that it has now reached epidemic proportions. Inactivity is an important contributing factor; however, obesity is the result of a combination of a number of other factors. Fewer than 5% of all cases of obesity can be linked to underlying disease.

Matt, age 14 years, seems to be always eating, although his weight is appropriate for his height. The best explanation for this is: a. This is normal because of increase in body mass. b. This is abnormal and suggestive of future obesity. c. His caloric intake would have to be excessive. d. He is substituting food for unfilled needs.

A In adolescence, nutritional needs are closely related to the increase in body mass. The peak requirements occur in the years of maximal growth. The caloric and protein requirements are higher than at almost any other time of life. This describes the expected eating pattern for young adolescents as long as weight and height are appropriate; obesity and substitution of food for unfilled needs are not concerns.

A 14-year-old boy and his parents are concerned about bilateral breast enlargement. The nurse's discussion of this should be based on knowing that: a. This is usually benign and temporary. b. This is usually caused by Klinefelter's syndrome. c. Administration of estrogen effectively reduces gynecomastia. d. Administration of testosterone effectively reduces gynecomastia.

A The male breast responds to hormone changes. Some degree of bilateral or unilateral breast enlargement occurs frequently in boys during puberty. This is not a manifestation of Klinefelter's syndrome. Administration of estrogen or testosterone will have no effect on the reduction of breast tissue and may aggravate the condition.

A useful skill that the nurse should expect a 5-year-old child to be able to master is to: a. Tie shoelaces. c. Hammer a nail. b. Use a knife to cut meat. d. Make change from a quarter.

A Tying shoelaces is a fine motor task typical of 5-year-olds. Using a knife to cut meat is a fine motor task of a 7-year-old. Hammering a nail and making change from a quarter are fine motor tasks of an 8- to 9-year-old.

C (explain to the parent that this is not harmful.)

A 2-year-old child has recently started having temper tantrums during which she holds her breath and sometimes faints. The nurse should: A. refer the child for respiratory evaluation. B. refer the child for psychologic evaluation. C. explain to the parent that this is not harmful. D. explain to the parent that the child is spoiled.

A (normal development This indicates normal development. Reflexive grasping occurs during the first 2 to 3 months and then gradually becomes voluntary. No evidence of developmental lag, delayed development, or neurologic dysfunction is present.)

A 3-month-old infant, born at 38 weeks of gestation, will hold a rattle if it is put in her hands; however, she will not voluntarily grasp it. The nurse should interpret this as: a. Normal development. b. Significant developmental lag. c. Slightly delayed development caused by prematurity. d. Suggestive of a neurologic disorder such as cerebral palsy.

C ("When your baby learns to roll over, you must supervise him whenever he is on a surface from which he might fall.")

A 4-month-old infant is brought to the clinic by his parents for a well-baby checkup. What should the nurse include at this time concerning injury prevention? A. "Never shake baby powder directly on your infant because it can be aspirated into his lungs." B. "Do not permit your child to chew paint from window ledges because he might absorb too much lead." C. "When your baby learns to roll over, you must supervise him whenever he is on a surface from which he might fall." D. "Keep doors of appliances closed at all times."

B (Vitamin D)

A mother is bringing her 4-month-old infant into the clinic for a routine well-baby check. The mother is exclusively breastfeeding. There are no other liquids given to the infant. What vitamin does the nurse anticipate the provider will prescribe for this infant? A. Vitamin B B. Vitamin D C. Vitamin C D. Vitamin K

21. The infant with congestive heart failure has a need for: 1. Increased calories. 2. Increased fluids. 3. Decreased protein. 4. Decreased fat.

ANS: 1 1. The metabolic rate of infants with congestive heart failure is greater because of poor cardiac function and increased heart and respiratory rates. Their caloric needs are greater than those of the average infants, yet their ability to take in the calories is diminished by their fatigue.

A, C , E (A. "We will put a plastic fillers in all electrical plugs." C. "we will place a gate at the top and bottom of stairways." E. "we will remove front knobs from the stove." By the time babies reach 6 months of age, they begin to become much more active, curious, and mobile. Putting plastic fillers on all electrical plugs can prevent an electrical shock. Putting gates at the top and bottom of stairways will prevent falls. Removing front knobs from the stove can prevent burns. Poisonous substances should be stored in a locked cabinet, not in a cabinet that children can reach when they begin to climb. The household hot water heater should be turned down to 120 degrees or less.)

A nurse has completed a teaching session for parents about "baby-proofing" the home. Which statements made by the parents indicate an understanding of the teaching (select all that apply)? a. "We will put plastic fillers in all electrical plugs." b. "We will place poisonous substances in a high cupboard." c. "We will place a gate at the top and bottom of stairways." d. "We will keep our household hot water heater at 130 degrees." e. "We will remove front knobs from the stove."

B, C, E (Certain groups of infants are at increased risk for SIDS: those with low birth weight, low Apgar scores, or recent viral illness, and those of male sex. Breastfed infants and infants of average or above-average weight are not at higher risk for SIDS.)

A nurse is conducting education classes for parents of infants. The nurse plans to discuss sudden infant death syndrome (SIDS). Which risk factors should the nurse include as increasing an infant's risk of a SIDS incident (select all that apply)? a. Breastfeeding b. Low Apgar scores c. Male sex d. Birth weight in the 50th or higher percentile e. Recent viral illness

B, C, D (The characteristics of preoperational thinking that occur for the toddler include egocentrism (views everything in relation to self), animism (believes that inert objects are alive), and magical thinking (believes that thinking something causes that event). Concrete thinking is seen in school-age children and ability to reason is seen with adolescents.)

A nurse is planning care for a hospitalized toddler in the preoperational thinking stage. Which characteristics should the nurse expect in this stage (select all that apply)? a. Concrete thinking b. Egocentrism c. Animism d. Magical thinking e. Ability to reason

B, C, E, F (B. Rotavirus (RV) C. Diptheria, tetanus, and acellular pertussis (DTaP) E. Haemophilus influenzae type b (HIB) F. Inactivated poliovirus (IPV) The recommended immunization schedule for a 4-month-old, up to date on immunizations, would be to administer the RV, DTaP, HIB, and IPV vaccinations. The MMR and varicella vaccinations would not be administered until the child is at least 1 year of age.)

A nurse is preparing to administer routine immunizations to a 4-month-old infant. The infant is currently up to date on all previously recommended immunizations. Which immunizations will the nurse prepare to administer (select all that apply)? a. Measles, mumps, and rubella (MMR) b. Rotavirus (RV) c. Diphtheria, tetanus, and acellular pertussis (DTaP) d. Varicella e. Haemophilus influenzae type b (HIB) f. Inactivated poliovirus (IPV)

B (Eliminate all secondhand smoke contact. To prevent and treat colic, teach parents that if household members smoke, they should avoid smoking near the infant; smoking activity should preferably be confined to outside of the home. A pacifier can be introduced for added sucking. The infant should be swaddled tightly with a soft, stretchy blanket and placed in an upright seat after feedings.)

A nurse is teaching parents about prevention and treatment of colic. Which should the nurse include in the teaching plan? a. Avoid use of pacifiers. b. Eliminate all secondhand smoke contact. c. Lay infant flat after feeding. d. Avoid swaddling the infant.

A (The nurse is using the toddler's inability to conserve. This is when the toddler is unable to understand the idea that a mass can be changed in size, shape, volume, or length without losing or adding to the original mass. Instead, toddlers judge what they see by the immediate perceptual clues given to them. A small glass means less amount of contrast. Magical thinking is believing that thoughts are all-powerful and can cause events. Centration is focusing on one aspect rather than considering all possible alternatives. Irreversibility is the inability to undo or reverse the actions initiated, such as being unable to stop doing an action when told.)

A nurse places some x-ray contrast the toddler is to drink in a small cup instead of a large cup. Which concept of a toddler's preoperational thinking is the nurse using? a. Inability to conserve b. Magical thinking c. Centration d. Irreversibility

C (6 months Between ages 6 and 8 months, fear of strangers and stranger anxiety become prominent and are related to the infant's ability to discriminate between familiar and nonfamiliar people. At age 2 months, the infant is just beginning to respond differentially to the mother. At age 4 months, the infant is beginning the process of separation individuation when the infant begins to recognize self and mother as separate beings. Twelve months is too late and requires referral for evaluation if the infant does not fear strangers at this age.)

A parent asks the nurse "At what age do most babies begin to fear strangers?" The nurse responds that most infants begin to fear strangers at age: a. 2 months. b. 4 months. c. 6 months d. 12 months.

D (The nurse should suggest to the parent that questions should be phrased with realistic choices rather than "yes" or "no" answers. This provides a sense of control for the toddler and reduces the opportunity for negativism. Negativism is not an indication of stubbornness or insolence and should not be punished. The negativism is not a function of attention; the child is testing limits to gain an understanding of the world. The toddler is too young to be asked to not always say "no.")

A parent asks the nurse about how to respond to negativism in toddlers. The most appropriate recommendation is to: a. Punish the child. b. Provide more attention. c. Ask child not always to say "no." d. Reduce the opportunities for a "no" answer.

D (The acellular pertussis vaccine is recommended by the American Academy of Pediatrics beginning at age 6 weeks. Infants are at greater risk for complications of pertussis. The vaccine is not given after age 7 years, when the risks of the vaccine become greater than those of pertussis. The infant is highly susceptible to pertussis, which can be a life-threatening illness in this age-group.)

A parent asks the nurse whether her infant is susceptible to pertussis. The nurse's response should be based on which statement concerning susceptibility to pertussis? a. Neonates will be immune the first few months. b. If the mother has had the disease, the infant will receive passive immunity. c. Children younger than 1 year seldom contract this disease. d. Most children are highly susceptible from birth.

A (Encourage parent to verbalize feelings. Colic is multifactorial, and no single treatment is effective for all infants. The parent is verbalizing concern and worry. The nurse should allow the parent to put these feelings into words. An empathic, gentle, and reassuring attitude, in addition to suggestions about remedies, will help alleviate the parent's anxieties. The nurse should reassure the parent that he or she is not doing anything wrong. The infant with colic is experiencing spasmodic pain that is manifested by loud crying, in some cases up to 3 hours each day. Telling the parent that it will eventually go away does not help him or her through the current situation.)

A parent of an infant with colic tells the nurse, "All this baby does is scream at me; it is a constant worry." The nurse's best action is: a. Encourage parent to verbalize feelings. b. Encourage parent not to worry so much. c. Assess parent for other signs of inadequate parenting. d. Reassure parent that colic rarely lasts past age 9 months.

C (The parent should be told that the best way to deal with temper tantrums is to ignore the behaviors, provided that the actions are not dangerous to the child. Tantrums are common during this age-group as the child becomes more independent and increasingly complex tasks overwhelm him or her. The parents and caregivers need to have consistent and developmentally appropriate expectations. Punishment and explanations will not be beneficial.)

A toddler's parent asks the nurse for suggestions on dealing with temper tantrums. The most appropriate recommendation is to: a. Punish the child. b. Leave the child alone until the tantrum is over. c. Ignore the behavior, provided that it is not injurious. d. Explain to child that this is wrong.

Injuries claim many lives during adolescence. Which factors contribute to early adolescents engaging in risk-taking behaviors (select all that apply)? a. Peer pressure b. A desire to master their environment c. Engagement in the process of separation from their parents d. A belief that they are invulnerable e. Impulsivity

A, D, E Peer pressure (including impressing peers) is a factor contributing to adolescent injuries. During early to middle adolescence, children feel that they are exempt from the consequences of risk-taking behaviors; they believe that negative consequences only happen to others. Feelings of invulnerability ("It can't happen to me") are evident in adolescence. Impulsivity places adolescents in unsafe situations. Mastering the environment is the task of young school-age children. Emancipation is a major issue for the older adolescent. The process is accomplished as the teenager gains an education or vocational training.

A 6-year-old child is having a generalized seizure in the classroom at school. Place in order the interventions the school nurse should implement starting with the highest-priority intervention sequencing to the lowest-priority intervention. Provide answer using lowercase letters separated by commas (e.g., a, b, c, d, e). a. Take vital signs. b. Ease child to the floor. c. Allow child to rest. d. Turn child to the side. e. Integrate child back into the school environment.

ANS: b, d, a, c, e The nurse should ease the child to the floor immediately during a generalized seizure. During (and sometimes after) the generalized seizure, the swallowing reflex is lost, salivation increases, and the tongue is hypotonic. Therefore, the child is at risk for aspiration and airway occlusion. Placing the child on the side facilitates drainage and helps maintain a patent airway. Vital signs should be taken next and the child should be allowed to rest. When feasible, the child is integrated into the environment as soon as possible.

15. A beneficial effect of administering digoxin (Lanoxin) is that it: 1. Decreases edema. 2. Decreases cardiac output. 3. Increases heart size. 4. Increases venous pressure.

ANS: 1 1. Digoxin has a rapid onset and is useful increasing cardiac output, decreasing venous pressure, and as a result, decreasing edema.

36. Which of the following is a major clinical manifestation of rheumatic fever? 1. Polyarthritis 2. Osler nodes 3. Janeway spots 4. Splinter hemorrhages of distal third of nails

ANS: 1 1. Polyarthritis, which is swollen, hot, red, and painful joints. The affected joints will change every 1 to 2 days. Primarily the large joints are affected.

11. Which of the following structural defects constitute tetralogy of Fallot? 1. Pulmonary stenosis, ventricular septal defect, overriding aorta, right ventricular hypertrophy 2. Aortic stenosis, ventricular septal defect, overriding aorta, right ventricular hypertrophy 3. Aortic stenosis, atrial septal defect, overriding aorta, left ventricular hypertrophy 4. Pulmonary stenosis, ventricular septal defect, aortic hypertrophy, left ventricular hypertrophy

ANS: 1 1. Tetralogy of Fallot has these four characteristics: pulmonary stenosis, ventricular septal defect, overriding aorta, and right ventricular hypertrophy.

8. Which of the following should be included in the instructions to an active adolescent who is going home after a cardiac catheterization? 1. Avoid tub baths but may shower. 2. Maintain strict bed rest for 3 days. 3. Leave pressure dressing on for 7 days. 4. Stay home from school until Band-Aid is removed.

ANS: 1 1. The catheterization site should be kept relatively dry with a adhesive bandage. Showers are recommended.

27. Which of the following should the nurse consider when preparing a school-age child and the family for heart surgery? 1. Unfamiliar equipment should not be shown. 2. Let child hear the sounds of an ECG monitor. 3. Avoid mentioning postoperative discomfort and interventions. 4. Explain that an endotracheal tube will not be needed if the surgery goes well.

ANS: 2 2. The child and family should be exposed to the sights and sounds of the intensive care unit (ICU). All positive, nonfrightening aspects of the environment are emphasized

24. An 8-month-old infant has a hypercyanotic spell while blood is being drawn. The nurse's first action should be which of the following? 1. Assess for neurologic defects. 2. Place the child in the knee-chest position 3. Begin cardiopulmonary resuscitation. 4. Prepare family for imminent death.

ANS: 2 2. The first action is to place the infant in the knee-chest position. Blow-by oxygen may be indicated.

34. The primary nursing intervention to prevent bacterial endocarditis is which of the following? 1. Institute measures to prevent dental procedures. 2. Counsel parents of high-risk children about prophylactic antibiotics. 3. Observe children for complications, such as embolism and heart failure. 4. Encourage restricted mobility in susceptible children.

ANS: 2 2. The objective of nursing care is to counsel the parents of high-risk children about both the need for prophylactic antibiotics for dental procedures and the necessity of maintaining excellent oral health. The child's dentist should be aware of the child's cardiac condition.

40. Which of the following is the leading cause of death after heart transplantation? 1. Infection 2. Rejection 3. Cardiomyopathy 4. Congestive heart failure

ANS: 2 2. The posttransplant course is complex. The leading cause of death after cardiac transplant is rejection.

19. Which of the following is a common sign of digoxin toxicity? 1. Seizures 2. Vomiting 3. Bradypnea 4. Tachycardia

ANS: 2 2. Vomiting is a common sign of digoxin toxicity

1. A chest radiograph film is ordered for a child with suspected cardiac problems. The child's parent asks the nurse, "What will the radiograph show about the heart?" The nurse's response should be based on knowledge that the x-ray film will do which of the following? 1. Show bones of chest but not the heart 2. Measure electrical potential generated from heart muscle 3. Provide permanent record of heart size and configuration 4. Provide computerized image of heart vessels and tissues

ANS: 3 3. A chest radiograph will provide information on the heart size and pulmonary blood-flow patterns. It will be provide a baseline for future comparisons

30. The nurse is caring for a child after heart surgery. Which of the following should the nurse do if evidence is found of cardiac tamponade? 1. Increase analgesia. 2. Apply warming blankets. 3. Immediately report this to physician. 4. Encourage child to cough, turn, and breathe deeply.

ANS: 3 3. If evidence is noted of cardiac tamponade, blood or fluid in the pericardial space constricting the heart, the physician is notified immediately of this life-threatening complication

29. Which of the following is an important nursing consideration when suctioning a young child who has had heart surgery? 1. Perform suctioning at least every hour. 2. Suction for no longer than 30 seconds at a time. 3. Administer supplemental oxygen before and after suctioning. 4. Expect symptoms of respiratory distress when suctioning.

ANS: 3 3. If suctioning is indicated, supplemental oxygen is administered with a manual resuscitation bag before and after the procedure to prevent hypoxia

18. A 6-month-old infant is receiving digoxin (Lanoxin). The nurse should notify the practitioner and withhold the medication if the apical pulse is less than which of the following? 1. 60 2. 70 3. 90 to 110 4. 110 to 120

ANS: 3 3. If the 1-minute apical is below 90 to 110, the digoxin should not be given to a 6-month-old.

31. Which of the following is an important nursing consideration when chest tubes will be removed from a child? 1. Explain that it is not painful. 2. Explain that only a Band-Aid will be needed 3. Administer analgesics before procedure. 4. Expect bright red drainage for several hours after removal.

ANS: 3 3. It is appropriate to prepare the child for the removal of chest tubes with analgesics. Short-acting medications can be used that are administered through an existing IV line

38. Which of the following actions by the school nurse is important in the prevention of rheumatic fever? 1. Encourage routine cholesterol screenings. 2. Conduct routine blood pressure screenings. 3. Refer children with sore throats for throat cultures. 4. Recommend salicylates instead of acetaminophen for minor discomforts.

ANS: 3 3. Nurses have a role in prevention; primarily in screening school-age children for sore throats caused by group A streptococci. This can be by actively participating in throat culture screening or by referring children with possible streptococcal sore throats for testing.

10. Which of the following defects results in increased pulmonary blood flow? 1. Pulmonic stenosis 2. Tricuspid atresia 3. Atrial septal defect 4. Transposition of the great arteries

ANS: 3 3. The atrial septal defect results in increased pulmonary blood flow. Blood flows from the left atrium (higher pressure) into the right atrium (lower pressure) and then to the lungs via the pulmonary artery.

13. Which of the following is best described as the inability of the heart to pump an adequate amount of blood to the systemic circulation at normal filling pressures? 1. Pulmonary congestion 2. Congenital heart defect 3. Congestive heart failure 4. Systemic venous congestion

ANS: 3 3. The definition of congestive heart failure is the inability of the heart to pump an adequate amount of blood to the systemic circulation at normal filling pressures to meet the metabolic demands of the body.

35. Which of the following is a common, serious complication of rheumatic fever? 1. Seizures 2. Cardiac arrhythmias 3. Pulmonary hypertension 4. Cardiac valve damage

ANS: 4 4. Cardiac valve damage is the most significant complication of rheumatic fever.

Which clinical manifestations would suggest hydrocephalus in a neonate? a. Bulging fontanel and dilated scalp veins b. Closed fontanel and high-pitched cry c. Constant low-pitched cry and restlessness d. Depressed fontanel and decreased blood pressure

ANS: A Bulging fontanels, dilated scalp veins, and separated sutures are clinical manifestations of hydrocephalus in neonates. Closed fontanel and high-pitched cry, constant low-pitched cry and restlessness, and depressed fontanel and decreased blood pressure are not clinical manifestations of hydrocephalus, but all should be referred for evaluation.

20. The parents of a young child with congestive heart failure tell the nurse that they are "nervous" about giving digoxin. The nurse's response should be based on which of the following? 1. It is a very safe, frequently used drug. 2. It is difficult to either overmedicate or undermedicate with digoxin. 3. Parents lack the expertise necessary to administer digoxin. 4. Parents must learn specific, important guidelines for administration of digoxin.

ANS: 4 4. Digoxin has a narrow therapeutic range. The margin of safety between therapeutic, toxic, and lethal doses is very small. Specific guidelines are available for parents to learn how to administer the drug safely and to monitor for side effects

39. When discussing hyperlipidemia with a group of adolescents, the nurse should explain that high levels of what are thought to protect against cardiovascular disease? 1. Cholesterol 2. Triglycerides 3. Low-density lipoproteins (LDLs) 4. High-density lipoproteins (HDLs)

ANS: 4 4. HDLs contain very low concentrations of triglycerides, relatively little cholesterol, and high levels of proteins. It is thought that HDLs protect against cardiovascular disease.

41. When caring for the child with Kawasaki disease, the nurse should know which of the following? 1. Child's fever is usually responsive to antibiotics within 48 hours. 2. Principal area of involvement is the joints. 3. Aspirin is contraindicated. 4. Therapeutic management includes administration of gamma globulin and aspirin.

ANS: 4 4. High-dose IV gamma globulin and aspirin therapy is indicated to reduce the incidence of coronary artery abnormalities when given within the first 10 days of the illness

4. Which of the following explanations regarding cardiac catheterization is appropriate for a preschool child? 1. Postural drainage will be performed every 4 to 6 hours after the test. 2. It is necessary to be completely "asleep" during the test. 3. The test is very short, usually taking less than 1 hour. 4. When the procedure is done, you will have to keep your leg straight for at least 4 hours.

ANS: 4 4. The child's leg will have to be maintained in a straight position for approximately 4 hours. Younger children can be held in the parent's lap with the leg maintained in the correct position

9. Surgical closure of the ductus arteriosus would do which of the following? 1. Stop the loss of unoxygenated blood to the systemic circulation 2. Decrease the edema in legs and feet 3. Increase the oxygenation of blood 4. Prevent the return of oxygenated blood to the lungs

ANS: 4 4. The ductus arteriosus allows blood to flow from the higher-pressure aorta to the lower-pressure pulmonary artery, causing a right-to-left shunt. If this is surgically closed, no additional oxygenated blood (from the aorta) will return to the lungs through the pulmonary artery

Meperidine (Demerol) is not recommended for children in sickle cell crisis because it: a. may induce seizures. b. is easily addictive. c. is not adequate for pain relief. d. is given by intramuscular injection.

ANS: A A metabolite of meperidine, normeperidine, is a central nervous system stimulant that produces anxiety, tremors, myoclonus, and generalized seizures when it accumulates with repetitive dosing. Patients with sickle cell disease are particularly at risk for normeperidine-induced seizures. Meperidine is no more addictive than other narcotic agents. Meperidine is adequate for pain relief. It is available for IV infusion.

A young child with leukemia has anorexia and severe stomatitis. The nurse should suggest that the parents try which intervention? a. Relax any eating pressures. b. Firmly insist that child eat normally. c. Begin gavage feedings to supplement diet. d. Serve foods that are either hot or cold.

ANS: A A multifaceted approach is necessary for children with severe stomatitis and anorexia. First, the parents should relax eating pressures. The nurse should suggest that the parents try soft, bland foods; normal saline or bicarbonate mouthwashes; and local anesthetics. The stomatitis is a temporary condition. The child can resume good food habits as soon as the condition resolves.

In which of the conditions are all the formed elements of the blood simultaneously depressed? a. Aplastic anemia b. Sickle cell anemia c. Thalassemia major d. Iron deficiency anemia

ANS: A Aplastic anemia refers to a bone marrowfailure condition in which the formed elements of the blood are simultaneously depressed. Sickle cell anemia is a hemoglobinopathy in which normal adult hemoglobin is partly or completely replaced by abnormal sickle hemoglobin. Thalassemia major is a group of blood disorders characterized by deficiency in the production rate of specific hemoglobin globin chains. Iron deficiency anemia results in a decreased amount of circulating red cells.

Olivia, age 5 years, tells the nurse that she needs a Band-Aid where she had an injection. The best nursing action is to: a. Apply a Band-Aid. b. Ask her why she wants a Band-Aid. c. Explain why a Band-Aid is not needed. d. Show her that the bleeding has already stopped.

ANS: A Children in this age-group still fear that their insides may leak out at the injection site, even if the bleeding has stopped. Provide the Band-Aid. No explanation should be required.

The nurse is administering an IV chemotherapeutic agent to a child with leukemia. The child suddenly begins to wheeze and have severe urticaria. Which is the most appropriate nursing action? a. Stop drug infusion immediately. b. Recheck rate of drug infusion. c. Observe child closely for next 10 minutes. d. Explain to child that this is an expected side effect.

ANS: A If an allergic reaction is suspected, the drug should be immediately discontinued. Any drug in the line should be withdrawn, and a normal saline infusion begun to keep the line open. Rechecking the rate of drug infusion, observing the child closely for next 10 minutes, and explaining to the child that this is an expected side effect can all be done after the drug infusion is stopped and the child is evaluated.

A previously potty-trained 30-month-old child has reverted to wearing diapers while hospitalized. The nurse should reassure the parents that this is normal because: a. Regression is seen during hospitalization. b. Developmental delays occur because of the hospitalization. c. The child is experiencing urinary urgency because of hospitalization. d. The child was too young to be potty-trained.

ANS: A Regression is expected and normal for all age-groups when hospitalized. Nurses should assure the parents this is temporary and the child will return to the previously mastered developmental milestone when back home. This does not indicate a developmental delay. The child should not be experiencing urinary urgency because of hospitalization and this would not be normal. Successful potty-training can be started at 2 years of age if the child is ready.

Latasha, age 8 years, is being admitted to the hospital from the emergency department with an injury from falling off her bicycle. What will help her most in her adjustment to the hospital? a. Explain hospital schedules such as mealtimes. b. Use terms such as honey and dear to show a caring attitude. c. Explain when parents can visit and why siblings cannot come to see her. d. Orient her parents, because she is young, to her room and hospital facility.

ANS: A School-age children need to have control of their environment. The nurse should offer explanations or prepare the child for experiences that are unavoidable. The nurse should refer to the child by the preferred name. Telling the child about all of the limitations of visiting does not help her adjust to the hospital. At the age of 8 years, the child and parents should be oriented to the environment.

The nurse is conducting a staff in-service on inherited childhood blood disorders. Which statement describes severe combined immunodeficiency syndrome (SCIDS)? a. There is a deficit in both the humoral and cellular immunity with this disease. b. Production of red blood cells is affected with this disease. c. Adult hemoglobin is replaced by abnormal hemoglobin in this disease. d. There is a deficiency of T and B lymphocyte production with this disease.

ANS: A Severe combined immunodeficiency syndrome (SCIDS) is a genetic disorder that results in deficits of both humoral and cellular immunity. Wiskott-Aldrich is an X-linked recessive disorder with selected deficiencies of T and B lymphocytes. Fanconi syndrome is a hereditary disorder of red cell production. Sickle cell disease is characterized by the replacement of adult hemoglobin with an abnormal hemoglobin S.

When both parents have sickle cell trait, which is the chance their children will have sickle cell anemia? a. 25% b. 50% c. 75% d. 100%

ANS: A Sickle cell anemia is inherited in an autosomal recessive pattern. If both parents have sickle cell trait (one copy of the sickle cell gene), then for each pregnancy, a 25% chance exists that their child will be affected with sickle cell disease. With each pregnancy, a 50% chance exists that the child will have sickle cell trait. Percentages of 75% and 100% are too high for the children of parents who have sickle cell trait.

Parents of a child with sickle cell anemia ask the nurse, What happens to the hemoglobin in sickle cell anemia? Which statement by the nurse explains the disease process? a. Normal adult hemoglobin is replaced by abnormal hemoglobin. b. There is a lack of cellular hemoglobin being produced. c. There is a deficiency in the production of globulin chains. d. The size and depth of the hemoglobin are affected.

ANS: A Sickle cell anemia is one of a group of diseases collectively called hemoglobinopathies, in which normal adult hemoglobin is replaced by abnormal hemoglobin. Aplastic anemia is a lack of cellular elements being produced. Thalassemia major refers to a variety of inherited disorders characterized by deficiencies in production of certain globulin chains. Iron deficiency anemia affects the size and depth of the color.

The school nurse is informed that a child with human immunodeficiency virus (HIV) will be attending school soon. Which is an important nursing intervention? a. Carefully follow universal precautions. b. Determine how the child became infected. c. Inform the parents of the other children. d. Reassure other children that they will not become infected.

ANS: A Universal precautions are necessary to prevent further transmission of the disease. It is not the role of the nurse to determine how the child became infected. Informing the parents of other children and reassuring children that they will not become infected is a violation of the childs right to privacy.

When caring for the child with Reye syndrome, the priority nursing intervention should be to: a. monitor intake and output. b. prevent skin breakdown. c. observe for petechiae. d. do range-of-motion exercises.

ANS: A Accurate and frequent monitoring of intake and output is essential for adjusting fluid volumes to prevent both dehydration and cerebral edema. Preventing skin breakdown, observing for petechiae, and doing range-of-motion exercises are important interventions in the care of a critically ill or comatose child. Careful monitoring of intake and output is a priority.

The nurse is caring for an adolescent with osteosarcoma being admitted to undergo chemotherapy. The adolescent had a right above-the-knee amputation 2 months ago and has been experiencing "phantom limb pain." Which prescribed medication is appropriate to administer to relieve phantom limb pain? a. Amitriptyline (Elavil) b. Hydrocodone (Vicodin) c. Oxycodone (OxyContin) d. Alprazolam (Xanax)

ANS: A Amitriptyline (Elavil) has been used successfully to decrease phantom limb pain. Opioids such as Vicodin or OxyContin would not be prescribed for this pain. A benzodiazepine, Xanax, would not be prescribed for this type of pain.

23. A child has an avulsed (knocked-out) tooth. In which medium should the nurse instruct the parents to place the tooth for transport to the dentist? a. Cold milk b. Cold water c. Warm salt water d. A dry, clean jar

ANS: A An avulsed tooth should be placed in a suitable medium for transport, either cold milk or saliva (under the child's or parent's tongue). Cold milk is a more suitable medium for transport than cold water, warm salt water, or a dry, clean jar.

39. An 18-month-old child is seen in the clinic with AOM. Trimethoprim-sulfamethoxazole (Bactrim) is prescribed. Which statement made by the parent indicates a correct understanding of the instructions? a. I should administer all the prescribed medication. b. I should continue medication until the symptoms subside. c. I will immediately stop giving medication if I notice a change in hearing. d. I will stop giving medication if fever is still present in 24 hours

ANS: A Antibiotics should be given for their full course to prevent recurrence of infection with resistant bacteria. Symptoms may subside before the full course is given. Hearing loss is a complication of AOM. Antibiotics should continue to be given. Medication may take 24 to 48 hours to make symptoms subside. It should be continued

43. A child with cystic fibrosis (CF) receives aerosolized bronchodilator medication. When should this medication be administered? a. Before chest physiotherapy (CPT) c. Before receiving 100% oxygen b. After CPT d. After receiving 100% oxygen

ANS: A Bronchodilators should be given before CPT to open bronchi and make expectoration easier. Aerosolized bronchodilator medications are not helpful when used after CPT. Oxygen administration is necessary only in acute episodes with caution because of chronic carbon dioxide retention

17. The ability to mentally understand that 1 + 3 = 4 and 4 − 3 = 1 occurs in which stage of cognitive development? a. Concrete operations stage b. Formal operations stage c. Intuitive thought stage d. Preoperations stage

ANS: A By 7 to 8 years of age, the child is able to retrace a process (reversibility) and has the skills necessary for solving mathematical problems. This stage is called concrete operations. The formal operations stage deals with abstract reasoning and does not occur until adolescence. Thinking in the intuitive stage is based on immediate perceptions. A child in this stage often solves problems by random guessing. In preoperational thinking, the child is usually able to add 1 + 3 = 4 but is unable to retrace the process.

6. An 8-year-old girl tells the nurse that she has cancer because God is punishing her for "being bad." She shares her concern that, if she dies, she will go to hell. How should the nurse interpret this belief? a. It is a belief common at this age. b. It is a belief that forms the basis for most religions. c. The belief is suggestive of excessive family pressure. d. The statement suggests a failed attempt to develop a conscience.

ANS: A Children at this age may view illness or injury as a punishment for a real or imagined mystique. The belief in divine punishment is common at this age.

34. A nurse caring for a child receiving chemotherapy notes that the child's urine specific gravity is 1.010. Which action by the nurse is the most appropriate? A. Document the findings in the child's chart. B. Increase the rate of the IV fluids per protocol. C. Notify the provider about the laboratory results. D. Prepare to administer an alkalizing agent.

ANS: A Children on chemotherapy should remain well hydrated to ensure the medications and any toxic by-products are flushed out. The urine specific gravity should remain at 1.012 or below. The nurse needs to take no further action after documenting the findings. The IV rate should be increased if the specific gravity is above that level. The provider does not need to be notified specifically about this normal finding. An alkalizing agent is not needed.

28. In providing nourishment for a child with cystic fibrosis (CF), which factor should the nurse keep in mind? a. Diet should be high in carbohydrates and protein. b. Diet should be high in easily digested carbohydrates and fats. c. Most fruits and vegetables are not well tolerated. d. Fats and proteins must be greatly curtailed.

ANS: A Children with CF require a well-balanced, high-protein, high-calorie diet because of impaired intestinal absorption. Enzyme supplementation helps digest foods; other modifications are not necessary. A wellbalanced diet containing fruits and vegetables is important. Fats and proteins are a necessary part of a wellbalanced diet.

10. An infants parents ask the nurse about preventing otitis media (OM). What should the nurse recommend? a. Avoid tobacco smoke. b. Use nasal decongestant. c. Avoid children with OM. d. Bottle-feed or breastfeed in supine position.

ANS: A Eliminating tobacco smoke from the childs environment is essential for preventing OM and other common childhood illnesses. Nasal decongestants are not useful in preventing OM. Children with uncomplicated OM are not contagious unless they show other upper respiratory infection symptoms. Children should be fed in an upright position to prevent OM

31. An appropriate nursing intervention when caring for a child with pneumonia is to: a. Encourage rest. b. Encourage the child to lie on the unaffected side. c. Administer analgesics. d. Place the child in the Trendelenburg position

ANS: A Encouraging rest by clustering care and promoting a quiet environment is the best intervention for a child with pneumonia. Lying on the affected side may promote comfort by splinting the chest and reducing pleural rubbing. Analgesics are not indicated. Children should be placed in a semi-erect position or position of comfort

42. A nurse is conducting an in-service on asthma. Which statement is the most descriptive of bronchial asthma? a. There is heightened airway reactivity. b. There is decreased resistance in the airway. c. The single cause of asthma is an allergic hypersensitivity. d. It is inherited

ANS: A In bronchial asthma, spasm of the smooth muscle of the bronchi and bronchioles causes constriction, producing impaired respiratory function. In bronchial asthma, there is increased resistance in the airway. There are multiple causes of asthma, including allergens, irritants, exercise, cold air, infections, medications, medical conditions, and endocrine factors. Atopy or development of an immunoglobulin E (IgE)mediated response is inherited but is not the only cause of asthma.

The nurse is teaching nursing students about childhood nervous system tumors. Which best describes a neuroblastoma? a. Diagnosis is usually made after metastasis occurs. b. Early diagnosis is usually possible because of the obvious clinical manifestations. c. It is the most common brain tumor in young children. d. It is the most common benign tumor in young children.

ANS: A Neuroblastoma is a silent tumor with few symptoms. In more than 70% of cases, diagnosis is made after metastasis occurs, with the first signs caused by involvement in the nonprimary site. In only 30% of cases is diagnosis made before metastasis. Neuroblastomas are the most common malignant extracranial solid tumors in children. The majority of tumors develop in the adrenal glands or the retroperitoneal sympathetic chain. They are not benign but metastasize.

5. Which statement characterizes moral development in older school-age children? a. They are able to judge an act by the intentions that prompted it rather than just by the consequences. b. Rules and judgments become more absolute and authoritarian. c. They view rule violations in an isolated context. d. They know the rules but cannot understand the reasons behind them.

ANS: A Older school-age children are able to judge an act by the intentions that prompted the behavior rather than just by the consequences. Rules and judgments become less absolute and authoritarian. Rule violation is likely to be viewed in relation to the total context in which it appears. Both the situation and the morality of the rule itself influence reactions.

A nurse is conducting a staff in-service on childhood cancers. Which is the primary site of osteosarcoma? a. Femur b. Humerus c. Pelvis d. Tibia

ANS: A Osteosarcoma is the most frequently encountered malignant bone cancer in children. The peak incidence is between ages 10 and 25 years. More than half occur in the femur. After the femur, most of the remaining sites are the humerus, tibia, pelvis, jaw, and phalanges.

An adolescent boy is brought to the emergency department after a motorcycle accident. His respirations are deep, periodic, and gasping. There are extreme fluctuations in blood pressure. Pupils are dilated and fixed. The nurse should suspect which type of head injury? a. Brainstem b. Skull fracture c. Subdural hemorrhage d. Epidural hemorrhage

ANS: A Signs of brainstem injury include deep, rapid, periodic or intermittent, and gasping respirations. Wide fluctuations or noticeable slowing of the pulse, widening pulse pressure, or extreme fluctuations in blood pressure are consistent with a brainstem injury. Skull fracture, subdural hemorrhage, and epidural hemorrhage are not consistent with brainstem injuries.

A child is upset because, when the cast is removed from her leg, the skin surface is caked with desquamated skin and sebaceous secretions. Which should the nurse suggest to remove this material? a. Soak in a bathtub. b. Vigorously scrub leg. c. Apply powder to absorb material. d. Carefully pick material off leg.

ANS: A Simple soaking in the bathtub is usually sufficient for the removal of the desquamated skin and sebaceous secretions. It may take several days to eliminate the accumulation completely. The parents and child should be advised not to scrub the leg vigorously or forcibly remove this material because it may cause excoriation and bleeding. Oil or lotion, but not powder, may provide comfort for the child.

A young girl has just injured her ankle at school. In addition to calling the child's parents, the most appropriate, immediate action by the school nurse is to: a. apply ice. b. observe for edema and discoloration. c. encourage child to assume a position of comfort. d. obtain parental permission for administration of acetaminophen or aspirin.

ANS: A Soft-tissue injuries should be iced immediately. In addition to ice, the extremity should be rested, be elevated, and have compression applied. Observing for edema and discoloration, encouraging the child to assume a position of comfort, and obtaining parental permission for administration of acetaminophen or aspirin are not immediate priorities. The application of ice can reduce the severity of the injury.

30. Abdominal thrusts (the Heimlich maneuver) are recommended for airway obstruction in children older than: a. 1 year. c. 8 years. b. 4 years. d. 12 years.

ANS: A The Heimlich maneuver is recommended for airway obstruction in children older than 1 year. In children younger than 1 year, back blows and chest thrusts are administered.

Four-year-old David is placed in Buck extension traction for Legg-Calvé-Perthes disease. He is crying with pain as the nurse assesses that the skin of his right foot is pale with an absence of pulse. Which action should the nurse take first? a. Notify the practitioner of the changes noted. b. Give the child medication to relieve the pain. c. Reposition the child and notify physician. d. Chart the observations and check the extremity again in 15 minutes.

ANS: A The absence of a pulse and change in color of the foot must be reported immediately for evaluation by the practitioner. Pain medication should be given after the practitioner is notified. Legg-Calvé-Perthes disease is an emergency condition; immediate reporting is indicated. The findings should be documented with ongoing assessment.

. A toddler fell out of a second-story window. She had a brief loss of consciousness and vomited four times. Since admission, she has been alert and oriented. Her mother asks why a computed tomography (CT) scan is required when she "seems fine." Which explanation should the nurse give? a. Your child may have a brain injury and the CT can rule one out. b. The CT needs to be done because of your child's age. c. Your child may start to have seizures and a baseline CT should be done. d. Your child probably has a skull fracture and the CT can confirm this diagnosis.

ANS: A The child's history of the fall, brief loss of consciousness, and vomiting four times necessitates evaluation of a potential brain injury. The severity of a head injury may not be apparent on clinical examination but will be detectable on a CT scan. The need for the CT scan is related to the injury and symptoms, not the child's age. The CT scan is necessary to determine whether a brain injury has occurred.

The nurse is taking care of a child who is alert but showing signs of increased intracranial pressure. Which test is contraindicated in this case? a. Oculovestibular response b. Doll's head maneuver c. Funduscopic examination for papilledema d. Assessment of pyramidal tract lesions

ANS: A The oculovestibular response (caloric test) involves the instillation of ice water into the ear of a comatose child. The caloric test is painful and is never performed on an awake child or one who has a ruptured tympanic membrane. Doll's head maneuver, funduscopic examination for papilledema, and assessment of pyramidal tract lesions can be performed on awake children.

The nurse is taking care of an adolescent with osteosarcoma. The parents ask the nurse about treatment. The nurse should make which accurate response about treatment for osteosarcoma? a. Treatment usually consists of surgery and chemotherapy. b. Amputation of affected extremity is rarely necessary. c. Intensive irradiation is the primary treatment. d. Bone marrow transplantation offers the best chance of long-term survival.

ANS: A The optimal therapy for osteosarcoma is a combination of surgery and chemotherapy. Intensive irradiation and bone marrow transplantation are usually not part of the therapeutic management.

The nurse should monitor for which effect on the cardiovascular system when a child is immobilized? a. Venous stasis b. Increased vasopressor mechanism c. Normal distribution of blood volume d. Increased efficiency of orthostatic neurovascular reflexes

ANS: A The physiologic effects of immobilization, as a result of decreased muscle contraction, include venous stasis. This can lead to pulmonary emboli or thrombi. A decreased vasopressor mechanism results in orthostatic hypotension, syncope, hypotension, decreased cerebral blood flow, and tachycardia. An altered distribution of blood volume is found with decreased cardiac workload and exercise tolerance. Immobilization causes a decreased efficiency of orthostatic neurovascular reflexes with an inability to adapt readily to the upright position and with pooling of blood in the extremities in the upright position.

1.Which diagnostic test allows visualization of the renal parenchyma and renal pelvis without exposure to external beam radiation or radioactive isotopes? a. Renal ultrasound b. Computed tomography c. Intravenous pyelography d. Voiding cystourethrography

ANS: A The transmission of ultrasonic waves through the renal parenchyma allows visualization of the renal parenchyma and renal pelvis without exposure to external beam radiation or radioactive isotopes. Computed tomography uses external radiation, and sometimes contrast media are used. Intravenous pyelography uses contrast medium and external radiation for x-ray films. Contrast medium is injected into the bladder through the urethral opening for voiding cystourethrography. External radiation for x-ray films is used before, during, and after voiding.

9. A young child diagnosed with leukemia is experiencing anorexia and severe stomatitis. The nurse should suggest that the parents try which intervention? a. Relax any eating pressures b. Firmly insist that child eat normally c. Begin gavage feedings to supplement diet d. Serve foods that are either hot or cold

ANS: A A multifaceted approach is necessary for children with severe stomatitis and anorexia. First, the parents should relax eating pressures. The nurse should suggest that the parents try soft, bland foods; normal saline or bicarbonate mouthwashes; and local anesthetics. The stomatitis is a temporary condition. The child can resume good food habits as soon as the condition resolves.

34. The nurse is implementing care for a school-age child admitted to the pediatric intensive care experiencing symptomology associated with diabetic ketoacidosis (DKA). Which prescribed intervention should the nurse implement first? a. Begin 0.9% saline solution intravenously as prescribed. b. Administer regular insulin intravenously as prescribed. c. Place child on a cardiac monitor. d. Place child on a pulse oximetry monitor.

ANS: A All patients with DKA experience dehydration (10% of total body weight in severe ketoacidosis) because of the osmotic diuresis, accompanied by depletion of electrolytes (sodium, potassium, chloride, phosphate, and magnesium). The initial hydrating solution is 0.9% saline solution. Insulin therapy should be started after the initial rehydration bolus because serum glucose levels fall rapidly after volume expansion. The child should be placed on the cardiac and pulse oximetry monitors after the rehydrating solution has been initiated.

24. What is the most appropriate nursing action when a child with a probable intussusception has a normal, brown stool? a. Notify the practitioner b. Measure abdominal girth c. Auscultate for bowel sounds d. Take vital signs, including blood pressure

ANS: A Passage of a normal brown stool indicates that the intussusception has reduced itself. This is immediately reported to the practitioner, who may choose to alter the diagnostic/therapeutic plan of care.

27. What food choice by the parent of a 2-year-old child with celiac disease indicates a need for further teaching? a. Oatmeal b. Rice cake c. Corn muffin d. Meat patty

ANS: A The child with celiac disease is unable to fully digest gluten, the protein found in wheat, barley, rye, and oats. Oatmeal contains gluten and is not an appropriate food selection. Rice is an appropriate choice because it does not contain gluten. Corn is digestible because it does not contain gluten. Meats do not contain gluten and can be included in the diet of a child with celiac disease.

25. An important nursing consideration in the care of a child with celiac disease is to facilitate which intervention? a. Refer to a nutritionist for detailed dietary instructions and education. b. Help the child and family understand that diet restrictions are usually only temporary. c. Teach proper hand washing and Standard Precautions to prevent disease transmission. d. Suggest ways to cope more effectively with stress to minimize symptoms.

ANS: A The main consideration is helping the child adhere to dietary management. Considerable time is spent in explaining to the child and parents the disease process, the specific role of gluten in aggravating the condition, and those foods that must be restricted. Referral to a nutritionist would help in this process. The most severe symptoms usually occur in early childhood and adult life. Dietary avoidance of gluten should be lifelong. Celiac disease is not transmissible or stress related.

19. The best chance of survival for a child with cirrhosis is: a. liver transplantation. b. treatment with corticosteroids. c. treatment with immune globulin. d. provision of nutritional support.

ANS: A The only successful treatment for end-stage liver disease and liver failure may be liver transplantation, which has improved the prognosis for many children with cirrhosis. Liver transplantation has revolutionized the approach to cirrhosis. Liver failure and cirrhosis are indications for transplantation. Liver transplantation reflects the failure of other medical and surgical measures, such as treatment with corticosteroids or immune globulin and nutritional support, to prevent or treat cirrhosis.

17. The nurse is taking care of an adolescent with osteosarcoma. The parents ask the nurse about treatment. The nurse should make which accurate response about treatment for osteosarcoma? a. Treatment usually consists of surgery and chemotherapy. b. Amputation of the affected extremity is rarely necessary. c. Intensive irradiation is the primary treatment. d. Bone marrow transplantation offers the best chance of long-term survival.

ANS: A The optimal therapy for osteosarcoma is a combination of surgery and chemotherapy. Intensive irradiation and bone marrow transplantation are usually not part of the therapeutic management.

21. The nurse, caring for a neonate with a suspected tracheoesophageal fistula, should include what intervention into the plan of care? a. Elevating the head to facilitate secrete drainage. b. Elevating the head for feedings only. c. Feeding glucose water only. d. Avoiding suctioning unless the infant is cyanotic.

ANS: A When a newborn is suspected of having tracheoesophageal fistula, the most desirable position is supine with the head elevated on an inclined plane of at least 30 degrees to maintain an airway and facilitate drainage of secretions. It is imperative that any source of aspiration be removed at once; oral feedings are withheld. Feeding of fluids should not be given to infants suspected of having tracheoesophageal fistulas. The oral pharynx should be kept clear of secretion by oral suctioning. This is to avoid the cyanosis that is usually the result of laryngospasm caused by overflow of saliva into the larynx.

The nurse is caring for an infant with developmental dysplasia of the hip. Which clinical manifestations should the nurse expect to observe? (Select all that apply.) a. Positive Ortolani click b. Unequal gluteal folds c. Negative Babinski sign d. Trendelenburg sign e. Telescoping of the affected limb f. Lordosis

ANS: A, B A positive Ortolani test and unequal gluteal folds are clinical manifestations of developmental dysplasia of the hip seen from birth to 2 to 3 months. Unequal gluteal folds, negative Babinski sign, and Trendelenburg sign are signs that appear in older infants and children. Telescoping of the affected limb and lordosis are not clinical manifestations of developmental dysplasia of the hip.

MULTIPLE RESPONSE 37.The nurse is caring for an infant with a suspected urinary tract infection. Which clinical manifestations would be observed (Select all that apply)? a. Vomiting b. Jaundice c. Failure to gain weight d. Swelling of the face e. Back pain f. Persistent diaper rash

ANS: A, C, F Vomiting, failure to gain weight, and persistent diaper rash are clinical manifestations observed in an infant with a urinary tract infection. Jaundice, swelling of the face, and back pain would not be observed in an infant with a urinary tract infection.

An adolescent with juvenile idiopathic arthritis (JIA) is prescribed abatacept (Orencia). Which should the nurse teach the adolescent regarding this medication? (Select all that apply.) a. Avoid receiving live immunizations while taking the medication. b. Before beginning this medication, a tuberculin screening test will be done. c. You will be getting a twice-a-day dose of this medication. d. This medication is taken orally.

ANS: A, B Abatacept reduces inflammation by inhibiting T cells and is given intravenously every 4 weeks. Possible side effects of biologics include an increased infection risk. Because of the infection risk, children should be evaluated for tuberculosis exposure before starting these medications. Live vaccines should be avoided while taking these agents.

4. A student is learning about the process of hematopoiesis and how it is affected by leukemia. Which information does the student discover? (Select all that apply.) A. Blast cells multiply faster than mature cells. B. Leukemia disrupts normal hematopoiesis. C. Lymphoid cells differentiate into B and T cells. D. Myeloid cells crowd out normal cells in bone marrow. E. Pancytopenia occurs from proliferation of mast cells.

ANS: A, B, C Blast, or immature, cells have an increased rate of proliferation and multiply at the expense of normal cells. Leukemia does disrupt normal hematopoiesis (production and development of blood cells). Lymphoid cells differentiate into B and T cells. Myeloid cells differentiate into red blood cells, monocytes, granulocytes, and platelets; they do not reproduce and crowd out "normal" cells in the marrow. Pancytopenia occurs when large numbers of blast cells reproduce and crowd out normal marrow components.

2. Peer victimization is becoming a significant problem for school-age children and adolescents in the United States. Parents should be educated regarding signs that a child is being bullied. These might include: (Select all that apply.) a. The child spends an inordinate amount of time in the nurse's office. b. Belongings frequently go missing or are damaged. c. The child wants to be driven to school. d. School performance improves. e. The child freely talks about his or her day.

ANS: A, B, C Signs that may indicate a child is being bullied are similar to signs of other types of stress and include nonspecific illness or complaints, withdrawal, depression, school refusal, and decreased school performance. Children expressed fear of going to school or riding the school bus, and their belongings often are damaged or missing. Very often, children will not talk about what is happening to them.

3. The nurse is teaching a community group about early warning signs of cancer. Which signs does the nurse include? (Select all that apply.) A. A sore that does not heal B. Change in bowel or bladder habits C. Difficulty swallowing or indigestion D. Nagging feeling that something is wrong E. Unusual bleeding or discharge

ANS: A, B, C, E The American Cancer Society uses the acronym CAUTION to describe common warning signs of cancer: change in bowel or bladder habits; a sore that does not heal; unusual bleeding or discharge; thickening or lump in the breast, testicles, or elsewhere; indigestion or trouble swallowing; obvious change in the size, color shape, or thickness of a wart, mole, or mouth sore; and nagging cough or hoarseness.

Which assessment findings should the nurse note in a school-age child with Duchenne muscular dystrophy (DMD)? (Select all that apply.) a. Lordosis b. Gower sign c. Kyphosis d. Scoliosis e. Waddling gait

ANS: A, B, E Difficulties in running, riding a bicycle, and climbing stairs are usually the first symptoms noted in Duchenne muscular dystrophy. Typically, affected boys have a waddling gait and lordosis, fall frequently, and develop a characteristic manner of rising from a squatting or sitting position on the floor (Gower sign). Lordosis occurs as a result of weakened pelvic muscles, and the waddling gait is a result of weakness in the gluteus medius and maximus muscles. Kyphosis and scoliosis are not assessment findings with DMD.

What is an age-appropriate nursing intervention to facilitate psychologic adjustment for an adolescent expected to have a prolonged hospitalization (select all that apply)? a. Encourage parents to bring in homework and schedule study times. b. Allow the adolescent to wear street clothes. c. Involve the parents in care. d. Follow home routines. e. Encourage parents to bring in favorite foods.

ANS: A, B, E Encouraging parents to bring in homework, street clothes, and favorite foods are all developmentally appropriate approaches to facilitate adjustment and coping for an adolescent who will be experiencing prolonged hospitalization. Involving parents in care and following home routines are important interventions for the preschool child who is in the hospital. Adolescents do not need parents to assist in their care. They are used to performing independent self-care. Adolescents may want their parents to be nearby, or they may enjoy the freedom and independence from parental control and routines.

1. The student nurse studying childhood cancers understands that neoplasms are caused by which factors? (Select all that apply.) A. Chromosomal/genetic abnormalities B. External stimuli or environment C. Maternal nutrition during gestation D. Substance abuse during pregnancy E. Viruses that alter the immune system

ANS: A, B, E Neoplasms are caused by one or a combination of the following: chromosomal or genetic abnormalities, external stimuli or the environment, and/or viruses that alter the immune system. Nutritional deficits and substance abuse by the mother can certainly lead to developmental and other health problems, but do not lead to childhood cancers.

3. A nurse is planning care for a 7-year-old child hospitalized with osteomyelitis. Which activities should the nurse plan to bring from the playroom for the child? (Select all that apply.) a. Paper and some paints b. Board games c. Jack-in-the-box d. Stuffed animals e. Computer games

ANS: A, B, E School-age children become fascinated with complex board, card, or computer games that they can play alone, with a best friend, or with a group. They also enjoy sewing, cooking, carpentry, gardening, and creative activities such as painting. Jack-in-the-box and stuffed animals would be appropriate for a toddler or preschool child.

39.A school-age child is admitted to the hospital with acute glomerulonephritis and oliguria. Which dietary menu items should be allowed for this child (Select all that apply)? a. Apples b. Bananas c. Cheese d. Carrot sticks e. Strawberries

ANS: A, D, E Moderate sodium restriction and even fluid restriction may be instituted for children with acute glomerulonephritis. Foods with substantial amounts of potassium and sodium are generally restricted during the period of oliguria. Apples, carrot sticks, and strawberries would be items low in sodium and allowed. Bananas are high in potassium and cheese is high in sodium. Those items would be restricted.

13. A child is getting induction therapy for Burkitt lymphoma. The nurse finds the child lethargic and complaining of side and back pain. The child's morning laboratory results indicate a serum calcium level of 7.2 mg/dL. What actions by the nurse are the most appropriate at this time? (Select all that apply.) A. Administer a dose of pain medication. B. Assess Chvostek and Trousseau signs. C. Call the rapid response team. D. Encourage an increased oral intake. E. Prepare to administer allopurinol (Aloprim).

ANS: A, B, E This child is manifesting signs of tumor lysis syndrome. The child is at risk due to the rapid destruction of cancer cells (induction therapy) and from the child's type of cancer (Burkitt lymphoma). Lethargy, flank pain, and hypocalcemia are common findings in this condition. The nurse should administer pain medication, assess for physical manifestations of hypocalcemia (Chvostek and Trousseau signs), and prepare to administer allopurinol. Adequate hydration is important as well, but because the child is lethargic, IV fluids should be given, not oral fluids. The rapid response team is not needed at this point.

The nurse is preparing to give oral care to a school-age child with mucositis secondary to chemotherapy administered to treat leukemia. Which preparations should the nurse use for oral care on this child? (Select all that apply.) a. Chlorhexidine gluconate (Peridex) b. Lemon glycerin swabs c. Antifungal troches (lozenges) d. Lip balm (Aquaphor) e. Hydrogen peroxide

ANS: A, C, D Preparations that may be used to prevent or treat mucositis include chlorhexidine gluconate (Peridex) because of its dual effectiveness against candidal and bacterial infections, antifungal troches (lozenges) or mouthwash, and lip balm (e.g., Aquaphor) to keep the lips moist. Agents that should not be used include lemon glycerin swabs (irritate eroded tissue and can decay teeth), hydrogen peroxide (delays healing by breaking down protein), and milk of magnesia (dries mucosa).

Ryan has just been unexpectedly admitted to the intensive care unit after abdominal surgery. The nursing staff has completed the admission process, and Ryans condition is beginning to stabilize. When speaking with the parents, the nurses should expect which stressors to be evident (select all that apply)? a. Unfamiliar environment b. Usual day-night routine c. Strange smells d. Provision of privacy e. Inadequate knowledge of condition and routine

ANS: A, C, E Intensive care units, especially when the family is unprepared for the admission, are a strange and unfamiliar place. There are many pieces of unfamiliar equipment, and the sights and sounds are much different from a general hospital unit. Also, with the childs condition being more precarious, it may be difficult to keep the parents updated and knowledgeable about what is happening. Lights are usually on around the clock, seriously disrupting the diurnal rhythm. There is usually little privacy available for families in intensive care units.

A child is being discharged from an ambulatory care center after an inguinal hernia repair. Which discharge interventions should the nurse implement (select all that apply)? a. Discuss dietary restrictions. b. Hold any analgesic medications until the child is home. c. Send a pain scale home with the family. d. Suggest the parents fill the prescriptions on the way home. e. Discuss complications that may occur.

ANS: A, C, E The discharge interventions a nurse should implement when a child is being discharged from an ambulatory care center should include dietary restrictions, being very specific and giving examples of clear fluids or what is meant by a full liquid diet. The nurse should give specific information on pain control and send a pain scale home with the family. All complications that may occur after an inguinal hernia repair should be discussed with the parents. The pain medication, as prescribed, should be given before the child leaves the building, and prescriptions should be filled and given to the family before discharge.

10. A nurse is explaining radiation side effects to the parents of a child for whom it has been ordered. Which side effects does the nurse include in the explanation? (Select all that apply.) A. Hair loss B. Leukocytosis C. Nausea D. Polycythemia E. Skin desquamation

ANS: A, C, E Common side effects of radiation include nausea, alopecia (hair loss), fatigue and malaise, low WBC, skin desquamation, and mucous membrane inflammation and irritation. Leukocytosis (high WBC count) and polycythemia (increased RBC count) are not seen.

11. A student nurse is preparing to administer odansetron (Zofran) to a child receiving chemotherapy. The child weighs 44 lb (20 kg). Which actions by the student nurse require intervention by the faculty? (Select all that apply.) A. Assesses the child's pain with a pediatric scale B. Discusses side effects with the parents/child C. Draws up 300 mg for IV administration D. Prepares to administer 3 mg IV push E. Withdraws a 200-mg suppository for use

ANS: A, C, E Odansetron is an anti-emetic (not a pain medication) and can be given IV at 0.15 mg/kg, making the correct dose 3 mg. The faculty member should intervene if the student assesses pain, not nausea; draws up 300 mg; or tries to obtain a suppository for the child. Odansetron can also be given PO. Discussing side effects is a responsibility when giving medications.

The treatment of brain tumors in children consists of which therapies? (Select all that apply.) a. Surgery b. Bone marrow transplantation c. Chemotherapy d. Stem cell transplantation e. Radiation f. Myelography

ANS: A, C, E Treatment for brain tumors in children may consist of surgery, chemotherapy, and radiotherapy alone or in combination. Bone marrow and stem cell transplantation therapies are used for leukemia, lymphoma, and other solid tumors where myeloablative therapies are used. Myelography is a radiographic examination after an intrathecal injection of contrast medium. It is not a treatment.

3. Which interventions should a nurse implement when caring for a child with hepatitis? (Select all that apply.) a. Provide a well-balanced, low-fat diet. b. Schedule playtime in the playroom with other children. c. Teach parents not to administer any over-the-counter medications. d. Arrange for home schooling because the child will not be able to return to school. e. Instruct parents on the importance of good hand washing.

ANS: A, C, E The child with hepatitis should be placed on a well-balanced, low-fat diet. Parents should be taught to not give over-the-counter medications because of impaired liver function. Hand hygiene is the most important preventive measure for the spread of hepatitis. The child will be in contact isolation in the hospital, so playtime with other hospitalized children is not scheduled. The child will be on contact isolation for a minimum of 1 week after the onset of jaundice. After that period, the child will be allowed to return to school.

1. Nursing care of a child diagnosed with syndrome of inappropriate antidiuretic hormone (SIADH) should include which interventions? (Select all that apply.) a. Weigh daily b. Encourage fluids c. Turn frequently d. Maintain nothing by mouth e. Restrict fluids

ANS: A, E Increased secretion of ADH causes the kidney to resorb water, which increases fluid volume and decreases serum osmolarity with a progressive reduction in sodium concentration. The immediate management of the child is to restrict fluids. The child should also be weighed at the same time each day. Encouraging fluids, turning frequently, and maintaining nothing by mouth are not associated with SIADH since they are not associated with managing/monitoring for fluid retention.

An infant with hydrocephalus is hospitalized for surgical placement of a ventriculoperitoneal shunt. Which interventions should be included in the child's postoperative care? (Select all that apply.) a. Observe closely for signs of infection. b. Pump the shunt reservoir to maintain patency. c. Administer sedation to decrease irritability. d. Maintain Trendelenburg position to decrease pressure on the shunt. e. Maintain an accurate record of intake and output. f. Monitor for abdominal distention.

ANS: A, E, F Infection is a major complication of ventriculoperitoneal shunts. Observation for signs of infection is a priority nursing intervention. Intake and output should be measured carefully. Abdominal distention could be a sign of peritonitis or a postoperative ileus. Pumping of the shunt may cause obstruction or other problems and should not be performed unless indicated by the neurosurgeon. Pain management rather than sedation should be the goal of therapy. The child is kept flat to avoid too rapid a reduction of intracranial fluid.

Chelation therapy is begun on a child with b-thalassemia major. The purpose of this therapy is to: a. treat the disease. b. eliminate excess iron. c. decrease risk of hypoxia. d. manage nausea and vomiting.

ANS: B A complication of the frequent blood transfusions in thalassemia is iron overload. Chelation therapy with deferoxamine (an iron-chelating agent) is given with oral supplements of vitamin C to increase iron excretion. Chelation therapy treats the side effect of the disease management. Decreasing the risk of hypoxia and managing nausea and vomiting are not the purposes of chelation therapy.

An appropriate nursing intervention to minimize separation anxiety in a hospitalized toddler is to: a. Provide for privacy. b. Encourage parents to room in. c. Explain procedures and routines. d. Encourage contact with children the same age.

ANS: B A toddler experiences separation anxiety secondary to being separated from the parents. To avoid this, the parents should be encouraged to room in as much as possible. Maintaining routines and ensuring privacy are helpful interventions, but they would not substitute for the parents. Contact with same-aged children would not substitute for having the parents present.

Which situation poses the greatest challenge to the nurse working with a child and family? a. Twenty-four-hour observation b. Emergency hospitalization c. Outpatient admission d. Rehabilitation admission

ANS: B Emergency hospitalization involves (1) limited time for preparation both for the child and family, (2) situations that cause fear for the family that the child may die or be permanently disabled, and (3) a high level of activity, which can foster further anxiety. Although preparation time may be limited with a 24-hour observation, this situation does not usually involve the acuteness of the situation and the high levels of anxiety associated with emergency admission. Outpatient admission generally involves preparation time for the family and child. Because of the lower level of acuteness in this setting, anxiety levels are not as high. Rehabilitation admission follows a serious illness or disease. This type of unit may resemble a home environment, which decreases the childs and familys anxiety.

The nurse is caring for a neonate born with a myelomeningocele. Surgery to repair the defect is scheduled the next day. The most appropriate way to position and feed this neonate is which position? a. Prone and tube-fed b. Prone, head turned to side, and nipple-fed c. Supine in an infant carrier and nipple-fed d. Supine, with defect supported with rolled blankets, and nipple-fed

ANS: B In the prone position, feeding is a problem. The infants head is turned to one side for feeding. If the child is able to nipple-feed, tube feeding is not needed. Before surgery, the infant is kept in the prone position to minimize tension on the sac and risk of trauma.

How much folic acid is recommended for women of childbearing age? a. 1.0 mg b. 0.4 mg c. 1.5 mg d. 2.0 mg

ANS: B It has been estimated that a daily intake of 0.4 mg of folic acid in women of childbearing age will prevent 50% to 70% of cases of neural tube defects; 1.0 mg is too low a dose; 1.5 to 2.0 mg are not the recommended dosages of folic acid.

A boy with leukemia screams whenever he needs to be turned or moved. Which is the most probable cause of this pain? a. Edema b. Bone involvement c. Petechial hemorrhages d. Changes within the muscles

ANS: B The invasion of the bone marrow with leukemic cells gradually causes a weakening of the bone and a tendency toward fractures. As leukemic cells invade the periosteum, increasing pressure causes severe pain. Edema, petechial hemorrhages, and changes within the muscles would not cause severe pain.

A school-age child is admitted in vasoocclusive sickle cell crisis. The childs care should include: a. correction of acidosis. b. adequate hydration and pain management. c. pain management and administration of heparin. d. adequate oxygenation and replacement of factor VIII.

ANS: B The management of crises includes adequate hydration, minimization of energy expenditures, pain management, electrolyte replacement, and blood component therapy if indicated. Hydration and pain control are two of the major goals of therapy. The acidosis will be corrected as the crisis is treated. Heparin and factor VIII is not indicated in the treatment of vasoocclusive sickle cell crisis. Oxygen may prevent further sickling, but it is not effective in reversing sickling because it cannot reach the clogged blood vessels.

The nurse is admitting a school-age child with suspected Guillain-Barr syndrome (GBS). Which is a priority in the care for this child? a. Monitoring intake and output b. Assessing respiratory efforts c. Placing on a telemetry monitor d. Obtaining laboratory studies

ANS: B Treatment of GBS is primarily supportive. In the acute phase, patients are hospitalized because respiratory and pharyngeal involvement may require assisted ventilation, sometimes with a temporary tracheotomy. Treatment modalities include aggressive ventilatory support in the event of respiratory compromise, intravenous (IV) administration of immunoglobulin (IVIG), and sometimes steroids; plasmapheresis and immunosuppressive drugs may also be used. Intake and output, telemetry monitoring and obtaining laboratory studies may be part of the plan of care but are not the priority.

40. A nurse is preparing to administer chemotherapy to a child who has an Infuse-a-Port. Which action by the nurse is the most appropriate? A. Flush the catheter with normal saline and heparin. B. Obtain a Huber needle prior to administration. C. Unclamp the catheter prior to flushing the line. D. Wrap the catheter in gauze so it doesn't pull out.

ANS: B A centrally implanted port, such as an Infuse-a-Port, must be accessed with a Huber needle. Prior to administering medication is not the time to flush with heparin. The port is entirely indwelling, so there is no catheter to unclamp, nor will the device pull out.

The nurse is teaching nursing students about childhood fractures. Which describes a compound skull fracture? a. Involves the basilar portion of the occipital bone b. Bone is exposed through the skin c. Traumatic separations of the cranial sutures d. Bone is pushed inward, causing pressure on the brain

ANS: B A compound fracture has the bone exposed through the skin. A basilar fracture involves the basilar portion of the frontal, ethmoid, sphenoid, temporal, or occipital bone. Diastatic skull fractures are traumatic separations of the cranial sutures. A depressed fracture has the bone pushed inward, causing pressure on the brain.

20. A child is being discharged after surgical resection of a retinoblastoma with enucleation. Which discharge instruction is most important based on the diagnosis? A. Encouraging healthy eating B. Irrigation of the surgical site C. Monitoring the child's temperature D. Pain assessment and control

ANS: B After enucleation (removal of the eye), the eye socket must be irrigated and a thin layer of antibiotic ointment applied. The other options are valid for all postoperative pediatric patients.

26. A child has liver cancer. The most recent results for the alpha-fetoprotein level show it has been reduced by 50%. Which statement by the nurse to the parents and child is most appropriate at this time? A. "Once the level gets to normal, we can resect the tumor." B. "This shows the cancer is responding to therapy." C. "Unfortunately, the chemotherapy is not working." D. "Your child will need a liver transplant soon."

ANS: B Alpha-fetoprotein (AFP) is a protein produced by both hepatoblastomas and hepatocellular carcinomas. Falling levels of AFP indicate that treatment is working. The other responses are not correct.

7. A child is admitted and is scheduled to receive intravenous asparginase (Elspar). Which action by the nurse is most important when administering this medication? A. Arranging an outpatient hearing test B. Having emergency drugs on hand C. Monitoring the child's intake and output D. Providing anti-emetic drugs as needed

ANS: B Anaphylaxis is a possible side effect of this drug. Emergency medications should be readily available. Ototoxicity can be caused by carboplatin (Paraplatin). Monitoring intake and output is important for any child on IV therapy. Anti-emetic drugs are important for any child receiving chemotherapy.

Kristin, age 10 years, sustained a fracture in the epiphyseal plate of her right fibula when she fell off of a tree. When discussing this injury with her parents, the nurse should consider which statement? a. Healing is usually delayed in this type of fracture. b. Growth can be affected by this type of fracture. c. This is an unusual fracture site in young children. d. This type of fracture is inconsistent with a fall.

ANS: B Detection of epiphyseal injuries is sometimes difficult, but fractures involving the epiphysis or epiphyseal plate present special problems in determining whether bone growth will be affected. Healing of epiphyseal injuries is usually prompt. The epiphysis is the weakest point of the long bones. This is a frequent site of damage during trauma.

41. Which consideration is the most important in managing tuberculosis (TB) in children? a. Skin testing annually c. Adequate nutrition b. Pharmacotherapy d. Adequate hydration

ANS: B Drug therapy for TB includes isoniazid, rifampin, and pyrazinamide daily for 2 months and 2 or 3 times a week for the remaining 4 months. Although skin testing and adequate nutrition and hydration are important, pharmacotherapy is the most important intervention for TB.

12. Which statement best describes fear in school-age children? a. They are increasingly fearful for body safety. b. Most of the new fears that trouble them are related to school and family. c. They should be encouraged to hide their fears to prevent ridicule by peers. d. Those who have numerous fears need continuous protective behavior by parents to eliminate these fears.

ANS: B During the school-age years, children experience a wide variety of fears, but new fears related predominantly to school and family bother children during this time. During the middle-school years, children become less fearful of body safety than they were as preschoolers. Parents and other persons involved with children should discuss their fear with them individually or as a group activity. Sometimes school-age children hide their fears to avoid being teased. Hiding the fears does not end them and may lead to phobias.

Which position should the nurse place a 10-year-old child after a large tumor was removed through a supratentorial craniotomy? a. On the inoperative side with the bed flat b. On the inoperative side with the head of bed elevated 20 to 30 degrees c. On the operative side with the bed flat and pillows behind the head d. On the operative side with the head of bed elevated 45 degrees

ANS: B If a large tumor was removed, the child is not placed on the operative side because the brain may suddenly shift to that cavity, causing trauma to the blood vessels, linings, and the brain itself. The child with an infratentorial procedure is usually positioned on either side with the bed flat. When a supratentorial craniotomy is performed, the head of bed is elevated 20 to 30 degrees with the child on either side or on the back. In a supratentorial craniotomy, the head elevation facilitates CSF drainage and decreases excessive blood flow to the brain to prevent hemorrhage. Pillows should be placed against the child's back, not head, to maintain the desired position.

The nurse is caring for a preschool child immobilized by a spica cast. Which effect on metabolism should the nurse monitor on this child related to the immobilized status? a. Hypocalcemia b. Decreased metabolic rate c. Positive nitrogen balance d. Increased production of stress hormones

ANS: B Immobilization causes a decreased metabolic rate with slowing of all systems and a decreased food intake. Immobilization leads to hypercalcemia and causes a negative nitrogen balance secondary to muscle atrophy. A decreased production of stress hormones occurs with decreased physical and emotional coping capacity.

26.Which statement is descriptive of renal transplantation in children? a. It is an acceptable means of treatment after age 10 years. b. It is preferred means of renal replacement therapy in children. c. Children can receive kidneys only from other children. d. The decision for transplantation is difficult since a relatively normal lifestyle is not possible.

ANS: B Renal transplantation offers the opportunity for a relatively normal lifestyle versus dependence on dialysis and is the preferred means of renal replacement therapy in end-stage renal disease. It can be done in children as young as age 6 months. Both children and adults can serve as donors for renal transplant purposes.

11. The diet of a child with nephrosis usually includes: a. High protein. b. Salt restriction. c. Low fat. d. High carbohydrate.

ANS: B Salt is usually restricted (but not eliminated) during the edema phase. The child has very little appetite during the acute phase. Favorite foods are provided (with the exception of high-salt ones) in an attempt to provide nutritionally complete meals.

25. Parents of a 12-year-old child ask the clinic nurse, "How many hours of sleep should our child get each night?" The nurse should respond that 12-year-old children need ____ hours of sleep at night. a. 8 b. 9 c. 10 d. 11

ANS: B School-age children usually do not require naps, but they do need to sleep approximately 11 hours at age 5 years and 9 hours at age 12 years each night.

32. The parent of a toddler calls the nurse, asking about croup. What is a distinguishing manifestation of spasmodic croup? a. Wheezing is heard audibly. c. It is bacterial in nature. b. It has a harsh, barky cough. d. The child has a high fever

ANS: B Spasmodic croup is viral in origin, is usually preceded by several days of symptoms of upper respiratory tract infection, and often begins at night. It is marked by a harsh, metallic, barky cough; sore throat; inspiratory stridor; and hoarseness. Wheezing is not a distinguishing manifestation of croup. It can accompany conditions such as asthma or bronchiolitis. A high fever is not usually present.

22. A nurse is assessing an older school-age child recently admitted to the hospital. Which assessment indicates that the child is in an appropriate stage of cognitive development? a. The child's addition and subtraction ability b. The child's ability to classify c. The child's vocabulary d. The child's play activity

ANS: B The ability to classify things from simple to complex and the ability to identify differences and similarities are cognitive skills of the older school-age child; this demonstrates use of classification and logical thought processes. Subtraction and addition are appropriate cognitive activities for the young school-age child. Vocabulary is not as valid an assessment of cognitive ability as is the child's ability to classify. Play activity is not as valid an assessment of cognitive function as is the ability to classify.

The nurse is closely monitoring a child who is unconscious after a fall and notices that the child suddenly has a fixed and dilated pupil. The nurse should interpret this as: a. eye trauma. b. neurosurgical emergency. c. severe brainstem damage. d. indication of brain death.

ANS: B The sudden appearance of a fixed and dilated pupil(s) is a neurosurgical emergency. The nurse should immediately report this finding. Although a dilated pupil may be associated with eye trauma, this child has experienced a neurologic insult. Pinpoint pupils or bilateral fixed pupils for more than 5 minutes are indicative of brainstem damage. The unilateral fixed and dilated pupil is suggestive of damage on the same side of the brain. One fixed and dilated pupil is not suggestive of brain death.

The vector reservoir for agents causing viral encephalitis in the United States is: a. tarantula spiders. b. mosquitoes. c. carnivorous wild animals. d. domestic and wild animals.

ANS: B Viral encephalitis, not attributable to a childhood viral disease, is usually transmitted by mosquitoes. The vector reservoir for most agents pathogenic for humans and detected in the United States are mosquitoes and ticks; therefore, most cases of encephalitis appear during the hot summer months. Tarantula spiders, carnivorous wild animals, and domestic and wild animals are not reservoirs for the agents that cause viral encephalitis.

16. When teaching injury prevention during the school-age years, the nurse should include what topic? a. Teaching the need to fear strangers. b. Teaching basic rules of water safety. c. Avoiding letting children cook in microwave ovens. d. Cautioning children against engaging in competitive sports.

ANS: B Water safety instruction is an important source of injury prevention at this age. The child should be taught to swim, select safe and supervised places to swim, swim with a companion, check sufficient water depth for diving, and use an approved flotation device. Teach stranger safety, not fearing strangers. This includes not going with strangers, not having personalized clothing in public places, having children tell parents if anyone makes them uncomfortable, and teaching children to say "no" in uncomfortable situations. Teach children safe cooking methods. Caution against engaging in hazardous sports, such as those involving trampolines.

An adolescent with a fractured femur is in Russell's traction. Surgical intervention to correct the fracture is scheduled for the morning. Nursing actions should include which action? a. Maintaining continuous traction until 1 hour before the scheduled surgery b. Maintaining continuous traction and checking position of traction frequently c. Releasing traction every hour to perform skin care d. Releasing traction once every 8 hours to check circulation

ANS: B When the muscles are stretched, muscle spasm ceases and permits realignment of the bone ends. The continued maintenance of traction is important during this phase because releasing the traction allows the muscle's normal contracting ability to again cause malpositioning of the bone ends. Continuous traction must be maintained to keep the bone ends in satisfactory realignment. Releasing at any time, either 1 hour before surgery, once every hour for skin care, or once every 8 hours would not keep the fracture in satisfactory alignment.

29. An adolescent is being seen in the clinic for evaluation of acromegaly. The nurse understands that which event occurs with acromegaly? a. There is a lack of growth hormone (GH) being produced. b. There is excess GH after closure of the epiphyseal plates. c. There is an excess of GH before the closure of the epiphyseal plates. d. There is a lack of thyroid hormone being produced.

ANS: B Excess GH after closure of the epiphyseal plates results in acromegaly. A lack of growth hormone results in delayed growth or even dwarfism. Gigantism occurs when there is hypersecretion of GH before the closure of the epiphyseal plates. Cretinism is associated with hypothyroidism.

11. A 4-month-old infant diagnosed with gastroesophageal reflux disease (GERD) is thriving without other complications. What should the nurse suggest to minimize reflux? a. Place in Trendelenburg position after eating. b. Thicken formula with rice cereal. c. Give continuous nasogastric tube feedings. d. Give larger, less frequent feedings.

ANS: B Giving small frequent feedings of formula combined with 1 teaspoon to 1 tablespoon of rice cereal per ounce of formula has been recommended. Milk thickening agents have been shown to decrease the number of episodes of vomiting and increase the caloric density of the formula. This may benefit infants who are underweight as a result of GERD. Placing the child in Trendelenburg position would increase the reflux. Continuous nasogastric feedings are reserved for infants with severe reflux and failure to thrive. Smaller, more frequent feedings are recommended in reflux.

6. Which is often administered to prevent or control hemorrhage in a child diagnosed with cancer? a. Nitrosoureas b. Platelets c. Whole blood d. Corticosteroids

ANS: B Most bleeding episodes can be prevented or controlled with the administration of platelet concentrate or platelet-rich plasma. Nitrosoureas, whole blood, and corticosteroids would not prevent or control hemorrhage.

27. Which laboratory finding confirms that a child with type 1 diabetes is experiencing diabetic ketoacidosis? a. No urinary ketones b. Low arterial pH c. Elevated serum carbon dioxide d. Elevated serum phosphorus

ANS: B Severe insulin deficiency produces metabolic acidosis, which is indicated by a low arterial pH. Urinary ketones, often in large amounts, are present when a child is in diabetic ketoacidosis. Serum carbon dioxide is decreased in diabetic ketoacidosis. Serum phosphorus is decreased in diabetic ketoacidosis.

6. Diabetes insipidus is a disorder of which organ? a. Anterior pituitary b. Posterior pituitary c. Adrenal cortex d. Adrenal medulla

ANS: B The principal disorder of posterior pituitary hypofunction is diabetes insipidus. The anterior pituitary produces hormones such as growth hormone, thyroid-stimulating hormone, adrenocorticotropic hormone, gonadotropin, prolactin, and melanocyte-stimulating hormone. The adrenal cortex produces aldosterone, sex hormones, and glucocorticoids. The adrenal medulla produces catecholamines.

3. An infant is brought to the emergency department with poor skin turgor, sunken fontanel, lethargy, and tachycardia. This is suggestive of which condition? a. Overhydration b. Dehydration c. Sodium excess d. Calcium excess

ANS: B These clinical manifestations indicate dehydration. Symptoms of overhydration are edema and weight gain. Regardless of extracellular sodium levels, total body sodium is usually depleted in dehydration. Symptoms of hypocalcemia are a result of neuromuscular irritability and manifest as jitteriness, tetany, tremors, and muscle twitching.

1. The nurse, caring for an infant whose cleft lip was repaired, should include which interventions into the infant's postoperative plan of care? (Select all that apply.) a. Postural drainage b. Petroleum jelly to the suture line c. Elbow restraints d. Supine and side-lying positions e. Mouth irrigations

ANS: B, C Apply petroleum jelly to the operative site for several days after surgery. Elbows are restrained to prevent the child from accessing the operative site for up to 7 to 10 days. The child should be positioned on back or side or in an infant seat. Postural drainage is not indicated. This would increase the pressure on the operative site when the child is placed in different positions. Mouth irrigations would not be indicated.

A nurse plans therapeutic play time for a hospitalized child. Which are the benefits of therapeutic play (select all that apply)? a. Serves as method to assist disturbed children b. Allows the child to express feelings c. The nurse can gain insight into the childs feelings d. The child can deal with concerns and feelings e. Gives the child a structured play environment

ANS: B, C, D Therapeutic play is an effective, nondirective modality for helping children deal with their concerns and fears, and at the same time, it often helps the nurse gain insights into childrens needs and feelings. Play and other expressive activities provide one of the best opportunities for encouraging emotional expression, including the safe release of anger and hostility. Nondirective play that allows children freedom for expression can be tremendously therapeutic. Play therapy is a structured therapy that helps disturbed children. It should not be confused with therapeutic play.

A nurse is interviewing the parents of a toddler about use of complementary or alternative medical practices. The parents share several practices they use in their household. Which should the nurse document as complementary or alternative medical practices (select all that apply)? a. Use of acetaminophen (Tylenol) for fever b. Administration of chamomile tea at bedtime c. Hypnotherapy for relief of pain d. Acupressure to relieve headaches e. Cool mist vaporizer at the bedside for stuffiness

ANS: B, C, D When conducting an assessment, the nurse should inquire about the use of complementary or alternative medical practices. Administration of chamomile tea at bedtime, hypnotherapy for relief of pain, and acupressure to relieve headaches are complementary or alternative medical practices. Using Tylenol for fever relief and a cool mist vaporizer at the bedside to reduce stuffiness are not considered complementary or alternative medical practices.

14. A nurse has a RN preceptor student working on the pediatric oncology unit. When teaching the student about oncological crises, what disorders does the nurse include? (Select all that apply.) A. Inferior vena cava infarction B. Neurogenic shock C. Perirectal abscess D. Pleural effusion E. Superior vena cava syndrome

ANS: B, C, D, E There are many oncologic emergencies, including neurogenic shock (and other types of shock), perirectal abscess, pleural effusion, and superior vena cava syndrome. Inferior vena cava infarction is not on the list of emergent conditions.

2. Which statements regarding hepatitis B are correct? (Select all that apply.) a. Hepatitis B cannot exist in a carrier state. b. Hepatitis B can be prevented by hepatitis B virus vaccine. c. Hepatitis B can be transferred to an infant of a breastfeeding mother. d. The onset of hepatitis B is insidious. e. Immunity to hepatitis B occurs after one attack.

ANS: B, C, D, E The vaccine elicits the formation of an antibody to the hepatitis B surface antigen, which is protective against hepatitis B. Hepatitis B can be transferred to an infant of a breastfeeding mother, especially if the mother's nipples are cracked. The onset of hepatitis B is insidious. Immunity develops after one exposure to hepatitis B. Hepatitis B can exist in a carrier state.

Which should the nurse teach about prevention of sickle cell crises to parents of a preschool child with sickle cell disease? (Select all that apply.) a. Limit fluids at bedtime. b. Notify the health care provider if a fever of 38.5 C (101.3 F) or greater occurs. c. Give penicillin as prescribed. d. Use ice packs to decrease the discomfort of vasoocclusive pain in the legs. e. Notify the health care provider if your child begins to develop symptoms of a cold.

ANS: B, C, E The most important issues to teach the family of a child with sickle cell anemia are to (1) seek early intervention for problems, such as a fever of 38.5 C (101.3 F) or greater; (2) give penicillin as ordered; (3) recognize signs and symptoms of splenic sequestration, as well as respiratory problems that can lead to hypoxia; and (4) treat the child normally. The nurse emphasizes the importance of adequate hydration to prevent sickling and to delay the adhesionstasisthrombosisischemia cycle. It is not sufficient to advise parents to force fluids or encourage drinking. They need specific instructions on how many daily glasses or bottles of fluid are required. Many foods are also a source of fluid, particularly soups, flavored ice pops, ice cream, sherbet, gelatin, and puddings. Increased fluids combined with impaired kidney function result in the problem of enuresis. Parents who are unaware of this fact frequently use the usual measures to discourage bedwetting, such as limiting fluids at night. Enuresis is treated as a complication of the disease, such as joint pain or some other symptom, to alleviate parental pressure on the child. Ice should not be used during a vasoocclusive pain crisis because it vasoconstricts and impairs circulation even more.

A clinic nurse is conducting a staff in-service for other clinic nurses about signs and symptoms of a rhabdomyosarcoma tumor. Which should be included in the teaching session? (Select all that apply.) a. Bone fractures b. Abdominal mass c. Sore throat and ear pain d. Headache e. Ecchymosis of conjunctiva

ANS: B, C, E The initial signs and symptoms of rhabdomyosarcoma tumors are related to the site of the tumor and compression of adjacent organs. Some tumor locations, such as the orbit, manifest early in the course of the illness. Other tumors, such as those of the retroperitoneal area, only produce symptoms when they are relatively large and compress adjacent organs. Unfortunately, many of the signs and symptoms attributable to rhabdomyosarcoma are vague and frequently suggest a common childhood illness, such as "earache" or "runny nose." An abdominal mass, sore throat and ear pain, and ecchymosis of conjunctiva are signs of a rhabdomyosarcoma tumor. Bone fractures would be seen in osteosarcoma and a headache is a sign of a brain tumor.

2. Which children admitted to the pediatric unit would the nurse monitor closely for development of syndrome of inappropriate antidiuretic hormone (SIADH)? (Select all that apply.) a. A newly diagnosed preschooler with type 1 diabetes. b. A school-age child returning from surgery for removal of a brain tumor. c. An infant with suspected meningitis. d. An adolescent with blunt abdominal trauma following a car accident. e. A school-age child with head trauma.

ANS: B, C, E The disorder that results from hypersecretion of ADH from the posterior pituitary hormone. Childhood SIADH usually is caused by disorders affecting the central nervous system, such as infections (meningitis), head trauma, and brain tumors. Type 1 diabetes and blunt abdominal trauma are not likely to cause SIADH since do not affect secretion of this hormone.

40.A school-age child has been admitted to the hospital with an exacerbation of nephrotic syndrome. Which clinical manifestations should the nurse expect to assess (Select all that apply)? a. Weight loss b. Facial edema c. Cloudy, smoky brown-colored urine d. Fatigue e. Frothy-appearing urine

ANS: B, D, E A child with nephrotic syndrome will present with facial edema, fatigue, and frothy-appearing urine (proteinuria). Weight gain, not loss, is expected because of the fluid retention. Cloudy, smoky brown-colored urine is seen with acute glomerulonephritis but not with nephrotic syndrome because there is no gross hematuria associated with nephrotic syndrome.

An adolescent will receive a bone marrow transplant (BMT). The nurse should explain that the bone marrow will be administered by which route? a. Bone grafting b. Bone marrow injection c. IV infusion d. Intra-abdominal infusion

ANS: C Bone marrow from a donor is infused intravenously, and the transfused stem cells will repopulate the marrow. Because the stem cells migrate to the recipients marrow when given intravenously, this is the method of administration.

During the first 4 days of hospitalization, Eric, age 18 months, cried inconsolably when his parents left him, and he refused the staffs attention. Now the nurse observes that Eric appears to be settled in and unconcerned about seeing his parents. The nurse should interpret this as which of the following? a. He has successfully adjusted to the hospital environment. b. He has transferred his trust to the nursing staff. c. He may be experiencing detachment, which is the third stage of separation anxiety. d. Because he is at home in the hospital now, seeing his mother frequently will only start the cycle again.

ANS: C Detachment is a behavioral manifestation of separation anxiety. Superficially it appears that the child has adjusted to the loss and transferred his trust to the nursing staff. Detachment is a sign of resignation, not contentment. Parents should be encouraged to be with their child. If parents restrict visits, they may begin a pattern of misunderstanding the childs cues and not meeting his needs.

What is the best action for the nurse to take when a 5-year-old child who requires another 2 days of intravenous (IV) antibiotics cries, screams, and resists having the IV restarted? a. Exit the room and leave the child alone until he stops crying. b. Tell the child big boys and girls dont cry. c. Let the child decide which color arm board to use with the IV. d. Administer a narcotic analgesic for pain to quiet the child.

ANS: C Giving the preschooler some choice and control, while maintaining boundaries of treatment, supports the childs coping skills. Leaving the child alone robs the child of support when a coping difficulty exists. Crying is a normal response to stress. The child needs time to adjust and support to cope with unfamiliar and painful procedures during hospitalization. Although administration of a topical analgesic is indicated before restarting the childs IV, a narcotic analgesic is not indicated.

Which type of seizure involves both hemispheres of the brain? a. Focal b. Partial c. Generalized d. Acquired

ANS: C Clinical observations of generalized seizures indicate that the initial involvement is from both hemispheres. Focal seizures may arise from any area of the cerebral cortex, but the frontal, temporal, and parietal lobes are most commonly affected. Partial seizures are caused by abnormal electric discharges from epileptogenic foci limited to a circumscribed region of the cerebral cortex. A seizure disorder that is acquired is a result of a brain injury from a variety of factors; it does not specify the type of seizure.

The nurse is teaching parents of an infant about the causes of iron deficiency anemia. Which statement best describes iron deficiency anemia in infants? a. It is caused by depression of the hematopoietic system. b. It is easily diagnosed because of an infants emaciated appearance. c. Clinical manifestations are similar regardless of the cause of the anemia. d. Clinical manifestations result from a decreased intake of milk and the premature addition of solid foods.

ANS: C In iron deficiency anemia, the childs clinical appearance is a result of the anemia, not the underlying cause. Usually the hematopoietic system is not depressed in iron deficiency anemia. The bone marrow produces red cells that are smaller and contain less hemoglobin than normal red cells. Children who are iron deficient from drinking excessive quantities of milk are usually pale and overweight. They are receiving sufficient calories, but are deficient in essential nutrients. The clinical manifestations result from decreased intake of iron-fortified solid foods and an excessive intake of milk.

Which statement best describes b-thalassemia major (Cooley anemia)? a. All formed elements of the blood are depressed. b. Inadequate numbers of red blood cells are present. c. Increased incidence occurs in families of Mediterranean extraction. d. Increased incidence occurs in persons of West African descent.

ANS: C Individuals who live near the Mediterranean Sea and their descendants have the highest incidence of thalassemia. An overproduction of red cells occurs. Although numerous, the red cells are relatively unstable. Sickle cell disease is common in persons of West African descent.

The nurse is caring for an adolescent who had an external fixator placed after suffering a fracture of the wrist during a bicycle accident. Which statement by the adolescent would be expected about separation anxiety? a. I wish my parents could spend the night with me while I am in the hospital. b. I think I would like for my siblings to visit me but not my friends. c. I hope my friends dont forget about visiting me. d. I will be embarrassed if my friends come to the hospital to visit.

ANS: C Loss of peer-group contact may pose a severe emotional threat to an adolescent because of loss of group status; friends visiting is an important aspect of hospitalization for an adolescent and would be very reassuring. Adolescents may welcome the opportunity to be away from their parents. The separation from siblings may produce reactions from difficulty coping to a welcome relief.

Emma, age 3 years, is being admitted for about 1 week of hospitalization. Her parents tell the nurse that they are going to buy her a lot of new toys because she will be in the hospital. The nurses reply should be based on an understanding that: a. New toys make hospitalization easier. b. New toys are usually better than older ones for children of this age. c. At this age children often need the comfort and reassurance of familiar toys from home. d. Buying new toys for a hospitalized child is a maladaptive way to cope with parental guilt.

ANS: C Parents should bring favorite items from home to be with the child. Young children associate inanimate objects with significant people; they gain comfort and reassurance from these items. New toys will not serve the purpose of familiar toys and objects from home. The parents may experience some guilt as a response to the hospitalization, but there is no evidence that it is maladaptive.

Parents of a hemophiliac child ask the nurse, Can you describe hemophilia to us? Which response by the nurse is descriptive of most cases of hemophilia? a. Autosomal dominant disorder causing deficiency in a factor involved in the blood-clotting reaction b. X-linked recessive inherited disorder causing deficiency of platelets and prolonged bleeding c. X-linked recessive inherited disorder in which a blood-clotting factor is deficient d. Y-linked recessive inherited disorder in which the red blood cells become moon-shaped

ANS: C The inheritance pattern in 80% of all of the cases of hemophilia is X-linked recessive. The two most common forms of the disorder are factor VIII deficiency, hemophilia A or classic hemophilia; and factor IX deficiency, hemophilia B or Christmas disease. The inheritance pattern is X-linked recessive. The disorder involves coagulation factors, not platelets, and does not involve red cells or the Y chromosomes.

A school-age child with leukemia experienced severe nausea and vomiting when receiving chemotherapy for the first time. Which is the most appropriate nursing action to prevent or minimize these reactions with subsequent treatments? a. Encourage drinking large amounts of favorite fluids. b. Encourage child to take nothing by mouth (remain NPO) until nausea and vomiting subside. c. Administer an antiemetic before chemotherapy begins. d. Administer an antiemetic as soon as child has nausea.

ANS: C The most beneficial regimen to minimize nausea and vomiting associated with chemotherapy is to administer the antiemetic before the chemotherapy is begun. The goal is to prevent anticipatory symptoms. Drinking fluids will add to the discomfort of the nausea and vomiting. Remaining until nausea and vomiting subside will help with this episode, but the child will have the discomfort and be at risk for dehydration. Administering an antiemetic as soon as the child has nausea does not prevent anticipatory nausea.

Which should the nurse include when teaching the mother of a 9-month-old infant about administering liquid iron preparations? a. They should be given with meals. b. They should be stopped immediately if nausea and vomiting occur. c. Adequate dosage will turn the stools a tarry green color. d. Allow preparation to mix with saliva and bathe the teeth before swallowing.

ANS: C The nurse should prepare the mother for the anticipated change in the childs stools. If the iron dose is adequate, the stools will become a tarry green color. The lack of the color change may indicate insufficient iron. The iron should be given in two divided doses between meals when the presence of free hydrochloric acid is greatest. Iron is absorbed best in an acidic environment. Vomiting and diarrhea may occur with iron administration. If these occur, the iron should be given with meals, and the dosage reduced, then gradually increased as the child develops tolerance. Liquid preparations of iron stain the teeth. They should be administered through a straw and the mouth rinsed after administration.

4. What describes moral development in younger school-age children? a. The standards of behavior now come from within themselves. b. They do not yet experience a sense of guilt when they misbehave. c. They know the rules and behaviors expected of them but do not understand the reasons behind them. d. They no longer interpret accidents and misfortunes as punishment for misdeeds.

ANS: C Children who are ages 6 and 7 years know the rules and behaviors expected of them but do not understand the reasons for them. Young children do not believe that standards of behavior come from within themselves but that rules are established and set down by others. Younger school-age children learn standards for acceptable behavior, act according to these standards, and feel guilty when they violate them. Misfortunes and accidents are viewed as punishment for bad acts.

The nurse is admitting a child with Werdnig-Hoffmann disease (spinal muscular atrophy type 1). Which signs and symptoms are associated with this disease? a. Spinal muscular atrophy b. Neural atrophy of muscles c. Progressive weakness and wasting of skeletal muscle d. Pseudohypertrophy of certain muscle groups

ANS: C Werdnig-Hoffmann disease (spinal muscular atrophy type 1) is the most common paralytic form of floppy infant syndrome (congenital hypotonia). It is characterized by progressive weakness and wasting of skeletal muscle caused by degeneration of anterior horn cells. Kugelberg-Welander disease is a juvenile spinal muscular atrophy with a later onset. Charcot-Marie-Tooth disease is a form of progressive neural atrophy of muscles supplied by the peroneal nerves. Progressive weakness is found of the distal muscles of the arms and feet. Duchenne muscular dystrophy is characterized by muscles, especially in the calves, thighs, and upper arms, which become enlarged from fatty infiltration and feel unusually firm or woody on palpation. The term pseudohypertrophy is derived from this muscular enlargement.

A 14-year-old boy is being admitted to the hospital for an appendectomy. Which roommate should the nurse assign with this patient? a. A 4-year-old boy who is first day post-appendectomy surgery b. A 6-year-old boy with pneumonia c. A 15-year-old boy admitted with a vaso-occlusive sickle cell crisis d. A 12-year-old boy with cellulitis

ANS: C When a child is admitted, nurses follow several fairly universal admission procedures. The minimum considerations for room assignment are age, sex, and nature of the illness. Age-grouping is especially important for adolescents. The 14-year-old boy being admitted to the unit after appendectomy surgery should be placed with a noninfectious child of the same sex and age. The 15-year-old child with sickle cell is the best choice. The 4-year-old boy who is post-appendectomy is too young, and the child with pneumonia is too young and possibly has an infectious process. The 12-year-old boy with cellulitis is the right age, but he has an infection (cellulitis).

Which neurologic diagnostic test gives a visualized horizontal and vertical cross-section of the brain at any axis? a. Nuclear brain scan b. Echoencephalography c. CT scan d. Magnetic resonance imaging (MRI)

ANS: C A CT scan provides a visualization of the horizontal and vertical cross-sections of the brain at any axis. A nuclear brain scan uses a radioisotope that accumulates where the blood-brain barrier is defective. Echoencephalography identifies shifts in midline structures of the brain as a result of intracranial lesions. MRI permits visualization of morphologic features of target structures and permits tissue discrimination that is unavailable with any other techniques.

A child has been seizure-free for 2 years. A father asks the nurse how much longer the child will need to take the antiseizure medications. The nurse includes which intervention in the response? a. Medications can be discontinued at this time. b. The child will need to take the drugs for 5 years after the last seizure. c. A step-wise approach will be used to reduce the dosage gradually. d. Seizure disorders are a lifelong problem. Medications cannot be discontinued.

ANS: C A predesigned protocol is used to wean a child gradually off antiseizure medications, usually when the child is seizure-free for 2 years and has a normal electroencephalogram (EEG). Medications must be gradually reduced to minimize the recurrence of seizures. Seizure medications can be safely discontinued. The risk of recurrence is greatest within the first year.

A 3-year-old child is hospitalized after a submersion injury. The child's mother complains to the nurse, "Being at the hospital seems unnecessary when he is perfectly fine." The nurse's best reply should be: a. "He still needs a little extra oxygen." b. "I'm sure he is fine, but the doctor wants to make sure." c. "The reason for this is that complications could still occur." d. "It is important to observe for possible central nervous system problems."

ANS: C All children who have a submersion injury should be admitted to the hospital for observation. Although many children do not appear to have suffered adverse effects from the event, complications such as respiratory compromise and cerebral edema may occur 24 hours after the incident. The mother would not think the child is fine if oxygen were still required. The nurse should clarify that different complications can occur up to 24 hours later and that observations are necessary.

25. A nurse is caring for four patients who have Hodgkin's lymphoma. Which child should the nurse see first? A. Anorexia for a week B. Enlarged cervical lymph nodes C. Fever of 102.1°F (38.9°C) D. Mediastinal mass

ANS: C All options are possible manifestations of Hodgkin's lymphoma. However, the child with a fever may have another cause for the temperature, including infection, that needs to be ruled out. This is especially true of a child receiving chemotherapy, a standard treatment for this disorder.

7. A child is diagnosed with influenza, probably type A disease. Management includes: a. Clear liquid diet for hydration. b. Aspirin to control fever. c. Amantadine hydrochloride to reduce symptoms. d. Antibiotics to prevent bacterial infection

ANS: C Amantadine hydrochloride may reduce symptoms related to influenza type A if administered within 24 to 48 hours of onset. It is ineffective against type B or C. A clear liquid diet is not necessary for influenza, but maintaining hydration is important. Aspirin is not recommended in children because of increased risk of Reyes syndrome. Acetaminophen or ibuprofen is a better choice. Preventive antibiotics are not indicated for influenza unless there is evidence of a secondary bacterial infection

35. The nurse is teaching parents of a child with chronic renal failure (CRF) about the use of recombinant human erythropoietin (rHuEPO) subcutaneous injections. Which statement indicates the parents have understood the teaching? a. "These injections will help with the hypertension." b. "We're glad the injections only need to be given once a month." c. "The red blood cell count should begin to improve with these injections." d. "Urine output should begin to improve with these injections."

ANS: C Anemia in children with CRF is related to decreased production of erythropoietin. Recombinant human erythropoietin (rHuEPO) is being offered to these children as thrice-weekly or weekly subcutaneous injections and is replacing the need for frequent blood transfusions. The parents understand the teaching if they say that the red blood cell count will begin to improve with these injections.

19. A major complication in a child with chronic renal failure is: a. Hypokalemia. b. Metabolic alkalosis. c. Water and sodium retention. d. Excessive excretion of blood urea nitrogen.

ANS: C Chronic renal failure leads to water and sodium retention, which contributes to edema and vascular congestion. Hyperkalemia, metabolic acidosis, and retention of blood urea nitrogen are complications of chronic renal failure.

A young child's parents call the nurse after their child was bitten by a raccoon in the woods. The nurse's recommendation should be based on which statement? a. Child should be hospitalized for close observation. b. No treatment is necessary if thorough wound cleaning is done. c. Antirabies prophylaxis must be initiated. d. Antirabies prophylaxis must be initiated if clinical manifestations appear.

ANS: C Current therapy for a rabid animal bite consists of a thorough cleansing of the wound and passive immunization with human rabies immune globulin (HRIG) as soon as possible. Hospitalization is not necessary. The wound cleansing, passive immunization, and immune globulin administration can be done as an outpatient. The child needs to receive both HRIG and rabies vaccine.

The nurse is monitoring a 7-year-old child post-surgical resection of an infratentorial brain tumor. Which vital sign findings indicate Cushing's triad? a. Increased temperature, tachycardia, tachypnea b. Decreased temperature, bradycardia, bradypnea c. Bradycardia, hypertension, irregular respirations d. Bradycardia, hypotension, tachypnea

ANS: C Cushing's triad is a hallmark sign of increased intracranial pressure (ICP). The triad includes bradycardia, hypertension, and irregular respirations. Increased or decreased temperature is not a sign of Cushing's triad.

9. A 7-year-old child presents to the emergency department, where the parent reports a 3-week history of pale skin, extreme fatigue, and dizziness. Which laboratory value would the nurse correlate with the patient's current condition? A. Hematocrit: 33% B. Hemoglobin: 13.2 g/dL C. Red blood cell count: 2.8/mm3 D. White blood cell count: 12.3/mm3

ANS: C For a child of this age, a normal RBC count is 4-5.2/mm3. Low RBCs can lead to pallor, fatigue, headaches, and dizziness, as tissues are not being oxygenated. The other laboratory values are normal.

32. The nurse is teaching the parent about the diet of a child experiencing severe edema associated with acute glomerulonephritis. Which information should the nurse include in the teaching? a. "You will need to decrease the number of calories in your child's diet." b. "Your child's diet will need an increased amount of protein." c. "You will need to avoid adding salt to your child's food." d. "Your child's diet will consist of low-fat, low-carbohydrate foods."

ANS: C For most children, a regular diet is allowed, but it should contain no added salt. The child should be offered a regular diet with favorite foods. Severe sodium restrictions are not indicated.

24. A nurse is teaching parents of first-grade children general guidelines to assist their children in adapting to school. Which statement by the parents indicates they understand the teaching? a. "We will only meet with the teacher if problems occur." b. "We will discourage hobbies so our child focuses on schoolwork." c. "We will plan a trip to the library as often as possible." d. "We will expect our child to make all As in school."

ANS: C General guidelines for parents to help their child in school include sharing an interest in reading. The library should be used frequently and books the child is reading should be discussed. Hobbies should be encouraged. The parents should not expect all As. They should focus on growth more than grades.

13.The most appropriate nursing diagnosis for the child with acute glomerulonephritis is: a. Risk for Injury related to malignant process and treatment. b. Deficient Fluid Volume related to excessive losses. c. Excess Fluid Volume related to decreased plasma filtration. d. Excess Fluid Volume related to fluid accumulation in tissues and third spaces.

ANS: C Glomerulonephritis has a decreased filtration of plasma. The decrease in plasma filtration results in an excessive accumulation of water and sodium that expands plasma and interstitial fluid volumes, leading to circulatory congestion and edema. No malignant process is involved in acute glomerulonephritis. A fluid volume excess is found. The fluid accumulation is secondary to the decreased plasma filtration, not fluid accumulation.

The most common clinical manifestation(s) of brain tumors in children is/are: a. irritability. b. seizures. c. headaches and vomiting. d. fever and poor fine motor control.

ANS: C Headaches, especially on awakening, and vomiting that is not related to feeding are the most common clinical manifestation(s) of brain tumors in children. Irritability, seizures, and fever and poor fine motor control are clinical manifestations of brain tumors, but headaches and vomiting are the most common.

A school nurse is conducting a staff in-service to other school nurses on idiopathic scoliosis. During which period of child development does idiopathic scoliosis become most noticeable? a. Newborn period b. When child starts to walk c. Preadolescent growth spurt d. Adolescence

ANS: C Idiopathic scoliosis is most noticeable during the preadolescent growth spurt. Idiopathic scoliosis is seldom apparent before age 10 years. Diagnosis usually occurs during the preadolescent growth spurt.

Which is the initial clinical manifestation of generalized seizures? a. Being confused b. Feeling frightened c. Losing consciousness d. Seeing flashing lights

ANS: C Loss of consciousness is a frequent occurrence in generalized seizures and is the initial clinical manifestation. Being confused, feeling frightened, and seeing flashing lights are clinical manifestations of a complex partial seizure.

An appropriate nursing intervention when caring for an unconscious child should be to: a. change the child's position infrequently to minimize the chance of increased ICP. b. avoid using narcotics or sedatives to provide comfort and pain relief. c. monitor fluid intake and output carefully to avoid fluid overload and cerebral edema. d. give tepid sponge baths to reduce fever because antipyretics are contraindicated.

ANS: C Often comatose patients cannot cope with the quantity of fluids that they normally tolerate. Overhydration must be avoided to prevent fatal cerebral edema. The child's position should be changed frequently to avoid complications such as pneumonia and skin breakdown. Narcotics and sedatives should be used as necessary to reduce pain and discomfort, which can increase ICP. Antipyretics are the method of choice for fever reduction.

26. A nurse planning care for a school-age child should take into account that which thought process is seen at this age? a. Animism b. Magical thinking c. Ability to conserve d. Thoughts are all-powerful

ANS: C One cognitive task of school-age children is mastering the concept of conservation. At an early age (5 to 7 years), children grasp the concept of reversibility of numbers as a basis for simple mathematics problems (e.g., 2 + 4 = 6 and 6 − 4 = 2). They learn that simply altering their arrangement in space does not change certain properties of the environment, and they are able to resist perceptual cues that suggest alterations in the physical state of an object. Animism, magical thinking, and believing that thoughts are all-powerful are thought processes seen in preschool children.

22. A parent brings a 10-year-old child to the clinic, reporting that the child fell while playing and now has a limp several days later. In completing a history, which other finding would the nurse correlate more with bone cancer than a minor trauma? A. Decreased appetite for the last month B. Fatigues easily when playing outdoors C. Limping several weeks prior to the fall D. Often has unexplained extremity bruises

ANS: C Pain and swelling are the most common manifestations of osteosarcoma. Often the child has a limp. The child also may have a dull pain at the tumor site, and if it is on a leg (weight-bearing), it could easily cause a limp that has lasted for several weeks before really being noticed. The other manifestations are vague and could be related to other problems.

A child is brought to the emergency department after experiencing a seizure at school. There is no previous history of seizures. The father tells the nurse that he cannot believe the child has epilepsy. The nurse's best response is: a. "Epilepsy is easily treated." b. "Very few children have actual epilepsy." c. "The seizure may or may not mean that your child has epilepsy." d. "Your child has had only one convulsion; it probably won't happen again."

ANS: C Seizures are the indispensable characteristic of epilepsy; however, not every seizure is epileptic. Epilepsy is a chronic seizure disorder with recurrent and unprovoked seizures. The treatment of epilepsy involves a thorough assessment to determine the type of seizure the child is having and the cause, followed by individualized therapy to allow the child to have as normal a life as possible. The nurse should not make generalized comments regarding the incidence of epilepsy until further assessment is made.

A neonate is born with bilateral mild talipes equinovarus (clubfoot). When the parents ask the nurse how this will be corrected, the nurse should give which explanation? a. Traction is tried first. b. Surgical intervention is needed. c. Frequent, serial casting is tried first. d. Children outgrow this condition when they learn to walk.

ANS: C Serial casting is begun shortly after birth before discharge from nursery. Successive casts allow for gradual stretching of skin and tight structures on the medial side of the foot. Manipulation and casting of the leg are repeated frequently (every week) to accommodate the rapid growth of early infancy. Serial casting is the preferred treatment. Surgical intervention is done only if serial casting is not successful. Children do not improve without intervention.

15. A clinic nurse notes that a child brought in for a physical has swelling and bruising around the eyes. The patient denies any trauma and the parent reports no environmental allergies. Which assessment is most important? A. Auscultate lungs bilaterally. B. Inspect skin on the back. C. Palpate abdomen and neck. D. Percuss abdomen and flank.

ANS: C Swelling and bruising around the face and eyes is often seen in children with neuroblastoma. Most commonly the tumor can be found by palpation of the abdomen or neck, where the tumor will present as a hard, painless mass that crosses the midline.

22. A parent whose two school-age children have asthma asks the nurse in what sports, if any, they can participate. The nurse should recommend: a. Soccer. c. Swimming. b. Running. d. Basketball.

ANS: C Swimming is well tolerated in children with asthma because they are breathing air fully saturated with moisture and because of the type of breathing required in swimming. Exercise-induced bronchospasm is more common in sports that involve endurance, such as soccer, running, and basketball. Prophylaxis with medications may be necessary

33. Which intervention for treating croup at home should be taught to parents? a. Have a decongestant available to give the child when an attack occurs. b. Have the child sleep in a dry room. c. Take the child outside. d. Give the child an antibiotic at bedtime.

ANS: C Taking the child into the cool, humid, night air may relieve mucosal swelling and improve symptoms. Decongestants are inappropriate for croup, which affects the middle airway level. A dry environment may contribute to symptoms. Croup is caused by a virus. Antibiotic treatment is not indicated

20. Identify the statement that is the most accurate about moral development in the 9-year-old school-age child. a. Right and wrong are based on physical consequences of behavior. b. The child obeys parents because of fear of punishment. c. The school-age child conforms to rules to please others. d. Parents are the determiners of right and wrong for the school-age child.

ANS: C The 7- to 12-year-old child bases right and wrong on a good-boy or good-girl orientation in which the child conforms to rules to please others and avoid disapproval. Children 4 to 7 years of age base right and wrong on consequences, the most important consideration for this age-group. Parents determine right and wrong for the child younger than 4 years of age.

16. A nurse notes in a patient's medical record high levels of vanillymandelic acid (VMA). Based on this information, which condition does the nurse prepare to educate the patient and family about? A. Ewing's sarcoma B. Hodgkin's lymphoma C. Neuroblastoma D. Wilms' tumor

ANS: C VMA and homovanillic acid (HVA) are tests used to measure the level of catecholamine metabolites in the urine. Neuroblastomas typically secrete catecholamines, so high levels of either substance are indicative of neuroblastoma.

17. A nursing student is caring for a child diagnosed with Wilms' tumor. Which action by the student causes the faculty member to intervene? A. Assesses urinary output per protocol B. Involves the parents in the child's care C. Palpates the abdomen in all four quadrants D. Provides frequent nutritious snacks

ANS: C Wilms' tumor is a solid, encapsulated mass that can rupture with palpation. Once the child is diagnosed with this cancer, palpation of the child's abdomen is prohibited. The other actions are appropriate.

28. A child diagnosed with hypopituitarism is being started on growth hormone (GH) therapy. Nursing considerations should be based on which information? a. Treatment is most successful if it is started during adolescence. b. Treatment is considered successful if children attain full stature by adulthood. c. Replacement therapy requires daily subcutaneous injections. d. Replacement therapy will be required throughout the child's lifetime.

ANS: C Additional support is required for children who require hormone replacement therapy, such as preparation for daily subcutaneous injections and education for self-management during the school-age years. Young children, obese children, and those who are severely GH deficient have the best response to therapy. When therapy is successful, children can attain their actual or near-final adult height at a slower rate than their peers. Replacement therapy is not needed after attaining final height. They are no longer GH deficient.

33. To help the adolescent deal with diabetes, the nurse must consider which characteristic of adolescence? a. Desire to be unique b. Preoccupation with the future c. Need to be perfect and similar to peers d. Need to make peers aware of the seriousness of hypoglycemic reactions

ANS: C Adolescence is a time when the individual wants to be perfect and similar to peers. Having diabetes makes adolescents different from their peers. Adolescents do not wish to be unique; they desire to fit in with the peer group and are usually not future oriented. Forcing peer awareness of the seriousness of hypoglycemic reactions would further alienate the adolescent with diabetes since the peer group would likely focus on the differences.

20. A parent asks the nurse why self-monitoring of blood glucose is being recommended for her child with diabetes. The nurse should base the explanation on what information? a. It is a less expensive method of testing. b. It is not as accurate as laboratory testing. c. Children need to learn to manage their diabetes. d. The parents are better able to manage the disease.

ANS: C Blood glucose self-management has improved diabetes management and can be used successfully by children from the time of diagnosis. Insulin dosages can be adjusted based on blood sugar results. Blood glucose monitoring is more expensive but provides improved management. It is as accurate as equivalent testing done in laboratories. The ability to self-test allows the child to balance diet, exercise, and insulin. The parents are partners in the process, but the child should be taught how to manage the disease.

15. Which statement is most descriptive of Meckel's diverticulum? a. It is more common in females than in males. b. It is acquired during childhood. c. Intestinal bleeding may be mild or profuse. d. Medical interventions are usually sufficient to treat the problem.

ANS: C Blood stools are often a presenting sign of Meckel's diverticulum. It is associated with mild-to-profuse intestinal bleeding. It is twice as common in males as in females, and complications are more frequent in males. Meckel's diverticulum is the most common congenital malformation of the gastrointestinal tract and is present in 2% of the general population. The standard therapy is surgical removal of the diverticulum.

11. The nurse is teaching the parents of a child who is receiving propylthiouracil for the treatment of hyperthyroidism (Graves' disease). Which statement made by the parent indicates a correct understanding of the teaching? a. "I would expect my child to gain weight while taking this medication." b. "I would expect my child to experience episodes of ear pain while taking this medication." c. "If my child develops a sore throat and fever, I should contact the physician immediately." d. "If my child develops the stomach flu, my child will need to be hospitalized."

ANS: C Children being treated with propylthiouracil must be carefully monitored for the side effects of the drug. Parents must be alerted that sore throat and fever accompany the grave complication of leukopenia. These symptoms should be immediately reported. Weight gain, episodes of ear pain, and stomach flu are not usually associated with leukopenia.

21. The parents of a child who has just been diagnosed with type 1 diabetes ask about exercise. The nurse should provide the parents with what information to address the child's safety needs? a. Exercise will increase blood glucose. b. Exercise should be restricted. c. Extra snacks are needed before exercise. d. Extra insulin is required during exercise.

ANS: C Exercise lowers blood glucose levels, which can be compensated for by extra snacks. Exercise is encouraged and not restricted unless indicated by other health conditions. Extra insulin is contraindicated because exercise decreases blood glucose levels.

1. Which statement best describes idiopathic hypopituitarism? a. Growth is normal during the first 3 years of life. b. Weight is usually more retarded than height. c. Skeletal proportions are normal for age. d. Most of these children have subnormal intelligence.

ANS: C In children with idiopathic hypopituitarism, the skeletal proportions are normal. Growth is within normal limits for the first year of life. Height is usually more delayed than weight. Intelligence is not affected by hypopituitarism.

1. Which is most descriptive of the pathophysiology of leukemia? a. Increased blood viscosity occurs. b. Thrombocytopenia (excessive destruction of platelets) occurs. c. Unrestricted proliferation of immature white blood cells (WBCs) occurs. d. First stage of coagulation process is abnormally stimulated.

ANS: C Leukemia is a group of malignant disorders of the bone marrow and lymphatic system. It is defined as an unrestricted proliferation of immature WBCs in the blood-forming tissues of the body. Increased blood viscosity may occur secondary to the increased number of WBCs. Thrombocytopenia may occur secondary to the overproduction of WBCs in the bone marrow. The coagulation process is unaffected by leukemia.

4. At what age is sexual development in boys and girls considered to be precocious? a. Boys, 11 years; girls, 9 years b. Boys, 12 years; girls, 10 years c. Boys, 9 years; girls, 8 years d. Boys, 10 years; girls, 9.5 years

ANS: C Manifestations of sexual development before age 9 in boys and age 8 in girls are considered precocious and should be investigated. Boys older than 9 years of age and girls older than 8 years of age fall within the expected range of pubertal onset.

7. Therapeutic management of the child with acute diarrhea and dehydration usually begins with what intervention? a. Clear liquids b. Adsorbents such as kaolin and pectin c. Oral rehydration solution (ORS) d. Antidiarrheal medications such as paregoric

ANS: C ORS is the first treatment for acute diarrhea. Clear liquids are not recommended because they contain too much sugar, which may contribute to diarrhea. Adsorbents are not recommended and neither are antidiarrheal because they do not get rid of pathogens.

5. The viral pathogen that frequently causes acute diarrhea in young children is: a. Giardia organisms. b. Shigella organisms. c. Rotavirus. d. Salmonella organisms.

ANS: C Rotavirus is the most frequent viral pathogen that causes diarrhea in young children. Giardia and Salmonella are bacterial pathogens that cause diarrhea. Shigella is a bacterial pathogen that is uncommon in the United States.

14. When caring for a child with probable appendicitis, the nurse should be alert to recognize what sign of perforation? a. Bradycardia b. Anorexia c. Sudden relief from pain d. Decreased abdominal distention

ANS: C Signs of peritonitis, in addition to fever, include sudden relief from pain after perforation. Tachycardia, not bradycardia, is a manifestation of peritonitis. Anorexia is already a clinical manifestation of appendicitis. Abdominal distention usually increases in addition to an increase in pain (usually diffuse and accompanied by rigid guarding of the abdomen).

26. What is the major focus of the therapeutic management for a child with lactose intolerance? a. Compliance with the medication regimen b. Providing emotional support to family members c. Teaching dietary modifications d. Administration of daily normal saline enemas

ANS: C Simple dietary modifications are effective in the management of lactose intolerance. Symptoms of lactose intolerance are usually relieved after instituting a lactose-free diet. Medications are not typically ordered in the management of lactose intolerance. Providing emotional support to family members is not specific to this medical condition. Diarrhea is a manifestation of lactose intolerance. Enemas are contraindicated for this alteration in bowel elimination.

16. What is the characteristic of the immune-mediated type 1 diabetes mellitus? a. Ketoacidosis is infrequent b. Onset is gradual c. Age at onset is usually younger than 18 years d. Oral agents are often effective for treatment

ANS: C The immune-mediated type 1 diabetes mellitus typically has its onset in children or young adults. Peak incidence is between the ages of 10 and 15 years. Infrequent ketoacidosis, gradual onset, and treatment with oral agents are more consistent with type 2 diabetes

12. What is the primary purpose of prescribing a histamine receptor antagonist for an infant diagnosed with gastroesophageal reflux? a. Prevent reflux b. Prevent hematemesis. c. Reduce gastric acid production. d. Increase gastric acid production.

ANS: C The mechanism of action of histamine receptor antagonists is to reduce the amount of acid present in gastric contents and may prevent esophagitis. None of the remaining options are modes of action of histamine receptor antagonists but rather desired effects of medication therapy.

5. A nurse is planning care for a school-age child diagnosed with type 1 diabetes. Which insulin preparations are either rapid or short acting? (Select all that apply.) a. Novolin N b. Lantus c. NovoLog d. Novolin R

ANS: C, D Rapid-acting insulin (e.g., NovoLog) reaches the blood within 15 minutes after injection. The insulin peaks 30 to 90 minutes later and may last as long as 5 hours. Short-acting (regular) insulin (e.g., Novolin R) usually reaches the blood within 30 minutes after injection. The insulin peaks 2 to 4 hours later and stays in the blood for about 4 to 8 hours. Intermediate-acting insulins (e.g., Novolin N) reach the blood 2 to 6 hours after injection. The insulins peak 4 to 14 hours later and stay in the blood for about 14 to 20 hours. Long-acting insulin (e.g., Lantus) takes 6 to 14 hours to start working. It has no peak or a very small peak 10 to 16 hours after injection. The insulin stays in the blood between 20 and 24 hours.

5. A nurse is conducting dietary teaching on high-fiber foods for parents of a child with constipation. Which foods should the nurse include as being high in fiber? (Select all that apply.) a. White rice b. Avocados c. Whole grain breads d. Bran pancakes e. Raw carrots

ANS: C, D, E High-fiber foods include whole grain breads, bran pancakes, and raw carrots. Unrefined (brown) rice is high in fiber but white rice is not. Raw fruits, especially those with skins or seeds, other than ripe banana or avocados are high in fiber

The nurse is preparing to admit a newborn with myelomeningocele to the neonatal intensive care nursery. Which describes this newborns defect? a. Fissure in the spinal column that leaves the meninges and the spinal cord exposed b. Herniation of the brain and meninges through a defect in the skull c. Hernial protrusion of a saclike cyst of meninges with spinal fluid but no neural elements d. Visible defect with an external saclike protrusion containing meninges, spinal fluid, and nerves

ANS: D A myelomeningocele is a visible defect with an external saclike protrusion, containing meninges, spinal fluid, and nerves. Rachischisis is a fissure in the spinal column that leaves the meninges and the spinal cord exposed. Encephalocele is a herniation of brain and meninges through a defect in the skull, producing a fluid-filled sac. Meningocele is a hernial protrusion of a saclike cyst of meninges with spinal fluid, but no neural elements.

Which clinical manifestation should the nurse expect when a child with sickle cell anemia experiences an acute vasoocclusive crisis? a. Circulatory collapse b. Cardiomegaly, systolic murmurs c. Hepatomegaly, intrahepatic cholestasis d. Painful swelling of hands and feet; painful joints

ANS: D A vasoocclusive crisis is characterized by severe pain in the area of involvement. If in the extremities, painful swelling of the hands and feet is seen; if in the abdomen, severe pain resembles that of acute surgical abdomen; and if in the head, stroke and visual disturbances occur. Circulatory collapse results from sequestration crises. Cardiomegaly, systolic murmurs, hepatomegaly, and intrahepatic cholestasis result from chronic vasoocclusive phenomena.

Which is a common clinical manifestation of Hodgkin disease? a. Petechiae b. Bone and joint pain c. Painful, enlarged lymph nodes d. Enlarged, firm, nontender lymph nodes

ANS: D Asymptomatic, enlarged, cervical or supraclavicular lymphadenopathy is the most common presentation of Hodgkin disease. Petechiae are usually associated with leukemia. Bone and joint pain are not likely in Hodgkin disease. The enlarged nodes are rarely painful.

The parents of a child with cerebral palsy ask the nurse whether any drugs can decrease their childs spasticity. The nurses response should be based on which statement? a. Anticonvulsant medications are sometimes useful for controlling spasticity. b. Medications that would be useful in reducing spasticity are too toxic for use with children. c. Many different medications can be highly effective in controlling spasticity. d. Implantation of a pump to deliver medication into the intrathecal space to decrease spasticity has recently become available.

ANS: D Baclofen, given intrathecally, is best suited for children with severe spasticity that interferes with activities of daily living and ambulation. Anticonvulsant medications are used when seizures occur in children with cerebral palsy. The intrathecal route decreases the side effects of the drugs that reduce spasticity. Few medications are currently available for the control of spasticity.

Which is the most effective pain-management approach for a child who is having a bone marrow aspiration? a. Relaxation techniques b. Administration of an opioid c. EMLA cream applied over site d. Conscious or unconscious sedation

ANS: D Effective pharmacologic and nonpharmacologic measures should be used to minimize pain associated with procedures. For bone marrow aspiration, conscious or unconscious sedation should be used. Relaxation, opioids, and EMLA can be used to augment the conscious or unconscious sedation.

The nurse is conducting reflex testing on infants at a well-child clinic. Which reflex finding should be reported as abnormal and considered as a possible sign of cerebral palsy? a. Tonic neck reflex at 5 months of age b. Absent Moro reflex at 8 months of age c. Moro reflex at 3 months of age d. Extensor reflex at 7 months of age

ANS: D Establishing a diagnosis of cerebral palsy (CP) may be confirmed with the persistence of primitive reflexes: (1) either the asymmetric tonic neck reflex or persistent Moro reflex (beyond 4 months of age) and (2) the crossed extensor reflex. The tonic neck reflex normally disappears between 4 and 6 months of age. The crossed extensor reflex, which normally disappears by 4 months, is elicited by applying a noxious stimulus to the sole of one foot with the knee extended. Normally, the contralateral foot responds with extensor, abduction, and then adduction movements. The possibility of CP is suggested if these reflexes occur after 4 months.

The nurse is reviewing first aid with a group of school nurses. Which statement made by a participant indicates a correct understanding of the information? a. If a child loses a tooth due to injury, I should place the tooth in warm milk. b. If a child has recurrent abdominal pain, I should send him or her back to class until the end of the day. c. If a child has a chemical burn to the eye, I should irrigate the eye with normal saline. d. If a child has a nosebleed, I should have the child sit up and lean forward.

ANS: D If a child has a nosebleed, the child should lean forward, not lie down. A tooth should be placed in cold milk or saliva for transporting to a dentist. Recurrent abdominal pain is a physiologic problem and requires further evaluation. If a chemical burn occurs in the eye, the eye should be irrigated with water for 20 minutes.

Several complications can occur when a child receives a blood transfusion. Which is an immediate sign or symptom of an air embolus? a. Chills and shaking b. Nausea and vomiting c. Irregular heart rate d. Sudden difficulty in breathing

ANS: D Signs of air embolism are sudden difficulty breathing, sharp pain in the chest, and apprehension. Air emboli should be avoided by carefully flushing all tubing of air before connecting to patient. Chills, shaking, nausea, and vomiting are associated with hemolytic reactions. Irregular heart rate is associated with electrolyte disturbances and hypothermia.

Which immunization should not be given to a child receiving chemotherapy for cancer? a. Tetanus vaccine b. Inactivated poliovirus vaccine c. Diphtheria, pertussis, tetanus (DPT) d. Measles, rubella, mumps

ANS: D The vaccine used for measles, mumps, and rubella is a live virus and can result in an overwhelming infection. Tetanus vaccine, inactivated poliovirus vaccine, and diphtheria, pertussis, tetanus (DPT) are not live virus vaccines.

The nurse is talking to a parent with a child who has a latex allergy. Which statement by the parent would indicate a correct understanding of the teaching? a. My child will have an allergic reaction if he comes in contact with yeast products. b. My child may have an upset stomach if he eats a food made with wheat or barley. c. My child will probably develop an allergy to peanuts. d. My child should not eat bananas or kiwis.

ANS: D There are cross-reactions between latex allergies and a number of foods such as bananas, avocados, kiwi, and chestnuts. Children with a latex allergy will not develop allergies to other food products such as yeast, wheat, barley, or peanuts.

The nurse is doing a prehospitalization orientation for Kayla, age 7, who is scheduled for cardiac surgery. As part of the preparation, the nurse explains that Kayla will not be able to talk because of an endotracheal tube but that she will be able to talk when it is removed. This explanation is: a. Unnecessary. b. The surgeons responsibility. c. Too stressful for a young child. d. An appropriate part of the childs preparation.

ANS: D This is a necessary part of preoperative preparation that will help reduce the anxiety associated with surgery. If the child wakes and is not prepared for the inability to speak, she will be even more anxious. It is a joint responsibility of nursing, medical staff, and child life personnel. This is a necessary component of preparation that will help reduce the anxiety associated with surgery.

Because of their striving for independence and productivity, which age-group of children is particularly vulnerable to events that may lessen their feeling of control and power? a. Infants b. Toddlers c. Preschoolers d. School-age children

ANS: D When a child is hospitalized, the altered family role, physical disability, loss of peer acceptance, lack of productivity, and inability to cope with stress usurp individual power and identity. This is especially detrimental to school-age children, who are striving for independence and productivity and are now experiencing events that lessen their control and power. Infants, toddlers, and preschoolers, although affected by loss of power, are not as significantly affected as are school-age children.

3. A nurse is reviewing a patient's chart and notes that the patient has a cancerous tumor that has invaded other organs. Based on this information, at which stage is this patient's cancer classified? A. Stage O B. Stage I C. Stage III D. Stage IV

ANS: D A stage IV cancer is one that has invaded other organs. Stage 0 is early cancer, present only in the cells in which it began. Stages I-III are more extensive, with larger tumors and spread to nearby lymph nodes or adjacent organs.

25. Cystic fibrosis (CF) is suspected in a toddler. Which test is essential in establishing this diagnosis? a. Bronchoscopy c. Urine creatinine b. Serum calcium d. Sweat chloride test

ANS: D A sweat chloride test result greater than 60 mEq/L is diagnostic of CF. Although bronchoscopy is helpful for identifying bacterial infection in children with CF, it is not diagnostic. Serum calcium is normal in children with CF. Urine creatinine is not diagnostic of CF.

31. A student nurse wants to provide nonpharmacological pain management interventions to a hospitalized child with cancer. Which action by the student causes the faculty member to intervene? A. Applying a moist heat pack B. Giving the child a massage C. Reading the child a story D. Using candles for aromatherapy

ANS: D Actions that have been reported by children to be effective pain control strategies are moist heat, massage, adequate rest and sleep, distraction (reading a story), and providing opportunities for social support. Open flames are prohibited in hospitals due to the risk of fire and explosion.

B (Sweets should be consumed with meals so the teeth can be cleaned afterward. This decreases the amount of time that the sugar is in contact with the teeth. Raisins, honey, and molasses are highly cariogenic and should be avoided.)

An appropriate recommendation in preventing tooth decay in young children is to: a. Substitute raisins for candy. b. Serve sweets after a meal. c. Use honey or molasses instead of refined sugar. d. Serve sweets between meals.

Which is beneficial in reducing the risk of Reye syndrome? a. Immunization against the disease b. Medical attention for all head injuries c. Prompt treatment of bacterial meningitis d. Avoidance of aspirin to treat fever associated with influenza

ANS: D Although the etiology of Reye syndrome is obscure, most cases follow a common viral illness, either varicella or influenza. A potential association exists between aspirin therapy and the development of Reye syndrome, so use of aspirin is avoided. No immunization currently exists for Reye syndrome. Reye syndrome is not correlated with head injuries or bacterial meningitis.

31. A mother asks the nurse what would be the first indication that acute glomerulonephritis is improving. The nurse's best response should be that the: a. Blood pressure will stabilize. . b. Child will have more energy. c. Urine will be free of protein d. Urinary output will increase.

ANS: D An increase in urinary output may signal resolution of the acute glomerulonephritis. If blood pressure is elevated, stabilization usually occurs with the improvement in renal function. The child having more energy and the urine being free of protein are related to the improvement in urinary output.

14. Skin testing for tuberculosis (the Mantoux test) is recommended: a. Every year for all children older than 2 years. b. Every year for all children older than 10 years. c. Every 2 years for all children starting at age 1 year. d. Periodically for children who reside in high-prevalence regions

ANS: D Children who reside in high prevalence regions for tuberculosis should be tested every 2 to 3 years. Annual testing is not necessary. Testing is not necessary unless exposure is likely or an underlying medical risk factor is present.

20. Which clinical manifestation would be seen in a child with chronic renal failure? a. Hypotension b. Massive hematuria c. Hypokalemia d. Unpleasant "uremic" breath odor

ANS: D Children with chronic renal failure have a characteristic breath odor resulting from the retention of waste products. Hypertension may be a complication of chronic renal failure. With chronic renal failure, little or no urine output occurs. Hyperkalemia is a concern in chronic renal failure.

Children taking phenobarbital (phenobarbital sodium) and/or phenytoin (Dilantin) may experience a deficiency of: a. calcium. b. vitamin C. c. fat-soluble vitamins. d. vitamin D and folic acid.

ANS: D Deficiencies of vitamin D and folic acid have been reported in children taking phenobarbital and phenytoin. Calcium, vitamin C, and fat-soluble vitamin deficiencies are not associated with phenobarbital or phenytoin.

27. Pancreatic enzymes are administered to the child with cystic fibrosis. Nursing considerations should include: a. Do not administer pancreatic enzymes if the child is receiving antibiotics. b. Decrease dose of pancreatic enzymes if the child is having frequent, bulky stools. c. Administer pancreatic enzymes between meals if at all possible. d. Pancreatic enzymes can be swallowed whole or sprinkled on a small amount of food taken at the beginning of a meal.

ANS: D Enzymes may be administered in a small amount of cereal or fruit or swallowed whole at the beginning of a meal, not between meals. Pancreatic enzymes are not contraindicated with antibiotics. The dose of enzymes should be increased if the child is having frequent, bulky stools.

35. Which vitamin supplements are necessary for children with cystic fibrosis? a. Vitamin C and calcium c. Magnesium b. Vitamins B6 and B12 d. Vitamins A, D, E, and K

ANS: D Fat-soluble vitamins are poorly absorbed because of deficient pancreatic enzymes in children with cystic fibrosis; therefore, supplements are necessary. Vitamin C and calcium are not fat soluble. Vitamins B6 and B12 are not fat-soluble vitamins. Magnesium is a mineral, not a vitamin.

27.Which intervention is appropriate when examining a male infant for cryptorchidism? a. Cooling the examiner's hands b. Taking a rectal temperature c. Eliciting the cremasteric reflex d. Warming the room

ANS: D For the infant's comfort, the infant should be examined in a warm room with the examiner's hands warmed. Testes can retract into the inguinal canal if the infant is upset or cold. Examining the infant with cold hands is uncomfortable for the infant and likely to cause the infant's testes to retract into the inguinal canal. It may also cause the infant to be uncooperative during the examination. A rectal temperature yields no information about cryptorchidism. Testes can retract into the inguinal canal if the cremasteric reflex is elicited. This can lead to an incorrect diagnosis.

The mother of a 1-month-old infant tells the nurse she worries that her baby will get meningitis like her oldest son did when he was an infant. The nurse should base her response on which statement? a. Meningitis rarely occurs during infancy. b. Often a genetic predisposition to meningitis is found. c. Vaccination to prevent all types of meningitis is now available. d. Vaccination to prevent Haemophilus influenzae type B meningitis has decreased the frequency of this disease in children.

ANS: D H. influenzae type B meningitis has been virtually eradicated in areas of the world where the vaccine is administered routinely. Bacterial meningitis remains a serious illness in children. It is significant because of the residual damage caused by undiagnosed and untreated or inadequately treated cases. The leading causes of neonatal meningitis are the group B streptococci and Escherichia coli organisms. Meningitis is an extension of a variety of bacterial infections. No genetic predisposition exists. Vaccinations are not available for all of the potential causative organisms.

9. Which statement is characteristic of acute otitis media (AOM)? a. The etiology is unknown. b. Permanent hearing loss often results. c. It can be treated by intramuscular antibiotics. d. It is treated with a broad range of antibiotics.

ANS: D Historically AOM has been treated with a range of antibiotics, and it is the most common disorder treated with antibiotics in the ambulatory setting. The etiology of AOM may be bacterial, such as Streptococcus pneumoniae, Haemophilus influenzae, and Moraxella catarrhalis, or a viral agent. Recent concerns about drugresistant organisms have caused authorities to recommend judicious use of antibiotics and that antibiotics are not required for initial treatment. Permanent hearing loss is not a frequent cause of properly treated AOM. Intramuscular antibiotics are not necessary. Oral amoxicillin is the treatment of choice.

17. The nurse is caring for a child with acute renal failure. What clinical manifestation should he or she recognize as a sign of hyperkalemia? a. Dyspnea b. Seizure c. Oliguria d. Cardiac arrhythmia

ANS: D Hyperkalemia is the most common threat to the life of the child. Signs of hyperkalemia include electrocardiographic anomalies such as prolonged QRS complex, depressed ST segments, peaked T waves, bradycardia, or heart block. Dyspnea, seizure, and oliguria are not manifestations of hyperkalemia.

5.The nurse is assisting the pediatric provider with a newborn examination. The provider notes that the infant has hypospadias. The nurse understands that hypospadias refers to: a. Absence of a urethral opening. b. Penis shorter than usual for age. c. Urethral opening along dorsal surface of penis. d. Urethral opening along ventral surface of penis.

ANS: D Hypospadias is a congenital condition in which the urethral opening is located anywhere along the ventral surface of the penis. The urethral opening is present, but not at the glans. Hypospadias does not refer to the size of the penis. When the urethral opening is along the dorsal surface of the penis, it is known as epispadias.

Which should cause a nurse to suspect that an infection has developed under a cast? a. Complaint of paresthesia b. Cold toes c. Increased respirations d. "Hot spots" felt on cast surface

ANS: D If hot spots are felt on the cast surface, they usually indicate infection beneath the area. This should be reported so that a window can be made in the cast to observe the site. The five Ps of ischemia from a vascular injury are pain, pallor, pulselessness, paresthesia, and paralysis. Paresthesia is an indication of vascular injury, not infection. Cold toes may be indicative of too tight a cast and need further evaluation. Increased respirations may be indicative of a respiratory tract infection or pulmonary emboli. This should be reported, and child should be evaluated.

The nurse is taking care of an adolescent diagnosed with kyphosis. Which describes this condition? a. Lateral curvature of the spine b. Immobility of the shoulder joint c. Exaggerated concave lumbar curvature of the spine d. Increased convex angulation in the curve of the thoracic spine

ANS: D Kyphosis is an abnormally increased convex angulation in the curve of the thoracic spine. Scoliosis is a complex spinal deformity usually involving lateral curvature, spinal rotation causing rib asymmetry, and thoracic hypokyphosis. Ankylosis is the immobility of a joint. Lordosis is an exaggerated concave lumbar curvature of the spine.

2.Which diagnostic finding is present when a child has primary nephrotic syndrome? a. Hyperalbuminemia b. Positive ASO titer c. Leukocytosis d. Proteinuria

ANS: D Large amounts of protein are lost through the urine as a result of an increased permeability of the glomerular basement membrane. Hypoalbuminemia is present because of loss of albumin through the defective glomerulus and the liver's inability to synthesize proteins to balance the loss. ASO titer is negative in a child with primary nephrotic syndrome. Leukocytosis is not a diagnostic finding in primary nephrotic syndrome.

The nurse should recommend medical attention if a child with a slight head injury experiences: a. sleepiness. b. vomiting, even once. c. headache, even if slight. d. confusion or abnormal behavior.

ANS: D Medical attention should be sought if the child exhibits confusion or abnormal behavior, loses consciousness, has amnesia, has fluid leaking from the nose or ears, complains of blurred vision, or has an unsteady gait. Sleepiness alone does not require evaluation. If the child is difficult to arouse from sleep, medical attention should be obtained. Vomiting more than three times requires medical attention. Severe or worsening headache or one that interferes with sleep should be evaluated.

8. Chronic otitis media with effusion (OME) is differentiated from acute otitis media (AOM) because it is usually characterized by: a. Fever as high as 40 C (104 F). c. Nausea and vomiting. b. Severe pain in the ear. d. A feeling of fullness in the ear.

ANS: D OME is characterized by an immobile or orange-discolored tympanic membrane and nonspecific complaints and does not cause severe pain. Fever and severe pain may be signs of AOM. Nausea and vomiting are associated with otitis media

24.The nurse is caring for an adolescent who has just started dialysis. The child seems always angry, hostile, or depressed. The nurse should recognize that this is most likely related to: a. Neurologic manifestations that occur with dialysis. b. Physiologic manifestations of renal disease. c. Adolescents having few coping mechanisms. d. Adolescents often resenting the control and enforced dependence imposed by dialysis.

ANS: D Older children and adolescents need control. The necessity of dialysis forces the adolescent into a dependent relationship, which results in these behaviors. Neurologic manifestations that occur with dialysis and physiologic manifestations of renal disease are a function of the age of the child, not neurologic or physiologic manifestations of the dialysis. Adolescents do have coping mechanisms, but they need to have some control over their disease management.

10. What is the characteristic of dishonest behavior in children ages 8 to 10 years? a. Cheating during games is now more common. b. Lying results from the inability to distinguish between fact and fantasy. c. They may steal because their sense of property rights is limited. d. They may lie to meet expectations set by others that they have been unable to attain.

ANS: D Older school-age children may lie to meet expectations set by others to which they have been unable to measure up. Cheating usually becomes less frequent as the child matures. In this age-group, children are able to distinguish between fact and fantasy. Young children may lack a sense of property rights; older children may steal to supplement an inadequate allowance, or it may be an indication of serious problems.

7. What is the role of the peer group in the life of school-age children? a. Gives them an opportunity to learn dominance and hostility. b. Allows them to remain dependent on their parents for a longer time. c. Decreases their need to learn appropriate sex roles. d. Provides them with security as they gain independence from their parents.

ANS: D Peer-group identification is an important factor in gaining independence from parents. Through peer relationships, children learn ways to deal with dominance and hostility. They also learn how to relate to people in positions of leadership and authority and explore ideas and the physical environment. Peer-group identification helps in gaining independence rather than remaining dependent. A child's concept of appropriate sex roles is influenced by relationships with peers.

9.A common side effect of corticosteroid therapy is: a. Fever. b. Hypertension. c. Weight loss. d. Increased appetite.

ANS: D Side effects of corticosteroid therapy include an increased appetite. Fever is not a side effect of therapy. It may be an indication of infection. Hypertension is not usually associated with initial corticosteroid therapy. Weight gain, not weight loss, is associated with corticosteroid therapy.

15. The mother of a toddler yells to the nurse, Help! He is choking to death on his food. The nurse determines that lifesaving measures are necessary based on: a. Gagging. c. Pulse over 100 beats/min. b. Coughing. d. Inability to speak.

ANS: D The inability to speak indicates a foreign-body airway obstruction of the larynx. Abdominal thrusts are needed for treatment of the choking child. Gagging indicates irritation at the back of the throat, not obstruction. Coughing does not indicate a complete airway obstruction. Tachycardia may be present for many reasons.

The nurse is conducting teaching to parents of a 7-year-old child who fractured an arm and is being discharged with a cast. Which instruction should be included in the teaching? a. Swelling of the fingers is to be expected for the next 48 hours. b. Immobilize the shoulder to decrease pain in the arm. c. Allow the affected limb to hang down for 1 hour each day. d. Elevate casted arm when resting and when sitting up.

ANS: D The injured extremity should be kept elevated while resting and in a sling when upright. This will increase venous return. Swelling of the fingers may indicate neurovascular damage and should be reported immediately. Permanent damage can occur within 6 to 8 hours. Joints above and below the cast on the affected extremity should be moved. The affected limb should not hang down for any length of time.

15.The most common cause of acute renal failure in children is: a. Pyelonephritis. b. Tubular destruction. c. Urinary tract obstruction. d. Severe dehydration.

ANS: D The most common cause of acute renal failure in children is dehydration or other causes of poor perfusion that may respond to restoration of fluid volume. Pyelonephritis and tubular destruction are not common causes of acute renal failure in children. Obstructive uropathy may cause acute renal failure, but it is not the most common cause.

The nurse is caring for a 4-year-old child immobilized by a fractured hip. Which complication should the nurse monitor related to the child's immobilization status? a. Metabolic rate increases b. Increased joint mobility leading to contractures c. Bone calcium increases, releasing excess calcium into the body (hypercalcemia) d. Venous stasis leading to thrombi or emboli formation

ANS: D The physiologic effects of immobilization, as a result of decreased muscle contraction, include venous stasis. This can lead to pulmonary emboli or thrombi. The metabolic rate decreases with immobilization. Loss of joint mobility leads to contractures. Bone demineralization with osteoporosis and hypercalcemia occur with immobilization.

10. A child has been diagnosed with chronic myelogenous leukemia (CML). Which statement by the nurse to the parents is most appropriate? A. "Radiation therapy is the standard treatment." B. "The prognosis for this disease is extremely poor." C. "There are lots of good medications for nausea." D. "We need to test siblings for a bone marrow match."

ANS: D The preferred treatment for CML is a bone marrow or stem cell transplant from a matching sibling, which can be curative in up to 80% of patients. Radiation therapy is not used. Although there are many good medications for nausea, this statement is not the best choice, because it is not specific to this child's condition.

7. The nurse is caring for a child with suspected diabetes insipidus. Which clinical manifestation would be observable? a. Oliguria b. Glycosuria c. Nausea and vomiting d. Polydipsia

ANS: D Excessive urination accompanied by insatiable thirst is the primary clinical manifestation of diabetes. These symptoms may be so severe that the child does little other than drink and urinate. Oliguria is decreased urine production and is not associated with diabetes insipidus. Glycosuria is associated with diabetes mellitus. Nausea and vomiting are associated with inappropriate antidiuretic hormone secretion.

17. Which symptom is considered a cardinal sign of diabetes mellitus? a. Nausea b. Seizures c. Impaired vision d. Frequent urination

ANS: D Hallmarks of diabetes mellitus are glycosuria, polyuria, and polydipsia. Nausea and seizures are not clinical manifestations of diabetes mellitus. Impaired vision is a long-term complication of the disease.

30. Careful hand washing before and after contact can prevent the spread of which condition in day care and school settings? a. Irritable bowel syndrome b. Ulcerative colitis c. Hepatic cirrhosis d. Hepatitis A

ANS: D Hepatitis A is spread person to person, by the fecal-oral route, and through contaminated food or water. Good hand washing is critical in preventing its spread. The virus can survive on contaminated objects for weeks. Irritable bowel syndrome is the result of increased intestinal motility and is not contagious. Ulcerative colitis and cirrhosis are not infectious.

2. Which type of dehydration results from water loss in excess of electrolyte loss? a. Isotonic dehydration b. Isosmotic dehydration c. Hypotonic dehydration d. Hypertonic dehydration

ANS: D Hypertonic dehydration results from water loss in excess of electrolyte loss. This is the most dangerous type of dehydration. It is caused by feeding children fluids with high amounts of solute. Isotonic dehydration occurs in conditions in which electrolyte and water deficits are present in balanced proportion. Isosmotic dehydration is another term for isotonic dehydration. Hypotonic dehydration occurs when the electrolyte deficit exceeds the water deficit, leaving the serum hypotonic.

33. An infant diagnosed with pyloric stenosis experiences excessive vomiting that can result in which condition? a. Hyperchloremia b. Hypernatremia c. Metabolic acidosis d. Metabolic alkalosis

ANS: D Infants with excessive vomiting are prone to metabolic alkalosis from the loss of hydrogen ions. Chloride ions and sodium are lost with vomiting. Metabolic alkalosis, not acidosis, is likely.

13. Which clinical manifestation would most suggest acute appendicitis? a. Rebound tenderness b. Bright red or dark red rectal bleeding c. Abdominal pain that is relieved by eating d. Abdominal pain that is most intense at McBurney point

ANS: D Pain is the cardinal feature. It is initially generalized and usually periumbilical. The pain localizes to the right lower quadrant at McBurney point. Rebound tenderness is not a reliable sign and is extremely painful to the child. Abdominal pain that is relieved by eating and bright or dark red rectal bleeding are not signs of acute appendicitis.

5. A child will start treatment for precocious puberty. This involves injections of which synthetic medication? a. Thyrotropin b. Gonadotropins c. Somatotropic hormone d. Luteinizing hormone-releasing hormone

ANS: D Precocious puberty of central origin is treated with monthly subcutaneous injections of luteinizing hormone-releasing hormone. Thyrotropin, gonadotropin, and somatotropic hormone are not appropriate therapies for precocious puberty.

22. A child eats some sugar cubes after experiencing symptoms of hypoglycemia. This rapid-releasing sugar should be followed by: a. saturated and unsaturated fat. b. fruit juice. c. several glasses of water. d. complex carbohydrate and protein.

ANS: D Symptoms of hypoglycemia are treated with a rapid-releasing sugar source followed by a complex carbohydrate and protein. Saturated and unsaturated fat, fruit juice, and several glasses of water do not provide the child with complex carbohydrate and protein necessary to stabilize the blood sugar.

3. Myelosuppression, associated with chemotherapeutic agents or some malignancies such as leukemia can cause bleeding tendencies because of what resulting outcome? a. Decrease in leukocytes b. Increase in lymphocytes c. Vitamin C deficiency d. Decrease in blood platelets

ANS: D The decrease in blood platelets secondary to the myelosuppression of chemotherapy can cause an increase in bleeding. The child and family should be alerted to avoid risk of injury. Decrease in leukocytes, increase in lymphocytes, and vitamin C deficiency would not affect bleeding tendencies.

32. The parent of a child diagnosed with diabetes mellitus asks the nurse when urine testing will be necessary. The nurse should explain that urine testing is necessary for which reason? a. Glucose is needed before administration of insulin. b. Glucose is needed 4 times a day. c. Glycosylated hemoglobin is required. d. Ketonuria is suspected.

ANS: D Urine testing is still performed to detect evidence of ketonuria. Urine testing for glucose is no longer indicated for medication administration because of the poor correlation between blood glucose levels and glycosuria. Glycosylated hemoglobin analysis is performed on a blood sample.

38.A child with secondary enuresis who complains of dysuria or urgency should be evaluated for what condition (Select all that apply)? a. Hypocalciuria b. Nephrotic syndrome c. Glomerulonephritis d. Urinary tract infection (UTI) e. Diabetes mellitus

ANS: D, E Complaints of dysuria or urgency from a child with secondary enuresis suggest the possibility of a UTI. If accompanied by excessive thirst and weight loss, these symptoms may indicate the onset of diabetes mellitus. An excessive loss of calcium in the urine (hypercalciuria) can be associated with complaints of painful urination, urgency, frequency, and wetting. Nephrotic syndrome is not usually associated with complaints of dysuria or urgency. Glomerulonephritis is not a likely cause of dysuria or urgency.

Which are clinical manifestations of increased intracranial pressure (ICP) in infants? (Select all that apply.) a. Low-pitched cry b. Sunken fontanel c. Diplopia and blurred vision d. Irritability e. Distended scalp veins f. Increased blood pressure

ANS: D, E Diplopia and blurred vision, irritability, and distended scalp veins are signs of increased ICP in infants. Diplopia and blurred vision is indicative of ICP in children. A high-pitched cry and a tense or bulging fontanel are characteristics of increased ICP. Increased blood pressure, common in adults, is rarely seen in children.

C (secondary circular reactions Infants are usually in the secondary circular reaction stage from age 4 months to 8 months. This stage is characterized by a continuation of the primary circular reaction for the response that results. For example, shaking of a rattle is performed to hear the noise of the rattle, not just for shaking. The use of reflexes is primarily during the first month of life. The primary circular reaction stage marks the replacement of reflexes with voluntary acts. The infant is in this stage from age 1 month to 4 months. The fourth sensorimotor stage is coordination of secondary schemata. This is a transitional stage in which increasing motor skills enable greater exploration of the environment.)

According to Piaget, the 6-month-old infant would be in what stage of the sensorimotor phase? a. Use of reflexes b. Primary circular reactions c. Secondary circular reactions d. Coordination of secondary schemata

D (Make a follow-up home visit to parents as soon as possible after the infant's death. A competent, qualified professional should visit the family at home as soon as possible after the death and provide the family with printed information about SIDS. An explanation of how SIDS could have been predicted and prevented is inappropriate. SIDS cannot be prevented or predicted. Discussions about the cause will only increase parental guilt. The parents should be asked only factual questions to determine the cause of death. Parents should be allowed and encouraged to make a last visit with their infant.)

An important nursing responsibility when dealing with a family experiencing the loss of an infant from sudden infant death syndrome (SIDS) is to: a. Explain how SIDS could have been predicted and prevented. b. Interview parents in depth concerning the circumstances surrounding the infant's death. c. Discourage parents from making a last visit with the infant. d. Make a follow-up home visit to parents as soon as possible after the infant's death.

C ("we will check the monitor several times a day to be sure the alarm is working." The parents should check the monitor several times a day to be sure the alarm is working and that it can be heard from room to room. The parents should not adjust the monitor to eliminate false alarms. Adjustments could compromise the monitor's effectiveness. The monitor should be placed on a firm surface away from the crib and drapes. The parents should not sleep in the same bed as the monitored infant.)

An infant experienced an apparent life-threatening event and is being placed on home apnea monitoring. The parents have understood the instructions for use of a home apnea monitor when they state: a. "We can adjust the monitor to eliminate false alarms." b. "We should sleep in the same bed as our monitored infant." c. "We will check the monitor several times a day to be sure the alarm is working." d. "We will place the monitor in the crib with our infant."

C ("What time did you find the infant?" During a SIDS incident, if the infant is not pronounced dead at the scene, he or she may be transported to the emergency department to be pronounced dead by a physician. While they are in the emergency department, the parents are asked only factual questions, such as when they found the infant, how he or she looked, and whom they called for help. The nurse avoids any remarks that may suggest responsibility, such as "Why didn't you go in earlier?" "Didn't you hear the infant cry out?" or "Was the head buried in a blanket?")

An infant has been pronounced dead from sudden infant death syndrome (SIDS) in the emergency department. Which is an appropriate question to ask the parents? a. "Did you hear the infant cry out?" b. "Why didn't you check on the infant earlier?" c. "What time did you find the infant?" d. "Was the head buried in a blanket?"

A (Cardiopulmonary resuscitation (CPR))

Apnea of infancy has been diagnosed in an infant who will soon be discharged with home monitoring. When teaching the parents about the infant's care, what is the most important information the nurse should include in the discharge teaching plan? A. Cardiopulmonary resuscitation (CPR) B. Administration of intravenous (IV) fluids C. Reassurance that the infant cannot be electrocuted during monitoring D. Advice that the infant not be left with other caretakers such as baby-sitters

C (8 months Sitting erect without support is a developmental milestone usually achieved by 8 months. At age 4 months, an infant can sit with support. At age 6 months, the infant will maintain a sitting position if propped. By 10 months, the infant can maneuver from a prone to a sitting position.)

At which age can most infants sit steadily unsupported? a. 4 months b. 6 months c. 8 months d. 10 months

The nurse is caring for a hospitalized 4-year-old boy, Ryan. His parents tell the nurse that they will be back to visit at 6 PM. When Ryan asks the nurse when his parents are coming, the nurse's best response is: a. "They will be here soon." b. "They will come after dinner." c. "Let me show you on the clock when 6 PM is." d. "I will tell you every time I see you how much longer it will be."

B A 4-year-old understands time in relation to events such as meals. Children perceive "soon" as a very short time. The nurse may lose the child's trust if his parents do not return in the time he perceives as "soon." Children cannot read or use a clock for practical purposes until age 7 years. This answer assumes that the child understands the concept of hours and minutes, which is not developed until age 5 or 6 years.

The parent of a 4-year-old son tells the nurse that the child believes "monsters and the boogeyman" are in his bedroom at night. The nurse's best suggestion for coping with this problem is to: a. Insist that the child sleep with his parents until the fearful phase passes. b. Suggest involving the child to find a practical solution such as a night-light. c. Help the child understand that these fears are illogical. d. Tell the child frequently that monsters and the boogeyman do not exist.

B A night-light shows a child that imaginary creatures do not lurk in the darkness. Letting the child sleep with parents or telling the child that these creatures do not exist will not get rid of the fears. A 4-year-old is in the preconceptual age and cannot understand logical thought.

Acyclovir (Zovirax) is given to children with chickenpox to: a. Minimize scarring. c. Prevent aplastic anemia. b. Decrease the number of lesions. d. Prevent spread of the disease.

B Acyclovir decreases the number of lesions, shortens duration of fever, and decreases itching, lethargy, and anorexia; however, it does not prevent scarring. Preventing aplastic anemia is not a function of acyclovir. Only quarantine of the infected child can prevent the spread of disease.

Which statement is the most appropriate advice to give parents of a 16-year-old girl who is rebellious? a. "You need to be stricter so that your teen stops trying to test the limits." b. "You need to collaborate with your daughter and set limits that are perceived as being reasonable." c. "Increasing your teen's involvement with her peers will improve her self-esteem." d. "Allow your teenager to choose the type of discipline that is used in your home."

B Allowing teenagers to choose between realistic options and offering consistent and structured discipline typically enhances cooperation and decreases rebelliousness. Structure helps adolescents to feel more secure and assists them in the decision-making process. Setting stricter limits typically does not decrease rebelliousness or decrease testing of parental limits. Increasing peer involvement does not typically increase self-esteem.

Which statement is most descriptive of central nervous system stimulants? a. They produce strong physical dependence. b. They can result in strong psychologic dependence. c. Withdrawal symptoms are life threatening. d. Acute intoxication can lead to coma.

B Central nervous system stimulants such as amphetamines and cocaine produce a strong psychologic dependence. This class of drugs does not produce strong physical dependence and can be withdrawn without much danger. Acute intoxication leads to violent, aggressive behavior or psychotic episodes characterized by paranoia, uncontrollable agitation, and restlessness.

The psychologic effects of being obese during adolescence include: a. Sexual promiscuity. b. Poor body image. c. Memory having no effect on eating behavior. d. Accurate body image but self-deprecating attitude.

B Common emotional consequences of obesity include poor body image, low self-esteem, social isolation, and feelings of depression and isolation. Sexual promiscuity is an unlikely effect of obesity. The obese adolescent often substitutes food for affection. Eating behaviors are closely related to memory. Memory and appetite are closely linked and can be modified over time with treatment. Obese adolescents most often have a very poor self-image.

Which behavior suggests appropriate psychosocial development in the adolescent? a. The adolescent seeks validation for socially acceptable behavior from older adults. b. The adolescent is self-absorbed and self-centered and has sudden mood swings. c. Adolescents move from peers and enjoy spending time with family members. d. Conformity with the peer group increases in late adolescence.

B During adolescence, energy is focused within. Adolescents concentrate on themselves in an effort to determine who they are or who they will be. Adolescents are likely to be impulsive and impatient. Parents often describe their teenager as being "self-centered" or "lazy." The peer group validates acceptable behavior during adolescence. Adolescents move from family and enjoy spending time with peers. Adolescents also spend time alone; they need this time to think and concentrate on themselves. Conformity becomes less important in late adolescence.

Which predisposes the adolescent to feel an increased need for sleep? a. An inadequate diet b. Rapid physical growth c. Decreased activity that contributes to a feeling of fatigue d. The lack of ambition typical of this age group

B During growth spurts, the need for sleep is increased. Rapid physical growth, the tendency toward overexertion, and the overall increased activity of this age contribute to fatigue.

The nurse is performing an assessment on a child and notes the presence of Koplik's spots. In which communicable disease are Koplik's spots present? a. Rubella c. Chickenpox (varicella) b. Measles (rubeola) d. Exanthema subitum (roseola)

B Koplik's spots are small, irregular red spots with a minute, bluish white center found on the buccal mucosa 2 days before systemic rash. Koplik's spots are not present with rubella, varicella, or roseola.

A teen asks a nurse, "What is physical dependence in substance abuse?" Which is the correct response by the nurse? a. Problem that occurs in conjunction with addiction b. Involuntary physiologic response to drug c. Culturally defined use of drugs for purposes other than accepted medical purposes d. Voluntary behavior based on psychosocial needs

B Physical dependence is an involuntary response to the pharmacologic characteristics of drugs such as opioids or alcohol. A person can be physically dependent on a narcotic/drug without being addicted; for example, patients who use opioids to control pain need increasing doses to achieve the same effect. Dependence is a physiologic response; it is not culturally determined or subject to voluntary control.

A 4-year-old boy is hospitalized with a serious bacterial infection. He tells the nurse that he is sick because he was "bad." The nurse's best interpretation of this comment is that it is: a. A sign of stress. b. Common at this age. c. Suggestive of maladaptation. d. Suggestive of excessive discipline at home.

B Preschoolers cannot understand the cause and effect of illness. Their egocentrism makes them think that they are directly responsible for events, making them feel guilt for things outside of their control. Children of this age show stress by regressing developmentally or acting out. Maladaptation is unlikely. This comment does not imply excessive discipline at home.

The nurse is completing a health history with a 16-year-old male. He informs the nurse that he has started using smokeless tobacco after he plays baseball. Which information regarding smokeless tobacco would be most correct for the nurse to provide to this teen? a. Not addicting. b. Proven to be carcinogenic. c. Easy to stop using. d. A safe alternative to cigarette smoking.

B Smokeless tobacco is a popular substitute for cigarettes and poses serious health hazards to children and adolescents. Smokeless tobacco is associated with cancer of the mouth and jaw. Smokeless tobacco is just as addictive as cigarettes. Although teens believe that it is easy to stop using smokeless tobacco, this is not the case. A popular belief is that smokeless tobacco is a safe alternative to cigarettes; this has been proven incorrect. Half of all teens who use smokeless tobacco agree that it poses significant health risks.

A 17-year-old tells the nurse that he is not having sex because it would make his parents very angry. This response indicates that the adolescent has a developmental lag in which area? a. Cognitive development c. Psychosocial development b. Moral development d. Psychosexual development

B The appropriate moral development for a 17-year-old would include evidence that the teenager has internalized a value system and does not depend on parents to determine right and wrong behaviors. Adolescents who remain concrete thinkers may never advance beyond conformity to please others and avoid punishment. Cognitive development is related to moral development, but it is not the pivotal point in determining right and wrong behaviors. Identity formation is the psychosocial development task. Energy is focused within the adolescent, who exhibits behavior that is self-absorbed and egocentric. Although a task during adolescence is the development of a sexual identity, the teenager's dependence on the parents' sanctioning of right or wrong behavior is more appropriately related to moral development.

The mean age of menarche in the United States is: a. 11.5 years c. 13.5 years b. 12.5 years d. 14 years

B The average age of menarche is 12 years and 4 months in North American girls, with a normal range of 10.5 to 15 years.

Parents tell the nurse that they found their 3-year-old daughter and a male cousin of the same age inspecting each other closely as they used the bathroom. Which is the most appropriate recommendation the nurse should make? a. Punish children so this behavior stops. b. Neither condone nor condemn the curiosity. c. Allow children unrestricted permission to satisfy this curiosity. d. Get counseling for this unusual and dangerous behavior.

B Three-year-olds become aware of anatomic differences and are concerned about how the other "works." Such exploration should not be condoned or condemned. Children should not be punished for this normal exploration. Encouraging the children to ask questions of the parents and redirecting their activity are more appropriate than giving permission. Exploration is age-appropriate and not dangerous behavior.

Which statement, made by a 4-year-old child's father, is true about the care of the preschooler's teeth? a. "Because the 'baby teeth' are not permanent, they are not important to the child." b. "My son can be encouraged to brush his teeth after I have thoroughly cleaned his teeth." c. "My son's 'permanent teeth' will begin to come in at 4 to 5 years of age." d. "Fluoride supplements can be discontinued when my son's 'permanent teeth' erupt."

B Toddlers and preschoolers lack the manual dexterity to remove plaque adequately, so parents must assume this responsibility. Deciduous teeth are important because they maintain spacing and play an important role in the growth and development of the jaws and face and in speech development. Secondary teeth erupt at about 6 years of age. If the family does not live in an area where fluoride is included in the water supply, fluoride supplements should be continued.

According to Erikson, the psychosocial task of adolescence is developing: a. Intimacy. c. Initiative. b. Identity. d. Independence.

B Traditional psychosocial theory holds that the developmental crises of adolescence lead to the formation of a sense of identity. Intimacy is the developmental stage for early adulthood. Initiative is the developmental stage for early childhood. Independence is not one of Erikson's developmental stages.

When is a child with chickenpox considered to be no longer contagious? a. When fever is absent c. 24 hours after lesions erupt b. When lesions are crusted d. 8 days after onset of illness

B When the lesions are crusted, the chickenpox is no longer contagious. This may be a week after onset of disease. The child is still contagious once the fever has subsided and after the lesions erupt, and may or may not be contagious any time after 6 days as long as all lesions are crusted over.

Which interventions should the nurse plan when caring for a child with a visual impairment (select all that apply)? a. Touch the child upon entering the room before speaking. b. Keep items in the room in the same location. c. Describe the placement of the eating utensils on the meal tray. d. Use color examples to describe something to a child who has been blind since birth. e. Identify noises for the child.

B, C, E

The nurse finds that a child spends several hours each day playing video games and lives in a home environment with limited access to safe playgrounds and parks. What health risks does the nurse expect based on these findings? Select all that apply. A Tooth decay B Hypertension C Diabetes D Growth delays E Hypercholesterolemia

B, C, E A child's home environment with limited access to safe outdoor play areas and abundant access to television and video games is a major contributing factor for childhood obesity. Childhood obesity increases the risk for hypertension, diabetes, and hypercholesterolemia, among other conditions. Tooth decay is associated with poor dental care, not lack of exercise contributing to childhood obesity. Growth delays are associated with malnutrition, not lack of exercise contributing to childhood obesity.

Which screening tests should the school nurse perform for the adolescent (select all that apply)? a. Glucose b. Vision c. Hearing d. Cholesterol e. Scoliosis

B, C, E The school nurse should perform vision, hearing, and scoliosis screening tests according to the school district's required schedule. Glucose and cholesterol screening would be performed in the medical clinic setting.

In terms of language and cognitive development, a 4-year-old child would be expected to have which traits (select all that apply)? a. Think in abstract terms. b. Follow directional commands. c. Understand conservation of matter. d. Use sentences of eight words. e. Tell exaggerated stories.

B, E Children ages 3 to 4 years can give and follow simple commands and tell exaggerated stories. Children cannot think abstractly at age 4 years. Conservation of matter is a developmental task of the school-age child. Five-year-old children use sentences with eight words with all parts of speech.

During the preschool period, the emphasis of injury prevention should be placed on: a. Constant vigilance and protection. b. Punishment for unsafe behaviors. c. Education for safety and potential hazards. d. Limitation of physical activities.

C Education for safety and potential hazards is appropriate for preschoolers because they can begin to understand dangers. Constant vigilance and protection is not practical at this age since preschoolers are becoming more independent. Punishment may make children scared of trying new things. Limitation of physical activities is not appropriate.

The parents of a 15-year-old girl are concerned that their adolescent spends too much time looking in the mirror. Which statement is the most appropriate for the nurse to make? a. "Your teenager needs clearer and stricter limits about her behavior." b. "Your teenager needs more responsibility at home." c. "During adolescence this behavior is not unusual." d. "The behavior is abnormal and needs further investigation."

C Egocentric and narcissistic behavior is normal during this period of development. The teenager is seeking a personal identity. Stricter limits are not an appropriate response for a behavior that is part of normal development. More responsibility at home is not an appropriate response for this situation. The behavior is normal and needs no further investigation.

The school nurse tells adolescents in the clinic that confidentiality and privacy will be maintained unless a life-threatening situation arises. This practice is: a. Not appropriate in a school setting. b. Never appropriate because adolescents are minors. c. Important in establishing trusting relationships. d. Suggestive that the nurse is meeting his or her own needs.

C Health professionals who work with adolescents should consider the adolescents' increasing independence and responsibility while maintaining privacy and ensuring confidentiality. However, circumstances may occur in which they are not able to maintain confidentiality, such as self-destructive behavior or maltreatment by others. Confidentiality and privacy are necessary to facilitate trust with this age group. The nurse must be aware of the limits placed on confidentiality by local jurisdiction.

Young people with anorexia nervosa are often described as being: a. Independent. c. Conforming. b. Disruptive. d. Low achieving.

C Individuals with anorexia nervosa are described as perfectionist, academically high achievers, conforming, and conscientious. Independent, disruptive, and low achieving are not part of the behavioral characteristics of anorexia nervosa.

In providing anticipatory guidance to parents whose child will soon be entering kindergarten, which is a critical factor in preparing a child for kindergarten entry? a. The child's ability to sit still b. The child's sense of learned helplessness c. The parent's interactions and responsiveness to the child d. Attending a preschool program

C Interactions between the parent and child are an important factor in the development of academic competence. Parental encouragement and support maximize a child's potential. The child's ability to sit still is important to learning; however, parental responsiveness and involvement are more important factors. Learned helplessness is the result of a child feeling that he or she has no effect on the environment and that his or her actions do not matter. Parents who are actively involved in a supportive learning environment will demonstrate a more positive approach to learning. Preschool and day care programs can supplement the developmental opportunities provided by parents at home, but they are not critical in preparing a child for entering kindergarten.

Imaginary playmates are beneficial to the preschool child because they: a. Take the place of social interactions. b. Take the place of pets and other toys. c. Become friends in times of loneliness. d. Accomplish what the child has already successfully accomplished.

C One purpose of an imaginary friend is to be a friend in time of loneliness. Imaginary friends do not take the place of social interactions but may encourage conversation. Imaginary friends do not take the place of pets or toys. They accomplish what the child is still attempting, not what has already been accomplished.

A 4-year-old child tells the nurse that she does not want another blood sample drawn because "I need all my insides, and I don't want anyone taking them out." Which is the nurse's best interpretation of this? a. Child is being overly dramatic. b. Child has a disturbed body image. c. Preschoolers have poorly defined body boundaries. d. Preschoolers normally have a good understanding of their bodies.

C Preschoolers have little understanding of body boundaries, which leads to fears of mutilation. The child is not capable of being dramatic at 4 years of age. She truly has fear. Body image is just developing in the school-age child. Preschoolers do not have good understanding of their bodies.

Which statement by the nurse is most appropriate to a 15-year-old whose friend has mentioned suicide? a. "Tell your friend to come to the clinic immediately." b. "You need to gather details about your friend's suicide plan." c. "Your friend's threat needs to be taken seriously and immediate help for your friend is important." d. "If your friend mentions suicide a second time, you will want to get your friend some help."

C Suicide is the third most common cause of death among American adolescents. A suicide threat from an adolescent serves as a dramatic message to others and should be taken seriously. Adolescents at risk should be targeted for supportive guidance and counseling before a crisis occurs. Instructing a 15-year-old to tell a friend to come to the clinic immediately provides the teen with limited information and does not address the concern. It is important to determine whether a person threatening suicide has a plan of action; however, the best information for the 15-year-old to have is that all threats of suicide should be taken seriously and immediate help is important. Taking time to gather details or waiting until the teen discusses it a second time may be too late.

The nurse is explaining Tanner staging to an adolescent and her mother. Which statement best describes Tanner staging? a. Predictable stages of puberty that are based on chronologic age b. Staging of puberty based on the initiation of menarche and nocturnal emissions c. Predictable stages of puberty that are based on primary and secondary sexual characteristics d. Staging of puberty based on the initiation of primary sexual characteristics

C Tanner sexual-maturing ratings are based on the development of stages of primary and secondary sexual characteristics. Tanner stages are not based on chronologic age. The age at which an adolescent enters puberty is variable. The puberty stage in girls begins with breast development. The puberty stage in boys begins with genital enlargement. Primary sexual characteristics are not the sole basis of Tanner staging.

The parents of a 14-year-old girl express concerns about the number of hours their daughter spends with her friends. The nurse explains that peer relationships become more important during adolescence because: a. Adolescents dislike their parents. b. Adolescents no longer need parental control. c. They provide adolescents with a feeling of belonging. d. They promote a sense of individuality in adolescents.

C The peer group serves as a strong support to teenagers, providing them with a sense of belonging and strength and power. During adolescence, the parent-child relationship changes from one of protection-dependency to one of mutual affection and quality. Parents continue to play an important role in personal and health-related decisions. The peer group forms the transitional world between dependence and autonomy.

The school nurse is discussing testicular self-examination with adolescent boys. Why is this important? a. Epididymitis is common during adolescence. b. Asymptomatic sexually transmitted diseases may be present. c. Testicular tumors during adolescence are generally malignant. d. Testicular tumors, although usually benign, are common during adolescence.

C Tumors of the testes are not common, but when manifested in adolescence, they are generally malignant and demand immediate evaluation. Epididymitis is not common in adolescence. Asymptomatic sexually transmitted disease would not be evident during testicular self-examination. The focus of this examination is on testicular cancer. Testicular tumors are most commonly malignant.

D, E (D. sphincter control is achieved. E. primary dentition is complete.)

Characteristics of physical development of a 30-month-old child are the: (Select all that apply.) A. anterior fontanel is open. B. birth weight has doubled. C. genital fondling is noted. D. sphincter control is achieved. E. primary dentition is complete.

A (Avoidance of eye contact. One of the clinical manifestations of nonorganic failure to thrive is the child's avoidance of eye contact with the health professional. A malabsorption defect would result in a physiologic problem, not behavioral. Weight (but not height) below the 5th percentile is indicative of failure to thrive. Developmental delays, including social, motor, adaptive, and language, exist.)

Clinical manifestations of failure to thrive caused by behavioral problems resulting in inadequate intake of calories include: a. Avoidance of eye contact. b. An associated malabsorption defect. c. Weight that falls below the 15th percentile. d. Normal achievement of developmental landmarks.

The nurse is guiding parents in selecting a day care facility for their child. When making the selection, it is especially important to consider: a. Structured learning environment. b. Socioeconomic status of children. c. Cultural similarities of children. d. Teachers knowledgeable about development.

D A teacher knowledgeable about development will structure activities for learning. A structured learning environment is not necessary at this age. Socioeconomic status is not the most important factor in selecting a preschool. Preschool is about expanding experiences with others; cultural similarities are not necessary.

By what age would the nurse expect that most children could understand prepositional phrases such as "under," "on top of," "beside," and "in back of"? a. 18 months c. 3 years b. 24 months d. 4 years

D At 4 years, children can understand directional phrases. Children 18 to 24 months and 3 years of age are too young.

Which characteristic best describes the language of a 3-year-old child? a. Asks meanings of words b. Follows directional commands c. Can describe an object according to its composition d. Talks incessantly, regardless of whether anyone is listening

D Because of the dramatic vocabulary increase at this age, 3-year-olds are known to talk incessantly, regardless of whether anyone is listening. A 4- to 5-year-old asks lots of questions and can follow simple directional commands. A 6-year-old can describe an object according to its composition.

Which is probably the most important criterion on which to base the decision to report suspected child abuse? a. Inappropriate parental concern for the degree of injury b. Absence of parents for questioning about child's injuries c. Inappropriate response of child d. Incompatibility between the history and injury observed

D Conflicting stories about the "accident" are the most indicative red flags of abuse. Inappropriate response of caregiver or child may be present, but is subjective. Parents should be questioned at some point during the investigation.

Which is the causative agent of scarlet fever? a. Enteroviruses b. Corynebacterium organisms c. Scarlet fever virus d. Group A -hemolytic streptococci (GABHS)

D GABHS infection causes scarlet fever. Enteroviruses do not cause the same complications. Corynebacterium organisms cause diphtheria. Scarlet fever is not caused by a virus.

In girls, the initial indication of puberty is: a. Menarche. c. Growth of pubic hair. b. Growth spurt. d. Breast development.

D In most girls, the initial indication of puberty is the appearance of breast buds, an event known as the larche. The usual sequence of secondary sexual characteristic development in girls is breast changes, rapid increase in height and weight, growth of pubic hair, appearance of axillary hair, menstruation, and abrupt deceleration of linear growth.

Which statement is most descriptive of bulimia during adolescence? a. Strong sense of control over eating behavior b. Feelings of elation after the binge-purge cycle c. Profound lack of awareness that the eating pattern is abnormal d. Weight that can be normal, slightly above normal, or below normal

D Individuals with bulimia are of normal weight or more commonly slightly above normal weight. Those who also restrict their intake can become severely underweight. Behavior related to this eating disorder is secretive, frenzied, and out of control. These cycles are followed by self-deprecating thoughts and a depressed mood. These young women are keenly aware that this eating pattern is abnormal.

When planning care for adolescents, the nurse should: a. Teach parents first, and they, in turn, will teach the teenager. b. Provide information for their long-term health needs because teenagers respond best to long-range planning. c. Maintain the parents' role by providing explanations for treatment and procedures to the parents only. d. Give information privately to adolescents about how they can manage the specific problems that they identify.

D Problems that teenagers identify and are interested in are typically the problems that they are the most willing to address. Confidentiality is important to adolescents. Adolescents prefer to confer privately (without parents) with the nurse and health care provider. Teenagers are socially and cognitively at the developmental stage where the health care provider can teach them. The nurse must keep in mind that teenagers are more interested in immediate health care needs than in long-term needs

Which common childhood communicable disease may cause severe defects in the fetus when it occurs in its congenital form? a. Erythema infectiosum c. Rubeola b. Roseola d. Rubella

D Rubella causes teratogenic effects on the fetus. There is a low risk of fetal death to those in contact with children affected with fifth disease. Roseola and rubeola are not dangerous to the fetus.

Which symptoms should the nurse expect to observe during the physical assessment of an adolescent girl with severe weight loss and disrupted metabolism associated with anorexia nervosa? a. Dysmenorrhea and oliguria b. Tachycardia and tachypnea c. Heat intolerance and increased blood pressure d. Lowered body temperature and brittle nails

D Symptoms of anorexia nervosa include lower body temperature, severe weight loss, decreased blood pressure, dry skin, brittle nails, altered metabolic activity, and presence of lanugo hair. Amenorrhea, rather than dysmenorrhea, and cold intolerance are manifestations of anorexia nervosa. Bradycardia, rather than tachycardia, may be present.

Steve, 14 years old, mentions that he now has to use deodorant but never had to before. The nurse's response should be based on knowledge that: a. Eccrine sweat glands in the axillae become fully functional during puberty. b. Sebaceous glands become extremely active during puberty. c. New deposits of fatty tissue insulate the body and cause increased sweat production. d. Apocrine sweat glands reach secretory capacity during puberty.

D The apocrine sweat glands, nonfunctional in children, reach secretory capacity during puberty. They secrete a thick substance as a result of emotional stimulation that, when acted on by surface bacteria, becomes highly odoriferous. They are limited in distribution and grow in conjunction with hair follicles in the axillae, genital and anal areas, and other areas. Eccrine sweat glands are present almost everywhere on the skin and become fully functional and respond to emotional and thermal stimulation. Sebaceous glands become extremely active at this time, especially those on the genitals and the "flush" areas of the body, such as face, neck, shoulders, upper back, and chest. This increased activity is important in the development of acne. New deposits of fatty tissue insulate the body and cause increased sweat production, but this is not the etiology of apocrine sweat gland activity.

Anorexia nervosa may best be described as: a. Occurring most frequently in adolescent males. b. Occurring most frequently in adolescents from lower socioeconomic groups. c. Resulting from a posterior pituitary disorder. d. Resulting in severe weight loss in the absence of obvious physical causes.

D The etiology of anorexia remains unclear, but a distinct psychologic component is present. The diagnosis is based primarily on psychologic and behavioral criteria. Anorexia nervosa is observed more commonly in adolescent girls and young women. It does not occur most frequently in adolescents from a lower socioeconomic group. In reality, anorexic adolescents are often from families of means who have high parental expectations for achievement. Anorexia is a psychiatric disorder

The most common cause of death in the adolescent age-group involves: a. Drownings. c. Drug overdoses. b. Firearms. d. Motor vehicles.

D The leading cause of all adolescent deaths in the United States is motor vehicle accidents. Drownings, firearms, and drug overdoses are major concerns in adolescence but do not cause the majority of deaths.

An adolescent boy tells the nurse that he has recently had homosexual feelings. The nurse's response should be based on knowledge that: a. This indicates that the adolescent is homosexual. b. This indicates that the adolescent will become homosexual as an adult. c. The adolescent should be referred for psychotherapy. d. The adolescent should be encouraged to share his feelings and experiences.

D These adolescents are at increased risk for health-damaging behaviors, not because of the sexual behavior itself, but because of society's reaction to the behavior. The nurse's first priority is to give the young man permission to discuss his feelings about this topic, knowing that the nurse will maintain confidentially, appreciate his feelings, and remain sensitive to his need to talk it. In recent studies among self-identified gay, lesbian, and bisexual adolescents, many of the adolescents report changing their self-labels one or more times during their adolescence.

In terms of fine motor development, what could the 3-year-old child be expected to do? a. Tie shoelaces. b. Use scissors or a pencil very well. c. Draw a person with seven to nine parts. d. Copy (draw) a circle.

D Three-year-olds are able to accomplish the fine motor skill of drawing a circle. Tying shoelaces, using scissors or a pencil very well, and drawing a person with multiple parts are fine motor skills of 5-year-old children.

Which accomplishment would the nurse expect of a healthy 3-year-old child? a. Jump rope b. Ride a two-wheel bicycle c. Skip on alternate feet d. Balance on one foot for a few seconds

D Three-year-olds are able to accomplish the gross motor skill of balancing on one foot. Jumping rope, riding a two-wheel bike, and skipping on alternate feet are gross motor skills of 5-year-old children.

Which medication may be given to high risk children after exposure to chickenpox to prevent varicella? a. Acyclovir b. Vitamin A c. Diphenhydramine hydrochloride d. Varicella zoster immune globulin (VZIG)

D VZIG is given to high risk children to help prevent the development of chickenpox. Immune globulin intravenous may also be recommended. Acyclovir is given to immunocompromised children to reduce the severity of symptoms. Vitamin A reduces morbidity and mortality associated with the measles. The antihistamine diphenhydramine is administered to reduce the itching associated with chickenpox.

The weight loss of anorexia nervosa is often triggered by: a. Sexual abuse. c. Independence from family. b. School failure. d. Traumatic interpersonal conflict.

D Weight loss may be triggered by a typical adolescent crisis such as the onset of menstruation or a traumatic interpersonal incident; situations of severe family stress such as parental separation or divorce; or circumstances in which the young person lacks personal control, such as being teased, changing schools, or entering college. There may in fact be a history of sexual abuse; however, this is not the trigger. These adolescents are often overachievers who are successful in school, not failures in school. The adolescent is most often enmeshed with his or her family.

A, B (Rolling from abdomen to back and placing the feet in the mouth when supine are developmentally appropriate for a 5-month-old infant. Rolling from back to abdomen is developmentally appropriate for a 6-month-old infant. An 8-month-old infant should be able to sit erect without support. A 10-month-old infant can usually move from a prone to a sitting position.)

In terms of gross motor development, what would the nurse expect a 5-month-old infant to do (select all that apply)? a. Roll from abdomen to back. b. Put feet in mouth when supine. c. Roll from back to abdomen. d. Sit erect without support. e. Move from prone to sitting position.

A (roll from abdomen to back Rolling from abdomen to back is developmentally appropriate for a 5-month-old infant. The ability to roll from back to abdomen usually occurs at 6 months old. Sitting erect without support is a developmental milestone usually achieved by 8 months. The 10-month-old infant can usually move from a prone to a sitting position.)

In terms of gross motor development, what would the nurse expect a 5-month-old infant to do? a. Roll from abdomen to back. b. Roll from back to abdomen. c. sit erect without support d. Move from prone to sitting position.

C (This is a good parent-child interaction. The 18-month-old is capable of building a tower of 3 or 4 blocks. The ability to build towers of blocks usually begins at age 15 months. With ongoing development, the child is able to build taller towers. At this age, children imitate others around them and no longer throw blocks.)

In the clinic waiting room, a nurse observes a parent showing an 18-month-old child how to make a tower out of blocks. In this situation the nurse should recognize that: a. Blocks at this age are used primarily for throwing. b. Toddlers are too young to imitate the behavior of others. c. Toddlers are capable of building a tower of blocks. d. Toddlers are too young to build a tower of blocks.

C (reassure the mother that this is very normal at this age Sucking is an infant's chief pleasure, and she may not be satisfied by bottle-feeding or breastfeeding alone. During infancy and early childhood, there is no need to restrict nonnutritive sucking. Dental damage does not appear to occur unless the use of the pacifier or finger persists after age 4 to 6 years. The nurse should explore with the mother her feelings about pacifier vs. thumb. This is a normal behavior to meet nonnutritive sucking needs. No data support that Latasha has sensory deprivation.)

Latasha is a breastfed infant being seen in the clinic for her 6-month checkup. Her mother tells the nurse that Latasha recently began to suck her thumb. The best nursing intervention is to: a. Recommend that the mother substitute a pacifier for Latasha's thumb. b. Assess Latasha for other signs of sensory deprivation. c. Reassure the mother that this is very normal at this age. d. Suggest that the mother breastfeed Latasha more often to satisfy sucking needs.

B (bowel control is accomplished before bladder control, so the parent should focus on bowel training first.)

One of the major tasks of toddlerhood is toilet training. In teaching the parents about a child's readiness for toilet training, it is important for the nurse to emphasize that: A. nighttime bladder control develops first, so parents should focus on that in the initial teaching with their toddler. B. bowel control is accomplished before bladder control, so the parent should focus on bowel training first. C. the toddler must have the gross motor skill to climb up to the adult toilet before training is begun. D. the universal age for toilet training to begin is 2 years, and the universal age for completion is 4 years.

C (Changing the infant's position frequently. Changing the infant's position frequently may be beneficial. The parent can walk holding the infant face down and with the infant's chest across the parent's arm. The parent's hand can support the infant's abdomen, applying gentle pressure. Gently massaging the abdomen is effective in some infants. Pacifiers can be used for meeting additional sucking needs. The infant should not be placed where monitoring cannot be done. The infant can be placed in the crib and allowed to cry. Periodically, the infant should be picked up and comforted.)

Parent guidelines for relieving colic in an infant include: a. Avoiding touching the abdomen. b. Avoiding using a pacifier. c. Changing the infant's position frequently. d. Placing the infant where the family cannot hear the crying.

D (This is a common and accepted practice, especially in some cultural groups. Co-sleeping or sharing the family bed, in which the parents allow the children to sleep with them, is a common and accepted practice in many cultures. Parents should evaluate the options available and avoid conditions that place the infant at risk. Population-based studies are currently underway; no evidence at this time supports or abandons the practice for safety reasons. This is the age at which children are just beginning to individuate. Increased daytime activity may help decrease sleep problems in general, but co-sleeping is a culturally determined phenomenon.)

Parents tell the nurse that their 1-year-old son often sleeps with them. They seem unconcerned about this. The nurse's response should be based on the knowledge that: a. Children should not sleep with their parents. b. Separation from parents should be completed by this age. c. Daytime attention should be increased. d. This is a common and accepted practice, especially in some cultural groups.

A (Most toddlers exhibit a physiologic anorexia in response to the decreased nutritional requirement associated with the slower growth rate. Parents should assist the child to develop healthy eating habits. The toddler is often unable to sit through a meal. Frequent nutritious snacks are a good way to ensure proper nutrition. To help with developing healthy eating habits, food should not be used as positive or negative reinforcement for behavior. The child may develop habits of overeating or eat nonnutritious foods in response.)

Parents tell the nurse that their toddler daughter eats little at mealtimes, only sits at the table with the family briefly, and wants snacks "all the time." The nurse should recommend that the parents: a. Give her planned, frequent, and nutritious snacks. b. Offer rewards for eating at mealtimes. c. Avoid snacks so she is hungry at mealtimes. d. Explain to her in a firm manner what is expected of her.

A (infants' temperaments are part of their unique characteristics Infant temperament has a strong biologic component. Together with interactions with the environment, primarily the family, the biologic component contributes to the infant's unique temperament. Children perceived as difficult may respond better to scheduled feedings and structured caregiving routines than to demand feedings and frequent changes in routines. Sara's temperament has been created by both biologic and environmental factors. The nurse should provide guidance in parenting techniques that are best suited to Sara's temperament.)

Sara, age 4 months, was born at 35 weeks' gestation. She seems to be developing normally, but her parents are concerned because she is a "more difficult" baby than their other child, who was term. The nurse should explain that: a. Infants' temperaments are part of their unique characteristics. b. Infants become less difficult if they are not kept on scheduled feedings and structured routines. c. Sara's behavior is suggestive of failure to bond completely with her parents. d. Sara's difficult temperament is the result of painful experiences in the neonatal period.

B (Toddlers need small, frequent meals. Nutritious selection throughout the day, rather than quantity, is more important with this age-group. Physiologically, growth slows and appetite decreases during the toddler period. Milk consumption should not exceed 16 to 24 oz daily. Juice should be limited to 4 to 6 oz per day. Increasing the amount of milk will only further decrease solid food intake. Supplemental vitamins are important for all children, but they do not increase appetite.)

The mother of a 14-month-old child is concerned because the child's appetite has decreased. The best response for the nurse to make to the mother is: a. "It is important for your toddler to eat three meals a day and nothing in between." b. "It is not unusual for toddlers to eat less." c. "Be sure to increase your child's milk consumption, which will improve nutrition." d. "Giving your child a multivitamin supplement daily will increase your toddler's appetite."

C (developmental/neurologic evaluation is needed.)

The nurse is assessing a 6-month-old infant who smiles, coos, and has a strong head lag. The nurse should recognize that: A. this assessment is normal. B. the child is probably cognitively impaired. C. developmental/neurologic evaluation is needed. D. the parent needs to work with the infant to stop head lag.

A, B, D, E (Temper tantrums are a common response to anger and frustration in toddlers. They occur more often when toddlers are tired, hungry, bored, or excessively stimulated. A nap prior to fatigue or a snack if mealtime is delayed will be helpful in alleviated the times when tantrums are most likely to occur. Tantrums may include screaming, kicking, throwing things, biting themselves, or banging their head. Effective management of tantrums includes safely isolating and ignoring the child. The child should learn that nothing is gained by having a temper tantrum. Giving in to the child's demands only increases the behavior.)

The nurse is assessing parental knowledge of temper tantrums. Which are true statements regarding temper tantrums (select all that apply)? a. Temper tantrums are a common response to anger and frustration in toddlers. b. Temper tantrums often include screaming, kicking, throwing things, and head banging. c. Parents can effectively manage temper tantrums by giving in to the child's demands. d. Children having temper tantrums should be safely isolated and ignored. e. Parents can learn to anticipate times when tantrums are more likely to occur.

D (encourage the toddler to do things for himself or herself when he or she is capable of doing them.)

The nurse is discussing toddler development with a parent. Which intervention will foster the achievement of autonomy? A. help the toddler complete tasks. B. provide opportunities for the toddler to play with other children. C. help the toddler learn the difference between right and wrong. D. encourage the toddler to do things for himself or herself when he or she is capable of doing them.

D (The decision about fluoride supplementation cannot be made until it is known whether the water supply contains fluoride and the amount. It is difficult to teach this age-group to spit out the mouthwash. Swallowing fluoridated mouthwashes can contribute to fluorosis. Fluoridated toothpaste is still indicated, but very small amounts are used. Fluoride supplementation is not recommended until after age 6 months.)

The nurse is discussing with a parent group the importance of fluoride for healthy teeth. The nurse should recommend that the parents: a. Use fluoridated mouth rinses in children older than 1 year. b. Have children brush teeth with fluoridated toothpaste unless fluoride content of water supply is adequate. c. Give fluoride supplements to breastfed infants beginning at age 1 month. d. Determine whether water supply is fluoridated.

A (a normal finding Because the anterior fontanel normally closes between ages 12 and 18 months, this is a normal finding, and no further intervention is required.)

The nurse is doing a routine assessment on a 14-month-old infant and notes that the anterior fontanel is closed. This should be interpreted as: a. A normal finding. b. A questionable finding—the infant should be rechecked in 1 month. c. An abnormal finding—indicates the need for immediate referral to a practitioner. d. An abnormal finding—indicates the need for developmental assessment.

A (is old enough to understand the word "No." By age 10 months, children are able to associate meaning with words. The child should be old enough to understand the word "No." The 10-month-old is too young to understand the purpose of an electrical outlet. The father is using both verbal and physical cues to teach safety measures and alert the child to dangerous situations. Physical discipline should be avoided.)

The nurse is interviewing the father of 10-month-old Megan. She is playing on the floor when she notices an electrical outlet and reaches up to touch it. Her father says "No" firmly and removes her from near the outlet. The nurse should use this opportunity to teach the father that Megan: a. Is old enough to understand the word "No." b. Is too young to understand the word "No." c. Should already know that electrical outlets are dangerous. d. Will learn safety issues better if she is spanked.

D (sudden infant death syndrome (SIDS).)

The nurse is interviewing the parents of a 4-month-old male infant brought to the hospital emergency department. The infant is dead on arrival, and no attempt at resuscitation is made. The parents state that the baby was found in his crib with a blanket over his head, lying face down in bloody fluid from his nose and mouth. They say he was "just fine" when they put him in his crib already asleep. The nurse should suspect his death was caused by: A. suffocation. B. child abuse. C. infantile apnea. D. sudden infant death syndrome (SIDS).

D (The 17-month-old is in the fifth stage of the sensorimotor phase: tertiary circular reactions. The child uses active experimentation to achieve previously unattainable goals. Trust is Erikson's first stage. Preoperations is the stage of cognitive development, usually present in older toddlers and preschoolers. Secondary circular reactions last from about ages 4 to 8 months.)

The nurse is planning care for a 17-month-old child. According to Piaget, in what stage would the nurse expect the child to be? a. Trust b. Preoperations c. Secondary circular reaction d. Tertiary circular reaction

B, E (B. Place iron toward the back side of the mouth with a dropper. E. Apply barrier ointment if needed to buttocks.)

The nurse is providing education to a parent of a 10-month-old infant receiving iron supplements. What will be included in the teaching? (Select all that apply.) A. Administer iron with meals. B. Place iron toward the back side of the mouth with a dropper. C. Mix iron with milk for greater absorption. D. Report black, tarry stools to health care provider. E. Apply barrier ointment if needed to buttocks.

A (The major reason for poisoning in the home is improper storage. Toddlers can climb, unlatch cabinets, and obtain access to high-security places. For medications, only a locked cabinet is safe. Toddlers can climb by using furniture. High places are not a deterrent to an exploring toddler. Toddlers are not able to generalize as dangerous all of the different forms of medications that may be available in the home. Teaching them not to touch medicines is not feasible. Many parents require medications for chronic illnesses. They must be taught safe storage for their home and when they visit other homes.)

The parent of 16-month-old Chris asks, "What is the best way to keep Chris from getting into our medicines at home?" The nurse should advise that: a. "All medicines should be locked securely away." b. "The medicines should be placed in high cabinets." c. "Chris just needs to be taught not to touch medicines." d. "Medicines should not be kept in the homes of small children."

C ("She may need to begin taking them at age 6 months." Fluoride supplementation is recommended by the American Academy of Pediatrics beginning at age 6 months if the child is not drinking adequate amounts of fluoridated water. The recommendation is to begin supplementation at 6 months, not at 2 weeks. The amount of water that is ingested and the amount of fluoride in the water are evaluated when supplementation is being considered.)

The parent of 2-week-old Sarah asks the nurse if Sarah needs fluoride supplements because she is exclusively breastfed. The nurse's best response is: a. "She needs to begin taking them now." b. "They are not needed if you drink fluoridated water." c. "She may need to begin taking them at age 6 months." d. "She can have infant cereal mixed with fluoridated water instead of supplements."

D ("Feeding himself will help foster his growth and development. Perhaps we can discuss ways to make the messes more tolerable.")

The parent of a 12-month-old infant says to the nurse, "He pushes the teaspoon right out of my hand when I feed him. I can't let him feed himself; he makes too much of a mess." The nurse's BEST response is: A. "It's important not to give in to this kind of temper tantrum at this age. Simply ignore the behavior and the mess." B. "You need to try different types of utensils, bowls, and plates. Some are specifically designed for young children." C. "It's important to let him make a mess. Just try not to worry about it so much." D. "Feeding himself will help foster his growth and development. Perhaps we can discuss ways to make the messes more tolerable."

D (Hot dogs must be cut into small, irregular pieces to prevent aspiration. Hot dogs are of a consistency, diameter, and round shape that may cause complete obstruction of the child's airway. If given to young children, the hot dog should be cut into small irregular pieces rather than served whole or in slices. The child's digestive system is mature enough to digest hot dogs. To eat the hot dog safely, the child should be sitting down, and the hot dog should be appropriately cut into irregularly shaped pieces.)

The parents of a 12-month-old child ask the nurse if the child can eat hot dogs. The nurse's reply should be based on knowing that: a. The child is too young to digest hot dogs. b. The child is too young to eat hot dogs safely. c. Hot dogs must be sliced into sections to prevent aspiration. d. Hot dogs must be cut into small, irregular pieces to prevent aspiration.

A (The baby talk is a sign of regression in the toddler. It should be ignored, while praising the child for developmentally appropriate behaviors. Regression is children's way of saying that they are expressing stress. The parents should not introduce new expectations and should allow the child to master the developmental tasks without criticism.)

The parents of a 2-year-old tell the nurse that they are concerned because the toddler has started to use "baby talk" since the arrival of their new baby. The nurse should recommend that the parents: a. Ignore the "baby talk." b. Explain to the toddler that "baby talk" is for babies. c. Tell the toddler frequently, "You are a big kid now." d. Encourage the toddler to practice more advanced patterns of speech.

D (Acceptable to encourage head control and turning over. These parents are implementing the guidelines to reduce the risk of SIDS. Infants should sleep on their backs and then be placed on their abdomens when awake to enhance development of milestones such as head control. The face-down position while awake and positioning on the back for sleep are acceptable because they reduce risk of SIDS and allow achievement of developmental milestones. These position changes encourage gross motor, not fine motor, development.)

The parents of a 3-month-old infant report that their infant sleeps supine (face up) but is often prone (face down) while awake. The nurse's response should be based on knowledge that this is: a. Unacceptable because of the risk of sudden infant death syndrome (SIDS). b. Unacceptable because it does not encourage achievement of developmental milestones. c. Unacceptable to encourage fine motor development. d. Acceptable to encourage head control and turning over.

B (Beginning to put her to bed while still awake)

The parents of a 5-month-old girl complain to the nurse that they are exhausted because she still wakes up as often as every 1 to 2 hours during the night. When she awakens, they change her diaper, and her mother nurses her back to sleep. What should the nurse suggest to help them deal with this problem? A. Putting her in parents' bed to cuddle B. Beginning to put her to bed while still awake C. Letting her cry herself back to sleep D. Giving her a bottle of formula instead of breastfeeding her so often at night

D (There is no need to restrain nonnutritive sucking during infancy.)

The parents of a 9-month-old infant tell the nurse that they are worried about their baby's thumb-sucking. What is the nurse's BEST reply? A. A pacifier should be substituted for the thumb. B. Thumb-sucking should be discouraged by age 12 months. C. Thumb-sucking should be discouraged when the teeth begin to erupt. D. There is no need to restrain nonnutritive sucking during infancy.

D (this is normal because of the immaturity of digestive processes at this age. The immaturity of the digestive tract is evident in the appearance of the stools. Solid foods are passed incompletely broken down in the feces. An excess quantity of fiber predisposes the child to large, bulky stools. This is a normal part of the maturational process, and no further investigation is necessary.)

The parents of a 9-month-old infant tell the nurse that they have noticed foods such as peas and corn are not completely digested and can be seen in their infant's stool. The nurse bases her explanation on knowing that: a. Children should not be given fibrous foods until the digestive tract matures at age 4 years. b. The infant should not be given any solid foods until this digestive problem is resolved. c. This is abnormal and requires further investigation. d. This is normal because of the immaturity of digestive processes at this age.

D (The arrival of a new infant represents a crisis for even the best-prepared toddler. Toddlers have their entire schedule and routines disrupted because of the new family member. The nurse should work with parents on ways to involve the toddler in the newborn's care and help focus attention on the toddler. The toddler does not hate the infant. This is an expected response to the changes in routines and attention that affect the toddler. This is a normal response. The toddler can be provided with a doll to tend to its needs when the parent is performing similar care for the newborn.)

The parents of a newborn say that their toddler "hates the baby . . . he suggested that we put him in the trash can so the trash truck could take him away." The nurse's best reply is: a. "Let's see if we can figure out why he hates the new baby." b. "That's a strong statement to come from such a small boy." c. "Let's refer him to counseling to work this hatred out. It's not a normal response." d. "That is a normal response to the birth of a sibling. Let's look at ways to deal with this."

C (Select an area that is safe and nonstimulating, such as a hallway.)

The parents of a toddler ask the nurse for suggestions about discipline. When discussing the use of timeouts, which of the following suggestions should the nurse include? A. Send the child to his or her room. B. If the child cries, refuses, or is more disruptive, try another approach. C. Select an area that is safe and nonstimulating, such as a hallway. D. The general rule for length of time is 1 hour per year of age.

B, C, D, E (B. Suggest that parents ignore the behavior as long as child is not harming self. C. Encourage the parents to provide comfort once the child has calmed down. D. Ask parents to praise the child for positive behavior when not having a tantrum. E. Tell parents not to give in to the original request that started the temper tantrum.)

The parents of a toddler ask the nurse how to handle their child's increasing number of temper tantrums. The nurse should include which positive reinforcement methods of reducing the number of tantrums? (Select all that apply.) A. Suggest that parents provide the child an "all or none" position. B. Suggest that parents ignore the behavior as long as child is not harming self. C. Encourage the parents to provide comfort once the child has calmed down. D. Ask parents to praise the child for positive behavior when not having a tantrum. E. Tell parents not to give in to the original request that started the temper tantrum.

D ("Becoming a fussy eater is expected during the toddler years.")

The parents of a toddler express frustration to the nurse because their child is a "fussy eater." The nurse's BEST response is: A. "You should provide larger servings of different foods. B. "Provide more bland food varieties as toddlers have few food preferences." C. "Table manners will improve if you provide finger foods." D. "Becoming a fussy eater is expected during the toddler years."

A (Using a transitional object.)

The parents of a toddler state their child is having trouble sleeping. What is the nurse's BEST suggestion to improve sleep habits? A. Using a transitional object. B. Varying the bedtime ritual. C. Restricting stimulating activities during the day. D. Explaining away fears.

D (If the need for basic trust has been satisfied, toddlers can give up dependence for control, independence, and autonomy. One of the tasks that the toddler is concerned with is the ability to withstand delayed gratification. Development of a conscience occurs during the preschool years. The recognition of sex differences occurs during the preschool years. The ability to get along with age mates develops during the preschool and school-age years.)

The psychosocial developmental tasks of toddlerhood include: a. Development of a conscience. b. Recognition of sex differences. c. Ability to get along with age mates. d. Ability to withstand delayed gratification.

B (Play develops from the solitary play of infancy to the parallel play of toddlers. The toddler plays alongside other children, not with them. This typical behavior of the toddler is not intentionally aggressive. Shared play is not within their cognitive development. Toddlers do not conceptualize shared play. Because the toddler cannot view the situation from the perspective of the other child, it is okay to take the toy. Therefore, no right or wrong is associated with taking a toy.)

Two toddlers are playing in a sandbox when one child suddenly grabs a toy from the other child. The best interpretation of this behavior is that: a. This is typical behavior because toddlers are aggressive. b. This is typical behavior because toddlers are egocentric. c. Toddlers should know that sharing toys is expected of them. d. Toddlers should have the cognitive ability to know right from wrong.

B (At this age the child is in the final sensorimotor stage. Children will now search for an object in several potential places, even though they saw only the original hiding place. Children have a more developed sense of objective permanence. They will search for objects even if they have not seen them hidden. Putting an object in a container but being unable to take it out indicates tertiary circular reactions. An embryonic sense of time exists; although toddlers may behave appropriately to time-oriented phrases, their sense of timing is exaggerated.)

What describes a toddler's cognitive development at age 20 months? a. Searches for an object only if he or she sees it being hidden b. Realizes that "out of sight" is not out of reach c. Puts objects into a container but cannot take them out d. Understands the passage of time such as "just a minute" and "in an hour"

C (Decreasing anxiety, particularly separation anxiety, is the function of a transitional object; it provides comfort to the toddler in stressful situations and helps make the transition from dependence to autonomy. A decrease in parental guilt (distress) is an indirect benefit of a transitional object. A transitional object may be part of a bedtime ritual, but it may not keep the child quiet at bedtime. A transitional object does not significantly affect negativity and tantrums, but it can comfort a child after tantrums.)

What is the primary purpose of a transitional object? a. It helps the parents deal with the guilt they feel when they leave the child. b. It keeps the child quiet at bedtime. c. It is effective in decreasing anxiety in the toddler. d. It decreases negativism and tantrums in the toddler.

A (Give the toddler a doll on which he or she can imitate parenting.)

What should the nurse recommend to help a toddler cope with the birth of a new sibling? A. Give the toddler a doll on which he or she can imitate parenting. B. Discourage the toddler from helping with care of new sibling. C. Prepare the toddler for upcoming changes about 1 to 2 weeks before birth of the sibling. D. Explain to the toddler that a new playmate will soon come home.

A (engage in parallel play.)

When completing the health assessment for a 2-year-old child, the nurse should expect the child to: A. engage in parallel play. B. fully dress self with supervision. C. have a vocabulary of at least 500 words. D. be one third of the adult height.

D (placing the car seat in the back seat of the car facing forward.)

When explaining the proper restraint of toddlers in motor vehicles to a group of parents, the nurse should include: A. wearing safety belts snugly over the toddler's abdomen. B. placing the car seat in the front passenger seat if there is an airbag. C. using lap and shoulder belts when the child is over 3 years of age. D. placing the car seat in the back seat of the car facing forward.

A, B, C, E (A. initiate an immunization record. B. confirm the hepatitis B status of the newborn's mother. C. obtain a syringe with a 25-gauge, 5/8-inch needle. E. confirm that the newborn's mother has signed the informed consent.)

When preparing to administer Hepatitis B vaccine to a newborn, the nurse should: (Select all that apply.) A. initiate an immunization record. B. confirm the hepatitis B status of the newborn's mother. C. obtain a syringe with a 25-gauge, 5/8-inch needle. D. assess the dorsogluteal muscle as the preferred site for injection. E. confirm that the newborn's mother has signed the informed consent.

C (actively searches for a hidden object During the first 6 months of life, infants believe that objects exist only as long as they can see them. When infants search for an object that is out of sight, this signals the attainment of object permanence, whereby an infant knows that an object exists even when it is not visible. Between ages 8 and 12 weeks, infants begin to respond differentially to their mothers. They cry, smile, vocalize, and show distinct preference for their mothers. This preference is one of the stages that influence the attachment process, but it is too early for object permanence. Recognizing familiar objects is an important transition for the infant, but it does not signal object permanence. The ability to understand cause and effect, such as pulling on a string to secure an object, is part of secondary schema development.)

Which behavior indicates that an infant has developed object permanence? a. Recognizes familiar face such as the mother b. Recognizes familiar object such as a bottle c. Actively searches for a hidden object d. Secures objects by pulling on a string

C (Children should first see the dentist 6 months after the first primary tooth erupts and no later than age 30 months. Toddlers need fluoride supplements when they use a water supply that is not fluorinated. Toddlers also require supervision with dental care. The parent should finish brushing areas not reached by the child. A small nylon bristle brush works best for cleaning toddlers' teeth.)

Which comments indicate that the mother of a toddler needs further teaching about dental care? a. "We use well water so I give my toddler fluoride supplements." b. "My toddler brushes his teeth with my help." c. "My child will not need a dental checkup until his permanent teeth come in." d. "I use a small nylon bristle brush for my toddler's teeth."

D (Toddlers continue to have the short, straight internal ear canal of infants. The lymphoid tissue of the tonsils and adenoids continues to be relatively large. These two anatomic conditions combine to predispose the toddler to frequent infections. The abdominal respirations and lowered pulse and respiratory rate of toddlers do not affect their susceptibility to infection. The defense mechanisms are more efficient compared with those of infancy.)

Which factor is most important in predisposing toddlers to frequent infections such as otitis media, tonsillitis, and upper respiratory tract infections? a. Respirations are abdominal. b. Pulse and respiratory rates are slower than those in infancy. c. Defense mechanisms are less efficient than those during infancy. d. Short, straight internal ear/throat structures and large tonsil/adenoid lymph tissue are present.

A, C, D (An 18-month-old child can jump in place with both feet, throw a ball overhand without falling, and pull and push toys. Taking a few steps on tiptoe and standing on one foot momentarily are not acquired until 30 months of age.)

Which gross motor milestones should the nurse assess in an 18-month-old child (select all that apply)? a. Jumps in place with both feet b. Takes a few steps on tiptoe c. Throws ball overhand without falling d. Pulls and pushes toys e. Stands on one foot momentarily

A (Give large push-pull toys for kinesthetic stimulation The 12-month-old child is able to pull to a stand and walk holding on or independently. Appropriate toys for a child of this age include large push-pull toys for kinesthetic stimulation. A cradle gym should not be placed across the crib. Finger paints are appropriate for older children. A 12-month-old child does not have the stability to use a stick horse.)

Which information could be given to the parents of a 12-month-old child regarding appropriate play activities for this age? a. Give large push-pull toys for kinesthetic stimulation. b. Place cradle gym across crib to facilitate fine motor skills. c. Provide child with finger paints to enhance fine motor skills. d. Provide stick horse to develop gross motor coordination

A, C, E (An important aspect of compassionate care for parents experiencing a SIDS incident is allowing them to say good-bye to their infant. These are the parents' last moments with their infant, and they should be as quiet, meaningful, peaceful, and undisturbed as possible. Because the parents leave the hospital without their infant, it is helpful to accompany them to the car or arrange for someone else to take them home. A debriefing session may help health care workers who dealt with the family and deceased infant to cope with emotions that are often engendered when a SIDS victim is brought into the acute care facility. An autopsy may clear up possible misconceptions regarding the death. When the parents return home, a competent, qualified professional should visit them after the death as soon as possible.)

Which interventions should the nurse implement when caring for a family of a sudden infant death syndrome (SIDS) infant (select all that apply)? a. Allow parents to say goodbye to their infant. b. Once parents leave the hospital, no further follow-up is required. c. Arrange for someone to take the parents home from the hospital. d. Avoid requesting an autopsy of the deceased infant. e. Conduct a debriefing session with the parents before they leave the hospital.

A (Establish a structured routine and follow it consistently. The infant with failure to thrive should have a structured routine that is followed consistently. Disruptions in other activities of daily living can have a great impact on feeding behaviors. Bathing, sleeping, dressing, playing, and feeding are structured. The nurse should talk to the infant by giving directions about eating. This will help the infant maintain focus. Young children should be held while being fed, and older children can sit at a feeding table. The infant should be fed in the same manner at each meal. The infant can engage in sensory and play activities at times other than mealtime.)

Which is an important nursing consideration when caring for an infant with failure to thrive? a. Establish a structured routine and follow it consistently. b. Maintain a nondistracting environment by not speaking to the infant during feeding. c. Place the infant in an infant seat during feedings to prevent overstimulation. d. Limit sensory stimulation and play activities to alleviate fatigue.

B (Large plastic blocks are appropriate for a toddler in isolation. Play objects for toddlers must still be chosen with an awareness of danger from small parts. Large, sturdy toys without sharp edges or removable parts are safest. Small plastic toys such as Legos can cause choking or can be aspirated. Balloons can cause significant harm if swallowed or aspirated. Coloring book and crayons would be too advanced for a toddler.)

Which play item should the nurse bring from the playroom to a hospitalized toddler in isolation? a. Small plastic Legos b. Set of large plastic building blocks c. Brightly colored balloon d. Coloring book and crayons

B, C, E (To prevent accidental poisoning, parents should be taught to promptly discard empty poison containers, to know the number of the nearest poison control center, and to caution the child against eating nonedible items, such as plants. Parents should place all potentially toxic agents, including cosmetics, personal care items, cleaning products, pesticides, and medications, in a locked cabinet, not in the garage. Parents should be taught to never remove labels from containers of toxic substances.)

Which should the nurse teach to parents of toddlers about accidental poison prevention (select all that apply)? a. Keep toxic substances in the garage. b. Discard empty poison containers. c. Know the number of the nearest poison control center. d. Remove colorful labels from containers of toxic substances. e. Caution child against eating nonedible items, such as plants.

B (Giving a bottle of milk or juice at naptime or bedtime predisposes the child to this syndrome.)

Which statement about early childhood caries (ECC) is correct? A. The syndrome is distinguished by protruding upper front teeth, resulting from sucking on a hard nipple. B. Giving a bottle of milk or juice at naptime or bedtime predisposes the child to this syndrome. C. The syndrome can be prevented by breastfeeding. D. Giving the child juice in the bottle instead of milk at bedtime prevents this syndrome.

B (Voluntary control of the anal and urethral sphincters is achieved sometime after the child is walking. The child must be able to recognize the urge to let go and to hold on. The child must want to please the parent by holding on rather than pleasing self by letting go. Bowel training precedes bladder training. Watching older siblings provides role modeling and facilitates imitation for the toddler. The child should be introduced to the potty chair or toilet in a nonthreatening manner.)

Which statement about toilet training is correct? a. Bladder training is usually accomplished before bowel training. b. Wanting to please the parent helps motivate the child to use the toilet. c. Watching older siblings use the toilet confuses the child. d. Children must be forced to sit on the toilet when first learning.

C (birth weight doubles by age 5 months and triples by age 1 year Growth is very rapid during the first year of life. The birth weight approximately doubles by age 5 to 6 months and triples by age 1 year. The anterior fontanel closes at age 12 to 18 months. Binocularity is not established until age 15 months. Maternal iron stores are usually depleted by age 6 months.)

Which statement best describes the infant's physical development? a. Anterior fontanel closes by age 6 to 10 months. b. Binocularity is well established by age 8 months. c. Birth weight doubles by age 5 months and triples by age 1 year. d. Maternal iron stores persist during the first 12 months of life.

C (Being persistent through 10 to 15 minutes of food refusal)

Which strategy might be recommended for an infant with failure-to-thrive to increase caloric intake? A. Using developmental stimulation by a specialist during feedings B. Avoiding solids until after the bottle is well accepted C. Being persistent through 10 to 15 minutes of food refusal D. Varying schedule of routine activities on a daily basis

A (Place the infant prone for 30 to 60 minutes per day. Prevention of positional plagiocephaly may begin shortly after birth by implementing prone positioning or "tummy time" for approximately 30 to 60 minutes per day when the infant is awake. Soft mattresses or sleeping with parents (co-sleeping) are not recommended because they put the infant at a higher risk for a sudden infant death incident. To prevent plagiocephaly, prolonged placement in car safety seats should be avoided.)

With the goal of preventing plagiocephaly, the nurse should teach new parents to: a. Place the infant prone for 30 to 60 minutes per day. b. Buy a soft mattress. c. Allow the infant to nap in the car safety seat. d. Have the infant sleep with the parents.

The nurse is talking to the parent of a 13-month-old child. The mother states, "My child does not make noises like 'da' or 'na' like my sister's baby, who is only 9 months old." Which statement by the nurse would be most appropriate to make? a. "I am going to request a referral to a hearing specialist." b. "You should not compare your child to your sister's child." c. "I think your child is fine, but we will check again in 3 months." d. "You should ask other parents what noises their children made at this age."

a. "I am going to request a referral to a hearing specialist."

A child with autism spectrum disorder (ASD) is admitted to the hospital with pneumonia. The nurse should plan which priority intervention when caring for the child? a. Maintain a structured routine and keep stimulation to a minimum. b. Place the child in a room with a roommate of the same age. c. Maintain frequent touch and eye contact with the child. d. Take the child frequently to the playroom to play with other children.

a. Maintain a structured routine and keep stimulation to a minimum.

The major consideration when selecting toys for a child who is cognitively impaired is: a. Safety. b. Age appropriateness. c. Ability to provide exercise. d. Ability to teach useful skills.

a. Safety

Which action best facilitates lipreading by the hearing-impaired child? a. Speaking at an even rate b. Exaggerating pronunciation of words c. Avoiding using facial expressions d. Repeating in exactly the same way if child does not understand

a. Speaking at an even rate

When a child with mild cognitive impairment reaches the end of adolescence, what characteristic would be expected? a. Achieves a mental age of 5 to 6 years b. Achieves a mental age of 8 to 12 years c. Is unable to progress in functional reading or arithmetic d. Acquires practical skills and useful reading and arithmetic to an eighth-grade level

b. Achieves a mental age of 8 to 12 years

A child with autism is hospitalized with asthma. The nurse should plan care so that the: a. Parents' expectations are met. b. Child's routine habits and preferences are maintained. c. Child is supported through the autistic crisis. d. Parents need not be at the hospital.

b. Child's routine habits and preferences are maintained.

A newborn assessment shows separated sagittal suture, oblique palpebral fissures, depressed nasal bridge, protruding tongue, and transverse palmar creases. These findings are most suggestive of: a. Microcephaly. c. Cerebral palsy. b. Down syndrome. d. Fragile X syndrome.

b. Down syndrome

A parent whose child has been diagnosed with a cognitive deficit should be counseled that intellectual impairment: a. Is usually due to a genetic defect. b. May be caused by a variety of factors. c. Is rarely due to first-trimester events. d. Is usually caused by parental intellectual impairment.

b. May be caused by a variety of factors.

Appropriate interventions to facilitate socialization of the cognitively impaired child include to: a. Provide age-appropriate toys and play activities. b. Provide peer experiences such as Special Olympics when older. c. Avoid exposure to strangers who may not understand cognitive development. d. Emphasize mastery of physical skills because they are delayed more often than verbal skills.

b. Provide peer experiences such as Special Olympics when older.

Distortion of sound and problems in discrimination are characteristic of which type of hearing loss? a. Conductive b. Sensorineural c. Mixed conductive-sensorineural d. Central auditory imperceptive

b. Sensorineural

A parent asks the nurse why a developmental assessment is being conducted for a child during a routine well-child visit. The nurse answers based on the knowledge that routine developmental assessments during well-child visits are: a. Not necessary unless the parents request them. b. The best method for early detection of cognitive disorders. c. Frightening to parents and children and should be avoided. d. Valuable in measuring intelligence in children.

b. The best method for early detection of cognitive disorders.

The nurse is discussing sexuality with the parents of an adolescent girl with moderate cognitive impairment. Which should the nurse consider when dealing with this issue? a. Sterilization is recommended for any adolescent with cognitive impairment. b. Sexual drive and interest are limited in individuals with cognitive impairment. c. Individuals with cognitive impairment need a well-defined, concrete code of sexual conduct. d. Sexual intercourse rarely occurs unless the individual with cognitive impairment is sexually abused.

c. Individuals with cognitive impairment need a well-defined, concrete code of sexual conduct.

A young child who has an intelligence quotient (IQ) of 45 would be described as: a. Within the lower limits of the range of normal intelligence. b. Mildly cognitively impaired but educable. c. Moderately cognitively impaired but trainable. d. Severely cognitively impaired and completely dependent on others for care.

c. Moderately cognitively impaired but trainable.

What should the nurse keep in mind when planning to communicate with a child who has autism? a. The child has normal verbal communication. b. The child is expected to use sign language. c. The child may exhibit monotone speech and echolalia. d. The child is not listening if she is not looking at the nurse.

c. The child may exhibit monotone speech and echolalia.

Which teaching guideline helps prevent eye injuries during sports and play activities? a. Restrict helmet use to those who wear eyeglasses or contact lenses. b. Discourage the use of goggles with helmets. c. Wear eye protection when participating in high-risk sports such as paintball. d. Wear a face mask when playing any sport or playing roughly.

c. Wear eye protection when participating in high-risk sports such as paintball.

A nurse is preparing a teaching session for parents on prevention of childhood hearing loss. The nurse should include that the most common cause of hearing impairment in children is: a. Auditory nerve damage. c. Congenital rubella. b. Congenital ear defects. d. Chronic otitis media.

d. Chronic otitis media.

A 2-year-old girl has excessive tearing and corneal haziness. The nurse knows that these symptoms may indicate: a. Viral conjunctivitis. c. Congenital cataract. b. Paralytic strabismus. c. Congenital cataract. d. Infantile glaucoma.

d. Infantile glaucoma.


Ensembles d'études connexes

Colorado Journeyman practice test 1

View Set

Summer Squash, las calabazas de verano, Winter Squash las calabazas de invierno

View Set

Instructions: Language of the Computer

View Set

Modern World, Test 3, Chapters 17 & 18

View Set

Chapter 1 - Introduction - Connect Questions

View Set